Você está na página 1de 230

DESIGUALDADES E

BASES DE NUMERAÇÃO

MÉDIAS Exercício Resolvido


Uma média de uma lista de números {x1, x2, x3, ..., xn} é um valor
01. Mostre que para os números reais positivos a, b, c.
que pode substituir todos os elementos da lista sem alterar uma de
( a + b + c ) 
suas características. 1 1 1
+ + ≥9
Se esta característica é a soma dos elementos desta lista, obtemos a b c
a Média Aritmética Simples dada por: Resolução:
x1 + x 2 + ⋅ ⋅ ⋅ + xn a+b+c 3
x= ≥
n Pela desigualdade das médias
3 1
a + b1 + 1
c

Se a característica a ser considerada for o produto dos seus  1 1 1


elementos, obteremos a Média Geométrica Simples dada por: Daí ( a + b + c )  + +  ≥ 9
a b c
=g n x1x 2 ⋅ ⋅ ⋅ xn
Se a característica for a soma dos inversos dos elementos da lista,
Exercício Resolvido
obteremos a Média Harmônica Simples que é dada por:
n 02. Sejam a e b números reais tais que a > b > 0. Determine o
h=
1 1 1 1
+ + ⋅⋅⋅ + menor valor possível de a + .
x1 x 2 xn b (a − b)

Se a característica for a soma dos quadrados dos elementos da Resolução:


lista, obteremos a Média Quadrática dada por:
Pela desigualdade das médias
x12 + x 22 +  + xn2
q= 1 1 1
n a+ = (a − b) + b + ≥ 3 ⋅ 3 (a − b) ⋅ b ⋅ = 3
b (a − b) b (a − b) b (a − b)
isto é, a média quadrática é igual à raiz quadrada da média
aritmética dos quadrados dos números da lista. 1
Com a igualdade ocorrendo se ( a − b ) =b = isto é, se
Finalmente, definimos a Média Aritmética Ponderada dos números b (a − b)
a = 2 e b = 1.
da lista com pesos respectivamente iguais a p1, p2, ..., pn como sendo:
p x + p2x 2 +  + pn xn
mp = 1 1
p1 + p2 +  + pn TEOREMA 2 (DESIGUALDADE DE CAUCHY-
SCHWARZ)
Sejam a1, a2, ..., an, b1, b2, ..., bn números reais. Então
DESIGUALDADES CLÁSSICAS
(a2
1 + a22 +  + a2n )(b21 + b22 +  + b2n ) ≥ ( a1b1 + a2b2 +  + anbn )
2

TEOREMA 1 (DESIGUALDADE DAS MÉDIAS Com a igualdade ocorrendo se, e somente se, aibj = ajbi para todos
MA-MG-MH) a1 a2 a
i, j isto é, = =  = n ou b1 = b2 = ... = bn = 0.
Sejam a1, a2, ..., an inteiros positivos. Então M.A ≥ M.G ≥ M.H para b1 b2 bn
estes números isto é,
Observação
a1 + a2 +  + an n n
≥ a1a2  an ≥
n 1 1 1 Basicamente a desigualdade de Cauchy-Schwarz afirma que “o
+ ++
a1 a2 an produto da soma dos quadrados é maior que o quadrado da soma
dos produtos”.
Com a igualdade ocorrendo se, e somente se a1 = a2 = ... = an.
Uma maneira fácil de memorizar a desigualdade de Cauchy-
Para n = 2, isto não é difícil provar: Schwarz é ∑a²∑b² ≥ (∑ab)².
2 2a a 2a1a2 2a1a2 2
= 12 = ≤ = a1a2 n n
 n

1 1 a1 + a2
+
a1 a2
( a1 − a2 )
2
+ 2 a1a2 2 a1a2 O que na realidade significa ∑ a ∑b ≥  ∑ ab 
=i 1=i 1
2 2

=i 1
i i

( a1 + a2 ) − ( a1 − a2 ) (=
a1 + a2 )
2 2 2
a1 + a2
= ≤
4 4 2

PROMILITARES.COM.BR 9
DESIGUALDADES E BASES DE NUMERAÇÃO

Exercício Resolvido aparece, sempre contado da direita para a esquerda, é de segunda


ordem, enquanto que o último é de quinta ordem. O 5 é primeira
03. Mostre que para os números reais positivos a, b, c. ordem, enquanto que o .1. é de quarta ordem.
Cada terna de ordens, também contadas da direita para a
( a + b + c ) 
1 1 1
+ + ≥9 esquerda, forma uma classe. As classes são, às vezes, separadas umas
a b c das outras por meio de um ponto.
Resolução: Damos a seguir os nomes das primeiras classes e ordens:
Aplicando-se a desigualdade de Cauchy-Schwarz, temos  unidades (1a ordem)


( a + b + c ) 
1 1 1
+ + = Classe das Unidades  dezenas ( 2a ordem )
a b c 
centenas ( 3 ordem )
a
2  1   1   1  
2 2 2

( a) ( ) ( )
+ b + c  
2 2
+ +
   a   b   c  
   unidades de milhar ( 4a ordem )
  

Classe do Milhar  dezenas de milhar (5a ordem )
2
 1 1 1 
≥ a⋅ + b⋅ + c⋅  =
9 
  centenas de milhar ( 6 ordem)
a b c a

unidades de milhão ( 7a ordem )




BASES DE NUMERAÇÃO Classe do Milhão  dezenas de milhão ( 8a ordem)

centenas de milhão ( 9 ordem )
a

INTRODUÇÃO
Chama-se Sistema de Numeração a um conjunto de regras que Um inteiro N qualquer pode ser representado sob a forma
nos permitam a representação de todos os números naturais mediante N = dn · 10n + dn-1 · 10n-1 + ... + d1 · 10 + d0
vários sinais ou várias palavras. onde d0, d1, ..., dn-1, dn são inteiros que podem ser representados
Sistemas muito conhecidos e de natureza muito diversas, são o por um único dígito 0, 1, 2, ..., 9.
romano e o decimal. O primeiro decompõe o número em uma soma Por exemplo,
ou diferença de vários outros, cada um quais está representado por
um símbolo especial a saber, I , V , X , L , C , D , M. 2 · 103 + 0 · 102 + 1 · 10 + 2
O segundo, em vez de introduzir novos símbolos para estes Pode ser abreviado para (2012)(10) ou simplesmente 2012 sempre
diversos somandos utiliza o princípio do valor relativo ou posicional tendo em mente o sistema decimal. Os dois símbolos “2“ estão
segundo o qual um mesmo símbolo representa valores distintos situados nas unidades simples e nas unidades de milhar e por isso
segundo o lugar que ocupa. seus significados são diferentes. Em geral, escreveremos um número
N como o dado pela expressão acima da seguinte forma:
Particularmente, nos interessam os sistemas baseados nos mesmos
princípios que o decimal isto é, aqueles que se baseiam no fato de que N = (dndn-1...d1d0)(10)
certa quantidade de unidades de uma ordem formam uma unidade para enfatizar o papel excepcional do 10. Esta é a chamada
de ordem imediatamente superior. Tal quantidade recebe o nome de notação posicional. Sua invenção, atribuída aos Babilônios e seu
base do sistema de numeração. Se a base escolhida for o número dois desenvolvimento posterior pelos Hindus teve um enorme significado
o sistema é dito binário. Se a base escolhida for o número dezesseis o para a civilização. No simbolismo Romano por exemplo, escreveríamos
sistema é dito hexadecimal. A maneira convencional de expressar os MMXII = (mil) + (mil) + (dez) + (um) + (um)
números é a decimal. Não há uma razão particular para o uso do dez
É claro que mais e mais novos símbolos tais como I, X, M são
como base de notação a não ser fato de que temos dez dedos nas
necessários para representar números maiores enquanto que com o
mãos. Computadores utilizam o dois para os cálculos internos e oito
sistema posicional Hindu utilizado atualmente precisamos apenas dos
ou dezesseis para expô-los.
numerais Arábicos 0, 1, 2, ..., 9 independente de quão grande seja o
número.
O SISTEMA DECIMAL CLÁSSICO POSICIONAL
Existe uma distinção importante entre os números e suas OUTROS SISTEMAS POSICIONAIS
representações. No sistema de numeração decimal, o zero e os nove
Os exercícios de contagem em diversas bases possuem, antes de
primeiros inteiros positivos são denotados pelos símbolos 0, 1, 2,
qualquer coisa, a finalidade de mostrar uma estrutura nos sistemas
..., 9 respectivamente. Estes símbolos são chamados algarismos ou
de numeração, uma melhor compreensão do sistema decimal,
dígitos e são utilizados para representar todos os inteiros. O sistema é
que na realidade é o utilizado universalmente. As propriedades do
também chamado posicional, pois cada algarismo, além do seu valor
sistema decimal estão presentes em todo mecanismo de cálculo que
intrínseco, possui um peso que lhe é atribuído em função da posição
praticamos. Antes não eram estudados com tanto destaque como o
que ele ocupa no número. Este peso, sempre uma potência de dez,
são atualmente. Um estudo feito em bases diferentes da base dez
varia do seguinte modo:
permite ressaltar essas propriedades que a nossa grande familiarização
O algarismo da extrema direita tem peso 1; o seguinte, sempre com elas quase não deixa perceber. Em  particular, merecem um
da direita para a esquerda, tem peso dez; o seguinte tem peso cem; o estudo especial as bases dois, oito e dezesseis devido ao seu emprego
seguinte tem peso mil, etc. em computadores.
Portanto, os números de um a nove são representados pelos A compreensão do princípio fundamental dos sistemas diferentes
algarismos de 1 a 9. O numero dez é representado por 10, o numero do sistema decimal, mostra desde logo que a base será sempre
cem por 100, o número mil por 1000. Por exemplo, o número 21025, representada por 10 e que não pode haver um algarismo no sistema
na base 10, é a representação de com o valor da base isto é, no sistema de base cinco, o valor cinco será
2 ⋅ 104 + 1⋅ 103 + 0 ⋅ 102 + 2 ⋅ 10 + 5 =⋅
2 104 + 1⋅ 103 + 2 ⋅ 10 + 5 representado por 10cinco e não há o algarismo 5, pois o de maior valor
absoluto é igual á base menos um. Matematicamente não há nada de
Cada algarismo de um número possui uma ordem contada da especial no sistema de numeração decimal. O uso do dez como base
direita para a esquerda. Assim, no exemplo acima, o primeiro 2 que remonta ao surgimento da civilização e é atribuído ao fato de termos

10 PROMILITARES.COM.BR
DESIGUALDADES E BASES DE NUMERAÇÃO

dez dedos nas mãos. Qualquer outro número pode ser usado como
3. Se a representação b – ária de um número natural N termina
base. Assim,
com n zeros então, ele é múltiplo de bn.
4. Se a representação b – ária de um número natural N é (dndn-1...
TEOREMA d2d1d0)b então este número é divisível por b – 1 se e somente se dn
Seja b > 1 um inteiro positivo. Todo inteiro positivo N pode ser + dn-1 + ... + d2 + d1 + d0 for divisível por b – 1.
univocamente representado sob a forma
5. Se a representação b – ária de um número natural N é
N = d0 + d1b + d2b² + ... + dnbn 
(aaaaaa)
 b com a = b – 1 então N = bn – 1.
onde os “dígitos” d0, d1, ..., dn são tais que 0 ≤ di ≤ b – 1 para n alg arismos

todo i. 6. Para passar da representação b – ária de um número natural N à


sua representação bk – ária, deve-se reunir, partindo da direita para
a esquerda, os algarismos b – ários de N em grupos de k algarismos
PROVA e substituir cada um destes grupos pelo algarismo bk – ário que
A prova é por indução sobre a representação do número N. o representa. A passagem inversa se efetua escrevendo em lugar
Claramente a representação 1 = 1 para 1 é única. Suponhamos de cada algarismo bk – ário de N a sua representação b – ária (por
indutivamente que todo inteiro 1, 2, ..., N – 1 seja univocamente meio dos algarismos b – ários). O resultado será a representação
representável e consideremos agora o inteiro N. Se d0 ≡ N(modb) b – ária de N.
N − d0
então N – d0 é divisível por b e seja N1 = . Como N1 < N, pela
b
hipótese de indução N1 é univocamente representável na forma EXERCÍCIOS DE

N1 =
N − d0
b
= d1 + d2b + d3b2 +  + dnbn −1 TREINAMENTO
E então claramente,
N=d0 + N1b =d0 + d1b + d2b2 + d3b3 +  + dnbn 01. Se a, b e c são reais positivos cuja soma é 1, determine o valor
1 1 1
mínimo de + + .
Finalmente, suponha que N possua alguma outra representação a b c
também desta forma isto é, a) 9

N = d0 + d1b + d2b2 +  + dnbn = c0 + c1b + c2b2 +  + cnbn b) 3


c) 1
Então, d0 = C0 = r uma vez que eles são iguais ao resto da divisão 1
de N por b. Agora o número d)
3
N−r
N1 = = d1 + d2b + d3b2 +  + dnbn −1 = c1 + c2b + c3b2 +  + cnbn −1 1
b e)
9
Possui duas representações diferentes o que contradiz hipótese
02. (UECE 2015) Se x é a média aritmética dos números reais a, b e c,
indutiva uma vez que supomos a veracidade do resultado para todo
y é a média aritmética de seus quadrados, então a média aritmética de
N1 < N.
seus produtos dois a dois ab, ac, bc, em função de x e y é
Sugestão: considere o quadrado da soma dos três números.
COROLÁRIO
3x 2 − y
Usaremos a notação N = (dndn-1...d1d0)(b) para expressar N = d0 + a) .
d1b + d2b² + ... + dnbn. Os dígitos di podem ser determinados através 2
de aplicações sucessivas do algoritmo da divisão da seguinte maneira 3x + y
b) .
N = q1b + d0 , ( 0 ≤ d0 < b ) 2
q1 = q2b + d1, ( 0 ≤ d1 < b ) c)
3x 2 + y
.
    2
qn = 0 ⋅ b + dn ( 0 ≤ dn < b ) 3x − y
d) .
2
Assim, por exemplo, o sistema posicional de base 5 utiliza os
03. Se a, b, c são números reais menores que 1 e a + b + c = 2 o valor
dígitos 0, 1, 2, 3, 4 e podemos então escrever
abc
2012(10) = 3·54 + 1·53 + 0·52 + 2·5 + 2 mínimo de é igual a:
(1 − a)(1 − b )(1 − c )
Entretanto, na era dos computadores, é o sistema binário (base 2) a) 3
que se apresenta como o mais importante. Nele temos somente dois b) 4
dígitos 0 e 1 e uma tábua de multiplicação muito simples embora no c) 5
sistema binário as representações dos números fiquem muito extensas
como por exemplo, d) 6
86(10) = 1·26 + 0·25 + 1·24 + 0·23 + 1·22 + 1·2 + 0 = 1010110(2) e) 8

Observações 04. Quando expresso na base 8, N! termina com 21 zeros. O maior


inteiro positivo N, escrito na base 10, com esta propriedade é igual a:
1. Se a representação b – ária de um número natural N consiste de
n algarismos então podemos afirmar que ele pertence ao intervalo a) 64
[bn-1,bn) isto é, 10n-1 ≤ n < 10n. b) 65
2. Se a representação b – ária de um número natural N consiste c) 66
do algarismo 1(um) seguido de n zeros então a sua representação d) 67
decimal é bn.
e) 68

PROMILITARES.COM.BR 11
DESIGUALDADES E BASES DE NUMERAÇÃO

05. (IME 2013) Considere as inequações abaixo: 11. O preço de um carro usado é mostrado, em reais, em 4 cartões
I. a² + b² + c² ≥ ab + bc + ca sobre o para-brisa. Cada cartão mostra um dígito. Se o cartão com o
dígito do milhar voar com o vento, o preço mostrado será 49 vezes
II. a³ + b³ ≥ a²b + ab² menor que o original. Qual o número no cartão do milhar?
III. (a² – b²) ≥ (a – b)4 a) 5
Está(ão) correta(s), para quaisquer valores reais positivos de a, b e c, b) 6
a(s) inequação(ões)
c) 7
a) II apenas.
d) 8
b) I e II apenas.
e) 9
c) I e III apenas.
d) II e III apenas. 12. (CN 2013) Os números (35041000)7, (11600)7 e (62350000)7
e) I, II e III. estão na base 7. Esses números terminam, respectivamente, com 3,
2 e 4 zeros. Com quantos zeros terminará o número na base decimal
06. A representação decimal de um natural a possui n algarismos n = 212012, na base 7?
enquanto que a representação decimal de a³ possui m algarismos. a) 2012
Dentre os valores abaixo, aquele que não pode ser possível para b) 2013
m + n é:
c) 2014
a) 2010
d) 2015
b) 2011
e) 2016
c) 2012
d) 2013 13. Para determinar uma grandeza desconhecida x, foram feitas
e) 2014 várias medições. Os resultados obtidos foram x1, x2, ..., xn tais que x1
≤ x2 ≤ ... ≤ xn. Qual a estimativa de x para a qual a média dos valores
07. (CN 1990) O cubo de 12(b) é 1750(b). A base de numeração b é: absolutos dos erros é mínima? Como aplicação numérica determine o
valor mínimo de |x – 1| + |x – 2| + ... + |x – 2011|
a) primo
a) 1013041
b) ímpar e não primo
b) 1013042
c) par menor que 5
c) 1013043
d) par entre 5 e 17
d) 1013044
e) par maior que 17
e) 1013045
08. (CN 2010) O número natural 198 está escrito na base 10. Em
quantas bases de numeração o número dado é escrito com três 14. (RÚSSIA) Seja N um número de 4 algarismos. Sabe-se que a soma
algarismos? dos quadrados dos algarismos extremos é igual a 13; a soma dos
quadrados dos algarismos do meio é 85; e, ao se subtrair 1089 de N,
a) 1
o resultado é um número que possui os mesmos algarismos de N, mas
b) 3 em ordem contrária. O produto dos algarismos de N:
c) 5 a) é maior que 28.
d) 7 b) é múltiplo de 33.
e) 9 c) é um quadrado perfeito.
d) possui 18 divisores positivos.
09. (OBM) A representação decimal de um número natural a consiste
de n algarismos enquanto que a representação decimal de a³ consiste
15. Sejam a, b, c números reais positivos tais que abc = 1. Prove que
de m algarismos. Assinale dentre os valores abaixo aquele que m + n
não pode assumir: 1 1 1 3
+ + ≥
a) 2007 a3 (b + c ) b3 ( c + a) c3 ( a + b ) 2
b) 2008
c) 2009 16. Após cair no chão, uma máquina de calcular passou a apresentar
d) 2010 o resultado das contas em outra base de numeração. Ao tentar utilizá-
e) 2011 la foram os seguintes os resultados:
3 × 5 = 17
10. O maior número real z tal que x + y + z = 5 e xy + yz + xz = 3 onde 6 × 5 = 36
x e y são reais é igual a: Se for feita a conta 4 × 5, qual será o resultado?
a) 4 n −1


n  n   2n − 2 
13 17. Para todo natural n > 2, prove que  ≤ 
b) k=0  k   n −1 
3
14 18. Considere as afirmativas e classifique-as em verdadeiro ou falso
c) justificando:
3
d) 5 1. O menor valor da função definida por f ( x ) =
( x + 10 )( x + 2) é igual
a 16. x +1
16
e)
3

12 PROMILITARES.COM.BR
DESIGUALDADES E BASES DE NUMERAÇÃO

12 ( xy − 4x − 3y ) (r + t ) < (r + t )
2. O valor máximo da função definida por f ( x,y ) = IV.
1 x 2y 3 s v
é igual a .
256 O número total de relações que estão corretas é:
9x 2sen2x + 4
3. O valor mínimo da função definida por f ( x ) = é a) 0
igual a 13. x ⋅ senx
b) 1
4. Se x, y são tais que 3x – y = 20 o menor valor de x 2 + y 2 é igual
c) 2
a 2 5.
d) 3
19. Se a, b e c são reais positivos cuja soma é 1, determine o valor e) 4
2 2 2
 1  1  1
mínimo de E =  a +  +  b +  +  c +  . 05. Viajando por uma estrada, Sá Bido passa por um marco
 a  b  c
a) 12 quilométrico onde está escrito um número de dois algarismos. Mais
b) 24 adiante, passa por outro marco, no qual os mesmos dois algarismos
estão escritos, mas na ordem contrária. Mais outro trecho de estrada
c) 30 e passa por um novo marco, este com três algarismos: os mesmos
d) 33 do primeiro marco separados por um zero. Ele notou que a distância
100 entre os dois primeiros marcos é igual à distância entre os dois últimos.
e) Qual a soma dos valores dos dois primeiros marcos?
3
a) 77
EXERCÍCIOS DE
b) 88

COMBATE c)
d) 66
99

e) 55
01. (CN 2016) Sejam x e y números reais tais que xy = 2 3. Sendo
assim, o valor mínimo de x8 + y8 é 06. Em uma cela, há uma passagem secreta que conduz a um porão
de onde partem três túneis. O primeiro túnel dá acesso à liberdade em
a) múltiplo de 18.
1 hora; o segundo, em 3 horas; o terceiro leva ao ponto de partida
b) um número primo. em 6 horas. Em média, os prisioneiros que descobrem os túneis
c) divisível por 5. conseguem escapar da prisão em:
d) divisível por 13. a) 3h 20min
e) par maior que 300. b) 3h 40min
c) 4h
02. (IME 2018) Seja x um número natural maior que 2. Se a d) 4h 30min
representação de um numeral N na base x é 1041 e na base x – 1 é
1431, então a sua representação na base binária é: e) 5h
a) 10001111
07. Considere as afirmativas:
b) 11011011 12 18
1. O valor mínimo de + + xy é igual a 18. Sendo x e y reais
c) 11100111 x y
positivos.
d) 11011110
e) 11110001 2. O valor mínimo de x 2 + 16 para todos os valores positivos de x é
igual a 12. x
03. (IME 2018) A soma dos algarismos de X com a soma dos quadrados 16
3. Para valores positivos de x o menor valor de 5x + + 21 é igual
dos algarismos de X é igual a X. Sabe-se que X é um número natural x
a 21 + 2 5 .
positivo. O menor X possível está no intervalo:
24
a) (0, 25] 4. O valor mínimo da expressão 6x + 2 , quando x > 0 é igual a 18.
x
b) (25, 50] Conclua que:
c) (50, 75] a) Todas são verdadeiras.
d) (75, 100] b) Somente (4) é falsa.
e) (100, 125] c) Somente (3) é falsa.
d) (3) e (4) são verdadeiras.
04. (IME 2010) Sejam r, s, t e v números inteiros positivos tais que
r t e) (1) e (3) são verdadeiras.
< . Considere as seguintes relações:
s v
08. (IME 2006) Seja N um número inteiro de 5 algarismos. O número P
I.
(r + s ) < ( t + v ) é construído agregando-se o algarismo 1 à direita de N e o número Q
s v é construído agregando-se o algarismo 1 à esquerda de N. Sabendo-se
r t que P é o triplo de Q, o algarismo das centenas do número N é:
II. <
(r + s ) ( t + v ) a) 0
b) 2
r (r + t )
III. < c) 4
s (s + v )
d) 6

PROMILITARES.COM.BR 13
DESIGUALDADES E BASES DE NUMERAÇÃO

e) 8 8abc.

09. Se a, b e c são reais positivos cuja soma é 1, determine o valor 16. Seja n = 999
  999 . Quantos “9” há na representação decimal
mínimo de a4 + b4 + c4. de n³? 2008 algarismos

1
a) 17. (IME 2002)
81
1 x+y+z 3
b) a) Sejam x, y e z números reais positivos. Prove que: ≥ x ⋅ y ⋅ z.
27 Em que condições se verifica a igualdade? 3
1 b) Considere um paralelepípedo de lados a, b e c, e área total S0.
c)
3 Determine o volume máximo desse paralelepípedo em função de
d) 1 S0. Qual a relação entre a, b e c para que o volume seja máximo?
Demonstre seu resultado.
e) 3

18. (IME 2011) Mostre que o determinante abaixo apresenta valor


10. (EN 2006) Um recipiente cilíndrico que deve ter 1 m³ de volume
menor ou igual a 16 para todos os valores de a, b e c, pertencentes
vai ser construído nas oficinas do Arsenal de Marinha, para atender
ao conjunto dos números reais, que satisfazem a equação a² + b² +
a um dos navios da MB. Na lateral e na tampa, será utilizado um
c² = 4.
material cujo preço é de R$1.000,00 por m² e, no fundo, um material
a+b b+c c +a
cujo preço é de R$2.000,00 por m². Que dimensões deve ter um
recipiente, para que a MB tenha a menor despesa possível? c +a a+b b+c
1 1 b+c c +a a+b
a) me m
3
3π 3π²
Nota:
1 1 a+b b+c c +a
b) me m
33 π 9π 3 π²
c + a a + b b + c =( a + b ) + (b + c ) + ( a + c ) − 3 ( a + b )(b + c )( a + c )
3 3 3

1 1 b+c c +a a+b
c) me m
π3 3 3
9π²
6
1 9  1  6 1 
d) m e 3 m x +  − x + 6  − 2
3
3π π 19. (PUTNAN) Encontre o valor mínimo de  x  x 
3
para x > 0.  1  3 1 
1 1 x +  + x + 3 
e) m e 3 m  x  x 
3
3π π 9π²

11. (IME 2014) Sejam p o semiperímetro de um triângulo, S sua 20. Observe que o número 399 quando escrito na base 5, ele se torna
área, r e R os raios de suas circunferências inscrita e circunscrita, 3044. A soma dos seus algarismos (abreviadamente “soma digital”) é
respectivamente. Demonstre que vale a seguinte desigualdade igual a 11. Com base nisto, certos números quando escritos na base
4, possuem soma digital igual a 17. Se k é a soma digital de um destes
2 3 2p2 tais números quando escritos na base 2, determine os valores mínimo
S ≤ r ⋅R ≤ e máximo de k.
9 27

GABARITO
12. (IME 2015) Seja n um inteiro positivo cuja representação decimal EXERCÍCIOS DE TREINAMENTO
é am...a1a0 e f a função que troca a posição dos dígitos a2i e a2i+1, de 01. A
forma que f(a2k +1 a2k  a1a0 ) = a2k a2k +1 a0a1. Por exemplo: Pela desigualdade entre as médias aritmética e harmônica, temos:
f(123456) = 214365 a+b+c 3  1 1 1 1 1 1
≥ ⇔ (a + b + c )  + +  ≥ 9 ⇒ + + ≥ 9
3 1 1 1  a b c  a b c
f(1034) = 143 + +
a b c
f(123) = 1032
1 1 1 1
f(10) = 1 Como para a= b= c= temos + + =9 , o valor mínimo é de
3 a b c
fato 9.
Determine o menor número maior que 99 que satisfaça à equação
x² = 9x + 9f(x) + (f(x))².
02. A
13. (IME 2014) Calcular o valor da expressão abaixo: Sabendo que a + b + c = 3x e a² + b² + c² = 3y, temos
370370...037 − 11...1 00...0 3(3x 2 − y)
3     (a + b + c)2 = a2 + b2 + c2 + 2(ab + ac + bc) ⇔ ab + ac + bc = .
89 alg arismos 30 alg s"1" 30 alg s"0"
2
Obs.: algs = algarismos Portanto, o resultado pedido é
ab + ac + bc 3x 2 − y
14. (IME) Demonstre que se M = (14641)b, então, independentemente = .
da base considerada, M é quadrado perfeito. 3 2

a) Determine a representação de M na base b + 1.


b) Determine a representação de M = (14654)b na base b + 1. 03. E
Seja x = 1− a, y =
1− b,z = 1− c . Então
15. Sejam a, b e c reais positivos. Mostre que (a + b)(b + c)(c + a) ≥ a + b + c = 2 ⇒ a = 2 − (1− y ) − (1− z ) = y + z

14 PROMILITARES.COM.BR
DESIGUALDADES E BASES DE NUMERAÇÃO

Analogamente temos b = z + x e c = x + y. Daí, a expressão original se Portanto, b = 8 que é um número par entre 5 e 17.
torna
( x + y )( y + z )( z + x ) 08. E
xyz Se o número 198 é escrito com 3 algarismos na base b, então
x+y b² ≤ 198 < b³.
Aplicando MA ≥ MG temos ≥ xy isto é, x + y ≥ 2 xy . b2 ≤ 198 ⇔ b ≤ 14
2
b ∈ *+ ⇒  3 ⇒ b ∈ {6,7,8, ,14}
Analogamente, y + z ≥ 2 yz e z + x ≥ 2 zx , consequentemente b > 198 ⇔ b ≥ 6

( x + y )( y + z )( z + x )

(2 xy )(2 yz )(2 zx ) =
8
A quantidade de bases de numeração é 14 – 6 + 1 = 9.

xyz xyz
09. C
10n−1 ≤ a < 10n ⇒ 103n−3 ≤ a3 < 103n ⇒ a³ possui 3n – 2, 3n – 1 ou
04. D
3n algarismos
N
2
( )
deve ser um múltiplo de 821 = 263 mas não de 822 = 266 . ( ) Então, m + n pode ser 4n – 2, 4n – 1 ou 4n, ou seja, m + n não pode
deixar resto 1 na divisão por 4.
Então, Logo, m + n não pode assumir o valor 2009.
N   N   N 
 2  +  22  +  23  +  =
63
      10. B
Se as parcelas são em ordem reversa 1 + 2 + 4 + 8 + 16 + 32 (=63), Uma vez que (x + y)² = (5 – z)² e xy = 3 – z(x + y) = 3 – z(5 – z) temos
N aplicando MA ≥ MG
temos que = 32 e isto nos dá o menor N possível a saber, N = 64.
2 13
Multiplicando 64! por 65 · 66 · 67 introduzimos apenas um fator 2 e
( x + y )2 ≥ 4xy ⇒ −3z2 + 10z + 13 ≥ 0 ⇒ −1 ≤ z ≤
3
portanto, o maior valor possível de N é 67.

05. B 11. B
Sejam a, b e c reais positivos. abcd =49 ⋅ bcd ⇔
I. Correta. ⇔ 1000a + 100b + 10c + d =49 ⋅ (100b + 10c + d)
Temos
⇔ 1000a =4800b + 480c + 48d =48 ⋅ (100b + 10c + d)
1
a2 + b2 + c2 − ab − bc − ca = ⋅ (2a2 + 2b2 + 2c2 − 2ab − 2bc − 2ca) = 6 (100b + 10c + d) ⇒ 6 | 125a ⇒ 6 | a =
⇔ 125a ⇒a 6
2
1
= ⋅ [(a − b)2 + (a − c)2 + (b − c)2 ] ≥ 0.
2 12. A
II. Correta. Segue que ( 3 ⋅ 7)2012 =
212012 =
n= 32012 ⋅ 72012
a3 + b3 − a2b − ab2= a2 (a − b) − b2 (a − b)  0 )7 , então n = 212012 termina com 2012 zeros
Como 72012 é (1 00

(a b)(a2 − b2 )
=− na base 7. 2012 zeros

= (a − b)2 (a + b) ≥ 0.
III. Incorreta. Fazendo a = 1 e b = 2, temos 13. B
(12 − 22 ) ≥ (1− 2)4 ⇔ −3 ≥ 1. Deseja-se encontrar x de modo que seja mínimo
n
Absurdo. f (x) = ∑ x−x
i=1
i = x − x1 + x − x 2 +  + x − xn

06. D Há dois casos a considerar. Quando n é ímpar, f(x) é decrescente para x


O fato da expansão decimal de um número natural a consistir de n menor que o termo central x (n+1) e crescente para x maior que este
algarismos, significa que 2 +1
n −1 n termo. Logo, atinge seu valor mínimo neste ponto, que é a mediana
10 ≤ a < 10
dos números x1, x2, ..., xn. Quando n é par, f(x) é decrescente para
Assim,
x < x n , constante para x n < x < x (n) e crescente para x > x (n) .
103n−3 ≤ a3 < 103n 2 2 2 +1 2+
1

Deste modo, Logo, o valor mínimo de f(x) ocorre para todo valor de x entre as
observações centrais.
m ∈ {3n − 2,3n − 1,3n}
e
n + m ∈ {4n − 2, 4n − 1, 4n} 14. D
N = abcd − 1089 = dcba ⇔
e daí,
1000a + 100b + 10c + d − 1089= 1000d + 100c + 10b + a
n + m ≡ 1(mod 4 ) ⇔ 999a + 90b − 90c − 999d = 1089 ⇔
⇔ 111( a − d) + 10 (b − c ) =
121
07. D ⇒ a − d= 1e b − c= 1
12(b) = 1⋅ b + 2 = b + 2
2
a2 + d2 = 13 ⇒ a2 + ( a − 1) = 13 ⇔
1750(b) = 1⋅ b3 + 7 ⋅ b2 + 5 ⋅ b + 0 = b3 + 7b2 + 5b
2
⇔ a − a − 6 =0 ⇒ a =−2 ou a = 3 ⇒ d = 2
Note que, como 7 é um dos algarismos, então b ≥ 8.
2
3 3
12(b)  = 1750(b) ⇔ (b + 2) = b + 7b + 5b ⇔
3 2 b2 + c2 = 85 ⇒ b2 + (b − 1) = 85 ⇔

⇔ b3 + 6b2 + 12b + 8 = b3 + 7b2 + 5b ⇔ ⇔ b2 − b − 42 =0 ⇒ b =−6 ou b = 7 ⇒ c = 6

⇔ b2 − 7b − 8 =0 ⇔ b =−1 ∨ b =8

PROMILITARES.COM.BR 15
DESIGUALDADES E BASES DE NUMERAÇÃO

⇒ a ⋅ b ⋅ c ⋅ d = 3 ⋅ 7 ⋅ 6 ⋅ 2 = 252 = 22 ⋅ 32 ⋅ 7
(3 + ( −1) ) ⋅ ( x
2 2 2
)
+ y 2 ≥ ( 3x − y )
2

⇒ d ( a ⋅ b ⋅ c ⋅ d) = ( 2 + 1)( 2 + 1)(1+ 1) = 18
e daí, x 2 + y 2 ≥ 2 10 .
15. Sejam x = ab, y = bc e z = ca. Então, x, y, z são números reais
positivos tais que xyz = (abc)² = 1. O lado esquerdo da desigualdade
original se torna 19.
2 2 2
1 1 1  1  1  1 1 1 1
+ + E =  a +  +  b +  +  c +  ⇔ E = a2 + b2 + c2 + 2 + 2 + 2 + 6
a2 ( ab + ac ) b2 (bc + ba) c2 ( ca + cb )  a  b  c a b c
y z x Pela desigualdade de Cauchy-Schwarz, temos:
= + +
xz ( x + z ) xy ( x + y ) yz ( y + z )
( a2 + b2 + c2 ) ⋅ (12 + 12 + 12 ) ≥ (a ⋅ 1+ b ⋅ 1+ c ⋅ 1)2 = 1 ⇔ a2 + b2 + c2 ≥ 1
x 2
y z 2 2 3
= + +
y+z x+z x+y Pela desigualdade entre as médias aritmética e geométrica, temos
também:
Pela desigualdade de Cauchy-Schwarz
a+b+c 3 1 1
 x2 y2 z2  ≥ abc ⇔ abc ≤ ⇔ ≥ 27
 ( y + z ) + ( x + z ) + ( x + y )  ≥ ( x + y + z )
2
 + + 3 27 abc
 y + z x + z x + y 
Mais uma vez usando a desigualdade entre as médias aritmética e
Daí geométrica, temos:
( x + y + z )= x + y +=
2
x2 y2 z2 z 3 x + y + z  3 1 3
+ + ≥   ≥ ( xyz )= 1 1 1 1
3

y + z x + z x + y 2( x + y + z) 2 2 3  2 2 + + ≥ 3 ⋅ 3 2 2 2 ≥ 27
a2 b2 c2 abc .
1 1 1 1 100
Dessa forma, E = a2 + b2 + c2 + 2 + 2 + 2 + 6 ≥ + 27 + 6 = .
16. a b c 3 3
3 × 5 = 17b ⇔ 15 = b + 7 ⇔ b = 8 1
Como para a= b= c= , temos que E = 100 , o valor mínimo é de
6 × 5 = 36b ⇔ 30 = 3b + 6 ⇔ b = 8 100 3 3
fato .
4 × 5 = 20 = 248 3

17. EXERCÍCIOS DE COMBATE


Basta aplicar a desigualdade das médias aos números 01. A
n n  n  02. E
  ,   , ,   , observando que sua soma é igual a 2 – 2;
n
1
   2  n − 1 03. D
04. D
18. 05. A
1. Verdadeira 06. C
9
Fazendo y = x + 1 podemos escrever f ( x ) = f ( y − 1) = y + 10 + 07. C
y
08. E
Aplicando a desigualdade das médias MA ≥ MG teremos:
09. B
9 9
y+ ≥ 2 y⋅ = 6 10. D
y y
3 3
e portanto, o valor mínimo da expressão dada é 16. 11. senA + senB + senC ≤
2
2. Verdadeira 12. 1110
Podemos escrever a função dada sob a forma 1030 − 1
13. R =
 2  2  3  2 2 3 3
f ( x, y )=   ⋅   ⋅   ⋅ 1− − − 
y y x  y y x 14. a) (100)b+1
Como a soma dos fatores do produto acima é constante, tal produto b) (10012)b+1
é máximo quando todos os fatores forem iguais, o que ocorre quando 15. Prove: (a + b)⋅(a + c)⋅(b + c) ≥ 8abc
1 a + b ≥ 2√ab
x = 12 e y = 8 e daí fmáx ( x, y ) = .
256 a + c ≥ 2√ac
3. Falsa b + c ≥ 2√bc
Podemos escrever a função dada sob a forma
4 Multiplique: (a + b)⋅(a + c)⋅(b + c) ≥ 8√a²b²c²
f (x) = 9x ⋅ senx +
x ⋅ senx 16. 4015
Aplicando a desigualdade das médias MA ≥ MG 17. a) x + y + z ≥ 3 3 x ⋅ y ⋅ z
4 4 S30
9x ⋅ senx + ≥ 2 9x ⋅ senx ⋅ =
12 b) V ≤
x ⋅ senx x ⋅ senx 6 6
18. Chió: ∆’ = 0 ⇔ x = -2 ou x = 2
4. Falsa
Em x = 2, há um máximo local (2,16)
Aplicando a desigualdade de Cauchy-Schwartz
Cauchy: x ∈ [-2√3, 2√3]
( a12 + a22 )⋅( b12 + b22 ) ≥ (a ⋅ b + a
1 1 2 ⋅ b2 )
2

19. 6
onde a1 =
3; a2 =
−1; b1 =
x; b2 =
y , temos 20. 9 ≤ k ≤ 17

16 PROMILITARES.COM.BR
TEORIA DOS NÚMEROS

TEOREMA (O ALGORITMO DA DIVISÃO) Prova:


Dados quaisquer inteiros a,b com a > 0, então existem inteiros Se n = 2, a afirmativa é verdadeira porque 2 é um número primo.
únicos q, r tais que Seja n > 2 e suponhamos que todo inteiro positivo menor do que
n seja um produto de números primos. Se o próprio n for primo a
b = qa + r, 0 ≤ r < a afirmativa é claramente verdadeira. Se n for composto, escrevamos
O número q é chamado quociente e o número r é chamado n = ab onde 1 < a,b < n. Mas por hipótese a e b são eles próprios
resto. um produtos de primos. Justapondo-se estes produtos temos uma
Assim, por exemplo, temos fatoração de n em primos.
35 = 3·11 + 2 e -51 = (-8)·7 + 5 Os números primos na decomposição de um número n > 1 não
são necessariamente distintos e nem estão arrumados em uma ordem
Definição:
particular. Se os arrumarmos em ordem crescente e agruparmos os
Um inteiro b é divisível por um inteiro a ≠ 0, se existe um inteiro conjuntos de primos iguais em um único fator e mudando para uma
c tal que b = ac. notação apropriada obtemos
Dizemos também que a é um divisor de b, que a divide b ou ainda
n = p1α1 ⋅ p2α2 ⋅  ⋅ pkαk
que b é múltiplo de a e escrevemos a|b. Escreveremos a|b para indicar
que a não divide b. onde α1 > 0,α2 > 0,... e p1 < p2 < ...
ou abreviadamente
PROPRIEDADES FUNDAMENTAIS DA k

RELAÇÃO DE DIVISIBILIDADE
n= ∏p
i =1
αi
i

(i) a|0, 1|a e a|a esta decomposição é chamada a decomposição canônica de n


(ii) se a|1 então a = ±1 e dizemos então que n está expresso na forma convencional ou na
(iii) se a|b e a|c então a|bx + cy. forma padrão.
(iv) se a|b então a|bc.
(v) se a|b e b|c então a|c. TEOREMA (EUCLIDES)
(vi) se a > 0 e b > 0 então a|b implica em a ≤ b. Existem infinitos números primos.
(vii) a|b e b|a implica em a = ± b. Prova:
(viii) Se quaisquer dois termos na igualdade a + b = c são divisíveis por Suponhamos que exista somente um número finito de primos
d, o terceiro também será divisível por d. {p1,p2,...,pr}. Formemos então o inteiro n = 1 + p1p2...pr. Uma vez que
n > pi para todo i ele deve ser composto. Seja q o menor primo de n.
Seja n um inteiro positivo. Denotaremos por d(n) o número de Como p1, p2,...pr representam todos os primos existentes então q é um
divisores de n. Conforme vimos 1 e n são sempre divisores de um deles digamos q = p1 e n = prm. Podemos então escrever
número n maior que 1 assim, d(1) = 1 e d(n) ≥ 2 para n > 1. 1 = n – p1p2...pr = p1m – p1p2...pr = p1(m – p2...pr)

Chegamos à conclusão que p1 > 1 é um fator de 1 o que é uma


DEFINIÇÃO contradição.
Um inteiro positivo n é primo se, e somente se, d(n) = 2. Um inteiro
n > 1 que não é primo é chamado um número composto. Observemos
que um inteiro positivo maior do que 1 é primo se, e somente se, PROPOSIÇÃO
seus únicos divisores positivos são ele mesmo e 1. Daqui para frente Existem n inteiros consecutivos compostos para todo n ≥ 1.
reservaremos a letra p para representarmos os números primos.
Prova:
Assim por exemplo, 2,3,5,7,11,13 são primos; 1,4,6,8,9,10 não
Podemos tomar, por exemplo, a sequência
são primos; 4,6,8,9,10 são números compostos.
(n + 1)! + 2, (n + 1)! + 3, (n + 1)! + 4, ..., (n + 1)! + (n + 1)
Lembrando que k! = 1·2·3·...·k, nenhum destes números é primo
TEOREMA FUNDAMENTAL DA porque i divide (n + 1)! + i para 2 ≤ i ≤ n + 1.
ARITMÉTICA
Todo inteiro positivo n > 1 pode ser expresso como um produto de
números primos (com talvez somente um fator), isto é: PROPOSIÇÃO (CRIVO DE
n = p1·p2·...·pm ERATOSTHENES)
onde p1 ≤ p2 ≤ ... ≤ pm são números primos. Esta fatoração é única Seja n um número composto. Então existe um número primo p
salvo a ordem dos fatores primos. tal que p|n e p ≤ n .

PROMILITARES.COM.BR 17
TEORIA DOS NÚMEROS

Prova: Teorema
Como n é composto, n = ab com 1 < a,b < n. Se a,b > n então O número de divisores positivos de n é
ab > n. Daí, pelo menos um fator, digamos a, satisfaz a a ≤ n . Daí d(n) = (α1 + 1)(α2 + 1)...(αk + 1)
todo divisor primo p de a será como desejado isto é, p|n e p ≤ n . Prova:
Deste modo, para nos certificarmos se um dado número inteiro De fato, temos exatamente αi + 1 possibilidades para escolher um
n > 1 é um número primo, é suficiente verificar se n é divisível por valor para βi a saber, 0,1,2,...,αi. Assim, o número total de divisores
qualquer número primo p ≤ n (senão n é um número primo). Isto será exatamente o produto (α1 + 1)(α2 + 1)...(αk + 1).
em geral economiza um monte de trabalho.
Por exemplo, será n = 271 um número primo? Bem 16 ≤ 271 < 17 DEFINIÇÃO
e assim 271 é um número primo ou existe um número primo p < 16 tal Os números kn onde k = 0, ±1, ±2,... são chamados múltiplos de
que p|271. Os números primos menores ou iguais a 16 são 2,3,5,7,11 n isto é, os elementos do conjunto {...,-kn,...-2n,-n,0,n,2n,...kn,...} que
e 13. Devemos agora ter algum trabalho para verificar se algum destes denotaremos por n.
primos divide exatamente 271. Como nenhum deles divide, então 271
é um número primo. É claro que todo múltiplo de = n p1α1 ⋅ p2α2 ⋅ ⋅ ⋅ pkαk possui a forma
A ideia acima pode ser posta em prática para gerar números m kp1γ1 ⋅ p2γ2 ⋅ ⋅ ⋅ pkγk , γ i ≥ αi ,=i 1,2, ,k onde k não possui nenhum
=
primos. O resultado obtido é o Crivo de Eratosthenes. Suponhamos dos primos p1, p2,..., pk em sua decomposição. O número de múltiplos
que desejemos encontrar todos os números primos menores do que de n é infinito.
60. Comecemos com uma lista dos primeiros 60 inteiros. Os primos
menores do que 60 são 2,3,5 e 7. Assim, eliminamos da lista a cada MÁXIMO DIVISOR COMUM
vez os números:1, depois cada segundo número começando pelo 4
Definição:
a seguir cada terceiro número começando pelo 9, depois cada quinto
começando pelo 25 e finalmente todo sétimo número começando pelo Sejam a e b dois inteiros não simultaneamente nulos. O máximo
49. Os números que restam, 17 no total, são os primos menores que 60. divisor comum de a e b é o inteiro positivo d = mdc (a,b) que satisfaz:
(1) d | a e d | b
1 5 9 13 17 21 25 29 33 37 41 45 49 53 57
2 6 10 14 18 22 26 30 34 38 42 46 50 54 58 (2) se c | a e c | b, então c ≤ d.
3 7 11 15 19 23 27 31 35 39 43 47 51 55 59 A condição (1) diz que d é um divisor comum de a e b e a condição
4 8 12 16 20 24 28 32 36 40 44 48 52 56 60 (2), que d é o maior dos divisores comuns.

A disposição dos números na tabela acima imaginada continuando MÉTODOS PARA O CÁLCULO DO MDC
indefinidamente para a direita sugere algumas questões sobre a
distribuição dos números primos. Sabemos que um número infinito
de colunas da tabela extendida conterá um número primo. O que não
1º MÉTODO: DECOMPOSIÇÃO SIMULTÂNEA
sabemos é se existe um número infinito de colunas contendo dois primos. Os números são decompostos em fatores primos comuns
O conjunto infinito de primos ímpares está dividido entre a primeira e simultaneamente até que os números resultantes sejam primos entre
a terceira linhas da tabela acima e pelo menos uma destas linhas deve si. O mdc é o produto dos fatores primos comuns obtidos.
conter um número infinito de primos. Qual é? Observe que a i-ésima linha
consiste dos inteiros da forma i + 4k, k = 0,1,2,.... Temos então: Exercício Resolvido

01. Calcular mdc(936, 588).


TEOREMA
Resolução:
Existe um número infinito de números k ≥ 0 tais que 3 + 4k é
primo.
936 588 2
Prova:
Sejam p1, p2,..., pk uma lista de primos da forma 3 + 4k e 468 294 2
n = 4p1p2...pk – 1. Como n é ímpar, todos os seus divisores primos são
234 147 3
da forma 1 + 4k ou 3 + 4k. Um exercício fácil mostra que qualquer
produto de inteiros da forma 1 + 4k possui também a forma 1 + 4k. 78 49
Uma vez que n não possui esta forma, ele deve ter um divisor primo q
mdc(7,8,49) = 1 ⇒ mdc(936,588) = 2² · 3 = 12
da forma 3 + 4k e como q|n, mas pi |N , o primo q deve ser diferente
de p1, p2, ..., pk e mostramos então que toda lista finita de números
primos da forma 3 + 4k está incompleta.
2º MÉTODO: DECOMPOSIÇÕES CANÔNICAS
MDC(a,b) é o produto dos fatores primos comuns às duas
NÚMERO DE DIVISORES INTEIROS decomposições tomados com seus menores expoentes.
POSITIVOS DE N
Exercício Resolvido
n p1α1 ⋅ p2α2 ⋅ ⋅ ⋅ pkαk onde os pi são primos distintos e os αi são
Seja =
inteiros positivos. Se d é um divisor de n isto é, n = dm para algum m 02. Calcular mdc(936, 588).
então n = dm = p1α1 ⋅ p2α2 ⋅  ⋅ pkαk
Resolução:
Como a decomposição em fatores primos de n é única, d não
588 = 22⋅3⋅72
pode ter em sua fatoração em primos um primo que não esteja entre
936 = 23⋅32⋅13
os primos p1,p2,...,pk. Além disso, um primo pi da decomposição em
mdc(588,936) = 22⋅3 = 12.
fatores primos de d não pode ter um expoente maior do que αi e,
portanto
= p1β1 ⋅ pβ22 ⋅  ⋅ pkβk ,
d 0 ≤ βi ≤ αi , =i 1,2, ,k

18 PROMILITARES.COM.BR
TEORIA DOS NÚMEROS

3º MÉTODO: DIVISÕES SUCESSIVAS OU 2º MÉTODO: DECOMPOSIÇÕES CANÔNICAS


ALGORITMO DE EUCLIDES MMC(a,b) é o produto dos fatores primos comuns e não-
Para encontrar o mdc de dois números, divide-se o maior pelo comuns às duas decomposições tomados com seus maiores
menor; em seguida, o menor pelo resto; depois o primeiro resto pelo expoentes.
segundo resto; e assim sucessivamente, até que se obtenha resto zero.
O mdc dos números será o último divisor. Exercício Resolvido
O método acima normalmente é apresentado através do 05. Calcular o mmc(936, 588).
dispositivo de cálculo a seguir:
Resolução:
q1 q2 q3 ... qn qn+1
588 = 22⋅3⋅72
a b r1 r2 ... rn–1 rn 936 = 23⋅32⋅13
mmc(588,936) = 23⋅32⋅72⋅13 = 45864.
r1 r2 r3 ... rn 0

O aparecimento do resto 0 indica rn = mdc(a, b).


MÍNIMO MÚLTIPLO COMUM E
Exercício Resolvido
MÁXIMO DIVISOR COMUM
03. Calcular o mdc(936, 588). Na decomposição de um número n em fatores primos, tem-se que
todos os expoentes de seus fatores primos são positivos, entretanto,
Resolução: algumas vezes é conveniente considerar alguns expoentes como sendo
iguais a zero. Isto é especialmente conveniente quando consideramos
1 1 1 2 4 2 a fatoração de dois números a e b para os quais estamos interessados
no mdc(a,b) e no mmc(a,b) onde podemos assumir que tanto a quanto
936 588 348 240 108 24 12 b possuem o mesmo conjunto de fatores primos.
348 240 108 24 12 0 Teorema
resto = 0 ⇒ mdc(936, 588) = 12. Sejam = b p1β1 ⋅ pβ22 ⋅ ⋅ ⋅ pkβk onde αi ≥ 0 e βi ≥ 0
a p1α1 ⋅ p2α2 ⋅ ⋅ ⋅ pkαk e =
são dois inteiros positivos quaisquer. Então,
mdc (=
a,b ) p1min( α1,β1) ⋅ pmin
2
( α2 ,β2 )
⋅  ⋅ pkmin( αk ,βk )
MÍNIMO MÚLTIPLO COMUM e
Definição: Sejam a e b dois inteiros não-nulos. Chama-se mínimo mmc (=
a,b ) p1max ( α1,β1) ⋅ pmax
2
( α2 ,β2 )
⋅  ⋅ pmax
k
( αk ,βk )

múltiplo comum de a e b o inteiro positivo m = mmc(a,b) que satisfaz


Além disso, mdc(a,b) · mmc(a,b) = a·b
as condições:
Prova:
(1) a | m e b | m
As fórmulas do mdc(a,b) e mmc(a,b) são consequências imediatas
(2) se a |c e b | c, com c > 0, então m ≤ c.
de nossas definições de divisor comum e de múltiplo comum. Par provar
A condição (1) diz que m é um múltiplo comum de a e b; a a última fórmula basta observar que min(αi,βi) + max(αi,βi) = αi + βi.
condição (2) diz que qualquer outro múltiplo comum será maior ou
A decomposição em fatores primos é uma coisa
igual a c.
computacionalmente difícil e não conhecemos ainda algoritmos fáceis
para isto. Felizmente, o máximo divisor comum dos números a e b
MÉTODOS PARA O CÁLCULO DO MMC podem ser encontrados sem conhecermos a decomposição em fatores
primos de a e b. Este algoritmo foi descoberto por Euclides.
1º MÉTODO: DECOMPOSIÇÃO SIMULTÂNEA
Os números são divididos pelos fatores primos comuns e não- TEOREMA (O ALGORITMO DE
comuns até que os números resultantes sejam todos 1. O mmc é o
produto dos fatores primos obtidos. EUCLIDES)
Sejam a e b inteiros positivos. Usaremos o algoritmo da divisão
Exercício Resolvido várias vezes para encontrar:
a = q1b + r1 0 < r1 < b,
04. Calcular mmc(936, 588).
b = q2r1 + r2 0 < r2 < r1,
Resolução:
r1= q3r2 + r3 0 < r3 < r2 ,
936 588 2  
468 294 2 rn −=2 qnrn −1 + rn 0 < rn < rn −1,
234 147 2 rn −1 = qn +1rn
117 147 3
Então, rn = mdc(a,b).
39 49 3
Prova:
13 49 7
Isto se baseia no fato de que se a = qb + r então mdc(a,b) =
13 7 7
mdc(b,r). De fato, se d é um divisor comum de a e b então a = a’d e
13 1 13 b = b’d e daí r = a – qb = a’d – qb’d = (a’ – qb’)d e portanto d é
1 1 também um divisor de b e r. Além disso, se d é um divisor comum de
b e r então b = b’d e r = r’d daí, a = qb + r = qb’d + r’d = (qb’ + r’)
mmc(936,588) = 2³ · 3² · 7² · 13 = 45864 d donde concluímos que d é um divisor comum de a e b. Fica claro
então que mdc ( a,b=) mdc (b,r1=) mdc (r1,r2=) = mdc (rn −1,rn=) rn

PROMILITARES.COM.BR 19
TEORIA DOS NÚMEROS

TEOREMA (O ALGORITMO DE DEFINIÇÃO


EUCLIDES EXTENDIDO) Se mdc(a,b) = 1 os números a e b são ditos primos entre si (ou
coprimos).
Escrevamos a seguinte tabela com duas linhas R1 e R2 e três
colunas. Lema
Se a e b são primos entre si isto é, se mdc(a,b) = 1 então tem-se que:
 a 1 0
  1. a e b não possuem fatores primos comuns em suas
 b 0 1
decomposições em fatores primos.
De acordo com o Algoritmo de Euclides, façamos as seguintes 2. Se c é um múltiplo comum de a e b então c é um múltiplo
operações com as linhas desta tabela. Primeiro criaremos a terceira de ab.
linha R3 subtraindo da primeira a segunda multiplicada por q1
e denotaremos isto por R3 = R1 – q1R2. Da mesma forma, criamos 3. Se ac é um múltiplo de b então c é um múltiplo de b.
a quarta linha R4 = R2 – q2R2 e continuamos com este processo da 4. Existem inteiros m,n tais que ma + nb = 1.
seguinte maneira: a criação de Rk é obtida subtraindo-se Rk-1 vezes qk-2
de Rk-2 o que pode ser escrito simbolicamente como Rk = Rk-2 – qk-2Rk-1.
Finalmente obtemos a tabela A FUNÇÃO φ DE EULER
Definição
a 1 0 
  Seja n um inteiro positivo. O número de inteiros positivos menores
b 0 1 
 r1 1 −q1  ou iguais a n que são primos com n é denotado por φ(n), Esta função
  é chamada a função φ de Euler.
 r2 −q2 1 + q1q2 
 Denotemos n = {0,1,2,...,n – 1} e por ∗n o conjunto dos inteiros
  
  não nulos de n que são primos com n. Então, φ(n) é o número de
r m n 
n 
elementos de  n isto é φ (n) =

# ( ∗n ) .
Então mdc(a,b) = rn = am + bn.
Assim por exemplo, se n = 20 então
Prova:
∗20 = {1,3,7,9,11,13,17,19} e φ(20) = 8.
Provaremos isto por indução. Suponha que a k-ésima linha da
tabela seja Rk = (uk, vk, wk) e que ui = avi + bwi para todo i < k. Lema
Isto certamente é verdade para i = 1,2. Então, pela hipótese de Se n = pk, onde p é um número primo, então
indução
 1
uk =uk − 2 − qkuk −1 =avk − 2 + bwk − 2 − qk ( avk −1 + bwk −1 ) = φ (n) =pk − pk −1 =pk 1 − 
 p
a ( vk − 2 − qk vk −1 ) + b ( wk − 2 − qk wk −1 ) =
avk + bwk .
Prova:
Assim a afirmativa ui = avi + bwi é verdadeira para todo i. Em
particular, isto é verdadeiro para a última linha o que nos dá rn = am É fácil listar todos os inteiros menores ou iguais a pk que não são
+ bn. primos com pk. São eles, p, 2p, 3p, ..., pk-1 · p. Temos exatamente
pk–1 deles daí, pk – pk–1 inteiros não nulos de n serão primos com n,
Assim por exemplo dados a = 321 e b = 843, determine o mdc (a,
portanto φ (n) = pk – pk–1.
b) o mmc (a, b) e escreva o máximo divisor comum de a e b como uma
combinação linear de a e b. Uma importante consequência do teorema do resto chinês é que
a função φ (n) é multiplicativo no seguinte sentido:
O Algoritmo de Euclides nos fornece:
Teorema
321 = 0 ⋅ 843 + 321
843 = 2 ⋅ 321 + 201 Sejam m e n dois inteiros positivos primos entre si. Então
321 = 1⋅ 201 + 120 φ (mn) = φ (m) φ (n)
201 = 1⋅ 120 + 81 Prova:
120 = 1⋅ 81 + 39 { } { }
Sejam ∗m = r1,r2 , ,rφ (m) e ∗n = s1,s2 , ,s φ (n) . Pelo teorema do
81 = 2 ⋅ 39 + 3 resto chinês, existe um único inteiro positivo Nij tal que 0 ≤ Nij < mn; Nij =
39= 13 ⋅ 3 + 0 qm + ri e Nij = kn + si em particular, para alguns inteiros a e b tem-se que
Nij = am + ri e Nij = bn + si
e portanto, mdc(321,843) = 3
321⋅ 843 Como vimos na prova do Algoritmo de Euclides, mdc(Nij,m) =
e mmc ( 321,843) = = 107 ⋅ 843 = 90201 mdc(m,ri) = 1 e mdc(Nij,n) = mdc(n,sj) = 1.
3
O Algoritmo de Euclides Extendido nos fornece: Isto é, Nij é primo com m e com n e como eles são primos entre si,
Nij é primo com mn e daí Nij ∈ ∗mn . Claramente diferentes pares (i,j) e
321 1 0
(k,l) produzem números diferentes Nij e Nkl.
843 0 1
Suponhamos um número N ≠ Nij para todos i e j. Então N = q1m + r
321 1 0 ∗ ∗
e N = q2n + s onde ou r não pertence a  m ou s não pertence a  n .
201 −2 1 Supondo que seja o primeiro, temos que mdc(r,m) > 1. Mas então
120 3 −1 mdc(N,m) = mdc(r,m) > 1 e N não pertence a ∗mn . Isto nos mostra que
81 −5 2 somente os números Nij formam ∗mn . Mas eles são exatamente φ(m)
39 8 −3 φ(n) dos números Nij, exatamente tantos quantos são os pares (ri,sj) e,
3 −21 8 portanto φ(mn) = φ(m)φ(n).
Teorema
Obtemos assim a combinação linear
Seja n um inteiro positivo tal que
mdc(321,843) = 3 = (-21)·321 + 8·843
n = p1α1 ⋅ p2α2 ⋅  ⋅ pkαk

20 PROMILITARES.COM.BR
TEORIA DOS NÚMEROS

Então lei do cancelamento o a pode ser cancelado e daí i ≡ j(modp) o que é


impossível. Portanto, estes restos são 1, 2, ..., p – 1 e
 1  1  1
φ (n) = n 1 −  1 −  1 −  a ⋅ 2a ⋅  ⋅ (p − 1) a ≡ (p − 1)!(modp )
 p1  p2   pk 
que é
Prova:
Pelo exposto no teorema e no lema anterior para determinarmos (p − 1)!⋅ ap −1 ≡ (p − 1)!(modp )
φ(n) temos: Como (p – 1)! é primo com p,pela lei do cancelamento, ap-1 ≡
φ (p
φ (n) = α1
1 ) φ (p ) φ (p )
α2
2
αk
k
1(modp).

ou

TEOREMA (O TEOREMA DE EULER)
1  1  1
φ (n)= p1α1 1 −  p2α2 1 − pkαk 1 −  Seja n um inteiro positivo. Então, aφ(n) ≡ 1(modn) para todo a
 p1  p2   pk 
primo com n.
e finalmente Prova:
 1  1  1
φ (n) = n 1 −  1 −  1 −  { }
Seja ∗n = z1,z2 , ,zφ (n) . Considere os números z1a, z2a, ..., zφ(n)a.
 p1   p2   pk  Como tanto zi quanto a são primos com n então, zia é também primo
com n. Suponhamos que zia ≡ ri(modn). Como mdc(zia,n) = mdc(ri,n)
Assim por exemplo, tem-se que ri ∈ ∗n e estes restos são todos diferentes. Além disso,
( ) ( )(
φ ( 264 ) =φ ( 23 ⋅ 3 ⋅ 11) =264 1 2 10
2 3
=
11 80 ) supondo que ri = rj para alguns 1 ≤ i < j ≤ n então zia ≡ zja(modn) e
pela lei do cancelamento a pode ser cancelado obtendo zi ≡ zj(modn)
o que é impossível. Portanto, os restos r1, r2, ..., zφ(n) coincidem com
z1, z2, ..., zφ(n) a menos da ordem na qual estão listados e assim
CONGRUÊNCIAS
Se a e b são inteiros quaisquer e m um inteiro positivo escrevemos z1a ⋅ z2a ⋅  ⋅ zφ (n)a ≡ r1 ⋅ r2 ⋅  ⋅ rφ (n) ≡ z1 ⋅ z2 ⋅  ⋅ zφ (n) (modn) que é igual
a ≡ b(mod m) e dizemos que a é congruente a b segundo o módulo m a Z·aφ(n) ≡ Z(modn) onde Z = z1·z2·...·zφ(n) e como Z é primo com n ele
se, e somente se, a e b deixam o mesmo resto quando divididos por m. pode ser cancelado e obtemos então aφ(n) ≡ 1(modn).
Assim por exemplo tem-se que 41 ≡ 80(mod13), 41 ≡ -37(mod13)
e 41≡ 7 (mod13) . TEOREMA DE WILSON
Lema Se p é primo se, e somente se, (p – 1)! ≡ –1(modp), onde n! =
Sejam a e b são dois inteiros e m um inteiro positivo então n · (n – 1)·...·3·2·1.
a. a ≡ b(modm) se, e somente se a – b é divisível por m; Exemplo:
b. Se a ≡ b(modm) e c ≡ d(modm) então, a + c ≡ b + d(modm); Para p = 7, temos que 6! ≡ -1(mód7), pois 6! + 1 = 721 = 7 · 103
c. Se a ≡ b(modm) e c ≡ d(modm) então, ac ≡ bd(modm);
d. Se a ≡ b(modm) e n é um inteiro positivo, então an ≡ bn(modm); EQUAÇÃO DIOFANTINA LINEAR
e. Se ac ≡ bc(modm) e os números c e m são primos entre si, É uma equação da forma ax + by = c, onde a, b e c são inteiros.
então a ≡ b(modm) Sejam a e b inteiros positivos e d = mdc(a,b). Se d /| c , então a equação
Prova: ax + by = c não possui nenhuma solução inteira. Se d|c, então ela
possui infinitas soluções e se x = x0 e y = y0 é uma solução particular,
(a) Pelo algoritmo da divisão tem-se que a = q1m + r1, 0 ≤ r1 < m então todas as soluções são dadas por
e b = q2m + r2, 0 ≤ r2 < m.
=x x 0 + (b d) k
Assim, a – b = (q1 – q2)m + (r1 – r2) onde –m < r1 – r2 < m. Vemos 
então que a – b é divisível por m se, e somente se, r1 – r2 é divisível y y 0 − ( a d) k
=
por m mas isto só ocorre,se e, e somente se, r1 – r2 = 0 isto é se r1 = r2.
onde k é um inteiro.
(b) é deixada como exercício.
(c) Se a ≡ b(modm) e c ≡ d(modm) então, m|(a – b) e m|(c – d) isto
é, a – b = k1m e c – d = k2m para alguns inteiros k1 e k2. Então CONGRUÊNCIA LINEAR
ac − bd = ( ac − bc ) + (bc − bd) = ( a − b ) c + b ( c − d) ou É uma congruência da forma ax ≡ b(modm), onde x é uma
incógnita, a,b ∈ , m ∈ *+ e mdc(a,m) = d. Se d /| b , a congruência ax
ac − bd = k1cm + k 2bm = (k1c + k2b ) m e daí, ac ≡ bd(modm). ≡ b(modm) não possui nenhuma solução e, quando d|b, a congruência
(d) é uma consequência imediata de (c). possui exatamente d solução não congruentes módulo m.
(e) Supondo que ac ≡ bc(modm) e mdc(c,m) = 1 então existem
inteiros u e v tais que cu + mv = 1 ou cu ≡ 1(modm). Então, por (c) TEOREMA CHINÊS DOS RESTOS
tem-se que a ≡ acu ≡ bcu ≡ b (modm) e portanto, a ≡ b(modm) como
Se mdc(ai,mi) = 1, mdc(mi,mj) = 1, ∀i ≠ j, e ci ∈ , então o sistema
queríamos.
a1x ≡ c1 (modm1 )

TEOREMA (O PEQUENO a2x ≡ c2 (mod m2 )

 
TEOREMA DE FERMAT) a x ≡ c (modm )
Seja p um número primo. Se um inteiro a não é divisível por p  k k k

então, ap-1 ≡ 1(modp). Além disso, ap ≡ a(modp) para todo a. possui uma solução única módulo m = m1 · m2 · ... · mk tal que
Prova: x ≡ b1 ⋅ M1 ⋅ x1 + b2 ⋅ M2 ⋅ x 2 +  + bk ⋅ Mk ⋅ xk (modm )
Seja a primo com p e considere os números: a, 2a, ..., (p – 1)a. m
onde bi é a solução de aix ≡ ci(modmi), Mi = e xi é a solução de
Todos possuem diferentes restos na divisão por p. De fato, supondo Mi · x ≡ 1(modmi). mi
que para alguns 1 ≤ i < j ≤ p – 1 tenhamos ia ≡ ja(modp) então pela

PROMILITARES.COM.BR 21
TEORIA DOS NÚMEROS

EXERCÍCIOS DE Conclua que

TREINAMENTO a) Quatro são falsas


b) Duas são verdadeiras e duas são falsas
c) Três são verdadeiras e uma é falsa
d) Três são falsas e uma é verdadeira
01. (ITA 2013) Seja n > 6 um inteiro positivo não divisível por 6. Se,
na divisão de n2 por 6, o quociente é um número ímpar, então o resto e) Todas são verdadeiras
da divisão de n por 6 é
a) 1. c) 3. e) 5. 11. Considere as afirmativas:
1. O último algarismo do número 31001·71002·131003 é igual a 9.
b) 2. d) 4. 7
7

2. O último algarismo do número 77 onde temos uma torre


02. O número de divisores positivos de 201010 que são menores que formada por 1001 setes é 3.
20105 é igual a:
 1099 
a) 7320 c) 7324 e) 7328 3. Os últimos três algarismos do número  33  onde  x é o
10 + 3 
b) 7322 d) 7326 maior inteiro que não supera x são 008.

03. Seja n = 247 × 343. O número de divisores positivos de n² menores Conclua que
que n, mas que não dividem n é igual a: a) Todas são falsas
a) 2020 c) 2022 e) 2024 b) Todas são verdadeiras
b) 2021 d) 2023 c) Somente (1) é verdadeira
d) Somente (1) e (2) são verdadeiras
04. (CN 1989) Se o mdc(a;b;c) = 100 e o mmc(a;b;c) = 600, podemos e) Somente (2) e (3) são verdadeiras
afirmar que o número de conjuntos de três elementos distintos a, b
e c é: 12. Seja k = 20082 + 22008. Qual é o algarismo das unidades de k2 + 2k?
a) 2 c) 6 e) 10 a) 0 c) 4 e) 8
b) 4 d) 8 b) 2 d) 6

05. Determine a soma dos algarismos do menor número inteiro 13. A soma de todos os divisores do número N = 1988 – 1 que são da
positivo n, que ao ser dividido por 10 deixa resto 9, ao ser dividido forma d = 2a · 3b com a,b > 0 é igual a:
por 9 deixa resto 8, ao ser dividido por 8 deixa resto 7, etc, e ao ser
dividido por 2 deixa resto 1. a) 740 c) 744 e) 748
a) 9 c) 17 e) 20 b) 742 d) 746
b) 13 d) 19
14. Seja n um número natural tal que MMC(n,MDC(18,2n)) = 4,
então 2n + n é igual a:
06. Se x e y são números naturais e 19x + 97y = 1997, então o menor
valor possível de x + y é a) 5n c) 4n e) 3n
a) 21 c) 38 e) 47 b) 7n d) 6n
b) 23 d) 41
15. A soma dos algarismos do número N de quatro algarismos
para o qual os restos das divisões de 21685 e 33509 por N sejam
07. Se m é um inteiro positivo que não excede 1000 e tal que a fração respectivamente iguais a 37 e 53 é:
m+4
seja irredutível, o número de valores possíveis de m é igual a: a) 20 c) 24 e) 28
m2 + 7
b) 22 d) 26
a) 951 c) 955 e) 959
b) 953 d) 957 16. (IME 2018) Determine todos os números primos p, q e r tais que
35p + 11pq + qr = pqr.
08. A divisão de 30 + 31 + 32 + ... + 32009 por 8 deixa resto:
a) 0 c) 2 e) 6 17. Entre os números naturais de 1 até n, pelo menos 11 são divisíveis
por 5 e no máximo 9 são divisíveis por 6. No máximo, quantos desses
b) 1 d) 4
números são divisíveis por 7?

09. Um número primo é chamado estranho se é um número primo de


18. Se x e y são inteiros positivos tais que x(x + 2 + 4 + 6 + ... + 4024)
um algarismo ou se tem dois ou mais algarismos e os dois números que
= 2013y, qual é o valor de y?
se obtêm, omitindo-se o primeiro ou o último algarismo do número,
são também estranhos. Quantos números primos estranhos existem?
19. Um retângulo, o qual não é um quadrado, tem lados com
a) 8 c) 10 e) infinitos comprimentos inteiros, medidos em centímetros. Se o seu perímetro
b) 9 d) 11 é n centímetros e sua área é n centímetros quadrados, determine n.

10. Considere as afirmativas: 20. (IME 2015) Os coeficientes a0, ..., a2014 do polinômio P(x) = x2015
1. O resto da divisão do número 22012 – 1 por 17 é igual a zero. + a2014x2014 + ... + a1x + a0 são tais que ai ∈ {0,1}, para 0 ≤ i ≤ 2014.
2. O resto da divisão do número (320 + 11)55 por 13 é igual a 6. a) Quais são as possíveis raízes inteiras de P(x)?
3. O resto da divisão do número 250 + 1 por 125 é igual a zero.
4. O resto da divisão de 248 – 1 por 105 é igual a zero. b) Quantos polinômios da forma acima têm duas raízes inteiras
distintas?

22 PROMILITARES.COM.BR
TEORIA DOS NÚMEROS

EXERCÍCIOS DE 07. (FUVEST 2017) Sejam a e b dois números inteiros positivos. Diz-

COMBATE
se que a e b são equivalentes se a soma dos divisores positivos de a
coincide com a soma dos divisores positivos de b.
Constituem dois inteiros positivos equivalentes:
a) 8 e 9. d) 15 e 20.
01. (IME 2010) A quantidade k de números naturais positivos, menores
b) 9 e 11. e) 16 e 25.
do que 1000, que não são divisíveis por 6 ou 8, satisfaz a condição.
c) 10 e 12.
a) k < 720 d) 780 ≤ k < 810
b) 720 ≤ k < 750 e) k ≥ 810 08. (FUVEST 2015) Na cidade de São Paulo, as tarifas de transporte
c) 750 ≤ k < 780 urbano podem ser pagas usando o bilhete único. A tarifa é de R$
3,00 para uma viagem simples (ônibus ou metrô/trem) e de R$ 4,65
02. O resto da divisão de um número inteiro positivo por 4 é igual a para uma viagem de integração (ônibus e metrô/trem). Um usuário
3 e o resto da divisão do mesmo número por 9 é 5. O resto da divisão vai recarregar seu bilhete único, que está com um saldo de R$12,50.
deste número por 36 é igual a: O menor valor de recarga para o qual seria possível zerar o saldo do
a) 23 c) 27 e) 35 bilhete após algumas utilizações é

b) 26 d) 30 a) R$ 0,85 c) R$ 1,45 e) R$ 2,80


b) R$ 1,15 d) R$ 2,50
03. (ITA 2003) O número de divisores de 17640 que, por sua vez, são
divisíveis por 3 é: 09. (AIME 1983) Seja an igual a 6n + 8n, então o resto de a83 quando
a) 24 c) 48 e) 72 dividido por 49 é

b) 36 d) 54 a) 0 c) 23 e) 41
b) 11 d) 35
04. (IME 2018) Se X e Y são números naturais tais que X2 – Y2 = 2017,
o valor de X2 + Y2 é: 10. (IME 2011) Seja x um número inteiro positivo menor ou igual a
a) 2008010 c) 2034145 e) 2052061 20.000. Sabe-se que 2x – x2 é divisível por 7. Determine o número de
possíveis valores de x.
b) 2012061 d) 2044145
a) 4237 c) 4923 e) 5716
05. Numa escola, ao longo de um corredor comprido estão b) 4516 d) 5247
enfileirados 2012 armários numerados consecutivamente de 1 a
2012 com suas portas fechadas. 2012 da escola também numerados 11. (ITA 2018) Quantos pares de números inteiros positivos (A,B)
consecutivamente de 1 a 2012 resolvem fazer a seguinte brincadeira: existem cujo mínimo múltiplo comum é 126 × 103? Para efeito de
O aluno de número 1 passa pelo corredor e abre todos os armários; contagem, considerar (A,B) ≡ (B,A).
em seguida, o aluno de número 2 passa e fecha todos os armários de
número par; depois passa o aluno de número 3 e inverte a posição 12. (IME 2009). Sabe-se que: a = [a] + {a}, ∀a ∈ , onde [a] é parte
das portas de todos os armários múltiplos de 3, isto é, ele os fecha inteira de a
se estiverem abertos e os abre se estiverem fechados; depois é a vez
do aluno de número 4 que inverte a posição das portas dos armários x + [ y ] + {z} =
4,2

múltiplos de 4 e assim sucessivamente. Após a passagem dos 2012 y + [ z] + {x} 3,6 , com x, y e z ∈ 
=
o número de armários que ficarão com as portas abertas é igual a : 
z + [ x ] + {y} =
2

Determine o valor de x – y + z.

13. (ITA 2017) Sejam A = {1,2,...,29,30} o conjunto dos números


inteiros de 1 a 30 e (a1,a2,a3) uma progressão geométrica crescente
com elementos de A e razão q > 1.

a) Determine todas as progressões geométricas (a1,a2,a3) de razão


3
q= .
a) 1968 c) 44 e) 42 2
b) 1006 d) 43 m
b) Escreva q = , com m, n ∈  e mdc(m,n) = 1. Determine o maior
n
06. (IME 2017) Um hexágono é dividido em 6 triângulos equiláteros. valor possível para n.
De quantas formas podemos colocar os números de 1 a 6 em cada
triângulo, sem repetição, de maneira que a soma dos números em
14. (IME 2010) Seja a equação pn + 144 = q2, onde n e q são números
três triângulos adjacentes seja sempre múltiplo de 3? Soluções obtidas
inteiros positivos e p é um número primo.
por rotação ou reflexão são diferentes, portanto as figuras abaixo
mostram duas soluções distintas. Determine os possíveis valores de n, p e q.

15. Calcule os dois últimos algarismos de 2222 – 1.


a) 12
b) 24 16. Em uma ilha deserta havia cinco homens e um macaco. Durante
c) 36 o dia os homens colheram cocos e deixaram a partilha para o dia
seguinte. Durante a noite, um dos homens acordou e resolveu pegar
d) 48 a sua parte. Dividiu a pilha de cocos em cinco partes iguais, observou
e) 96 que sobrava um coco, deu este coco para o macaco, retirou e guardou

PROMILITARES.COM.BR 23
TEORIA DOS NÚMEROS

a sua parte. Mais tarde, o segundo homem acordou e fez a mesma 04. B
coisa que o primeiro, dando também um coco para o macaco.  a b c 
Sucessivamente, cada um dos três homens restantes fez o mesmo que mdc ( a;b;c ) =
100 ⇒ mdc  , , =1
 100 100 100 
os outros dois isto é, dividindo os cocos existentes em cinco partes
iguais, dando um coco para o macaco e guardando a sua parte. No  a b c  600
dia seguinte os cinco homens repartiram os cocos restantes em cinco mmc ( a;b;c ) =
600 ⇒ mmc  , , = =
6
 100 100 100  100
partes iguais, observaram que sobrou um coco, deram-no para o
macaco e cada um pegou a sua parte. Se N é o menor número de a b c
Como , e são distintos, temos a menos da ordem os
cocos que a pilha inicial poderia ter então determine a soma de seus 100 100 100
algarismos. seguintes valores (1,2,6), (1,3,6), (1,2,3) e (2,3,6).
Logo, temos 4 conjuntos que satisfazem às condições do enunciado.
17. (IME 2016) Sabendo-se que m e n são inteiros positivos tais que
3m + 14400 = n², determine o resto da divisão de m + n por 5.
05. C
18. (IME 2016) Seja a equação n² – 7m² = (5m – 2n)² + 49. Determine n= 10k10 + 9= 9k 9 + 8= 8k 8 + 7= = 2k 2 + 1
todos os pares inteiros (m,n) que satisfazem a esta equação. 1 10 (k10 + 1=
⇒ n += ) 9 (k9 + 1=) 8 (k8 + 1=) = 2 (k2 + 1)
Logo, n+1 é múltiplo de 10, 9 , 8, ..., 2. O menor número com essa
19. (FUVEST 2018) Considere a sequência a1 = 6, a2 = 4, a3 = 1, a4 = 2 propriedade é:
e an = an-4, para n ≥ 5. Defina Skn = an + an +1 +  + an + k para k ≥ 0, isto é, n + 1 = mmc ( 2,3, 4, ,8,9,10 ) = 23 ⋅ 32 ⋅ 5 ⋅ 7 = 2520 ⇒ n = 2519
Skn é a soma de k + 1 termos consecutivos da sequência começando
do n – ésimo, por exemplo, S12 = 4 + 1 = 5.
06. B
a) Encontre n e k tal que Skn = 20. 19x + 97y =1997 ⇒ x =100 + 97t ∧ y =1− 19t, t ∈ 
b) Para cada inteiro j, 1 ≤ j ≤ 12, encontre n e k tal que Skn = j. 100 + 97t ≥ 0 ⇒ t ≥ −1
c) Mostre que, para qualquer inteiro j, j ≥ 1, existem inteiros n ≥ 1 e 1− 19t ≥ 0 ⇒ t ≤ 0
k ≥ 0 tais que Skn = j. ⇒ t =−1 ∨ t =0

20. Sejam r e s ∈  (inteiro). Prove que (2r + 3s) é múltiplo de 17 se e y 101+ 78t ⇒ ( x + y )MIN= 101+ 78 ⋅ ( −1=
x+= ) 23
somente se (9r + 5s) é múltiplo de 17.
07. D
m+ 4 m2 + 7
GABARITO Observe que é irredutível se e somente se o for e como
2
m +7 m+ 4
EXERCÍCIOS DE TREINAMENTO
01. C m2 + 7 m2 − 16 23 23
= + = m−4 + e a fração só não estará na
Todo número inteiro positivo n que não é múltiplo de 6 poderá ser m+ 4 m+ 4 m+ 4 m+ 4
escrito utilizando uma das formas abaixo: forma irredutível quando m + 4 for um múltiplo de 23 compreendido
entre 5 e 1004. O primeiro é 23 = 23 x 1 e o último é 989 = 23 x 43
n = 6k + 1 ⇒ n2 = 6.(6K2 + 2K) + 1
e portanto existem 43 tais múltiplos e a resposta é 1000 – 43 = 957.
n = 6k + 2 ⇒ n2 = 6.(6K2 + 4K) + 4
n = 6k + 3 ⇒ n2 = 6.(6K2 + 6K + 1) + 3 08. D
n = 6k + 4 ⇒ n2 = 6.(6K2 + 12K + 2) + 4 30 ≡ 1
8 
n = 6k + 5 ⇒ n2 = 6.(6K2 + 10K + 4) + 1 31 ≡ 3 ⇒ 32k + 32k +1 ≡ 4, k ∈  ⇒ 34k + 34k +1 + 34k + 2 + 34k + 3 ≡ 0, k ∈ 
Dos números acima, os únicos cujos quadrados terão quociente ímpar 8

8 8

quando divididos por 6 são os da forma 6k + 3; logo, o resto da 32 ≡ 1


8 
divisão de n por 6 será 3.
( 30 + 31 + 32 + 33 ) +  + ( 32004 + 32005 + 32006 + 32007 ) + 32008 + 32009 ≡ 1+ 3 ≡ 4
8 8
02. A
201010 = 210 × 310 × 510 × 6710 possui 14641 divisores positivos
5 10 09. B
incluindo 2010 = 2010 . Existe uma correspondência um a Os números estranhos de 1 algarismo são: 2, 3, 5 e 7. Os números
um entre os divisores d que são menores que 20105 e aqueles que estranhos de 2 algarismos são os números primos entre: 23, 32,
são maiores que 20105. Daí, existem 21 (14641− 1) =7320 divisores 25, 52, 27, 72, 35,53,37,73,57 e 75. Eles são 23, 37, 53 e 73. Os
positivos menores que 20105. números estranhos de três algarismos são os números primos entre:
237, 373, 537 e 737 Somente 373 é estranho. Nenhum número de
03. B quatro algarismos pode começar e terminar por 373 logo não há
Seja = n p1α × p2β onde p1 e p2 são primos distintos. Então, outro número estranho e portanto, existem exatamente 9 números
=n p12α × p22β e deste modo, n2 possui (2α + 1)(2β + 1) divisores
2 estranhos: 2, 3, 5, 7, 23, 37, 53, 73 e 373.
positivos. Para cada divisor positivo de n2 menor do que n, existe
um divisor correspondente maior do que n. Excluindo-se o próprio 10. C
( 2α + 1)( 2β + 1) − 1 (1) (F)
n vemos que existem = 2αβ + α + β divisores
− 1 16503 − 1 ≡ ( −1)
503
2 22012 = −1
positivos de n² que são menores do que n. Uma vez que n possui
≡ −2 (mod17) =15 (mod17)
(α + 1)(β + 1) divisores positivos, e como todos eles são também
divisores de n² então existem 2αβ + α + β – [(α + 1)(β + 1) – 1] = αβ (2) (V)
divisores positivos de n² que são menores do que n e que não dividem Como 3= 3
27 ≡ 1(mod13) então,
n. Quando α = 47 e β = 43 existem αβ = 2021 tais divisores.

24 PROMILITARES.COM.BR
TEORIA DOS NÚMEROS

55 15. C
(3 ) ≡  32( ) ⋅ 32 − 2
50 55 6
+ 11 = 755 = 4927 ⋅ 7 Podemos escrever
 
≡ 1027 ⋅ 7 ≡ ( −3) ⋅ 7 ≡ −7 ≡ 6 (mod13)
27 = Nq1 + 37
 21685  21648 = Nq1
 ou 
= Nq2 + 53
33509 33450 = Nq2
(3) (V)
Como N, q1 e q2 são números inteiros, vê-se que N é um divisor
250=
+1 (2 ) =
10 5
+ 1 1024 + 1 ≡ 24 + 1 5 5
comum a 21685 e 33509. Dentre os divisores comuns, o maior é
= ( 24 + 1) ( 24 − 24 + 24 − 24 + 1) (mod125)
4 3 2 o mdc(21685,33509) = 1968 de onde conclui-se que N ou é 1968
ou um de seus divisores. Como o maior deles é 984 que possui três
O primeiro fator é 25; o segundo fator é divisível por 5 uma vez que algarismos então N = 1968.
24 ≡ –1(mod5) e portanto o produto é divisível por 125.
(4) (V) 16.
Observe que 105 = 3 · 5 · 7. Agora, De 35p + 11pq + qr = pqr,
248 − 1 ≡ ( −1) − 1 ≡ 0 (mod3) ;
48
q 35p
48
2 −1
= 4 24
− 1 ≡ ( −1)
924
− 1 ≡ 0 (mod5) ; q 5⋅7⋅p

248 − 1= 816 − 1 ≡ 116 − 1 ≡ 0 (mod7) Então,


q = 5 ou q = 7 ou q = p
Assim, 248 – 1 é divisível por 3, 5 e 7. Primeiro caso: q = p
35p + 11p ⋅ p + p ⋅ r = p ⋅ p ⋅ r
11. C 35p + 11p2 + pr =
p2r
(1) (V)
p ( 35 + 11p + r ) =p ⋅ pr
31001 ⋅ 71002 ⋅ 131003
= 31000 ⋅ 911002 ⋅ 3 ⋅ 13
35 + 11p + r =pr
= 81250 ⋅ 91102 ⋅ 39 ≡ 9 (mod10 )
35 + 11p =pr − r
2. (V) r (p − 1) = 35 + 11p
7
77 35 + 11p
Uma vez que 74 ≡ 1(mod10) obtemos 7 ≡ 3 (mod 4 ) e observando r=
1000 7's p −1
7
7 11p − 11+ 46
que 72k ≡ 1(mod4) e 72k+1 ≡ 3(mod4) então 7
7
≡ 73 ≡ 3 (mod10 ) . r=
1001 7's
p −1
3. (V) 11⋅ (p − 1) + 46
Observe que
r= (1)
p −1
 1099  1099 + 33 − 33 
 33 = 33  (p − 1) 46
10 + 3   10 + 3  p – 1 = 1 ou p – 1 = 2 ou p – 1 = 23 ou p – 1 = 46
 33  De p – 1 = 1, p = 2
= 1066 − 3 ⋅ 1033 + 32 − 33 
 10 + 3  De p – 1 = 2, p = 3
De p – 1 = 23, p = 24 (Não é primo)
= 1066 − 3 ⋅ 1033 + 32 − 1 De p – 1 = 46, p = 47
E portanto, os três últimos algarismos são 008. Então, p = 2 ou p = 3 ou p = 47

12. D Substituindo p = 2 na equação (1),


11⋅ ( 2 − 1) + 46
21 ≡ 2 (mod10 ) =r = 57 (Não é primo)
2 −1
22 ≡ 4 (mod10 )
Substituindo p = 3 na equação (1),
23 ≡ 8 (mod10 ) 11⋅ ( 3 − 1) + 46
=r = 34 (Não é primo)
24 ≡ 6 (mod10 ) 3 −1
25 ≡ 2 (mod10 ) Substituindo p = 47 na equação (1),
22008 ≡ 24⋅502 ≡ 6 (mod10 ) 11⋅ ( 47 − 1) + 46
=r = 12 (Não é primo)
k ≡ 20082 + 22008 2 ( ) 2 (
≡ 8 + 6 ≡ 4 + 6 ≡ 0 mod10 ⇒ k ≡ 0 mod10 ) 47 − 1
Não há solução nesse caso.
Como k = 20082 + 22008 é múltiplo de 4, então 2k ≡ 6(mod10). Logo,
seu algarismo das unidades é 6.
Segundo caso: q = 5
35p + 11⋅ p ⋅ 5 + 5r = p ⋅ 5 ⋅ r
13. C 35p + 55p + 5r = 5pr
14. A 18p + r =pr
M.D.C. (18, 2n)= 2 ⋅ M.D.C. ( 9, n)
r (p − 1) =
18p
M.D.C. ( 9, n) ∈ {1,3,9}
18p
M.M.C. (n , M.D.C. (18, 2n) ) =
4 ⇒ M.D.C. (18, 2n) =
2 ⋅ M.D.C. ( 9, n) | 4 ⇒ M.D.C. ( 9, n) | 2 r= ( 2)
p −1
M.D.C. ( 9, n) ∈ {1,3,9}
 ⇒ M.D.C. ( 9, n) =
1
M.D.C. ( 9, n) | 2  (p − 1) 18
⇒ M.M.C. (n , M.D.C. (18, 2n) ) = 4 ⇒ M.M.C. (n , 2) = 4 ⇒ n = 4 p – 1 = 1 ou p – 1 = 2 ou p – 1 = 3 ou p –1 = 6 ou p – 1 = 9 ou
⇒ 2n + n = 24 + 4 = 20 = 5 ⋅ 4 = 5n p – 1 = 18
De p – 1 = 1, p = 2
De p – 1 = 2, p = 3

PROMILITARES.COM.BR 25
TEORIA DOS NÚMEROS

De p – 1 = 3, p = 4 (Não é primo)
De p – 1 = 6, p = 7 19. Sejam a < b os lados do retângulo. Então 2(a + b) = ab = n, e
De p – 1 = 9, p = 10 (Não é primo) ab – 2a – 2b = 0 é equivalente a (a – 2)(b – 2) = 4. Como a e b são
De p – 1 = 18,p = 19 diferentes, a – 2 = 1 e b – 2 = 4, de modo que a = 3 e b = 6, e n =
Substituindo p = 2 na equação (2), 2(3 + 6) = 18.
18 ⋅ 2
=r = 36 (Não é primo)
2 −1 20. a) Se considerarmos a0= a= 1 a=2 ...= a2014= 0, então
Substituindo p = 3 na equação (2), P(x) = x 2015 , cuja única solução possível é zero. Sabe-se que as
18 ⋅ 3 soluções inteiras devem ser divisíveis por a0, para que a divisão seja
=r = 27 (Não é primo)
3 −1 possível. Logo, a0 só poderia ser zero, +1 ou –1. Se a0 = +1, os outros
Substituindo p = 7 na equação (2), coeficientes devem ser negativos, o que é impossível pois ai ∈ {0, 1}.
18 ⋅ 7 Assim, as duas raízes inteiras possíveis para P(x) são= S {0 ; −1}.
=r = 21 (Não é primo)
7 −1
b) Sabe-se que P(0) = 0 e p(–1) = 0, logo:
Substituindo p = 19 na equação (2), P(0) =0 → a0 =0
18 ⋅ 19 P( −1) = 0 → −1+ a2014 − a2013 + a2012 − a2011 +  + a4 − a3 + a2 − a1 + a0 = 0
=r = 19
19 − 1
Reescrevendo a equação acima, tem-se:
Solução nesse caso: p = 19, q = 5 e r = 19
a2014 + a2012 +  + a4 + a2 =1+ a2013 + a2011 +  + a3 + a1
Terceiro caso: q = 7 1007 1006
35p + 11p ⋅ 7 + 7r = p ⋅ 7 ⋅ r
5p + 11p + r = pr
=∑
a2i
=i 1=i 1
∑a 2i+1 + 1

16p + r = pr Assim, como ai ∈ {0, 1}, no lado esquerdo da equação deve-se escolher
r (p − 1) =
16p n dos 1007 coeficientes ai para ser 1 e o restante para ser zero. Ou
1007

r=
16p
p −1
( 3)
1007 
seja, há 
 n 
 possibilidades para
i=1
∑a2i.

(p − 1) 16 De modo semelhante, no lado direito da equação deve-se escolher


p – 1 = 1 ou p – 1 = 2 ou p – 1 = 4 ou p – 1 = 8 ou p – 1 = 16 n – 1 dos 1007 coeficientes ai para ser 1 e o restante para ser zero. Ou
1006
1007 
De p – 1, p = 2
De p – 1 = 2. p = 3
seja, há 
 n − 1
 possibilidades para

∑a2i+1.
i=1
De p – 1 = 4, p = 5
De p – 1 = 8, p = 9 (Não é primo) 1007  1007 
Considerando essas duas possibilidades, há  ⋅  possíveis
De p – 1 = 16, p = 17  n   n −1 
Substituindo p = 2 na equação (3), valores para os coeficientes ai. Ou seja, o número de possíveis
1007
1007  1007 
=r
16 ⋅ 2
= 32 (Não é primo)
2 −1
polinômios com duas raízes inteiras distintas será
Pela Fórmula de Euler sabe-se que: n =1
∑  ⋅
 n −1   n 
.

Substituindo p = 3 na equação (3), m + h


p
m  h 
=
16 ⋅ 3
r = 24 (Não é primo)
=
 p 
 ∑  i  ⋅  p − i
i= 0
3 −1
Substituindo p = 5 na equação (3), Sendo p = 1006, m = 1007 e h = 1007, tem-se:
1006 1007
1007 + 1007   1007  1007  1007  1007 
=r
16 ⋅ 5
= 20 (Não é primo)
5 −1


=
1006
=
 n 0=

 1006∑− n
 ⋅
  n
=
  ∑
 ⋅
 n 1  n −1   n 

Substituindo p = 17 na equação (3), Assim, o número de possíveis polinômios com duas raízes inteiras
16 ⋅ 17  2014 
=r = 17 distintas será N = 
17 − 1 .
 1006 
Solução nesse caso: p = 17, q = 7 e r = 17 EXERCÍCIOS DE COMBATE
Resposta: p = 19, q = 5 e r = 19 ou p = 17, q = 7 e r = 17 01. C 09. D q = 15
02. A 10. E 15. 03
17. A primeira condição nos diz que n ≥ 55 e a segunda que n ≤ 59. Assim,
03. C 11. 473 16. 15
teremos no máximo 8 múltiplos de 7 quando n = 56, 57, 58 ou 59.
04. C 12. -0,5 17. resto 4
18. A equação dada é equivalente a x ( x + 2013 ⋅ 2012) = 2013y . 05. C 13. a) (4,6,9), 18. (37,99), (-37,-99)
Veja que 2013 | x ( x + 2013 ⋅ 2012) ⇒ 2013 | x 2 ⇒ 2013 | x , pois 06. D (8,12,18), (12,18,27) (13,51), (-13,-51)
2013 =3 ⋅ 11⋅ 61 é livre de quadrados. Fazendo então x = 2013k, 07. E b) n = 4 (7,21), (-7,-21)
t e m o s 20132k (k + 2012) = 2013y ⇔ k (k + 2012) = 2013y −2 ( y > 2) . 08. B 14. n = 4, p = 3,
Se y = 3, temos k = 1, o que nos dá uma solução. Suponhamos
agora y > 3 e daí se d | k e d | k + 2012 , teríamos d | 2012 , 19. a) n = 2 e k = 6
o que nos dá d = 1, pois 2012 e 2013 são primos entre si. Logo, b) Snk = 1 n=3 k=0 Snk = 5 n=2 k=1 Snk = 9 n=3 k=2
podemos escrever k = ay −2 e k + 2012 = by–2, onde a e b são inteiros Snk = 2 n=4 k=0 Sn = 6 n=1 k=0
k
Snk = 10 n=1 k=1
positivos tais que ab = 2013. Então, by −2 − ay − 2 = 2012 . Temos que Snk = 3 n=3 k=1 Sn = 7 n=2 k=2
k
Snk = 11 n=1 k=2
by − 2 − ay − 2 é função crescente de y e, portanto, 2012 ≥ b2 − a2 , com Snk = 4 n=2 k=0 Sn = 8 n=4 k=1
k
Snk = 12 n=4 k=2
ab = 2013. Temos poucas possibilidades para a e b: podemos ter c) Snk = 13q + r
( a,b ) = (1,2013) , ( 3,671) , (11,183) , ( 33,61) . Em todos os casos, temos 20. 17|(9r + 5s)
que b2 − a2 > 2012 , contradição. Portanto, y = 3 de fato.

26 PROMILITARES.COM.BR
GEOMETRIA PLANA I

TEOREMA DE PITOT ARCO-SOMA


“Em qualquer quadrilátero convexo circunscritível a um círculo, as
somas dos lados opostos são iguais.” A
C AM × BC = AC × BM + AB × CM
y B a x 2 R ⋅ x = bm + an
AB + CD = x bm + an
A B x=
n
(x + y) + (z + t) y m
2R
x
BC + AD = b − 4R2 − a2 + a 4R2 − b2
x=
(y + z) + (x + t) 2R
z
AB + CD = BC + AD t
t z C ARCO-DIFERENÇA
D
B
TEOREMA DE HIPARCO E PTOLOMEU x AC × BM = BC × AM + AB × CM
a C
“Em todo quadrilátero convexo inscritível num círculo o produto b
b ⋅ m = x ⋅ 2R + a ⋅ n
das diagonais é igual à soma dos produtos dos lados opostos”. A m n
bm − an
A A x=
2R
M
θ θ
a b a b x=
b − 4R2 − a2 − a 4R2 − b2
2R
β
j j
B B
D α D
d c
c TEOREMA DE PTOLOMEU
d
“A razão entre as diagonais de um quadrilátero convexo inscritível
C C num círculo é igual à razão entre as somas dos produtos dos lados
que concorrem respectivamente nas extremidades de cada diagonal”.
JD a ac (1)
∆ AJD ~ ∆ ABC → = ∴ JD =
c β β m q m q
∆ AIB ~ ∆ CID → = → = (1)
a c ad cd
JB b bd (2)
∆ ABJ ~ ∆ ACD → = ∴ JB =
d β β q n q n
∆ BIC ~ ∆ AID → = → = (2)
d b cd bc
Somando (1) e (2), temos:

ac + bd ac + bd n p n p
∆ AIB ~ ∆ CID → = → = (3)
=
JD + JB = ou α c a cb ab
β β
αβ = ac + bd Comparando (1), (2) e (3), temos:

m q n p
APLICAÇÃO = = = ou
ad cd bc ab
Calcular a corda correspondente ao arco-soma ou arco-diferença
de dois raios cujas cordas a e b são dadas num círculo de raio R. m+n p+q
= ou
ad + bc cd + ab

AC ad + bc
=
BD ab + cd

PROMILITARES.COM.BR 27
GEOMETRIA PLANA I

B CÁLCULO DO PRODUTO CONSTANTE


Produto constante: PM . PN
2
PM ⋅ PN = (d – R)(d + R) = d2 – R2 = PT
a
b
QUADRADO DA TANGENTE

A I n T
m C
q c
d
P M R R
D d

APLICAÇÃO
Calcular as diagonais de um quadrilátero convexo inscritível num
círculo sabendo-se que os lados são dados. 2º caso: P interior ao círculo
C
αβ= ac + bd
 B
 α ad + bc E
 =
 β ab + cd
P
A F
RELAÇÕES MÉTRICAS NOS CÍRCULOS
TEOREMA
“A perpendicular traçada de um ponto qualquer de um círculo em
relação a um diâmetro é média geométrica entre os segmentos que
determina sobre o diâmetro”.
“Qualquer corda de um círculo é média geométrica entre o D
diâmetro que passa por uma de suas extremidades e a sua projeção
sobre este diâmetro”. ∆ PAC ~ ∆ PBD
ou
P PA PC ou
=
PD PB

∆ APB é um retângulo PA ⋅ PB = PC ⋅ PD
2
PH= AH ⋅ HB A B
H 0 Generalizando, temos:
2
AP= AH ⋅ AB PA ⋅ PB = PC ⋅ PD = PE ⋅ PF = ... = cte

CÁLCULO DO PRODUTO CONSTANTE


T

TEOREMA
“Se um círculo é interceptado por um feixe de retas que concorrem
num ponto P, os produtos dos segmentos determinados em cada reta 0
e compreendidos entre o ponto P e o círculo é um produto constante”. M N
d
1º caso: P exterior ao círculo. R

A
Produto constante: PM . PN
2
P PM ⋅ PN = (R – d)(R + d) = R2 – d2 = PT
C D Quadrado da metade da corda mínima que passa por P.
E
F
∆ PCB ~ ∆ PAD
PC PB
=
PA PD

28 PROMILITARES.COM.BR
GEOMETRIA PLANA I

POTÊNCIAS Observação
Potência de um produto P em relação a um segmento AB, colinear 1) O eixo radical está sempre mais próximo do centro do menor
com P é o produto PA . PB. círculo.

P A M B 2) O eixo radical dos dois círculos exteriores contém os pontos


médios das tangentes comuns interiores e exteriores.
a a
d
M
A B
C
p = PA ⋅ PB = (d – a)(d + a) = d2 – a2
N
positiva se d > a o o’

A potência pode ser: negativa se d < a
nula se d = a D

Potência de um ponto em relação a um círculo
MA = MB → P M(o) = P N(o’)
É o produto PA . PB, sendo A e B dois pontos do círculo colineares
com P. NA = NB → P N(o) = P N(o’)
2 3) O eixo radical de dois círculos tangentes interiormente ou
p – PA ⋅ PB = PC ⋅ PD = ... = PT = d – R 2 2
exteriormente é a tangente comum que passa pelo ponto de
Se P é exterior ao círculo, a potência é positiva, isto é, contato.
d > R. (FIG. 1).
B
T D
A D A A
C A
P E
F P
C
T B O ponto A é potência mula em relação aos dois círculos.
4) O eixo radical de dois círculos secantes é a reta suposta de corda
(figura 1) (figura 2) comum.
A
2
p = PA ⋅ PB = PC ⋅ PD = ... = PT = R2 – d2
Se P é interior ao círculo, a potência é negativa, isto é,
d < R. (FIG. 2) o o

EIXO RADICAL B
É o lugar geométrico dos pontos que possuem igual potência em
relação a dos círculos dados.
Os pontos A e B possuem potência numa em relação aos dois
Seja P um ponto do lugar. círculos, e são suficientes para definir o eixo radical.
Devemos ter: 5) Para os pontos exteriores aos círculos o eixo radical é o lugar
P geométrico dos pontos de onde se podem tirar tangentes iguais
em relação aos dois círculos.

D d

0 M H 0’
r
R

O eixo radical é o lugar geométrico do centro dos círculos que são


P0 = D2 – R2 simultaneamente ortogonais aos dois círculos dados.
P0 = d2 – r2
Logo:
D2 – R² = d² – r²

PROMILITARES.COM.BR 29
GEOMETRIA PLANA I

TEOREMA APLICAÇÃO
“Se os centros de três círculos não são colineares os eixos radicais Determinar graficamente o eixo radical de dois círculos.
dos três círculos tomados dois a dois concorrem num mesmo ponto,
chamado Centro Radical”. c
c

o2

o1 o3
o1 o2

Como “c” pertence ao eixo radical dos círculos o1 e o3, temos:


Pc (o1) = pc (o3)
o3
Como “c” pertence ao eixo radical dos círculos o2 e o3 temos:
Traçar um círculo qualquer que intercepte os dois círculos dados.
Pc (o2) = pc (o3) Em seguida determinar o centro radical “c” de três círculos. O eixo
radical procurado é a reta perpendicular ao segmento o1 e o2
Logo: passando por “c”.
Pc (o1) = pc (o3)
POLÍGONOS REGULARES
e o ponto “c” pertence também ao eixo radical.
Espécie de um polígono
Ângulo externo de um polígono é o menor ângulo de que se deve
Observação girar um lado para que ele coincida com o lado seguinte, inclusive em
1) Se três círculos são tangentes exteriormente dois a dois, as sentido.
tangentes comuns interiores concorrem mesmo ponto. B
Ae
Be
o2 A De Ae

Ce
o1 Be
De
o3 C
Ce

O ponto “c” é também o centro do círculo inscrito no triângulo: 1 volta completa


o1 o2 o3
2) Se três círculos são secantes dois a dois, as cordas comuns D
concorrem num mesmo ponto.
De
Ae
B Ce Ae
A
Be Ee
De
Be
Ce
C Ee E

2 voltas completas
O nº de voltas descritas quando se tomam sucessivamente os
ângulos externos de um polígono em torno de um ponto arbitrário é
3) Se o centro radical é exterior aos círculos, ele é o único ponto de
o número que define a espécie do polígono.
onde se podem traçar tangentes iguais em relação aos três círculos.
O centro radical é ainda o centro do único círculo simultaneamente Soma dos ângulos externos
ortogonal aos três círculos dados. Se = 360° k

30 PROMILITARES.COM.BR
GEOMETRIA PLANA I

Soma dos ângulos internos


Ai + Ae = 180°
Bi + Be = 180°
.. ..
.. ..
.. ..
_____________
0

Si + Se = 180° n
Si = 180 n – 360 k
A M B
Si = 180 (n – 2k)

EXPRESSÕES NOS POLÍGONOS A’ N B’


REGULARES
Ln
CÁLCULO DO APÓTEMA
∆ OAB ~ OA'B'

AB OM  n an R n
= ou = ∴ Ln =
A'B' ON Ln R an

0
R BA QUADRADO

A B
H A

n 4
→ ∆ OAH
2
2
a=
n
  n
R2 −   o4
 2 
D o B
=an
1
2
4 R2 − 2n R

CÁLCULO DO LADO DO POLÍGONO DE 2n LADOS

C
0
A an
B ∆ OAB →  4 = R 2
M
AB ou R 2
∆ ABD → a4 = a4 =
2 2
n
N
OCTÓGONOS
n
Aplicando  2n = 2R2 − R 4R2 − 2n
onde:
2 onde: =
n =
4 R 2
  n
 + (R − an )
2
AMN ⇒ =
2
2n 
 2  =
8 R 2 − 2
1 Há dois octógonos regulares:
=an 4 R2 − 2n
2 • octógono regular convexo

 2n = 2 R2 − R 4 R2 − 2n • octógono regular estrelado de 3ª espécie

PROMILITARES.COM.BR 31
GEOMETRIA PLANA I

8 AD ou 6 = R
→ AB =
A 2
AD
M B → BD = 3 ou 3 = R 3
2

R
0 2
N R
8* 2

R
2
E
R
2
2 2 2
∆ ABE → BE = AE − AB
28 4R2 − 28
= R 3

*8 R 2 + 2
= AB R
OM = ou a3 =
Os apótemas são: 2 2

BE *8 R BD R 3
a8 = OM = = ⇒=
a8 2+ 2 ON = ou a6 =
2 2 2 2 2

AB  8 R
a*8 ON
= = = ⇒=
a8 2− 2
2 2 2 DODECÁGONO
Aplicando 2n em função de n temos:
CÁLCULO DO LADO DO POLÍGONO REGULAR
DE 2K LADOS  2n = 2R2 − R 4R2 − 2n
k=2 n=4 4 = R 2
onde:
k=3 n=8 8 = R 2 − 2
=
n =
6 R
k=4 n = 16 16 = R 2 − 2 + 2 =
12 R 2 − 3
Há dois decágonos regulares: um de 1ª espécie e um de 5ª
n = 2k ⇒  2k = R 2 − 2 + 2 + ... espécie.
(k – 1) radicais.
12
A

TRIÂNGULO EQUILÁTERO E HEXÁGONO M B

012
A 6
0
60° 012* N
B
12*
06

0 E
03
M 2 2
3 No ∆ABG temos: =
BG ( 2R )2 − AB
30° onde:
AB = =
12 R 2 − 3

D BG = =
12 R 2 + 3
Os apótemas são:
BG *12 R
OM = = ou =
a12 2+ 3
∆ ABD 2 2 2

32 PROMILITARES.COM.BR
GEOMETRIA PLANA I

R
ON = AB = 12 ou =
a12 2− 3
2 2 2 A 10
5 B
DECÁGONOS E PENTÁGONOS
= =
Há dois decágonos 
n 10 p 1 convexo
05 010
= =
n 10 p 3 estrelado
J
» = 36°
AB
A AB = l 10
0 0*5
¼ = 3 × 36°
A
AD AD = l *
10 0*10
A *5

36° B *10
36° 72°
F
I

36° 72°
∆ AJF:
72° =
*2
4R2 −  102
0  5
onde: 
36° =10

R
2
5 +1 ( )
D
R
=
5 10 − 2 5
2

∆ ABF:
=
*2
4R2 −  102
 5
onde: 
∆ OAB ~ ∆ AIB =10

R
2
5 −1 ( )
AB OB (1)
= ∴ AB2 =OB ⋅ IB R
BI AB =
 *5 10 + 2 5
2
= AI
OI = AB = 10
Mas  Da figura temos ainda:
IB =OB − OI =R − 10
10 10  *5 5
=a5 = a*5 = a10 = a*10
2
Logo: =
10 R (R − 10 ) 2 2 2 2

=2
10
R
2
( 5 −1) =
a5
R
2
( 5 +1) =
a*5
R
2
( 5 −1 )
O lado do decágono regular convexo é então o segmento áureo
interno do raio. =
a10
R
4
( 10 + 2 5 ) =
a*10
R
4
( 10 − 2 5 )
Ainda *10 = AD = AI + ID = 10 + R

TEOREMA
*10
=
R
2
( 5 +1 ) Os lados do hexágono regular e do decágono regular convexo são
O lado do decágono regular estrelado é então o segmento áureo os catetos de um triângulo retângulo cuja hipotenusa é igual ao lado
externo do raio. do pentágono regular convexo.
= n 5=p 1 convexo A demonstração algébrica é fácil de fazer:
Há dois pentágonos regulares 
= n 5=p 5 estrelado 6 = R
Dividindo uma circunferência em 10 partes temos:
 2 R2 R2
2 2
 6 + 10 = R +
 4
( )
5 −1 =
4
(
10 − 2 5 = 25 )
AB = 36° AB = 10
BF = 144° BF = 5*
FJ = 108° FJ = *10
=
10
R
2
( 5 −1 )
JA = 72° JA = 5
360°

PROMILITARES.COM.BR 33
GEOMETRIA PLANA I

DEMONSTRAÇÃO GRÁFICA x 2 + n2 − c2 b2 − m2 − x 2
= ⇔
Tomamos AP = R e deste modo OP = 5 2nx 2mx
⇔ mx 2 + mn2 − mc2 = nb2 − nm2 − nx 2
T
⇔ nb2 − x 2 (m + n) + mc
= 2
mn (m + n) ⇔
P
⇔ nb2 − ax 2 + mc2 =
amn
B b2 x 2 c2
⇔ − + =
1
R am mn an

10
0 EXERCÍCIOS DE

A
TREINAMENTO
01. (IME 2017) Dado um quadrado ABCD, de lado a, marcam-se os
pontos E sobre o lado AB, F sobre o lado BC, G sobre o lado CD e H
sobre o lado AD, de modo que os segmentos formados AE, BF, CG, e
Por P traçamos a tangente PT e temos:
DH tenham comprimento igual a 3a .
2 2 4
= R (R − 10 )
PT= PA ⋅ PB ⇒ PT
A área do novo quadrilátero formado pelas interseções dos segmentos
Como já vimos anteriormente: AF, BG, CH, e DE mede:
2
=
10 R (R − 10 ) , logo PT = 10 a2 a2
a) d)
Do ∆ OPT temos: 25 9
2 2 2
= OT + PT
OP a2 2a2
b) e)
2
=
5 26 + 10
2
18 9

a2
RELAÇÃO DE STEWART c)
16
02. (ITA 2017) Seja ABC um triângulo cujos lados AB, AC e BC
medem 6 cm, 8 cm e 10 cm, respectivamente. Considere os pontos
M e N sobre o lado BC tais que AM é a altura relativa a BC e N é o
ponto médio de BC. A área do triângulo AMN, em cm2, é
a) 3,36 d) 4,48
b) 3,60 e) 6,72
c) 4,20

03. (IME 2016) Considere quatro pontos distintos coplanares. Das


distâncias entre esses pontos, quatro delas valem a e duas delas valem
2
b
b. O valor máximo da relação   é
a
a) 2 1+ 2 2
d)
b) 1+ 3
e) 2+2 3
b2 x 2 c2 2+ 3
− + =
1 c)
am mn an
04. (ITA 2016) Sejam λ uma circunferência de raio 4 cm e PQ uma
DEMONSTRAÇÃO corda em λ de comprimento 4 cm. As tangentes a λ em P e em Q
Seja APB =ˆ
ˆ = θ , então APC 180 − θ . Aplicando a lei dos cossenos interceptam-se no ponto R exterior a λ. Então, a área do triângulo em
aos triângulos APB e APC, temos: PQR, em cm2, é igual a

x 2 + n2 − c2 a) 2 3 d) 2 3
c2= n2 + x 2 − 2nx cos θ ⇔ cos θ= . .
2nx 3 5
2
b= m2 + x 2 − 2mx cos 180 − θ= ( ) 3 2 4 3
2 2
b) . e) .
= m + x + 2mx cos θ 2 3
b2 − m2 − x 2
⇔ cos θ = 6
2mx c) .
2

34 PROMILITARES.COM.BR
GEOMETRIA PLANA I

05. (ITA 2014) Em um triângulo isósceles ABC, cuja área mede 48 cm2, 09. No triângulo ABC, AB = 20, AC = 21 e BC = 29. Os pontos D e
2 E sobre o lado BC são tais que BD = 8 e EC = 9. A medida do ângulo
a razão entre as medidas da altura AP e da base BC é igual a . Das
3 DÂE, em graus, é igual a:
afirmações abaixo: a) 30 d) 60
I. As medianas relativas aos lados AB e AC medem 97 cm; b) 40 e) 75
II. O baricentro dista 4 cm do vértice A; c) 45
III. Se α é o ângulo formado pela base BC com a mediana BM,
3 10. (ITA 1978) Os catetos b e c de um triângulo retângulo de altura
relativa ao lado AC, então cos α = , h (relativa à hipotenusa), são dados pelas seguintes expressões:
97
é (são) verdadeira(s) 1 1
=
b k + e= c k − onde k é um número real maior que 1. Então
a) Apenas I. d) Apenas I e III. k k
b) Apenas II. e) Apenas II e III. o valor de h em função de k é:
c) Apenas III. a) k2 − 1 c) 1+ k2 e) k4 − 1
2k −1− k 2 2k 3
06. (EN 2012) O triângulo da figura abaixo é equilátero, AM
= MB
= 5
b) k2 − 1
d) 2 (k 2 − 1)
e CD = 6. A área do triângulo MAE vale
k2 − 2 2k

11. (ITA 1979) Considere o triângulo ABC, onde AD é a mediana relativa


ao lado BC. Por um ponto arbitrário M do segmento BD, tracemos o
segmento MP paralelo a AD, onde P é o ponto de intersecção desta
paralela com o prolongamento do lado AC (figura 1). Se N é o ponto
de intersecção de AB com MP, podemos afirmar que:

a) 200 3 d) 200 2
11 11

b) 100 3 e) 200 2
11 2

c) 100 2
2

a) MN + MP = 2BM d) MN + MP = 2AD
07. (ITA 2007) Seja Pn um polígono regular de n lados, com n > 2. b) MN + MP = 2CM e) MN + MP = 2AC
Denote por an o apótema e por bn o comprimento de um lado de Pn. c) MN + MP = 2AB
O valor de n para o qual valem as desigualdades bn ≤ an e bn-1 > an-1,
pertence ao intervalo
12. (ITA 2000) Considere a circunferência inscrita num triângulo
a) 3<n<7 d) 10 < n < 13 isósceles com base de 6 cm e altura de 4 cm. Seja t a reta tangente
a esta circunferência e paralela à base do triângulo. O segmento de t
b) 6<n<9 e) 12 < n < 15
compreendido entre os lados do triângulo mede
c) 8 < n < 11
a) 1 cm d) 2,5 cm
08. (ITA 2007) Sejam p1 e p2 octógonos regulares. O primeiro está b) 1,5 cm e) 3 cm
inscrito e o segundo circunscrito a uma circunferência de raio R. Sendo c) 2 cm
A
A1 a área de p1 e A2 a área de p2, então a razão 1 é igual a 13. Seja um triângulo ABC, onde as alturas AP, BQ e CR se interceptam
A2
no ponto H interno ao triângulo. Sabendo-se que H é o ponto médio
de AP e que CH é o dobro de HR, pode-se afirmar que a medida do
a) 5 d) 4 2 +1 ˆ é:
ângulo ABC
8 8
a) O triplo da medida de ACR.ˆ

9 2 2+ 2 b) O dobro da medida de CÂP.


b) e)
16 4 c) ˆ
Um terço da medida de AHC.
d) Metade da medida de BÂC.
c) 2 ( 2 − 1) e) ˆ
O dobro da medida de ABH.

PROMILITARES.COM.BR 35
GEOMETRIA PLANA I

14. Qual é o perímetro de um quadrilátero convexo inscrito em 20. O triângulo XYZ tem lados de comprimentos 3, 4 e 5. P é um
uma circunferência de raio unitário, sabendo-se que foi construído ponto no interior do triângulo para o qual XPY ˆ
= =ˆ
YPZ ˆ .
ZPX
utilizando-se, pelo menos uma vez e somente, os lados do triângulo As distâncias de P a X, Y e Z são a, b e c, respectivamente. Determine
equilátero, quadrado e hexágono regular inscritos nessa circunferência? o valor de a2 + b2 + c2 é
a) 3+ 2+2 d) 3+2 2+2 EXERCÍCIOS DE
2 ( 3 + 2 + 1)
COMBATE
b) 3 + 2 2 +1 e)
c) 2 3 + 2 +1

15. (CN 1990) O quadrilátero ABCD está inscrito num círculo de raio
unitário. Os lados AB, BC e CD são respectivamente, os lados do 01. (IME 2018) Seja um heptágono regular de lado  cuja menor
triângulo equilátero, do quadrado e do pentágono regular inscrito no diagonal vale d. O valor da maior diagonal satisfaz a qual das
círculo. Se x é a medida do lado AD do quadrilátero, pode-se afirmar expressões?
que:
Observação: CD é aproximadamente igual a 1,2. a) ⋅d c) ⋅d e) 3⋅d
d− d+ 2
a) 1,0 < x < 1,2 d) 1,6 < x < 1,8
b) 1,2 < x < 1, 4 e) 1,8 < x < 2,0 b) d2 d) 2
d− d+
c) 1, 4 < x < 1,6

16. (Olimpíada de Maio-2003) Seja ABCD um retângulo de lados AB = 02. (ITA 2018) Os lados de um triângulo de vértices A, B e C medem
4 e BC = 3. A perpendicular à diagonal BD traçada por A corta BD no AB = 3 cm, BC = 7 cm e CA = 8 cm. A circunferência inscrita no
ponto H. Chamamos de M o ponto médio de BH e de N o ponto médio triângulo tangencia o lado AB no ponto N e o lado CA no ponto K.
de CD. Calcule a medida do segmento MN. Então, o comprimento do segmento NK, em cm, é

17. Na figura a seguir, AM, BN e CP são paralelos. a) 2. d) 2 3.

N b) 2 2. 7
e) .
2
M c) 3.

P 03. (ITA 2017) Considere o triângulo ABC, em que os segmentos


AC, CB e AB medem, respectivamente, 10 cm, 15 cm e 20 cm. Seja
D um ponto do segmento AB de tal modo que CD é bissetriz do
ângulo ACB  e seja E um ponto do prolongamento de CD, na direção
A C B  = DCB.
de D, tal que DBE  A medida, em cm, de CE é

Prove que 1 1 1 11 6 20 6
+ = a) . d) .
AM BN CP 3 3

ˆ mede 135O e os b) 13 6 e) 25 6
18. (ITA 2011) Num triângulo AOB, o ângulo AOB . .
3 3
lados AB e OB medem 2 cm e 2 – 3 cm , respectivamente. A
circunferência de centro em O e raio igual à medida de OB intercepta 17 6
c) .
AB no ponto C (≠B). 3
a) ˆ mede 15O.
Mostre que OAB 04. (ITA 2017) Seis circunferências de raio 5 cm são tangentes entre
b) Calcule o comprimento de AC . si duas a duas e seus centros são vértices de um hexágono regular,
conforme a figura abaixo.
19. (EFOMM 2010) Os lados de um triângulo ABC são tais
que BC é a média aritmética de AC e AB , onde AC < AB .
Os ângulos internos  , B̂ e Ĉ desse triângulo possuem a seguinte
propriedade:

sen2 Aˆ + sen2 Bˆ − sen2 Cˆ − 2 ⋅ sen Aˆ ⋅ senBˆ ⋅ cos Cˆ =cos2 Cˆ

Se o perímetro do triângulo ABC mede 3 3 m , sua área, em m2, é


igual a
3 3 c) 9
a)
4 8
3 d) 2
b)
4 e) 4

36 PROMILITARES.COM.BR
GEOMETRIA PLANA I

O comprimento de uma correia tensionada que envolve externamente a) 5 d) 10


as seis circunferências mede, em cm, b) 6 e) 15
a) 18 + 3π d) 60 + 10π c) 8
b) 30 + 10π e) 36 + 6π
c) 18 + 6π 10. (ITA 2015) Seja ABCD um trapézio isósceles com base maior AB
medindo 15, o lado AD medindo 9 e o ângulo ADB ˆ reto. A distância
05. (IME 2016) Em um triângulo ABC, o ponto D é o pé da bissetriz entre o lado AB e o ponto E em que as diagonais se cortam é
AB = α.
relativa ao ângulo Â. Sabe-se que= =
AC AD, r e que C 21 37
AC a) . d) .
8 8
Portanto o valor de sen2α é
b) 27 e) 45
. .
3r − 1 3r + 1 8 8
a) d)
4 4r 35
c) .
8
b) 3r − 1 e) 3r + 1
11. (IME 2018) Considere um triângulo ABC onde BC = a, AB = c,
4r 4
AC = b, c > b.O círculo inscrito a esse triângulo tangencia BC, em D e
r+3
DE é um diâmetro desse círculo. A reta que tangencia o círculo e que
c) passa por E intercepta AB em P e AC em Q. A reta AE intercepta BC
4 no ponto R. Determine os segmentos de reta EQ e DR em função dos
06. (EN 2016) Um triângulo inscrito em um círculo possui um lado lados do triângulo: a, b e c.
de medida 24 3 oposto ao ângulo de 15o. O produto do apótema
do hexágono regular pelo apótema do triângulo equilátero inscritos 12. (ITA 2018) Uma reta r separa um plano π em dois semiplanos π1 e π2.
nesse círculo é igual a: Considere pontos A e B tais que A ∈ π1 e B ∈ π2 de modo que d(A,r) =
3, d(B,r) = 6 e d(A,B) = 15. Uma circunferência contida em π passa pelos
a) 3( 3 + 2) d) 2(2 3 + 3) pontos A e B e encontra r nos pontos M e N. Determine a menor distância
possível entre os pontos M e N.
b) 4(2 3 + 3) e) 6( 2 + 1)
13. (IME 2017) Em um triângulo ABC, a medida da bissetriz interna
c) 8 3 + 12 AD é a média geométrica entre as medidas dos segmentos BD e DC,
e a medida da mediana AM é a média geométrica entre os lados AB
07. (ITA 2016) Sejam λ uma circunferência de raio 4 cm e PQ uma e AC. Os pontos D e M estão sobre o lado BC de medida a. Pede-se
corda em λ de comprimento 4 cm. As tangentes a λ em P e em Q determinar os lados AB e AC do triângulo ABC em função de a.
interceptam-se no ponto R exterior a λ. Então, a área do triângulo em
PQR, em cm2, é igual a 14. (IME 2016) Uma corda intercepta o diâmetro de um círculo de
centro O no ponto C’ segundo um ângulo de 45O. Sejam A e B os
pontos extremos desta corda, e a distância AC’ igual a 3 + 1cm.
a) 2 3 d) 2 3 O raio do círculo mede 2 cm, e C é a extremidade do diâmetro mais
. .
3 5 distante de C’. O prolongamento do segmento AO intercepta BC em
A’. Calcule a razão em que A’ divide BC.
b) 3 2 e) 4 3
. .
2 3 15. (ITA 2013) Em um triângulo de vértices A, B e C, a altura, a
bissetriz e a mediana, relativamente ao vértice C, dividem o ângulo
6 ˆ
BCA em quatro ângulos iguais. Se  é a medida do lado oposto ao
c) .
2 vértice C, calcule:
08. (ITA 2016) Seja Pn um polígono convexo regular de n lados, com a) A medida da mediana em função de .
n ≥ 3. Considere as afirmações a seguir: b) ˆ
Os ângulos CAB, ABC ˆ
ˆ e BCA.
I. Pn é inscritível numa circunferência.
16. (ITA 2011) Considere um triângulo equilátero cujo lado mede
II. Pn é circunscritível a uma circunferência.
2 3cm . No interior deste triangulo existem 4 círculos de mesmo raio
III. Se  n é o compromisso de um lado de Pn e an é o comprimento de r. O centro de um dos círculos coincide com o baricentro do triângulo.
a Este círculo tangência externamente os demais e estes, por sua vez,
um apótema de Pn , então n ≤ 1 para todo n ≥ 3.
n tangenciam 2 lados do triângulo.
É (são) verdadeira(s) a) Determine o valor de r.
a) apenas I. d) apenas I e II. b) Calcule a área do triângulo não preenchida pelos círculos.
b) apenas II. e) I, II e III. c) Para cada circulo que tangencia o triângulo, determine a distancia
c) apenas III. do centro ao vértice mais próximo.

09. (ITA 2015) Num triângulo PQR, considere os pontos M e N 17. O octógono da figura é obtido reunindo-se oito trapézios isósceles
pertencentes aos lados PQ e PR, respectivamente, tais que o congruentes. Se os três menores lados de cada trapézio possuem
comprimento 1, qual a medida de cada lado do octógono exterior?
segmento MN seja tangente à circunferência inscrita ao triângulo
PQR. Sabendo-se que o perímetro do triângulo PQR é 25 e que a
medida de QR é 10, então o perímetro do triângulo PMN é igual a

PROMILITARES.COM.BR 37
GEOMETRIA PLANA I

3a
= BF
AE = CG
= DH
=
4
a
= FC
EB = GD = AH =
4
∆AED ≡ ∆BFA ≡ ∆CGB ≡ ∆DHC
Quadrilátero amarelo → quadrado de lado x
PE AD x a a a2
= → = →x= → Área = x 2 =
DG DE a 2 5 25
4  3a 
a2 +  
4

02. A
Calculando:
6, 8, 10 ⇒ Pitagórico
18. Determine o lado do hexágono equilátero inscrito numa 24
semicircunferência do círculo de raio r e centro O, onde uma de suas 10 ⋅ h = 6 ⋅ 8 ⇒ h =
5
bases está sobre o diâmetro, é:
( ) 576
2 2
∆AMB ⇒ 62 = h2 + 5 − MN ⇒ 36 = + 25 − 10MN + MN ⇒ MN = 1, 4
25
24
1, 4 ⋅
MN ⋅ h 5 ⇒S
S∆AMN = = ∆AMN = 3,36
2 2

03. C
As distâncias mencionadas podem formas triângulos ou quadriláteros.
No caso de quadriláteros, podem ser: quadrado, retângulo ou
trapézios, conforme figuras a seguir:

19. No triângulo retângulo isósceles ABC, B̂ = 90 , AD é a mediana


relativa ao lado BC. Seja AB = BC = a. Se BE ⊥ AD, interceptando AC
em E, e EF ⊥ BC em F, determine EF.

ˆ = 30 .
ˆ = 45 e ACB ∆ABC =
∆ABD =
∆BCD =
∆ACD
20. Considere um triângulo ABC com BAC
2
ˆ
Se M é o ponto médio do lado BC, mostre que AMB = 45 e que b
b2 = a2 + a2 →   = 2
BC ⋅ AC =2 ⋅ AM ⋅ AB . a

GABARITO
EXERCÍCIOS DE TREINAMENTO
01. A
Pode-se desenhar, segundo o enunciado:

OC + OD > a

OA + OB > a
OA + OB + OC + OD > 2a
(OA + OD) + (OB + OC) > 2a
2a > 2a → não é possível

38 PROMILITARES.COM.BR
GEOMETRIA PLANA I

∆AOB → x > 90° 


 → não é possível
∆AOC → x < 90°

OC + OD > b

OA + OB > b
OA + OB + OC + OD > a + b
(OA + OC) + (OB + OD) > a + b
a + b > a + b → não é possível
∆ABC ≡ ∆ABO
=
ABC ABO → não é possível

∆ABC ≡ ∆ACD
2
b
b2 = a2 + a2 − 2 ⋅ a ⋅ a ⋅ cos 30° →   = 2 − 3
a

No caso de triângulos, podem ser conforme figuras a seguir:


∆ABO ≡ ∆AOC
2
b
b2 = a2 + a2 − 2 ⋅ a ⋅ a ⋅ cos150° →   = 2 + 3
a

2
Portanto, o maior valor de   é 2 + 3.
b
a
04. E

x < 60° 
 → não é possível
=
x 75°

ˆ = 90° − 60°= 30°


OPR

PROMILITARES.COM.BR 39
GEOMETRIA PLANA I

No triângulo PMR, temos: Portanto, sabendo que MÂE = 60O, encontramos


1 
h 3 h 2 3 (MAE) = ⋅ MA ⋅ EA ⋅ senMAE
tg30°= ⇒ = ⇒ h= cm 2
2 3 2 3
1 80 3
= ⋅5⋅ ⋅
Logo, a área do triângulo PQR será dada por: 2 11 2
100 3
= .
1 2⋅ 3 4⋅ 3 11
A= ⋅4⋅ ⇒A=
2 3 3
07. B
05. A
I. Verdadeira. Sabendo que a área do triângulo ABC mede 48 π
2
cm2 e que AP= ⋅ BC , vem an 2
3

1 1 2
(ABC) = ⋅ BC ⋅ AP ⇔ 48 = ⋅ BC ⋅ ⋅ BC
2 2 3 bn
⇔ BC =32 ⋅ 42 2
⇒ BC =
12cm
π b
tg   = n
Logo,  n  2an
2
AP = ⋅ 12 =8 cm 1 1 π π π
3 2 −1< < ⇒ tg < tg < tg ⇒ n =
7
2 3 8 n 6
Como P é ponto médio de BC, é imediato, pelo Teorema de
08. E
= AC
Pitágoras aplicado no triângulo APC, que AB = 10 cm .
Portanto, sendo M o pé da mediana relativa ao lado AC, tem-se Podemos considerar que os vértices de p1 são os pontos de
tangência de p2 com a circunferência.
1
(
2 2
BM = ⋅ 2 ⋅ AB + BC − AC2
2
) l 2/2

1
( )
= ⋅ 2 ⋅ 102 + 122 − 102
π/8
2 l 2/2
= 122 − 25
= 97cm
l1
π/8
II. Falsa. De fato, sendo G o baricentro do triângulo ABC, temos

2 2
AG = ⋅ AP = ⋅ 12 =8cm 2
3 3 π l1 / 2 π l  A
cos = ⇔ cos2 = 1  = 1
8 l2 / 2 8  l2  A2
III. Falsa. Sabendo que BM = 97 cm , vem π 2
2 2 97 cos +1 +1
BG = ⋅ BM = cm . Assim, do triângulo BGP, obtemos A1
⇔ = 4 = 2 = 2+ 2
3 3 A2 2 2 4
BP 6 9
cos =
α = =
BG 2 97 97 09. C
3

06. B
Pelo Teorema de Menelaus, aplicado no triângulo ABC,vem

MA DB EC 5 16 EC
⋅ ⋅ =⇔
1 ⋅ ⋅ =1
MB DC EA 5 6 EA
EC 3
⇔ =
EA 8

Agora, como AB = AC = AE + EC = 10, pela propriedade das


proporções, temos

EC 3 EC + EA 3 + 8
= ⇔ = BC2 = AB2 + AC2 ⇒ ∆ABC é retângulo em A
EA 8 EA 3
80 ⇒ x + θ + y = 90o
⇔ EA =.
11 DE = BC - BD -EC = 29 - 8 - 9 = 12
BE = BA = 20 ⇒ ∆ABE é isósceles de vértice B

40 PROMILITARES.COM.BR
GEOMETRIA PLANA I

⇒ BÊA = BÂE = θ + x AF DE
t  BC ⇒ ∆ADE ~ ∆ABC ⇒ = ⇔
CD = CA = 21 ⇒ ∆ACD é isósceles de vértice C AH BC
ˆ = CAD
⇒ CDA ˆ = θ+y 3
4 − 2⋅
⇔ 2 =DE ⇔ DE =3 =1,5 cm
θ + ( θ + y ) + ( θ +=
x ) 180 ⇒ 2θ + ( θ + x +=
y ) 180 4 6 2
∆ACE:
90 180 ⇔
⇒ 2θ + = = θ 45
13. C
10. E

Teorema de Pitágoras:
Seja M o ponto médio de HC.
2 2 21 1
a = b + c = k + + k − = 2k ⇔
k k RH ≡ MH
2
a> 0 ˆ ≡ MHP
RHA ˆ ⇔RHA ≡MHP (L.A.L ) ⇒ ARH
ˆ = PMH
ˆ = 90°.
⇔ a = 2k ⇔ a = 2k
AH ≡ PH
Usando a relação a ⋅ h = bc, temos:
PM é a mediana relativa à hipotenusa do triângulo HPC, logo
1 1 2 1 PM ≡ MC ≡ MH ≡ HR.
k+ . k− k − 2
b⋅c k k k k4 − 1
=h = = =
a 2k 2k 2k 3 PM ≡ MH
⇒ MHP ˆ = 45° ⇒ ABC
ˆ = 45° ⇒ MCP ˆ = 45°.
ˆ = 90°
PMH
11. D
ˆ = BAP
AHC ˆ + ABP ˆ = 45° + 45° + 45=
ˆ + BCR ° 135°
MN BM
∆BMN ~ ∆BDA ⇒ =
DA BD
ˆ
ˆ = AHC .
Assim ABC
MP MC 3
∆CDA ~ ∆CMP ⇒ =
DA DC
14. B
MN + MP BM CM BM + MC BC
⇒ = + = = =2
DA BD DC BD BD

⇒ MN + MP = 2AD

12. B

Suponhamos sem perder generalidade que AB é o lado do


quadrado inscrito na circunferência, que BC é o lado do triângulo
inscrito na circunferência e que CD é o lado do hexágono inscrito na
circunferência.
Logo,
 + BC
AB  + CD
= 90° + 120° + 60°= 270° ⇒ AD
= 90° .
Daí, o perímetro do quadrilátero formado é
AB = AC = 42 + 32 = 5 ⇒ 2p = 5 + 5 + 6 = 16 ⇒ p = 8 2l4 + l3 + l6= 2 2 + 3 + 1 .

a⋅h 6⋅4 3
SABC = p ⋅ r = ⇔ 8 ⋅r = ⇔r=
2 2 2

PROMILITARES.COM.BR 41
GEOMETRIA PLANA I

15. B Utilizando semelhança de triângulos na figura acima temos:


CP CB e CP AC
= =
AM AB BN AB
CP CP AC + CB
Somando temos: += = 1
AM BN AB
1 1 1
Daí, + =
AM BN CP

18.

O lado do triângulo equilátero é =


L3 R =
3 3 ≈ 1,7 ,
o lado do quadrado é =
L4 R =
2 2 ≈ 1, 4 e o lado do pentágono

regular 10 − 2 5
é L5 R= 10 − 2 5
=  1,2 .
2 2
O lado AD de medida x é uma corda que determina um arco de
78o.
Mas, 72o < 78o < 90o ⇒ L5 < x < L4 ⇒ 1,2 < x < 1,4.
Observe que não era necessário conhecer a expressão do lado
do triângulo equilátero, pois não foi usada, e nem a do pentágono
regular, pois sua medida foi dada. O importante nesse problema era
identificar o ângulo central. a) Aplicando a lei dos senos no ∆AOB , temos:
senOABˆ ˆ
sen AOB
= ⇔
16. OB AB
A B ˆ
senOAB sen135° 22
⇔ = = ⇔
2− 3 2 2

ˆ =2− 3
⇔ senOAB
M 2
ˆ < 180° − 135°= 45° ⇒ OAB
0 < OAB ˆ = 15°
H
θ b) OBA ˆ − OAB
ˆ= 180° − AOB ˆ= 180° − 135° − 15=
° 30°
D N C ˆ = OBC
OCB ˆ = OBA ˆ = 120°
ˆ = 30° ⇒ BOC
ˆ = AOB
⇒ AOC ˆ − BOC
ˆ = 135° − 120°= 15°
Aplicando o Teorema de Pitágoras no ∆ABD encontramos que BD = 5
BH AB BH 4 16 ⇒ OAC ˆ = 15° ⇒ AC= OC= OB=
ˆ = AOC 2 − 3 cm
∆ABH ~ ∆ABD ⇒ = ⇒ = ⇒ BH =
AB BD 4 5 5
19. C
8 17 Lei dos Senos
Portanto: DM = BD – BM = 5 − = . No ∆ABD: cos θ = 4/5.
Lei dos Cossenos em ∆DMN: 5 5
AC BC AB
= = = 2R
MN² = DM² + DN² – 2 ∙ DM ∙ DN ∙ cos θ ⇒ senBˆ sen Aˆ senCˆ
BC ˆ AC , senC
ˆ AB
289 17 4 3 13 ⇒ sen=Aˆ =
, senB =
2
MN= +4−2 . . 2 . ⇒ MN= 2R 2R 2R
25 5 5 5
sen2 Aˆ + sen2 Bˆ − sen2 Cˆ − 2sen Aˆ senBcos
ˆ Cˆ = cos2 Cˆ ⇔
17.
sen A + sen B − 2sen Aˆ senBcos
2 ˆ 2 ˆ ˆ Cˆ =sen Cˆ + cos2 Cˆ =
2
1⇔
N 2 2
 BC   AC  BC AC ˆ
 2R  +  2R  − 2 ⋅ 2R ⋅ 2R ⋅ cos C = 1 ⇔
M    

(BC) + ( AC)
2 2
− 2 ⋅ BC ⋅ AC ⋅ cos Cˆ =4R2
P

A expressão do lado esquerdo é a Lei dos Cossenos, então:

( AB)
2
= 4R2 ⇔ AB = 2R
A C B

42 PROMILITARES.COM.BR
 ab   ac 
a⋅  a⋅ 
b + c  b + c 
GEOMETRIA PLANA I
( )
2
b2 bc c2 a 2
− + =1 → b − c =
a ⋅ a2 ( a2) a ⋅ a2
2
2

Como AB = 2R , o triângulo ABC é retângulo de hipotenusa AB . 3a 2


b + c =a 2 b=
OC' AB A'C  x 2 2y A'C 4
AC ⋅ ⋅ 13. = 1 → ⋅ → ⋅ = 1
AC + AB  = 3 −k OC AC' A'B b − c =2a 2x + y A'Ba 2
BC =    2 c=
2  ⇒  =BC 3 A'C x + y 3 +1 A'B 3 ⋅ 24
3 3 
AC + AB + BC =  = = → =
= 3 +k
AB A'B yx 2 3⋅ 2 A'C 3 +1
A'B 6 14. 3 2 − 6
= =
( ) ( ) ( )
2 2 2
⇒ 3 −k + 3 = 3 +k ⇔ A'C 3 +1 2

3 AB 
⇔ 2 3 ⋅ 2k = 3 ⇔ k = =
15. a) Como o triângulo ABC é retângulo em C, temos CM = .
4 2 2

3 3 5 3  = 90° − ACH
b) BAC  e ABC 
 = 90° − BAC

= AC = , BC =
3, AB
4 4 = 90° − 22°30' = 90° − 67°30'
= 67°30' = 22°30'.
Logo, a área do triângulo é

1 1 3 3 9 16. a) Como G é o baricentro temos GM = 2r, já que AG = 4r;


SABC = ⋅ AC ⋅ BC = ⋅ ⋅ 3 = m2
2 2 4 8 1
Logo, 6r = 3 ⇔ r =
2
20.
(2 3 ) .
2
3  1
2
b) A = − 4 π.  = (3 3 − π)sm
2

4  2
 1
c) 2r = 2⋅   = 1cm  1+ 2 − 2  2− 2
L = (R +12) ⋅2sen22,5 =  ⋅2
 2− 2  2
 
CBG ˆ = α  17.=+
ˆ = BOC 1 2− 2 BC CG
 ⇒ ∆BCG ≡ ∆OBC ⇒ =
ˆ = BCO
BCG ˆ  OC BC

r r 2
⇔ BC = OC ⋅ CG18.
⇔L= r⋅ =
2 2

Aplicando a lei dos cossenos nos triângulos XPY, YPZ e ZPX, temos:
EF FC 1 1 a
AB ⇒ BMAB AM= = 19.
BC⇔ BM
⇒ EF = AB =
3 ⋅ AC = 3AM ⋅ AB3 ⇔
9 = a2 + b2 − 2ab cos120 = =
 BC AB AC
2 2 
25 = b + c − 2bc cos120 BC
 2 2  ⇔ ⋅ AC =AM ⋅ AB20.
⇔ BC ⋅ AC = 2 ⋅ AM ⋅ AB
16 = a + c − 2ac cos120 2
 1
⇒ 50 = 2 ( a2 + b2 + c2 ) − 2 ( ab + ac + bc ) ⋅  − 
 2 ANOTAÇÕES
1
2 2 2 (
⇒ a + b + c = 25 − ab + ac + bc )
2

SXYZ = SXPY + SYPZ + SZPX


3 ⋅ 4 ab bc ac
⇒= sen120 + sen120 + sen120
2 2 2 2
1 3
⇔6 = ( ab + ac + bc ) ⋅ ⇔ ab + ac + bc= 8 3
2 2

1
⇒ a2 + b2 + c2 = 25 − ⋅ 8 3 = 25 − 4 3
2

EXERCÍCIOS DE COMBATE
01. A 06. A
02. A 07. E
03. E 08. D
04. D 09. A
05. D 10. E

11. EQ =
(b + c − a) ⋅ ( a + c − b ) e DR = c - b.
2 ⋅ (a + b + c )

12. A menor distância possível entre os pontos M e N é 10 2.

PROMILITARES.COM.BR 43
GEOMETRIA PLANA I

ANOTAÇÕES

44 PROMILITARES.COM.BR
GEOMETRIA PLANA II

ÁREAS DAS FIGURAS PLANAS TRIÂNGULO


Área de uma figura plana é a porção do plano ocupado pela B
figura. B’
Duas figuras planas são equivalentes quando possuem a mesma
área, embora possam ter formas inteiramente diferentes.

RETÂNGULO
h

h
A C
b
1

1 b SABB'C b ⋅h
SABC = ⇒ SABC =
2 2
S = (b x h)
Observação
Sretângulo ⇒ S= b ⋅ h
A área de um triângulo permanece constante quando um dos seus
vértices se desloca paralelamente ao lado oposto.
QUADRADO
h = b =  ⇒ Squadrado = 2 LOSANGO

PARALELOGRAMO SABCD =  ⋅ h

d
A

= SABD + SBCD
SABCD
BD ⋅ AO BD ⋅ OC
h =
SABCD +
d D B 2 2
0 BD ⋅ AC
SABCD = ou
2
 h

C
D A’ t C
dd'
A ABCD =
2

SABCD = SABB’A’ ⇒ SABCD= b ⋅ h

PROMILITARES.COM.BR 45
GEOMETRIA PLANA II

TRAPÉZIO α ⋅ R2
α → graus ⇒ S =
b 2
A B α → radianos
Podemos obter outra fórmula em função do comprimento do
arco.
360° → 2 π R
h α→
α
= ⋅πR
180°
Logo:
D C
b’
α π R2  α  R R
= SABC + SACD
SABCD
S= =  ⋅ π ⋅ R=
⋅
360°  180°  2 2
b ⋅ h b'⋅ h
=
SABCD + R
2 2 S=
2
b + b' A área de um setor circular é então equivalente à área de um
S=
ABCD ⋅h
2 triângulo cuja base é igual ao comprimento do arco do setor e cuja
altura é igual ao raio do círculo.

Observação
A área do trapézio permanece constante quando deslocamos
COROA CIRCULAR
qualquer das bases sobre seus suportes.

POLÍGONOS REGULARES
B
 S=b ⋅ SAOD =
n⋅
a⋅
2
SC = π R2 − π R2 = π R2 − r2 ( ) R
C R
 =
S
n⋅
2
⋅a (
π R2 − r2
S= ) d
R
α
α
A ou
α
S= p ⋅ a
 2π R + 2π r
S=π (R + r )(R − r ) π (R + r=
)d ⇒ S ⋅d
2
2π r
d
2π r
CÍRCULO
SC = lim SP
n→∞ A área da coroa circular é então equivalente à área de um trapézio
cujas bases são iguais aos comprimentos das duas circunferências e
No limite: a → R cuja altura é igual à distância entre os círculos.
p→πR
SC = π ⋅ R2 Setor de coroa circular ou trapézio circular
2
360° → π R − r
2
( )
SETOR CIRCULAR α → S
É a porção do círculo compreendida entre dois raios.
S=
(
α ⋅ π R2 − r2 )
360°
ou
360° → R
2
 α 2π R + 2π r
R α → S S= ⋅ ⋅d
360° 2

0 α  ou ou
α α
R ⋅πR + ⋅π r
α ⋅ π R2 =S 180 ° 180 ° ⋅d
S= 2
360°
ou
 +L
=S ⋅d
2

46 PROMILITARES.COM.BR
GEOMETRIA PLANA II

A área de um triângulo é igual ao produto de seu semiperímetro


pelo raio do círculo inscrito nesse triângulo.

0 α  L (S) d

L
d

A área de um setor de coroa circular é então equivalente à área


de um trapézio cujas bases são iguais aos comprimentos dos arcos do
setor da coroa circular e cuja altura é igual à distância entre os círculos SABC= p ⋅ r
da coroa circular.
Demonstração:
TRIÂNGULO SABC = SABI + SACI + SBCI =
As fórmulas já obtidas para o triângulo podem ser relacionadas
c ⋅r b ⋅r a⋅r  a + b + c 
das seguinte formas: = + + =   ⋅r = p ⋅r
2 2 2  2 
a ⋅ ha b ⋅ hb c ⋅ hc
=S = = =
2 2 2
abc A área de um triângulo é igual ao produto da diferença entre o
= p(p − a)(p − b)(p − c)=
4R semiperímetro e um dos seus lados pelo raio do círculo ex-inscrito
= p ⋅ r = (p − a) ⋅ ra = (p − b) ⋅ rb = (p − c) ⋅ rc relativo a esse lado.

Vamos demonstrar os formatos acima e outros muito interessantes.


A área de um triângulo é igual à metade do produto de dois lados
adjacentes multiplicado pelo seno do ângulo entre eles.

SABC = (p − a) ⋅ ra = (p − b ) ⋅ rb = (p − c ) ⋅ rc

Demonstração:
Na figura AD = p – a e AT = p.
b⋅c a⋅c a⋅b
=
SABC = sen Aˆ = senBˆ senCˆ ID AD
2 2 2 ∆ADI ~ ∆ATIA ⇒ =
IA T AT
r p−a
Demonstração: ⇒ = ⇔ p ⋅r = (p − a) ⋅ ra
ra p
⇒ SABC = (p − a) ⋅ ra

A área de um triângulo é igual aos produtos dos três lados dividido


pelo quádruplo do raio do círculo circunscrito a esse triângulo.

AC ⋅ BE
Seja BE a altura relativa ao lado AC do ∆ABC, então SABC =
. 2

BE
No triângulo retângulo ABE, temos sen Aˆ = ⇔ BE = AB ⋅ sen Aˆ .
AB
a⋅b ⋅ c
AC ⋅ BE AC b⋅c SABC =
Logo, SABC = =⋅ ABsen Aˆ =sen Aˆ . 4R
2 2 2

PROMILITARES.COM.BR 47
GEOMETRIA PLANA II

Demonstração:

SABC = p ⋅ (p − a) ⋅ (p − b ) ⋅ (p − c )

a+b+c
onde p = .
2
ˆ = 90° .
AOF é um diâmetro do círculo circunscrito, então ACF Demonstração:

ABC ˆ =AC ⇒ BAD
ˆ =AFC ˆ =CAF
ˆ
2
AB AD
∆BDA ~ ∆FCA ⇒ = ⇒ b ⋅ c = hA ⋅ 2R
AF AC
a⋅h a bc abc
SABC = A = ⋅ =
2 2 2R 4R

A área de um triângulo é igual ao dobro do quadrado do raio


do círculo circunscrito multiplicado pelo produto dos senos de seus
ângulos.

Aplicando o teorema de Pitágoras nos ∆ABD e ∆ACD , temos:


h2A + x 2 =
c2
2
h2A + ( a − x ) =
b2
2
x 2 − ( a − x ) = c2 − b2 ⇔ a ⋅ ( 2x − a) = c2 − b2
a − b2 + c2
2
⇔x=
2a

⇒ h2A = c2 − x 2 = ( c + x ) ( c − x ) =
 a2 − b2 + c2   a2 − b2 + c2 
= c + c − =
 2a  2a 
1 ( 2 )(
= 2ac + a + c − b 2ac − a − c2 +=
2 2 2
b2 )
SABC =2R2 ⋅ sen Aˆ ⋅ senBˆ ⋅ senCˆ
4a2
1 (
 a + c ) − b  b − ( a − c=
2 2 2 2 )
= 
Demonstração: 4a2
Aplicando a lei dos senos ao ∆ABC , temos: 1 (
= a + c + b ) ( a + c − b ) (b + a − c ) (b + c −=a)
a b c 4a2
= = = 2R 1
sen Aˆ senBˆ senCˆ = ⋅ 2p ⋅ ( 2p − 2b )( 2p − 2c )( 2p − 2a) =
4a2
a ⋅ b ⋅ c 2R sen Aˆ ⋅ 2R senBˆ ⋅ 2R senCˆ 4
⇒ SABC = = = = 2 ⋅ p ⋅ (p − a) ⋅ (p − b ) ⋅ (p − c )
4R 4R a
=2R2 ⋅ sen Aˆ ⋅ senBˆ ⋅ senCˆ
2
⇒ hA = ⋅ p ⋅ (p − a) ⋅ (p − b ) ⋅ (p − c )
Fórmula de Heron: A área de um triângulo é igual à raiz quadrada a
do produto do semiperímetro pela diferença entre o semiperímetro e a ⋅ hA
⇒ SABC = = p ⋅ (p − a) ⋅ (p − b ) ⋅ (p − c )
cada um dos lados do triângulo. 2

48 PROMILITARES.COM.BR
GEOMETRIA PLANA II

A área de um triângulo é igual à raiz quadrada do produto do raio


do círculo inscrito e dos três raios dos círculos ex-inscritos ao triângulo.

= BT ⋅ TC
SABC

Demonstração:
BT =−
p b; CT =−
p c; r =−
p a; SABC =⋅
p r
SABC = p ⋅ (p − a) ⋅ (p − b ) ⋅ (p − c )
⇒ S2ABC =p ⋅ (p − a) ⋅ (p − b ) ⋅ (p − c ) =
=p ⋅ r ⋅ BT ⋅ CT =SABC ⋅ BT ⋅ CT
⇔ SABC =BT ⋅ CT

Figuras equivalentes são aquelas que possuem a mesma área.


Se dois triângulos possuem bases e alturas congruentes, então
eles são equivalentes.
SABC = r ⋅ ra ⋅ rb ⋅ rc

Demonstração:
SABC = p ⋅ r = (p − a) ⋅ ra = (p − b ) ⋅ rb = (p − c ) ⋅ rc

⇒ S4ABC= p (p − a)(p − b )(p − c ) ⋅ r ⋅ ra ⋅ rb ⋅ r=


c

= S2ABC ⋅ r ⋅ ra ⋅ rb ⋅ rc
⇒ S2ABC = r ⋅ ra ⋅ rb ⋅ rc

⇒ SABC = r ⋅ ra ⋅ rb ⋅ rc

A área de um triângulo é igual à raiz quadrada da metade do


produto do raio do círculo circunscrito por cada uma das três alturas
do triângulo.

SABC = SA'BC

Se dois triângulos são semelhantes, então a razão entre suas áreas


é o quadrado da razão de semelhança.

R ⋅ hA ⋅ hB ⋅ hC
SABC =
2

Teorema de Burlet: Em um triângulo retângulo, a área é igual SABC


∆ABC ~ ∆A'B'C' ⇒ k2
=
ao produto dos segmentos determinados pelo círculo inscrito sobre a SA'B'C'
hipotenusa.
onde k é a razão de semelhança.

PROMILITARES.COM.BR 49
GEOMETRIA PLANA II

a h Se dois triângulos possuem base comum e o vértice de um deles


∆ABC ~ ∆A'B'C' ⇒ = =k pertence a uma ceviana do outro partindo do vértice oposto à base
a' h'
a⋅h comum, então a razão entre a área do maior e do menor deles é igual
SABC a h à razão entre a medida da ceviana e a medida da parte entre o vértice
⇒ = 2 = . =k 2 do menor e a base comum.
SA'B'C' a'⋅ h' a' h'
2
Note que essa propriedade vale para quaisquer figuras
semelhantes, não só para triângulos.
Se dois triângulos possuem bases sobre a mesma reta e vértice
comum, então a razão entre suas áreas é igual à razão entre suas
bases.

SABC AE
=
SBCD DE

Demonstração:
SABE AE S AE
= ∧ ACE =
SABC a SBDE DE SCDE DE
=
SAB'C' a' S + SACE AE
⇒ ABE =
SBDE + SCDE DE
Demonstração: S AE
⇒ ABC =
Seja h a distância do ponto A à reta r, então h é altura do ∆ABC SBCD DE
a⋅h
SABC 2 a Uma mediana divide o triângulo em duas regiões equivalentes.
e do ∆AB'C' . Assim, = = .
SAB'C' a'⋅ h a'
2
Uma consequência imediata da proposição anterior é que a razão
entre as áreas em que uma ceviana divide um triângulo é igual à razão
entre as medidas dos segmentos em que essa ceviana divide o lado.

Seja AM a mediana relativa ao lado BC do ∆ABC .

SABD BD SABD BD SACD CD SABC


= ; = ; = S=
ABM S=
ACM
SACD CD SABC BC SABC BC 2

SABD SACD SABC As três medianas de um triângulo dividem esse triângulo em seis
= =
BD CD BC triângulos equivalentes.

50 PROMILITARES.COM.BR
GEOMETRIA PLANA II

EXERCÍCIOS DE

TREINAMENTO
01. Seja P um ponto interior do triângulo ABC e trace retas a partir
dos vértices passando por P até os lados opostos. Sejam a, b, c e d as
medidas dos segmentos indicados na figura. Se a + b + c =43 e d = 3 ,
o valor do produto a · b · c é

Sejam AM, BM e CP as medianas do ∆ABC, então


SABC
= S=
SAGN AGP = S=
SBGM BGP = SCGN
SCGM =
6

TEOREMA
A razão entre as áreas de duas figuras semelhantes é igual ao
quadrado da razão de semelhança das figuras. a) 129 d) 441
A b) 258 e) 516
c) 387
A’
02. (IME 1995) Três círculos de mesmo raio R se interceptam dois
h
h’
a dois, como é mostrado na figura abaixo, constituindo três áreas
comuns que formam um trevo.
B’ C’ B C
b’ b

1° caso) Sejam dois triângulos semelhantes:


b' h'
= = k
b h
b'h'
S' 2 S' b' h'
= ou = ×
S bh S b h
2
S'
= k2
S

2° caso) Sejam dois polígonos semelhantes:


A'B' B'C' O perímetro do trevo e sua área em função de R e da área S do
= = ...= k
AB BC triângulo IJK são dados por:

A a) π ⋅ R e π ⋅ R2 − S d) 2π ⋅ R e π ⋅ R2 − S

b) π ⋅ R e 2π ⋅ R2 − S e) 2π ⋅ R e π ⋅ R2 − 2S
A’
c) π ⋅ R e π ⋅ R2 − 2S

03. (CN 1976) Sobre os lados de um hexágono regular de 4 cm de


S1 B
lado, e exteriormente a ele, constroem-se quadrados, de modo que
S’1 B’ S2 cada quadrado tenha um lado em comum com o hexágono. Calcular
S’2 a área do dodecágono cujos vértices são os vértices dos quadrados
que não são vértices do hexágono:
D’ C’ D C

S'1= S1 ⋅ k 2
a) 48 ( )
3 + 2 cm2 d) 192 cm²
S'
+  = k2 b) 50 ( 3 + 2) cm2 e) 36 cm²
S'=
2 S2 ⋅ k 2 S
s’ = Sk2
c) 24 ( )
3 + 4 cm2

PROMILITARES.COM.BR 51
GEOMETRIA PLANA II

04. (IME 2005) Considere um triangulo ABC de área S. Marca-se o a) 2 d) 2

ponto P sobre o lado AC tal que PA = q e o ponto Q sobre o lado b) 3 e) 3


PC
QB 3
BC de maneira que = r . As cevianas AQ e BP encontram-se em T, c)
QC 2
conforme ilustrado na figura. A área do triangulo ATP é. 08. Supondo que o segmento de reta AB tem comprimento 3 e C
está sobre AB com AC = 2. Os triângulos equiláteros acf e CBE são
construídos do mesmo lado de AB. Se K é o ponto médio de FC,
determine a área do triângulo AKE.

a) d) 1
3
2

b) 3 e) 1
2 4

c) 3
4
09. Seis círculos de raio 1cm são inseridos no paralelogramo MNPQ,
Sq2 Sq de área X cm2 , de acordo com a figura abaixo.
a) d)
( q + 1)( q + r + qr ) ( q + 1)2 ( q + r + qr )
Sq Sq2
b) e)
( q + 1)( q + r + qr ) ( q + 1)( q + r − qr )
S ( q + 1)
2
c)
q ( q + r − qr )

05. (ITA-94) Sejam a, b e c as medidas dos lados de um triângulo


e A, B e C os ângulos internos opostos, respectivamente, a cada
um destes lados. Sabe-se que a, b, c nesta ordem, formam uma
progressão aritmética. Se o perímetro do triângulo mede 15 cm e
cos A cos B cos C 77 então sua área, em cm², mede:
+ + = , Sabendo-se que os seis círculos são tangentes entre si e com os lados
a b c 240
do paralelogramo, a área X, em cm2 , é
a) (15 7 )/4 c) (4 5 )/5 e) (3 5 )/4
a) 11+ 6 3. d) 11− 6 3.
b) (4 5 )/3 d) (4 7 )/7

b) 30 + 14 3 e) 36 + 20 3
06. (CN 2003) Em um trapézio, cujas bases medem a e b, os ponto . .
3 3
M e N pertencem aos lados não paralelos. Se MN divide esse trapézio
em dois outros trapézios equivalentes, então a medida do segmento
c) 10 + 5 3.
MN corresponde a:
10. O quadrilátero da figura está inscrito em uma circunferência de
a) média aritmética de a e b. raio 1. A diagonal desenhada é um diâmetro dessa circunferência.
b) média geométrica das bases.
c) raiz quadrada da média aritmética de a² e b².
d) raiz quadrada da média harmônica de a² e b².
e) média harmônica de a e b

07. (CN 2002) As diagonais AC, BD, CE, DF, EA, e FB de um hexágono
regular ABCDEF interceptam-se formando outro hexágono
A'B'C'D'E'F' conforme a figura abaixo. Qual a razão entre as áreas
do maior e a do menor hexágono?

Sendo x e y as medidas dos ângulos indicados na figura, a área da


região cinza, em função de x e y, é:
a) π + sen (2x) + sen (2y) cos (2x) + cos (2y)
d) π −
2
b) π − sen (2x) − sen (2y)
sen (2x) + sen (2y)
e) π −
c) π − cos (2x) − cos (2y) 2

52 PROMILITARES.COM.BR
GEOMETRIA PLANA II

11. A figura representa uma semicircunferência de diâmetro CD, A área da parte do círculo não encoberta pelas dobras, sombreada na
perfeitamente inscrita no retângulo ABCD. Sabe-se que P é um ponto figura 2, é igual a
de AB, e que AP é diâmetro da circunferência que tangencia a
semicircunferência maior em T. a)
1
3
( 96 − 16π ) cm2 d)
1
3
( )
16π + 12 3 cm2

b) 1
3
(16π − 48 ) cm2 e)
1
3
( )
48 3 − 16π cm2

c)
1
3
( )
16π − 12 3 cm2

14. (CN 2002) Observe a figura abaixo que representa três


semicircunferências de centros M, N e P, tangentes duas a duas,
respectivamente, nos pontos A, B e C. Os segmentos MM’, NN’, BB’ e
PP’são perpendiculares à reta r. Se a medida do segmento BB’ é 6 cm,
a área do triângulo M’ N’ P’ em cm², é igual a:

Se CD = 8 cm, a área sombreada na figura é, em cm², igual a

64 − 15π
a) d) 32 − 9π
2

b) 32 − 8π e) 16 − 4 π

c) 64 − 15π
4
12. Na figura, o retângulo ABCD tem lados de comprimento AB = 4
e BC = 2. Sejam M o ponto médio do lado BC e N o ponto médio do
lado CD. Os segmentos AM e AC interceptam o segmento BN nos
a) 9 d) 18
pontos E e F, respectivamente.
b) 10 e) 36
c) 12

15. (CN 2002) Na figura abaixo, o ponto P do menor arco AB dista


6 cm e 10 cm , respectivamente, das tangentes AQ e BQ. A distância,
em cm, do ponto P à corda AB é igual a:

A área do triângulo AEF é igual a


24 61 23
a) c) e)
25 60 20
29 16
b) d)
30 15

13. (EFOMM 2010) João construiu um círculo de papel com centro O


e raio 4cm (Figura 1). Traçou dois diâmetros AC e BD perpendiculares
e, em seguida, dobrou o papel fazendo coincidir A, O e C, conforme
sugere Figura 2. a) 30

b) 2 15
c) 16
d) 18

e) 6 10

16. (OBM Jr-95) ABCD é um quadrilátero convexo; M e N são pontos


médios dos lados AB e CD. Prove que se o segmento MN divide o
quadrilátero em dois de mesma área, então ABCD é um trapézio.

PROMILITARES.COM.BR 53
GEOMETRIA PLANA II

17. (IME 2002) Sobre uma reta r são marcados os pontos A, B, C e D. 02. (ITA 2011) Sejam ABCD um quadrado e E um ponto sobre AB .
São construídos os triângulos equiláteros ABE, BCF e CDG, de forma Considere as áreas do quadrado ABCD, do trapézio BEDC e do
que os pontos E e G encontram-se do mesmo lado da reta r, enquanto triângulo ADE. Sabendo que estas áreas definem, na ordem em que
que o ponto F se encontra do lado oposto, conforme mostra a figura. estão apresentadas, uma progressão aritmética cuja soma é 200 cm²,
Calcule a área do triângulo formado pelos baricentros de ABE, BCF
a medida do segmento AE , em cm, é igual a
e CDG, em função dos comprimentos dos segmentos AB, BC e CD.
10 25
E a)
3
d)
3
G b) 5 e) 10

c) 20
3
A D 03. (ITA 2005) Considere o triângulo de vértices A, B e C, sendo D
B C um ponto do lado AB e E um ponto do lado AC. Se m(AB) = 8 cm,
m(AC) = 10 cm, m(AD) = 4 cm e m(AE) = 6 cm, a razão das áreas dos
F triângulos ADE e ABC é
a) 1/2. c) 3/8. e) 3/4.
18. (IME 2003) Considere um hexágono regular de 6 cm de lado. b) 3/5. d) 3/10.
Determine o valor máximo da área de um triângulo XYZ, sabendo-se
que:
04. (ITA 2003) Sejam r e s duas retas paralelas distando entre si
a) os pontos X,Y e Z estão situados sobre lados do hexágono; 5 cm. Seja P um ponto na região interior a estas retas, distando 4 cm
b) a reta que une os pontos X e Y é paralela a um dos lados do de r. A área do triângulo equilátero PQR, cujos vértices Q e R estão,
hexágono. respectivamente, sobre as retas r e s, é igual, em cm2, a:
 15 
a) 3 15 d)   3
19. No triângulo ABC a circunferência inscrita divide a mediana AM  2
em três partes iguais. Determine os lados desse triângulo sabendo que b) 7 3
7
sua área é igual a 6 14 . c) 5 6 e)   15
 2

20. (IME-02) Considere um quadrado XYZW de lado a. Dividindo- 05. (CN 2011) Em um triângulo acutângulo não equilátero, os
se cada ângulo desse quadrado em quatro partes iguais, obtém-se o três pontos notáveis (ortocentro, circuncentro e baricentro) estão
octógono regular representado na figura abaixo. alinhados. Dado que a distância entre o ortocentro e o circuncentro
é ‘k’, pode-se concluir que a distância entre o circuncentro e o
X Y baricentro será

a) 5k c) 4k e) k
2 5 3
A
b) 4k d) k
H B 3 2

G C 06. (ITA 2009) Do triângulo de vértices A, B e C, inscrito em uma


circunferência de raio R = 2 cm, sabe-se que o lado BC mede 2 cm e o
ângulo interno ABC mede 30°. Então, o raio da circunferência inscrita
F D neste triângulo tem o comprimento, em cm, igual a

a) 2− 3 . c) 2 3 − 3 ..
E

b) 1. d) 1.
W 3 2
Z
Determine o lado e a área desse octógono em função de a. As c) 2.
respostas finais não podem conter expressões trigonométricas. 4

EXERCÍCIOS DE 07. Na figura, o valor de x é

COMBATE
01. (ITA 2011) Um triângulo ABC está inscrito numa circunferência de
raio 5 cm. Sabe-se ainda que AB é o diâmetro, BC mede 6 cm e a
 intercepta a circunferência no ponto D. Se α
bissetriz do ângulo ABC
e a soma das áreas dos triângulos ABC e ABD e β é a área comum aos
dois, o valor de α – 2β , em cm², é igual a
a) 14. d) 17. a) 22 d) 4
b) 15. e) 18. b) 16 e) a figura é incompatível
c) 16. c) 7

54 PROMILITARES.COM.BR
GEOMETRIA PLANA II

08. (IME 2011) Seja o triângulo retângulo ABC com os catetos 13. (ITA 2012) As retas r1 e r2 são concorrentes no ponto P, exterior a
medindo 3 cm e 4 cm. Os diâmetros dos três semicírculos, traçados na
um círculo ω . A reta r1 tangencia ω no ponto A e a reta r2 intercepta
figura abaixo, coincidem com os lados do triângulo ABC. A soma das
áreas hachuradas, em cm² é: ω nos pontos B e C diametralmente opostos. A medida do arco AC 

é 60º e PA mede 2 cm . Determine a área do setor menor de ω


.
definido pelo arco AB

14. (IME 89/90) Os lados de um triângulo estão em progressão


aritmética e o lado intermediário mede l. Sabendo-se que o maior
ângulo excede o menor em 90º, calcule a razão entre os lados.

15. (IME 1989) Numa circunferência de centro O e de diâmetro


AB = 2R, prolonga-se o diâmetro AB até um ponto M, tal que BM = R.
Traça-se uma secante MNS tal que MN = NS, onde N e S são os pontos
de interseção da secante com a circunferência. Determine a área do
a) 6 c) 10 e) 14 triângulo MOS.
b) 8 d) 12
16. Seja ABC um triângulo de lados medindo 8, 10, e 12. Sejam M, N e
09. (IME 2015) Seja um trapézio retângulo de bases a e b com P os pés das alturas traçadas dos vértices sobre os lados desses triângulos.
diagonais perpendiculares. Determine a área do trapézio. Sendo assim, o raio do círculo circunscrito ao triângulo MNP é

ab a+b 17. (ITA 2016) Um hexágono convexo regular H e um triângulo


a) c) a+b 2
2   ab e)  a b equilátero T estão inscritos em circunferência de raios RH e R T ,
 2   2  respectivamente. Sabendo-se que H e T têm mesma área, determine
2
b) a+b
d)  2a + b  a razão RH .
    ab RT
 2   2 

18. (ITA 2008) Um triângulo acutângulo de vértices A, B e C está


10. (IME 2010) Seja ABC um triângulo de lados AB, BC e AC iguais a
26, 28 e 18, respectivamente. Considere o círculo de centro O inscrito inscrito numa circunferência de raio 5 2 . Sabe-se que AB mede
nesse triângulo. A distância AO vale: 3
2 5 e BC mede 2 2 . Determine a área do triângulo ABC.

a) 104 c) 2 104 e) 3 104 19. Considere os quadrados ABCD e DEFG (E em CD e G no


6 3
prolongamento de AD). Calcular a medida do menor dos ângulos
b) 104 d) formados pela interseção de BF com CG.
104
3

11. Em um triângulo retângulo ABC, o cateto AC e a hipotenusa


BC medem, respectivamente, 10 e 40. Sabe-se que os segmentos
CX, CY e CZ dividem o ângulo ABC em quatro ângulos de medidas
iguais, e que AX, XY, YZ e ZB são segmentos consecutivos contidos
internamente no segmento AB. Se S1, S2, S3, e S4 são, respectivamente,
as áreas dos triângulos CAX,CXY, CYZ e CZB, qual será o valor da
SS
razão 1 3 ?
S2S4

12. (IME 1990) Como mostrado na figura, o triângulo ABC foi dividido
em seis triângulos menores por cevianas que passam por um mesmo
ponto interior ao triângulo. A área de quatro desses triângulos estão 20. (CN 1985) Na figura, o diâmetro AB mede 8 3 cm e a corda CD
indicadas na figura. Calcule a área do triângulo ABC. forma um ângulo de 30° com AB. Se E é o ponto médio de AO, onde
O é o centro do círculo, calcule a área da região sombreada, em cm².

PROMILITARES.COM.BR 55
GEOMETRIA PLANA II

GABARITO Logo, o perímetro do trevo é 2π ⋅ R .


EXERCÍCIOS DE TREINAMENTO
01. D
S (BPC) d S ( CPA ) d S ( APB) d
= ; = ; =
S ( ABC) a + d S ( ABC) b + d S ( ABC) c + d

S (BPC) + S ( CPA ) + S ( APB) = S ( ABC)


(
S BPC ) (
S CPA ) S APB)
(
⇔ + + =1
( )
S ABC S ABC S ( ABC)
( )
d d d
⇒ + + = 1
a+d b+d c +d
⇔ d [(b + d ) ( c + d ) + ( a + d ) ( c + d) + ( a + d) (b + d)] =
= ( a + d ) (b + d ) ( c + d)
⇔ 3d3 + 2 ( a + b + c ) d2 + (bc + ac + ab ) d =
= d3 + ( a + b + c ) d2 + ( ab + ac + bc ) d + abc
Considera as regiões OAB, OCD e OEF, cada uma delas igual a um dos
⇔ 2d3 + ( a + b + c ) d2 − abc =0
segmentos circulares.
3 2
⇒ abc =2 ⋅ 3 + 43 ⋅ 3 =441
A área do trevo é o dobro da soma das regiões OAB, OCD e OEF.
Somando os três setores circulares de ângulos α, β e γ é igual à soma
02. E das regiões OAB, OCD e OEF mais á área do triângulo IJK.
A soma das áreas dos três setores circulares de ângulos α, β e γ é

α ⋅ R2 β ⋅ R2 γ ⋅ R2 R2 π ⋅ R2 .
+ + = (α + β + =
γ)
2 2 2 2 2
2
Logo, π ⋅ R = Strevo + S ⇔ Strevo = π ⋅ R2 − 2S
2 2

03. A

= OJ
Seja O a interseção dos três círculos, então OI = OK = R eOéo
centro do círculo circunscrito ao triângulo também de raio R.
Seja A o outro ponto de interseção dos círculos de centros I e K, então
OK = OI = AK = AI= R , o quadrilátero OKAI é um losango, OA ⊥ IK e IK
divide os arcos que ligam O a A ao meio.
Sendo assim, a parte do trevo interior ao triângulo é idêntica à parte
exterior. Logo, basta calcular o perímetro e a área da parte interior e
depois multiplicar por 2.
A área do dodecágono é a área de uma hexágono de lado 4, mais
a área de 6 quadrados de lado 4 e mais 6 triângulos equiláteros de
lado 4.
A área do hexágono de lado 4 é igual à área de 6 triângulos equiláteros
de lado 4.
42 ⋅ 3 42 ⋅ 3
Sdodecágono = 6 ⋅ + 6 ⋅ 42 + 6 ⋅ = 48 ( 3 + 2) cm2
4 4

04. A

Sejam SCTP = a e SCTQ = b , temos:


SATP PA
= = q ⇒ SATP =
q⋅a
SCTP PC

Na figura acima, os ângulos α, β e γ estão em radianos, então temos: SBTQ QB


= = r ⇒ SBTQ =
r ⋅b
SCTQ QC
med ( AD
 ) + med ( CF
 ) + med (BE
 )= R ⋅ α + R ⋅ β + R ⋅ γ = R ⋅ ( α + β + γ )= π ⋅ R

56 PROMILITARES.COM.BR
GEOMETRIA PLANA II

SAQC QC 1 S 07. E
= = ⇒ SAQC =
SABC BC r + 1 r +1

SBPC CP 1 S
= = ⇒ SBPC =
SABC AC q + 1 q +1
S
SAQC = SATP + SCTP + SCTQ = ( q + 1) ⋅ a + b = ⇒ (r + 1) ( q + 1) ⋅ a + (r + 1) ⋅ b = S (1)
r +1

S
SBPC = SBTQ + SCTQ + SCTP = (r + 1) ⋅ b + a = ( 2)
q +1

Subtraindo (2) de (1), temos:


S q⋅S
( qr + q + r ) ⋅ a = S − ⇔a=
q +1 ( q + 1)( qr + q + r )
q2 ⋅ S
Logo, SATP = q ⋅ a = .
( q + 1)( qr + q + r )
05. Como os lados estão em uma PA de soma 15 : a = 5 – r b = 5 c
=5+r
Lei dos Cossenos: a² = b² + c² – 2 ∙ b∙ c ∙ cos A b² = a² + c² – 2 ∙ a ∙ c ∙ Seja o hexágono ABCDEF inscrito em uma circunferência de raio R,
cos B c² = a² + b² – 2 ∙ a ∙ b ∙ cos C então o lado do hexágono é igual a R.
Deste modo, desenvolvendo a expressão: Os triângulos AA'F' , BA’B’, CB'C' , DC'D' , ED'E' , FE'F' são
cos A cos B cos C b2 + c2 − a2 a2 + c2 − b2 a2 + b2 − c2 a2 + b2 + c2 77 triângulos equiláteros congruentes. Logo, = = C'D .
BB' B'C'
+ + = + + = = ⇒
a b c 2 . a . b . c 2 . a . b . c 2 . a . b . c 2 . a . b . c 240
A corda BD é o lado do triângulo equilátero inscrito na circunferência
120(a² + b² + c²) = 77 ∙ a ∙ b ∙ c ⇒ 120(25 – 10r + r² + 25 + 10r + r²) R 3
+ 77 ∙ 5 ∙ (25 – r²) ⇒ 24 (75 + 2r²) = 77(25 – r²) ⇒ r² = 1 ⇒ r = 1 ⇒ de raio R, então BD = 3x = R 3 ⇔ x = e o lado do hexágono
3
a=4b=5c=6
15  15   15   15  15 7 5 3 15 7 regular A'B'C'D'E'F' é igual a R 3 .
∴=
S p(p − a)(p − b)(p − c)
=  − 4   − 5   − 6=
 ⇒=
S 3
2 2  2  2  2 222 4

06. C Os dois hexágonos regulares são polígonos semelhantes, portanto a


razão entre suas áreas é igual ao quadrado da razão de semelhança.
Assim, temos:
2
SABCDEF  R 
= = 3
SA'B'C'D'E'F'  R 3 3 

08. C

Prolongam-se os lados não paralelos do trapézio até o ponto E.


FC 2
AB  MN  CD ⇒ ∆EAB ~ ∆EMN ~ ∆EDC KC= = = 1
2 2
Como a razão entre as área de figuras semelhantes é o quadrado da
razão de semelhança, temos: = EC
Como KC ˆ = 180 − 60 − 60 = 60 , então o ∆CEK é
= 1 e ECK
SEDC SEMN SEAB ˆ ˆ
= = = k equilátero, KE = 1 e KE  AB (pois EKC
= =
KCA 60 ).
a2 x2 b2
A altura do ∆AKE é igual à distância ente os segmentos KE e AB que é
SABNM = SMNCD ⇔ SEAB − SEMN = SEMN − SEDC
⇔ 2 ⋅ SEMN= SEAB + SEDC igual à altura do triângulo equilátero ∆CEK , ou seja, 1⋅ 3 = 3 .
2 2
a2 + b2
⇒ 2 ⋅ kx 2= kb2 + ka2 ⇔ x= 1⋅
3
2 2 3
Portanto, a área do ∆AKE é dada por =
SAKE = unidades
2 2 de área. 2 4
Logo, MN = x é a raiz quadrada da média aritmética de a e b .

PROMILITARES.COM.BR 57
GEOMETRIA PLANA II

09. E Assim, temos:

Portanto,
Na figura, temos: 6 + 4 3 12 + 4 3
X= ⋅ ⋅ sen 60°
3 3
λ1 ( A, 1)
6 + 4 3 12 + 4 3 3
λ 2 (B, 1) X= ⋅ ⋅
3 3 2
λ 3 ( C, 1)
36 + 20 3 2
X= cm
λ 4 (D, 1) 3

10. B
= AC
ABC é um triângulo equilátero, pois AB = BC
= 2. A diagonal do quadrilátero o divide em dois triângulos retângulos.
Sendo 2 sen x e 2 cos x os catetos do primeiro e 2 sen y e 2 cos y os
T1 é ponto de tangência entre λ1 e MQ, logo, AT1 ⊥ MQ
catetos do segundo, podemos concluir que o resultado é
T2 é ponto de tangência entre λ 2 e MQ, logo, BT2 ⊥ MQ 1 1
π ⋅ 12 − ⋅ 2sen x ⋅ 2cos x − ⋅ 2sen y ⋅ 2cos y =π − sen2x − sen2y.
2 2
T3 é ponto de tangência entre λ 2 e QP, logo, BT3 ⊥ QP

T4 é ponto de tangência entre λ 4 e QP, logo, DT4 ⊥ QP 11. A


T5 é ponto de tangência entre λ 4 e NP, logo, DT5 ⊥ NP

Como AT1T=
2 BT2T=
1 90°, AT1 / /BT2.

=
Como AT1 / /BT2 , AT1 =
BT2 1 e AT1T=
2 BT2T=
1 90°, AT1T2B é um
retângulo, logo, AB / /MQ.

ˆ = 60°.
Analogamente, BD / /QP, portanto, MQP

Os triângulos QBT3 e QBT2 são congruentes, pelo caso LAL, logo,

ˆ = ˆ = 60°
BQT 3 BQT 2 = 30°.
2 Considerando o triângulo retângulo desenhado em vermelho na
No triângulo BQT3 , figura acima, e sendo r o raio da circunferência menor e R o raio da
1 circunferência maior, pode-se escrever:
tg30° =
QT3 ( R + r )2 = R 2 + ( R − r )2
3 1
=
3 QT3 mas R = 4
QT3 = 3 ( 4 + r )2 = 42 + ( 4 − r ) ⇒ 16 + 8r + r2 = 16 + 16 − 8r + r2 ⇒ 16r = 16 ⇒ r = 1
2

π ⋅ 42 π ⋅ 12 π 64 − 15π
Shachurada = 4 ⋅ 8 − + = 32 − 8π + =
Os triângulos PDT4 e PDT5 são congruentes, pelo caso LAL, logo, 2 2 2 2

ˆ = T5PD
ˆ= 120°
T4PD = 60°. 12. D
2
De acordo com o enunciado:
No triângulo PT4D,
1
tg60° =
PT4
1
3=
PT4
3
PT4 =
3

58 PROMILITARES.COM.BR
2 x
= → y = 2x
4 y GEOMETRIA PLANA II
x = 2
 3
x + y = 2 → x + 2x = 2 → 
y = 4 3
∆NFC  ∆AFB ∆MEN  ∆MAN Sejam PC, PD e PE as perpendiculares a AQ, BQ e AB, respectivamente.
2 x 1 a Traçam-se PA, PB, EC e ED.
= → y = 2x = → b = 4a
4 y 4 b ˆ + AEP
ACP ˆ = 90º +90º = 180º ⇒ # ACPE é inscritível
x = 2 a = 1 ˆ + BEP
BDP ˆ = 90º +90º = 180º ⇒ #BDPE é inscritível
 3  5
x + y = 2 → x + 2x = 2 →  a + b = 1 → a + 4a = 1 → 
 y = 4  b = 4 
AP
3 5 ˆ =CAP
⇒ CEP ˆ = ˆ =EDP
=ABP ˆ
∆ MEN  ∆ MAN
Assim, a área do triângulo AEF será: 2
S1=
∆AEFa S∆ABF − S∆ABE 
BP
= → b = 4a ˆ = DBP
⇒ DEP ˆ = ˆ = ECP
= BAP ˆ
4 b 4y 4b 4 ⋅ 4 4⋅4 2
S∆AEF = − = 3− 5 8 8 16
 1 = − → S∆AEF =
2 2 2  = 25 3 5
a 15
a + b = 1 → a + 4a = 1 →  ˆ = EDP
CEP ˆ e ECP ˆ ⇒ ∆CEP ~ ∆EDP ⇒ PE= PC ⇔ PE2= PC ⋅ PD
ˆ = DEP
b = 4 5 PD PE
13. E Nesse caso, temos PC = 6 cm e PD = 10 cm , logo
A corda formada na Figura 2 divide o raio da circunferência ao meio, PE2 =6 ⋅ 10 ⇔ PE =2 15 cm
logo essa corda é igual ao raio do triângulo equilátero inscrito na
circunferência.
16. B
Dessa foram, a área pedida pode ser obtida retirando-se da área da
circunferência, a área de 4 segmentos circulares de 120°. N C
=
S SCIRC − 4 ⋅ S =
D
SEG120

 πr2 r2
=πr2 − 4 ⋅ 

− sen120  =
h3
3 2 h1 h2
 
 4 π 3   π
= r2  π − + 2⋅ = 2
 r  3 − 3  A M B
 3 2 
Para facilitar o cálculo das áreas, separemos o quadrilátero em quatro
Como o raio do círculo é r = 4 cm, temos: triângulos: ∆ADM, ∆DMN, ∆CMN e ∆BCN.
π 1
=

S 42  3 − =


3 3
48 3 − 16π cm2 ( ) Se MN divide o quadrilátero inicial em dois quadriláteros de mesma
área temos:
(AMND) = (BCNM) ⇒ (ADM) + (DMN) = (CMN) + (BCM) = AM.h1/2 +
DN.h3/2 = BM.h2/2 + DN.h3/2 = BM.h2/2 + CN.h3/2
14. A
Entretanto, como N é o ponto médio de CD, então DN = CN, e como
SM'N'P' = SMM'N'N + SPP'N'N − SMM'P'P
M é o ponto médio de AB, AM = BM. Desta forma: AM.h1 + DN.h3 =
Seja r o raio da circunferência de centro N, a o centro da circunferência de AM.h2 + DN.h3 ⇒ h1 = h2.
centro M e b o centro da circunferência de centro P. Sendo h1 = h2, o segmento CD é paralelo ao segmento AB,
⇒ 2r = 2a + 2b ⇔ r = a + b caracterizando um trapézio.
(MM'+ NN') ⋅ MN (r + a) (r − a) ( a + b + a) ( a + b − a) ( 2a + b ) b
=
SMM'N'N = = =
2 2 2 2 17. B
(NN'+ PP') ⋅ NP (r + b ) (r − b ) ( a + b + b ) ( a + b − b ) ( a + 2b ) a
=
SPP'N'N = = = E
2 2 2 2 G
(MM'+ PP') MP ( a + b ) ( a + b ) ( a + b )2
=
SMM'P'P = =
2 2 2 H
J
( 2a + b ) b ( a + 2b ) a ( a + b )2 2ab + b2 + a2 + 2ab − a2 − b2 − 2ab
SM'N'P' = + − = = ab A D
2 2 2 2
B C
O triângulo AB’C é retângulo em B' e BB' é a altura relativa I
à hipotenusa, então (BB')2 = AB ⋅ BC ⇒ 62 = 2a ⋅ 2b ⇔ ab = 9 F
⇒ SM'N'P' = 9 cm2 .
ab = Sejam H, I, J os baricentros dos triângulos ABE, BCF e CDG,
respectivamente. Como a bissetriz interna de um ângulo de um
triângulo equilátero é também mediana, passando pelo baricentro,
15. B
ˆ
=
temos ABH =ˆ BCI
CBI ˆ DCJ
= ˆ
= 30º , logo H, B, I e I, C, J são colineares
=
e H ˆ ˆ
IJ B=
IC 120º.
2 3 3
BH mede 2 da altura do triângulo ABE,= logo BH = . AB AB .
3 3 2 3
3 3
Analogamente, BI = IC = BC e CJ = C
D . Assim,
3 3
1
Área de HIJ = . HI . IJ . sen 120º =
2
1 3 3  3 3 3
=  AB + BC   BC + CD . =
2  3 3   3 3  2

3
= (AB + BC)(BC + CD).
12

PROMILITARES.COM.BR 59
GEOMETRIA PLANA II

18. B Seja AM = m. Utilizando as potências de A e de M em relação à


circunferência inscrita temos:
A X B m 2m
⋅ = AD2 = ME2
3 3
a a
Como AD = ME temos p − a = − (p − b) , ou seja 2p − a − b = ,
F C ou ainda, a = 2c. 2 2

2m2
Y Como = ME2 temos
Z2 9
2 2
2m2  a a + b + c  b − c 
E Z1 D = − + b  =
9 2 2   2 
2 1 b + c2 − 2bc
2

Fig. 1
9 4
(
⋅ 2b2 + 2c2 − a2 = ) 4
Condições:
4b2 + 4c2 − 2a2 = 9b2 + 9c2 − 18bc
a) X, Y e Z estão nos lados do hexágono.
4b2 + 4c2 − 2 ⋅ 4c2= 9b2 + 9c2 − 18bc
b) XY é paralela a um dos lados.
5b2 − 18bc + 13c2 =
0
Supondo X ∈ AB , como XY é paralela a um dos lados, Y pertence ao 2
b b
5   − 18   + 13 =
0
lado CD ou EF. Pela simetria, vamos supor Y e EF. Supondo XY fixo, c c
para atingir a área máxima (maior altura), o ponto Z deve pertencer
As raízes são 1 e 13/5.
ao lado CD. (Figura 1)
b b 13
Sabendo que Z pertence a CD, paralelo a XY, Z pode ser qualquer um = 1 , ou seja, b = c é impossível pois a = 2c. Logo, = , ou seja,
c c 5
dos pontos de CD.
13c .
b=
h 5
13c
Y O triângulo ABC tem lados 2c, e c. O triângulo ABC é, portanto,
A B 5
60º semelhante ao triângulo T de lados 10, 13 e 5. Vamos então calcular
H a área de T.
r
Como o semiperímetro de T é 14, temos:
F 3 3 C
Área de T = 14 ⋅ 4 ⋅ 1⋅ 9 =6 14 , que é a área de ABC. Como o
X triângulo ABC é congruente a T, os lados de ABC são 10, 13 e 5.
60º

E D 20. B
Seja O o ponto de encontro das diagonais do Quadrado.
X Y
Fig. 2
Posição XY que torna a área máxima:
A
Seja XY = 2t
B
a altura do triângulo XYZ é a distância entre as retas paralelas H

XY e CD : 45° r
G C
h = r + 3 3 (figura 2)
O
Pelo triângulo YHB: (H - projeção de Y em BE)
r F D
tg 60º = ⇒ r = (6 − t) 3 ⇒ h = 3 (9 − t)
6−t
E
⇒ SXYZ
= 3 t (9 −=
t) 3(9t − t2 )

9 81 3 W Z
SXYZ MAX ⇔ t = ⇒ SMAX = cm2
2 4 Pela simetria da figura, O também é o centro do Octógono regular.
Logo OB = r (raio do círculo circunscrito do octógono regular)
19. B Analisando o triângulo XOB,
π OB r a 2 π
tg = = ⇔ r= tg
8 OX a 2 2 8
2
π
• Cálculo de tg
8
π
2tg
8 π π π π π
=
tg =1 ⇔ 2tg =1− tg² ⇒ tg =2 − 1 (pois tg > 0)
π 4 8 8 8 8
1− tg 2

60 PROMILITARES.COM.BR
GEOMETRIA PLANA II

então:
ANOTAÇÕES
r=
a 2
2
( )
2 −1=
a
2
(
2− 2 )
O lado do octógono regular é obtido no triângulo OAB:
π
AB = 2r sen .
8
π
• Cálculo de sen
8

cos
π
4
=
π π 1
1− 2sen² ⇔ sen =2 − 2
8 8 2
( )
então:
3
⇒ AB = a (2 – 2) 2
2
A área do octógono é: S = 8 x S∆ AOB
Como
π 1 a²
1 2 a²
( ) ( )
2
S∆ AOB= r² sen = 2− 2 ⋅ = 2 6−4 2 ,
2 4 24 2 16
2
a
=
temos S 8x = (6 2 – 8) a2 (3 2 – 4)
16

EXERCÍCIOS DE COMBATE
01. A 06. D
02. C 07. A
03. D 08. A
04. B 09. C
05. E 10. D

11. 0,25
Nessa questão utilizamos o seguinte conceito: A área de um triângulo
que possui dois lados de medida a e b adjacentes ao ângulo θ é
a⋅b
=S sen θ .
2

12. SABC = 315 u.a.

2
1  2 2π
⋅ ( π ⋅ R2 )= ⋅13.
π ⋅ Ssetor120º
= cm2
3  3  9

b a c
=
14. =
1 7− 7 7+ 7
7 7

1 1( ) 15 R2 15
15. S=
MOS OM ⋅ OS ⋅ sen
= θ 2R ⋅ R ⋅ =
2 2 4 4

8⋅ 7
16. r =
7

3 3RH 2x 3 3 2
= 17.= = = =
2 6 ⋅ 3 3 ⋅ R2T 2 3 2 6 ⋅ 3 3⋅ 2 2
y⋅ ⋅
3 2 3
18. 6

19. θ = 45º

20. Área de T = 14 ⋅ 4 ⋅ 1⋅ 9 =6 14 , que é a área de ABC. Como o


triângulo ABC é congruente a T, os lados de ABC são 10, 13 e 5.

PROMILITARES.COM.BR 61
GEOMETRIA PLANA II

ANOTAÇÕES

62 PROMILITARES.COM.BR
FUNÇÃO AFIM E QUADRÁTICA

FUNÇÃO AFIM FUNÇÃO QUADRÁTICA


É uma função de  em  definida por
f(x) = ax + b DEFINIÇÃO
onde a e b são constantes reais e a ≠ 0. É uma função da forma f(x) = ax² + bx + c, com a, b, c ∈  e a ≠ 0.
A função identidade (a = 1 e b = 0) e a função linear (b = 0) são Onde a, b, c são constantes ∈  e x é uma variável.
casos particulares da função afim.
A função afim é uma função polinomial do 1º grau, seu gráfico é GRÁFICO
uma reta não paralela a nenhum dos eixos coordenados e sua imagem
é o conjunto dos números reais: Im = . O gráfico de todas as funções do segundo grau, ou Função
O coeficiente a é chamado coeficiente angular e representa a taxa quadrática, são parábolas.
∆y Parábola é uma das seções cônicas que é gerada pela interseção
de variação média da função que é igual à tangente do ângulo
∆x de uma superfície cônica  e um plano paralelo à reta geratriz do cone,
de inclinação da reta. Sendo o ângulo de inclinação da reta, tem-se sendo que o plano não contém esta.
Parábola pode também ser definida como o lugar geométrico
tg θ = a dos pontos do plano que que são equidistantes de um ponto dado
(chamado de foco) e de uma reta dada (chamada de diretriz).
a > 0 → θ é agudo → função crescente
Devido as propriedades geométricas das parábolas muitas
a < 0 → θ é obtuso → função decrescente
aplicações práticas são encontradas em diversas áreas da vida real.
Alguns exemplos são as antenas parabólicas, faróis de automóveis e
O coeficiente b é chamado coeficiente linear e é o ponto onde a
radares.
reta cruza o eixo Oy, ou seja, a reta passa no ponto (0, b).
A parábola representativa da função quadrática f(x) = ax² + bx + c
O gráfico intercepta o eixo dos x em um único ponto que é a raiz
terá concavidade voltada para cima quando a > 0 e concavidade
da equação f(x) = 0 dada por x = − b . voltada para baixo quando a < 0.
a
a>0
Abaixo são mostrados gráficos da função afim para a negativo e
positivo. Concavidade voltada para cima

a<0
Concavidade voltada para baixo

PROMILITARES.COM.BR 75
FUNÇÃO AFIM E QUADRÁTICA

ProBizu FORMA CANÔNICA


Para memorizar mais facilmente a concavidade da parábola, Considere o trinômio
podemos pensar no seguinte “macete”:
 b c
Quando a > 0, pensamos que “a parábola está feliz” e assim a ax 2 + bx +=
c a  x2 + x + =

 a a
concavidade é para CIMA.
Quando a < 0, pensamos que “a parábola está triste” e assim a  b b2 c b2 
= a  x2 + 2 x + 2 + − 2  ⇔
concavidade é para BAIXO.  2a 4a a 4a 
 b
2
4ac − b2 
ax 2 + bx +=
c a x +  + .
 2a  4a2 
PONTO DE INTERSEÇÃO DA PARÁBOLA COM O EIXO
DAS ORDENADAS Agora podemos escrever a função quadrática da seguinte forma:
 b
2
4ac − b2 
Quando x = 0 temos f(0) = c portanto o ponto em que a parábola f(x)= ax 2 + bx + c= a   x +  + =
corta o eixo das ordenadas é o ponto (0, c).   2a  4a2 
2 2
 b  4ac − b2   b  ∆
=a  x +  +   ⇔ f(x) =a  x +  +  −  .
 2a   4a   2a   4a 

Portanto a partir da forma canônica podemos chegar a alguns


fatos muito importantes:
I. Se a > 0, a função quadrática y = ax² + bx + c admite valor
mínimo y V = − ∆ (“y do vértice”) e tal valor mínimo ocorre
4a
para x = x V = − b (“x do vértice”). Neste caso, o “x do
2a
vértice” é dito minimante.
II. Se a < 0, a função quadrática y = ax² + bx + c admite valor
máximo y V = − ∆ (“y do vértice”) e tal valor máximo ocorre
4a
b
para x = x V = − (“x do vértice”). Neste caso, o “x do
2a
vértice” é dito maximante.

Observação
Acima estão representados os gráficos das funções:
 b
O ponto ( x V , y V ) =− ∆
em azul o gráfico de f(x) = x 2.  , −  é chamado de vértice da parábola.
2a 4a 
= x2 − 2
em verde o gráfico de g(x)
= x2 + 1
em vermelho o gráfico de g(x)
ProBizu
A variação do valor de c faz com que o gráfico da função
quadrática se desloque na vertical. Para memorizar mais facilmente a concavidade da parábola,
podemos pensar no seguinte “macete”:
ZEROS, RAÍZES-PONTOS DE INTERSEÇÃO DA Quando a > 0, pensamos que “a parábola está feliz” e assim a
concavidade é para CIMA.
PARÁBOLA COM O EIXO x
Quando a < 0, pensamos que “a parábola está triste” e assim a
Zeros ou raízes são os valores de x reais para os quais f(x) se anula, concavidade é para BAIXO.
sendo, portanto, as soluções da equação do 2º grau ax² + bx + c e as
abscissas dos pontos onde a parábola intersecta o eixo Ox. Exemplo:
As raízes da função f(x) = ax² + bx + c são dadas por A função f(x) = x² – 4x + 5 possui concavidade para cima e portanto
−b ± ∆ − ( −4 )
x1,2 = admite mínimo, que ocorre para = x V = 2 . O discriminante
2a 2
desta função quadrática é ∆ = (–4)² – 4 ⋅ 1 ⋅ 5 = –4 e portanto o
onde ∆ = b² – 4ac. − ( −4 )
valor mínimo=é y V = 1 . Pelo PROBIZU, poderíamos calcular o
A análise do sinal do discriminante ∆ permite identificar o número 4
de raízes reais do trinômio: mínimo substituindo x por 2, o que nos daria 2² – 4 ⋅ 2 + 5 = 1, como
encontrado pelo método do “y do vértice”.
Se ∆ > 0, a função possui duas raízes reais distintas dadas por
−b ± ∆
x1,2 = e intersecta o eixo Ox em dois pontos distintos.
2a III. Eixo de Simetria
Se ∆ = 0, a função tem uma raiz real dupla (ou duas raízes reais Pontos do gráfico de uma função quadrática com abscissas
−b equidistantes do xv estão à mesma altura, ou seja, a reta x = xv é um
iguais) dada por x1,2 = e tangencia o eixo Ox.
2a eixo de simetria da parábola.
Se ∆ < 0, a função não possui raízes reais e não intersecta o eixo Ox. Basta substituir em f(x) = a(x – xv)² + yv os pontos

76 PROMILITARES.COM.BR
FUNÇÃO AFIM E QUADRÁTICA

x=
1 xv + d Esse estudo de sinais serve de referência para a resolução das
inequações do 2º grau.
x=
2 xv − d
teremos que f(x1) = f(x 2 ). Exemplo:
Isto é, quando temos pontos x1 e x 2 que Para resolver a inequação x² + 5x – 24 ≥ 0, basta considerar a
função f(x) = x² + 5x – 24 . O primeiro passo é determinar as suas
estão a mesma distância de x v , f(x1) = f(x 2 ).
raízes.
∆= 52 − 4 ⋅ 1⋅ ( −24 )= 121 > 0
−5 − 11 −5 + 11
⇒=x1 = −8 ∧= x2 = 3
2 2
Como a = 1 > 0 a parábola tem concavidade voltada para cima,
logo a função será positiva fora das raízes e nula nas raízes e o
conjunto solução da inequação será S = ] −∞, −8] ∩ [ 3, +∞[ .

EXERCÍCIOS DE

TREINAMENTO
01. Quatro carros A, B, C, D viajam a velocidades constantes na mesma
estrada. A ultrapassa B e C às 8 horas e 9 horas, respectivamente, e
encontra D às 10 horas; D encontra B e C às 12 horas e 14 horas,
respectivamente. Determine a que horas B ultrapassa C.
ProBizu a) 10: 20 h
b) 10: 30 h
Isso implica que pontos, cujas abscissas equidistam do vértice,
possuem o mesmo valor de ordenada. c) 10: 40 h
f ( x V − k=
) f ( x V + k ) , ∀k ∈  d) 11:00 h
e) 11:20 h
Observe ainda que a abscissa do vértice é a média aritmética das
b x +x
raízes: x V =
− =1 2 . 02. Para cada número real x, seja f(x) = min{4x + 1, x + 2, –x + 6.
2a 2 Calcule o valor máximo de f(x).
a) 5 7
d)
b) 4 3
FORMA FATORADA 8 e) 2
c)
Se f(x) = ax² + bx + c possuir raízes r1 e r2, podemos fatorar 3
f(x) = a(x – r1)(x – r2).
03. (FUVEST 2004) Um estacionamento cobra R$ 6,00 pela primeira
Exemplo: hora de uso, R$ 3,00 por hora adicional e tem uma despesa diária de
Seja f(x) = 6x² – 5x –1. As raízes desta função quadrática R$ 320,00. Considere-se um dia em que sejam cobradas, no total, 80
1 horas de estacionamento. O número mínimo de usuários necessário
são 1 e − . Assim, podemos fatorar tal função como
6 para que o estacionamento obtenha lucro nesse dia é:
 1
6 ( x − 1)  x +  =( x − 1)( 6x + 1) . a) 25 d) 28
 6
b) 26 e) 9
c) 27
ESTUDO DO SINAL DA FUNÇÃO 04. Determine o valor de a tal que as raízes da equação
A análise dos gráficos da função f(x) = ax² + bx + c acima permite x 2 − (3a + 1) ⋅ x + (2a2 − 3a − 2) =0 são reais e a soma dos seus
realizar o estudo do sinal da função. quadrados é mínima.
Se ∆ < 0, a função tem sempre o mesmo sinal de a. 6 d) −9 + 6 2
a) −
5 e) 0
b
Se ∆ = 0, a função é nula em x = − e tem o mesmo sinal de a
2a 11
nos outros valores de x. b) −
5
Se ∆ > 0, a função tem o sinal de a no intervalo exterior às raízes e c) −9 − 6 2
sinal contrário ao de a entre as raízes.

PROMILITARES.COM.BR 77
FUNÇÃO AFIM E QUADRÁTICA

05. As raízes do trinômio do 2° grau y = ax² + bx + c são –100 e


12. As raízes reais da equação 5− 5−x =x pertencem ao
300. Sabendo que quando x vale −1050 o valor numérico de y é 513,
intervalo:
quando x vale −950 o valor numérico de y é 425 e quando x vale –850
o valor numérico de y é 345, qual é o valor numérico de y quando x a) (0, 5 ) d) ( 7, 8 )
vale 1050?
a) 513 d) 427,67 b) ( 5, 6 ) e) ( 8,3 )
b) 425 e) 345 c) ( 6, 7 )
c) −513
13. (ITA 2018) Se x é um número real que satisfaz x³ = x + 2, então
06. (EN 1991) Representemos por min(a,b) o menor dos números x10 é igual a
a, se a ≤ b a) 5x² + 7x + 9. d) 7x² + 5x + 9.
a e b, isto é, min ( a,b) =  . A solução da inequação
b, se a > b b) 3x² + 6x + 8. e) 9x² + 3x + 10.
min (2x + 3, 3x – 5) < 4 é:
c) 13x² + 16x + 12.
1 1
a) x < d) x >
2 2 14. (ITA 2011) O produto das raízes reais da equação
b) x < 3 e) x>3 |x2 – 3x + 2| = |2x – 3| é igual a
1 a) –5. d) 2.
c) <x<3
2 b) –1. e) 5.
07. A equação y = x² + 2ax + a representa uma parábola para todos c) 1.
os valores reais de a. Todas essas parábolas passam por um ponto
comum. A somas das coordenadas desse ponto comum é: 15. Considere as funções quadráticas reais f(x) = 2x² + 5x + c e
1 1 g(x) = 2x² + 5x + d. Determine, sem usar cálculo integral, a área
a) 0 c) – e) – localizada entre os gráficos de f e g no trecho de x = n até x = m, m > n.
2 4
1
b) 1
2 d) 16. Dadas as funções F(x) = ax² + bx + c e G(x) = cx² + bx + a, onde
4
F(0) ≤ 1, F(1) ≤ 1 e F(−1) ≤ 1. Prove que, para x ≤ 1, (i) F(x) ≤ 5/4; (ii)
08. Julgue as afirmativas a seguir em relação à equação G(x) ≤ 2.
(x ) ( )
2 2
− 3x − 2 − 3 x 2 − 3x − 2 − 2 − x =
0:
I. As raízes da equação x² – 4x – 2 = 0 são raízes da equação inicial; 17. Determine todos os x e y reais tais que 20x² + 10y² + 4xy + 12x
– 10y + 5 = 0.
II. O produto das raízes da equação inicial é 8;
III. Todas as raízes da equação inicial são reais. 18. (UNICAMP 1992) Calcule a e b positivos na equação da reta
Agora marque a alternativa que apresenta a seqüência obtida: ax + by = 6 de modo que ela passe pelo ponto (3,1) e forme com os
a) F – F – F d) V – V – V eixos coordenados um triângulo de área igual a 6.
b) V – F – V e) F – V – V
19. O maior inteiro que não supera a solução da equação
c) V–V–F
2x − 5 + 2 x 2 − 5x + 2 x − 5 + 2 x =48 é igual a:
09. (ITA 1986) Sejam a, b, c números reais dados com a < 0. Suponha a) 11 d) 17
que x1 e x2 sejam as raízes da função y = ax² + bx + c e x1 < x2. Sejam b) 13 e) 19
b 2
x3 = − e x 4 = − 2b + b − 4ac . Sobre o sinal de y podemos c) 15
2a 4a
afirmar que: EXERCÍCIOS DE

a) y < 0, ∀x ∈ , x1 < x < x3


b) y < 0, ∀x ∈ , x4 < x < x2
COMBATE
c) y > 0, ∀x ∈ , x1 < x < x4
d) y > 0, ∀x ∈ , x > x4 01. (ITA 2014) Considere as funções f, g:  → , f(x) = ax + m,
e) y < 0, ∀x ∈ , x < x3 g(x) = bx + n, em que a, b, m e n são constantes reais. Se A e B são as
imagens de f e de g, respectivamente, então, das afirmações abaixo:
10. Existe um único par de números inteiros positivos x e y que I. Se A = B, então a = b e m = n;
satisfazem a equação x² + 84x + 2008 = y². O valor de x + y é: II. Se A = Z, então a = 1;
a) 108 d) 80 III. Se a, b, m, n ∈ Z, com a = b e m = –n, então A = B,
b) 96 e) 72 é (são) verdadeira(s)
c) 84 a) apenas I. d) apenas I e II.
b) apenas II. e) nenhuma.
11. O módulo da diferença entre as raízes irracionais da equação
c) apenas III.
3x 4 − 11x 2 + 11x − 3 =0 é da forma m onde m e n são primos entre
n
si. O valor de m + n é igual a: 02. (ITA 1995) Os dados experimentais da tabela a seguir
a) 87 d) 90 correspondem às concentrações de uma substância química medida
em intervalos de 1 segundo. Assumindo que a linha que passa pelos
b) 88 e) 91 três pontos experimentais é uma parábola, tem-se que a concentração
c) 89 (em moles) após 2,5 segundos é:

78 PROMILITARES.COM.BR
FUNÇÃO AFIM E QUADRÁTICA

a) 52 d) 45
TEMPO (s) CONCENTRAÇÃO (MOLES)
b) 51 e) 42
1 3,00 c) 46
2 5,00
08. (EN 2013) Uma loja está fazendo uma promoção na venda de
3 1,00 bolas: “Compre x bolas e ganhe x% de desconto”. A promoção é
válida para compras de até 60 bolas, caso em que é concedido o
a) 3,60 c) 3,70 e) 3,80 desconto máximo de 60%. Julia comprou 41 bolas e poderia ter
comprado mais bolas e gasto a mesma quantia. Quantas bolas a mais
b) 3,65 d) 3,75 Julia poderia ter comprado?
a) 10
03. (ITA 2018) Para que o sistema
b) 12
x + y = 1
 3 c) 14
3
 x + y = c2
d) 18
admita apenas soluções reais, todos os valores reais de c pertencem e) 24
ao conjunto

a)  1 09. (EN 2013) Numa vidraçaria há um pedaço de espelho, sob a forma


 − ∞, − 4  . de um triângulo retângulo de lados 30 cm, 40 cm e 50 cm. Deseja-
 1  1  se, a partir dele, recortar um espelho retangular, com a maior área
b)  − ∞, − 4  ∪  4 , ∞  . possível, conforme figura abaixo. Então as dimensões do espelho são

 1 1
c)  − 2 , − 4  .

d) 1 
 2 , ∞  .
 1  1 
e)  − ∞, − 2  ∪  2 , ∞  .

04. (ITA 2018) O lugar geométrico das soluções da equação


x² + bx + 1 = 0, quando | b |< 2, b ∈ , é representado no plano
complexo por
a) dois pontos. a) 25 cm e 12 cm
b) um segmento de reta. b) 20 cm e 15 cm
c) uma circunferência menos dois pontos. c) 10 cm e 30 cm
d) uma circunferência menos um ponto. d) 12,5 cm e 24 cm
e) uma circunferência. e) 10 3 cm e 10 3 cm

05. (ITA 1996) Seja á um número real tal que α > 2(1 + 2 ) e 10. (IME 2012) Seja a, b e c números reais e
considere a equação x2 - αx + α + 1 = 0. Sabendo que as raízes reais distintos. Ao simplificar a função real, de variável real,
dessa equação são as cotangentes de dois dos ângulos internos de um ( x − b) ( x − c ) 2 ( x − c ) ( x − a) 2 ( x − a) ( x − b)
f ( x ) = a2 +b +c , obtém-se
triângulo, então o terceiro ângulo interno desse triângulo vale: ( a − b) ( a − c ) (b − c ) (b − a) ( c − a) ( c − b)
a) 30° c) 60° e) 120° f(x) igual a:
b) 45° d) 135° a) x² – (a + b + c)x + abc
b) x² + x – abc
06. (EN 2014) Uma bolinha de aço é lançada a partir da origem e segue c) x²
urna trajetória retilínea até atingir o vértice de um anteparo parabólico
d) –x²
 − 3 2
representado pela função real de variável= real f(x)
 3  x + 2 3x. e) x² – x + abc
 
Ao incidir no vértice do anteparo é refletida e a nova trajetória retilínea 11. (IME 2012) Um curso oferece as disciplinas A, B, C e D. Foram
feitas as matriculas dos alunos da seguinte forma:
é simétrica à inicial, em relação ao eixo da parábola. Qual é o ângulo
de incidência (ângulo entre a trajetória e o eixo da parábola)? – 6 alunos se matricularam na disciplina A;

a) 30º d) 75º – 5 alunos se matricularam na disciplina B;

b) 45º e) 90º – 5 alunos se matricularam na disciplina C; e

c) 60º – 4 alunos se matricularam na disciplina D.


Sabe-se que cada aluno se matriculou em, no mínimo, 3 disciplinas.
07. (EN 2014) Um restaurante a quilo vende 200 quilos de comida Determine a quantidade mínima de alunos que se matricularam nas
por dia, a 40 reais o quilo. Uma pesquisa de opinião revelou que, a 4 disciplinas.
cada aumento de um real no preço do quilo, o restaurante perde 8 a) 0 d) 3
clientes por dia, com um consumo médio de 500 gramas cada. Qual b) 1 e) 4
deve ser o valor do quilo de comida, em reais, para que o restaurante
tenha a maior receita possível por dia? c) 2

PROMILITARES.COM.BR 79
FUNÇÃO AFIM E QUADRÁTICA

a b
12. (ITA 2015) Considere a equação − 5, com a e b
= GABARITO
1 − x 2 x − 1/ 2
números inteiros positivos. Das afirmações: EXERCÍCIOS DE TREINAMENTO
I. Se a = 1 e b = 2 então x = 0 é uma solução da equação. 01. C
1 Desenhe os gráficos dos movimentos dos carros (linhas retas)
II. Se x é solução da equação, então x ≠ , x ≠ –1 e x ≠ 1. considerando 8 horas como o tempo inicial.
2
2
III. x= não pode ser solução da equação.
3
É (são) verdadeira(s)
a) apenas II. d) apenas II e III.
b) apenas I e II. e) I, II e III.
c) apenas I e III.

13. (IME 2015) Encontre as soluções reais da equação:


x + 4x − 4 + x − 4x − 4 = x+3

14. (EPCAR 2012 - ADAPTADA) Considere a parábola que representa



a igualdade y = ax² + bx + c, de eixo de simetria PV , e o quadrado
ABCD indicados na figura abaixo.

Suponha que B passe por C no momento T. Na figura, os pontos X, Y,


Z, M, N, G correspondem às ultrapassagens entre pares de carros. É
fácil ver que M e N são pontos médios de YZ e XY, respectivamente, e
que XM e ZN são medianas do ∆XYZ. Isso implica que sua interseção
G, correspondente à ultrapassagem de C por B, é o baricentro do
triângulo. Logo, a abscissa de G é a média aritmética das abscissas de
X, Y e Z, ou seja, T = 1 (8 + 10 + 14) = 10 2 . Logo, B ultrapassa C às
10:40 horas. 3 3

Sabendo-se que os pontos A e B pertencem à parábola e ao eixo Ox
e sendo V o ponto onde a parábola tangencia o segmento DC, qual é
o valor de D = b² – 4ac? 02. B

15. (IME) Resolva a equação 5− 5−x =x, sabendo-se que x > 0.

16. (ITA 2005) Determine todos os valores reais de a para os quais a


equação (x - 1)2 = |x - a| admita exatamente três soluções distintas.

17. (ITA 2003) Determine a área do polígono, situado no primeiro


quadrante, que é delimitado pelos eixos coordenados e pelo conjunto
{(x, y) ∈ 2: 3x2 + 2y2 + 5xy - 9x - 8y + 6 = 0}.

18. (OMERJ 1990) No triângulo ABC abaixo, a é a base, h a altura


relativa à esta base, e b o lado oposto ao ângulo de 45º.  1
4x + 1, se x ≤ 3

1
f ( x ) = x + 2, se <x≤2
3

 − x + 6, se x > 2
Analisando o gráfico, conclui-se que o valor máximo de f é 4.

03. C
nº de usuários = x ⇒ nº de primeiras horas = x
Se a + h = 4, QUAL É o valor mínimo de b²? nº de horas adicionais = y
6x + 3y > 320
19. (IME 2007) Sejam x1 e x2 as raízes da equação x² + (m – 15)x + m = 0.  ⇒ 6x + 3 ⇒ (80 − x) > 320 ⇒ x > 80 ≈ 26,7
x + y = 80
3
Sabendo que x1 e x2 são números inteiros, determine o conjunto de
valores possíveis para m. Logo, o nº mínimo para que haja lucro é 27.

20. (IME 1985) Uma reta m1 passa pelo ponto fixo P1(–1, –3)
e intercepta a reta m2: 3x + 2y –  6 = 0 no ponto A e a reta m3:
y – 3 = 0 no ponto B. Determinar a equação do lugar geométrico do
ponto médio do segmento retilíneo AB à medida que a reta m1 gira
em torno do ponto P1.

80 PROMILITARES.COM.BR
FUNÇÃO AFIM E QUADRÁTICA

04. D 07. E
= (3a + 1)2 − 4 ⋅ 1⋅ (2a2 − 3a − 2)
∆ = a ≠ b ⇒ x 2 + 2ax + a =x 2 + 2bx + b
2 2 2
= 9a + 6a + 1 − 8a + 12a + 8 = a + 18a + 9 ≥ 0 1
⇔ 2 ( a − b) x =b − a ⇔ x =−
⇒ a ≤ −9 − 6 2 ou a ≥ −9 + 6 2 2
2
 1  1 1 1
r12 + r22 = (r1 + r2 ) − 2rr
2
12 = ⇒ y =  −  + 2a  −  + a = − a + a =
 2  2 4 4
= (3a + 1)2 − 2(2a2 − 3a − 2) = 5a2 + 12a + 5
12 Logo, todas as parábolas passam pelo ponto  − 1 , 1  .
aMINIMANTE = − = −1,2  
2⋅5 2 4
y = 5a2 + 12a + 5 08. D
a = −9 − 6 2 ≈ −17,5 a = −9 + 6 2 ≈ −0,5 (I) Verdadeira
x 2 − 4x − 2 = 0 ⇔ x 2 − 3x − 2 = x

(x ) ( )
2 2
− 3x − 2 − 3 x 2 − 3x − 2 − 2 − x = x 2 − 3x − 2 − x =
= x−x = 0
Logo, as raízes de x² – 4x – 2 = 0 são raízes de
(x ) ( )
2 2 2
− 3x − 2 − 3 x − 3x − 2 − 2 − x =
0.

(II) Verdadeira
Termo independente = 2² – 3⋅ (–2) – 2 = 8
Pelas relações de Girard o produto das raízes é (–1)4 ⋅ 8 = 8.

Observando os intervalos acima, o valor mais próximo do a minimante


(III) Verdadeira
para o qual as raízes são reais é −9 + 6 2 .
x 4 − 6x 3 + 2x 2 + 20x + 8 = (x 2 − 4x − 2)(x 2 − 2x − 4)
05. E
1a resolução: Os dois fatores possuem D > 0.

−100 + 300
=xV = 100 09. C
2
⇒ f (1050) = f (100 + 950) = f (100 − 950) = Como a < 0, b2 − 4ac > 0 e x1 < x2, então
f ( 850) =
=− 345
−b + b2 − 4ac −b − b2 − 4ac
=x1 = e x2
2a 2a
06. B x1 < x3 < x2, pois x3 é a média das raízes
2b + b2 − 4ac b b2 − 4ac
x4 =− =− −
4a 2a 4a
b2 − 4ac b2 − 4ac
Como 0 < − <− , então x3 < x4 < x2.
4a 2a
Como a parábola possui concavidade voltada para baixo, então y > 0
quando x1 < x < x4.

10. D
y 2 =x 2 + 84x + 2008 =( x + 42) + 244
2

⇔ 244 = y 2 − ( x + 42) = ( y − x − 42)( y + x + 42)


2

Como y + x e y – x possuem a mesma paridade, então (y + x + 42) e


(y – x – 42) também possuem a mesma paridade.
Como 244 é par, então (y + x + 42) e (y – x – 42) são ambos pares.
Como 244 = 2² ⋅ 61 e x, y > 0, temos y – x – 42 = 2 e y + x + 42 = 122.
2x + 3 = 3x − 5 ⇔ x = 8, y = 19
A última equação implica x + y = 80 e resolvendo encontramos
3x − 5, se x < 8 (x,y) = (18,62).
⇒ min ( 2x + 3, 3x − 5) = 
2x + 3, se x ≥ 8
min ( 2x + 3, 3x − 5) < 4 11. B
⇒ 3x − 5 < 4 ⇒ x < 3 A equação proposta é uma equação recíproca de 2ª espécie e de grau
par logo, 1 é raiz da equação. Dividamos por x – 1

3 –11 0 11 –3

1 3 –8 –8 3 0

PROMILITARES.COM.BR 81
FUNÇÃO AFIM E QUADRÁTICA

x2 – 3x + 2 = -2x + 3 ⇒ x2 + x - 1 = 0, temos o produto das raízes


Chegamos à equação 3x 3 − 8x 2 − 8x + 3 = 0 que é recíproca de 1ª
igual a-1.
espécie e de grau ímpar, dai –1 é raiz desta equação. Dividamos por x + 1
Logo, o produto total das raízes é -1 ⋅ 5 = -5
3 –8 –8 3
–1 3 –11 3 0 15. (m−n) ⋅ |c−d|
Observe que os dois gráficos têm a mesma forma, diferindo entre si
Obtemos assim a equação do segundo grau 3x² – 11x + 3 = 0 cujas
apenas pela sua posição vertical.
11± 85
raízes são e portanto, o conjunto solução da equação Na figura , representamos a área procurada pela área entre os
6
segmentos AB e CD.
 
proposta é S =  −1, +1, 11 − 85 , 11+ 85  . Note, entretanto, que a área acima da parábola de cima e dentro do
 6 6  paralelogramo é igual à área acima da parábola de baixo e fora do
paralelogramo.
12. Elevando ao quadrado ambos os lados obtemos Dessa forma, a área procurada é igual à área do paralelogramo ABCD.
5 − x 2 = 5 − x . Elevando ao quadrado novamente tem-se
S = |c – d| ⋅ (m – n)
( )( )
x 4 − 10x 2 + x + 20 = 0 ⇔ x 2 + x − 5 x 2 − x − 4 = 0 . Fazendo x² – x – 4
1± 17
igual a zero obtemos x = . Entretanto nenhum destes dois
2
números é uma solução da equação original(eles foram introduzidos
quando elevamos ao quadrado). Fazendo x² + x – 5 igual a zero,
−1± 21
obtemos x = e destes somente x = −1+ 21 é uma solução.
2 2
A equação proposta é uma equação recíproca de 2ªespécie e de grau
par logo, 1 é raiz da equação. Dividamos por x – 1

3 –11 0 11 –3
1 3 –8 –8 3 0

Chegamos à equação 3x³ – 8x² – 8x + 3 = 0 que é recíproca de 1ª


espécie e de grau ímpar, dai –1 é raiz desta equação. Dividamos por
x+1

3 –8 –8 3
–1 3 –11 3 0

Obtemos assim a equação do segundo grau 3x² – 11x + 3 = 0 cujas


raízes são 11± 85 e portanto, o conjunto solução da equação 16. A função quadrática F(x) é determinada por três pares de valores
6 (x, F(x)). Nós escolhemos os pares (-1, F(-1}), (0,F(0)) e (1,F(1)) e
  obtemos
proposta é S =  −1, +1, 11 − 85 , 11+ 85  .
6 6 x(x − 1) (x + 1) x
  (1)
= F(x) .F( −1) − (x + 1)(x − 1)F(0) + F(1).
2 2
13. C desde que |F(–1)|, |F(0)| e |F(1)| são ≤ 1, nós temos

De x³ = x + 2, 2 F(x) ≤ x(x − 1) + 2 x 2 − 1 + x(x + 1) .

( x )=
3 3
( x + 2)3 o intervalo –1 ≤ x ≤ 1, nós também temos
x 9 = x 3 + 3 ⋅ x 2 ⋅ 2 + 3 ⋅ x ⋅ 22 + 23 0 ≤ 1+ x ≤ 2, 0 ≤ 1 − x ≤ 2 e 0 ≤ 1 − x 2 ≤ 1.
x 9 =x 3 + 6x 2 + 12x + 8 2 F(x) ≤ x {1 − x + 1+ x} + 2(1 − x 2 ) − 2{ x + 1 − x 2 }.
Mas, x³ = x + 2, logo, 1 5 5
F(x) ≤ −( x − )2 + ≤ .
9 2
x = x + 2 + 6x + 12x + 8 2 4 4
x 9 = 6x 2 + 13x + 10 Para comprovar esta equação considere
 1 5
De x = 6x² + 13x + 10,
9
F(x) = 1+ x − x 2 em [−1,1] : F  = .
 2 4
(
x ⋅ x 9 =x ⋅ 6x 2 + 13x + 10 )
(2) se então F(x) = ax² + bx + c,
x10 =6x 3 + 13x 2 + 10x
 1  a b 
Mais uma vez lembremos que x³ = x + 2 portanto, x 2 F   = x 2  2 + + c = cx 2 + bx + a= G(x).
 x x x 
x10 = 6 ⋅ ( x + 2) + 13x 2 + 10x
Por (1) = F =
1 1
[(1 − x)F( −1) − 2(1 − x 2 )F(0) + (1+ x)F(1)].
x10 = 13x 2 + 16x + 12  x  2x 2
então para –1 ≤ x ≤ 1
14. A 2 G(x) ≤ 1 − x + 2 1 − x 2 + 1 − x = 4 − 2x 2 ,
x – 3X + 2 = 2x – 3 ⇒ x – 5x + 5 = 0, temos o produto das raízes
2 2
G(x) ≤ 2 − x 2 ≤ 2.
igual a 5.

82 PROMILITARES.COM.BR
FUNÇÃO AFIM E QUADRÁTICA

17. x = −5/14 e y = 4/7


ANOTAÇÕES
20x 2 + (4y + 12)x + (10y 2 − 10y + 5) =
0
∆ = (4y + 12)2 − 4 ⋅ 20 ⋅ (10y 2 − 10y + 5) = −16(7y − 4)2

Para que as raízes sejam reais, deve-se ter ∆ ≥ 0


4
⇒ ∆ = −16(7y − 4)2 ≥ 0 ⇒ 7y − 4 = 0 ⇒ y =
7
5
⇒ x =−
14

18. a = 1 e b = 3

A reta cruza os eixos ordenados nos pontos  0,  e  ,0 .


6 6
 b a 
6 6
O triângulo formado é um triângulo retângulo de catetos e .
a b
1 6 6
S = ⋅ ⋅ = 6 ⇔ ab = 3 (*)
2 a b
Se a reta passa pelo ponto (3, 1), então
3a + b = 6 (**)
Substituindo (*) em (**), vem:
3
3a + = 6 ⇔ a2 − 2a + 1 = 0 ⇔ a = 1e b = 3
a

19. A
O domínio de definição é x ≥ 5. Sob esta definição temos x ≥ 0 e
x – 5 ≥ 0 e portanto, 2 x 2 − 5x= )
x ( x − 5= x x −5
Uma vez que 2x = x + x, a equação proposta pode ser escrita como
x + x − 5 + 2 x x − 5 + 2 x − 5 + 2 x − 48 =0 ou

( x) ( ) ( )
2 2
+2 x x −5 + x −5 +2 x − 5 + x − 48 =0

Isto é,

( ) ( )
2
x −5 + x +2 x − 5 + x − 48 =0

Fazendo y = x − 5 + x obtemos y² + 2y – 48 = 0 donde obtemos


y1 = 6 e y2 = –8 e assim, nosso problema reduz-se a resolver as duas
equações
x −5 + x =6 e x − 5 + x =−8
2
A primeira tem solução x =  41 e a segunda não tem solução.
 12

EXERCÍCIOS DE COMBATE
01. E  7
13. 1,  .
02. D  3
03. E 14. 16
04. C 21 − 1
15. x =
05. D 2
06. A 16. a = 3/4 ou a = 1 ou a = 5/4
07. D 17. 5
08. D 2
09. C 18. 16
5
10. C
19. {0, 7, 9, 25, 27, 34}.
11. C
20. 6xy + 4y2 + 3y − 45 = 0
12. E

PROMILITARES.COM.BR 83
FUNÇÃO AFIM E QUADRÁTICA

ANOTAÇÕES

84 PROMILITARES.COM.BR
FUNÇÃO EXPONENCIAL
E LOGARITMO

FUNÇÃO EXPONENCIAL 2º caso: 0 < a < 1 (função decrescente)

DEFINIÇÃO
Seja a ∈ , tal que 0 < a ≠ 1, a função exponencial de base a é a
função f:  →  tal que
f(x) = ax

PROPRIEDADES
1) Como f(0) = a0 = 1, o par ordenado (0, 1) pertence ao gráfico
da função exponencial.
2) Quando 0 < a < 1, a função f(x) = ax é decrescente. Já quando
a > 1, a função f(x) = ax é crescente. FUNÇÃO DO TIPO EXPONENCIAL
0 < a < 1: x1 < x2 ⇒ f(x1) > f(x2)
Uma função de crescimento ou decaimento exponencial é
a > 1: x1 < x2 ⇒ f(x1) < f(x2) uma função que cresce ou decresce em uma taxa de crescimento de
Essa propriedade tem aplicação na resolução das inequações porcentagem constante. A equação pode ser escrita na forma
exponenciais. f(x) = a(1 + r)x ou onde f(x) = abx
3) A função f(x) = ax, com 0 < a ≠ 1 é injetora. Onde b = 1 + r
f(x1) = f(x2) ⇔ x1 = x2 a é o valor inicial ou de início da função
Essa propriedade respalda a solução das equações exponenciais. r é o percentual de crescimento ou taxa de decaimento, escrito
como um decimal
4) A função f(x) = ax, com 0 < a ≠ 1 é ilimitada superiormente e
a sua imagem é o conjunto dos números reais positivos (+*). b é o fator de crescimento ou multiplicador de crescimento. Como
as potências dos números negativos se comportam estranhamente,
limitamos b a valores positivos.
GRÁFICO
Exemplo:
O gráfico da função exponencial f(x) = ax, com 0 < a ≠ 1, tem as Um certificado de depósito (CD) é um tipo de conta de poupança
seguintes características: oferecida pelos bancos, normalmente oferecendo uma taxa de juros
• está todo acima do eixo Ox; mais elevada em troca de um período de tempo fixo que você vai
deixar o seu dinheiro investido. Se um banco oferece um CD de 24
• corta o eixo Oy no ponto de ordenada 1;
meses com uma taxa de juros anual de 12% ao ano capitalizados
• é crescente para a > 1 e decrescente para 0 < a < 1. mensalmente, para quanto um investimento de $1000 irá crescer
• o eixo x é assíntota do gráfico. após esses 24 meses?
Primeiro, devemos notar que a taxa de juros é uma taxa anual, mas
é capitalizada mensalmente, significando que os juros são calculados
1º caso: a > 1 (função crescente)
e adicionados à conta mensalmente. Para encontrar a taxa de juros
mensal, dividimos a taxa anual de 12% em 12, uma vez que há 12
meses em um ano: 12%/12 = 1%. Cada mês, vamos ganhar 1% de
juros. A partir disso, podemos configurar uma função exponencial,
com a nossa quantidade inicial de $1000 e uma taxa de crescimento
de r = 0,01, e nossa entrada m medido em meses.
m
 0.12
=
f(m) 1000 1+ 
 12 
f(m) 1000(1+ 0.01)m
=
Após 24 meses, a conta terá crescido para f(24) =
1000(1 + 0,01)24 = $ 1269,73

PROMILITARES.COM.BR 85
FUNÇÃO EXPONENCIAL E LOGARÍTMO

TRANSFORMAÇÕES DE GRÁFICOS EXPONENCIAIS


Utilizando f(d) = 100 −100 ⋅ e−0,2d e o gráfico acima, a empresa
Embora as funções exponenciais possam ser transformadas pode prever que o funcionário alcançará a produção de 87 peças
seguindo as mesmas regras que qualquer função, há algumas num mesmo dia, quando d for igual a:
características interessantes de transformações que podem ser a) 5
identificadas. O primeiro que uma reflexão horizontal é equivalente a
b) 10
uma mudança no fator de crescimento.
c) 15
Em seguida, considere o efeito de uma mudança horizontal em
uma função exponencial. Deslocando a função quatro unidades d) 20
para a esquerda. Poderíamos reescrever isso assim: f(x) = 3(2)x,
f(x + 4) = 3(2)x+4, f(x + 4) = 3(2)x+4 = 3(2)x (24) = 48(2)x. Resolução: B
Curiosamente, verifica-se que uma mudança horizontal de uma f(d) = 100 −100⋅e−0,2d = 87 ⇔ e−0,2⋅d = 0,13
função exponencial corresponde com uma alteração no valor inicial No gráfico dado, temos 0,13 = e−2, então
da função.
e−0,2⋅d = e−2 ⇔ −0,2d = −2 ⇔ d = 10
Por fim, considere o efeito de uma mudança vertical em uma
função exponencial. Deslocando para baixo 4 unidades daria a
equação, produzindo o gráfico f(x) = 3(2) x, f(x) = 3(2) x – 4.
LOGARITMO

DEFINIÇÃO
Sejam a e b números reais positivos e a ≠ 1, define-se logaritmo
de b na base a como o expoente x que satisfaz ax = b.
loba b =x ⇔ ax =b
onde b é chamado logaritmando, a é a base e x é o logaritmo.
O logaritmando também é chamado antilogaritmo de x na base
a e indicado por:
=
b antiloga x ⇔= ax b
x loga b ⇔ =
Observe que esse gráfico é substancialmente diferente do Assim, antilog2= 3
3 2= 8.
gráfico exponencial básico. Ao contrário de um exponencial básico,
este gráfico não tem uma assíntota horizontal em y = 0; devido ao Convenciona-se que a omissão da base, escrevendo-se apenas
deslocamento vertical, a assíntota horizontal também mudou para logb, indica que trata-se dos logaritmos decimais cuja base é a = 10.
y = – 4. Podemos ver isso como x → ∞, f(x) → ∞ e como x → – ∞,
f(x) → – 4. CONDIÇÃO DE EXISTÊNCIA
O logaritmo de b na base a somente é definido quando
NÚMERO DE EULER
 1
k
a > 0 e a ≠ 1
e é a letra usada para representar o valor que se 1+  aproxima 
como k fica grande.
 k b > 0

e ≈ 2.718282 Exemplo:
Porque e é muitas vezes usado como a base de um exponencial, Para que valores de x está definido log(x +1) (3 − x) .
mais científico, gráficos e calculadoras têm um botão que pode logaritmando: 3 −x > 0 ⇔ x < 3
calcular potências de e, geralmente rotulados ex.
base: x +1 > 0 ⇔ x > −1
Exercício Resolvido x +1 ≠ 1 ⇔ x ≠ 0
O logaritmo está definido para x ∈ ]−1, 3[ − {0}.
01. (UERJ 1998) Uma empresa acompanha a produção diária de um
funcionário recém-admitido, utilizando uma função f(d), cujo valor
corresponde ao número mínimo de peças que a empresa espera CONSEQUÊNCIAS IMEDIATAS
que ele produza em cada dia (d), a partir da data de sua admissão.
Considere o gráfico auxiliar abaixo, que representa a função y = ex. Sejam a,b,c ∈ *+ e a ≠ 1 e k ∈  , então:
i) loga 1 = 0
ii) loga a = 1
iii) loga ak = k
iv) aloga b = b
v) loga= b loga c ⇔=
b c

Exemplo:
Resolver 2x = 10 para x.
Ao reescrever esta expressão como um logaritmo, obtemos
x = log2 (10) .

86 PROMILITARES.COM.BR
FUNÇÃO EXPONENCIAL E LOGARÍTMO

PROPRIEDADES f(x) = logax


O domínio da função logaritmo é *+ e a imagem é *.+
Sejam a,b,c ∈ *+ e a ≠ 1 e α,β k ∈∈ e n ∈ , n ≥ 2, então:
i) loga (b ⋅ =
c) loga b + loga c
PROPRIEDADES
 b
ii) loga=
  loga b − loga c
c 1) Como f(1) = loga1 = 0, o par ordenado (1, 0) pertence ao
iii) loga (bα ) = α ⋅ loga b gráfico da função exponencial.
1 2) Quando 0  <  a  <  1, a função f(x) = logax é decrescente. Já
iv) log(aβ ) b= ⋅ loga b quando a > 1, a função f(x) = logax é crescente.
β
α 0 < a < 1 : x1 < x 2 ⇒ f ( x1) > f ( x 2 )
v) log(aβ ) (bα=
) ⋅ loga b
β a > 1 : x1 < x 2 ⇒ f ( x1) < f ( x 2 )

3) A função f(x) = logax, com 0 < a ≠ 1 é injetora.


CONSEQUÊNCIAS IMEDIATAS
GRÁFICO
 1
i) loga   = − loga b
 b O gráfico da função logarítmica f(x) = logax, com 0 < a ≠ 1,
ii) log 1 b = − loga b • está todo à direita do eixo Oy;
 a 
• corta o eixo Ox no ponto de abscissa 1;
1
iii) loga n b= ⋅ loga b • é crescente para a > 1 e decrescente para 0 < a < 1.
n
iv) logn a b= n ⋅ loga b • o eixo Oy é assíntota do gráfico.
Os gráficos da função logarítmica estão exemplificados abaixo:
Exemplo:
1º caso: a > 1
Escreva log3 (5) + log3 ( 8) − log3 ( 2) como um único logaritmo.
Usando a soma de logs nos dois primeiros termos,
log3 (5) + log3 (=
8) log3 (5 ⋅=
8) log3 ( 40)

Isso reduz a nossa expressão original para log3 ( 40) − log3 ( 2)


Em seguida, usando a diferença de logs,

) log3  = log3 ( 20)


40
log3 ( 40) − log3 ( 2=
2
A expressão –logab é comumente chamada cologaritmo de b na
base a, o que significa que o cologaritmo é o oposto do logaritmo.
 1
cologa b = − loga b =loga   =log 1 b
 b  
a

MUDANÇA DE BASE
Sejam a, b, c ∈ *+ e a, c ≠ 1, temos:
logc b 2º caso: 0 < a < 1
loga b =
logc a
Calcule log5(100) usando a alteração da fórmula base.
Podemos reescrever essa expressão usando qualquer outra base.
Se nossas calculadoras são capazes de avaliar o logaritmo decimal,
poderíamos reescrever usando o log comum, base 10.
log10 100 2
log5 (100) = ≈ = 2.861
log10 5 0.69897

CONSEQUÊNCIAS

1
i) loga b =
logb a
ii) logc a ⋅ loga b =
logc b
iii) loga b ⋅ logb c ⋅ logc d ⋅  ⋅ logy z =
loga z

FUNÇÃO LOGARITMO A função logarítmica de base a e a função exponencial de mesma


base a são inversas uma da outra.
Seja a ∈ , a ≠ 1, a função logarítmica de base a é a função de
*+
*+ em *+, definida por: f(x) =
y=loga x ⇔ f −1(y) =
ay =
x

PROMILITARES.COM.BR 87
FUNÇÃO EXPONENCIAL E LOGARÍTMO

Nos gráficos podemos notar que a função exponencial e a função In2 = 5r


logarítmica são simétricas em relação à reta y = x. ln2
=r ≈ 0.1386
5
CARACTERÍSTICAS GRÁFICAS DO LOGARITMO
Agora que temos uma equação, podemos N(t) = 120e0,1386t prever
Graficamente, na função g(x) = loga(x) o número de usuários após 12 meses:
O gráfico tem uma intercepção horizontal em (1,0) N(12) = 120e0,1386(12) = 633140 mil usuários.
O gráfico tem uma assíntota vertical em x = 0
EXERCÍCIOS DE

TREINAMENTO
O gráfico está aumentando e côncavo para baixo
O domínio da função é x > 0, ou (0, ∞)

LOGARITMOS DECIMAIS
01. (EFOMM 2010) Sabendo que log303 = a e log305, que opção
Os logaritmos decimais, também conhecidos como logaritmos de representa log102?
Briggs, são aqueles de base a = 10. Nesse caso,
1− a − b 1− a − b 1− a − b
a) c) e)
logb =⇔
x 10x =b 2+ a 1+ a 1− a
Isso permite concluir que log b somente tem como resultado um 1− a − b 1− a − b
número inteiro se b for uma potência de 10. b) d)
a −1 2−a
Qualquer que seja o número positivo b, ele se encontra entre duas
potências inteiras e consecutivas de 10, portanto log  b deve estar
situado entre os logaritmos decimais dessas potências. 02. (ESPCEX (AMAN) 2019) A figura mostra um esboço do gráfico da
função f(x) = ax + b, com a e b reais, a > 0, a ≠ 1 e b ≠ 0. Então, o valor
c c +1 c c +1
10 ≤ b < 10 ⇒ log10 ≤ logb < log10 de f(2) – f(–2) é igual a
⇒ c ≤ logb < c + 1

Isso indica que b é igual à soma de um número inteiro c com uma


parcela m não negativa e menor que 1.
logb = c + m
onde c → característica de log b
m → mantissa de log b (0 ≤ m < 1)
A característica é o maior número inteiro que não supera o
logaritmo.
A mantissa é sempre um número não negativo e menor que 1.
Se dois números só diferem pela posição da vírgula, seus logaritmos
decimais possuem a mesma mantissa. 3 1 35
a) − . c) − . e) − .
4 4 6
LOGARITMOS NEPERIANOS 15
d) 7
b) − . − .
4 6
Os logaritmos neperianos ou logaritmos naturais são os que têm
como base o número irracional e ≅ 2,7182.
03. (IME 2019) O número de soluções reais da equação abaixo é:
Inb = logeb 2
(cos x)2018= 2 − 2(x π )
Da definição temos que Ine = 1.
O número é também pode ser definido como: a) 0 d) 3

 1
n b) 1 e) 4
=e lim 1+ 
n→∞  n c) 2
ou
04. (IME 2017) Seja a equação ylog3 3y = ylog3 3y − 6, y > 0.
1 1 1 1
e = 1+ 1+ + + ++ + O produto das raízes reais desta equação é igual a:
2 6 24 n!
1
Exemplo: a) d) 2
3
O uso de um novo site de redes sociais vem crescendo 1
exponencialmente, com o número de novos membros dobrando a b) e) 3
2
cada 5 meses. Se o site atualmente tem 120.000 usuários e esta
tendência continua, quantos usuários o site terá em 1 ano? 3
c)
Se usarmos uma equação de crescimento contínuo, seria 4
N(t) = 120ert, medido em milhares de usuários após t meses. Com 05. (ITA 2016) Se x é um número natural com 2015 dígitos, então o
base no tempo de duplicação, haveria 240.000 usuários após 5 meses.
número de dígitos da parte inteira de 7 x é igual a
Isso nos permite resolver para a taxa de crescimento contínuo:
a) 285. d) 288.
240 = 120er5
b) 286. e) 289.
2 = er5
c) 287.

88 PROMILITARES.COM.BR
FUNÇÃO EXPONENCIAL E LOGARÍTMO

06. (ITA 2016) Seja (a1, a2, a3, ...) a sequência definida da seguinte 1 1 1
forma: a1 = 1000 e an = log10(1 + an–1) para n ≥ 2. Considere as a) c) e)
4 16 64
afirmações a seguir:
I. A sequência (an) é decrescente. 1 1
b) d)
8 32
II. an > 0 para todo n ≥ 1.
III. an < 1 para todo n ≥ 3.
10. Na calculadora obtiveram-se os resultados seguintes:
É (são) verdadeira(s) log 6 = 0,778 e ln 6 = 1,791. Com estes dados, sem ajuda da
a) apenas I. d) I, II e III. calculadora, é verdade que log e, com aproximação de três casas
b) apenas I e II. e) apenas III. decimais, é: (Notação log6 = log106 e In6 = loge6)
c) apenas II e III. a) 0,434 c) 0,791 e) 1,791
b) 0,778 d) 1,778
07. (ITA 2016) Considere as seguintes afirmações:
x
x − 1 11. (IME 2014) Sabe-se y ⋅ z ⋅ z ⋅ x = x ⋅ y 3 ⋅ z2 =
I. A função f(x) = log10  é estritamente crescente no intervalo = e , em
 x  z⋅ y⋅z
]1, +∞[. que e é a base dos logaritmos naturais. O valor de x + y + z é
II. A equação 2x + 2 = 3x −1 possui uma única solução real. a) e³ + e² + 1 d) e³ + e–2 + e
III. A equação (x + 1)x = x admite pelo menos uma solução real b) e² + e–1 + e e) e³ + e–2 + e–1
positiva. c) e³ + 1
É (são) verdadeira(s)
a) apenas I. d) I, II e III. 3
+ (log3 x ) =
2
12. (IME 2013) Considere a equação log3x 1. A soma
b) apenas I e II. e) apenas III. x
dos quadrados das soluções reais dessa equação está contida no
c) apenas II e III. intervalo
a) [0, 5) c) [10, 15) e) [20, ∞)
08. (UERJ 2005) Um pesquisador, interessado em estudar uma
determinada espécie de cobras, verificou que, numa amostra de b) [5, 10) d) [15, 20)
trezentas cobras, suas massas M, em gramas, eram proporcionais ao
cubo de seus comprimentos L, em metros, ou seja M = a . L³, em que 13. (FUVEST 2005) Os pontos D e E pertencem ao gráfico da função
a é uma constante positiva. Observe os gráficos abaixo. y = log a x com a > 1 (figura abaixo). Suponha que B = (x, 0),
C = (x +1, 0) e A = (x −1, 0). Então, o valor de x para o qual a área do
trapézio BCDE é o triplo da área do triângulo ABE, é:

1+ 5 1 1
a) c) + 5 e) +2 5
2 2 2

5 d) 1+ 5
b) 1+
Aquele que melhor representa log M em função de log L é o 2
indicado pelo número:
a) I c) III 14. (IME 2010) Seja f(x) =
| 3 − log(x) |, x ∈ . Sendo n um número inteiro
b) II d) IV
n−3
positivo, a desigualdade f(x) + 2f(x) + 4f(x) + … + 2 f(x) + … ≤ 9
09. (IME 2018) Determine o valor de a na expressão abaixo, sabendo- 4 12 36 3n −1 4
se que 0 < a < 1,
somente é possível se:
co log 65 256
(a(2 ) )
log 4 256 Obs.: log representa a função logarítmica na base 10.
1 colog 2 256 (a )
loga 256 (a )
= Im {Z} a) 0 ≤ x ≤ 106
16
onde Z é um número complexo que satisfaz a equação: b) 10–6 ≤ x ≤ 108
c) 103 ≤ x ≤ 106
24.033 Z2 − 22.017 Z + 1 =0.
d) 100 ≤ x ≤ 106
Obs.: Im(Z) é a parte imaginária do número complexo Z.
e) 10–6 ≤ x ≤ 106

PROMILITARES.COM.BR 89
FUNÇÃO EXPONENCIAL E LOGARÍTMO

15. (OBM 1999) As representações decimais dos números 21999 e 51999 =


03. (ITA 1992) O domínio da função f(x) log2x2 − 3x +1(3x 2 − 5x + 2) é:
são escritas lado a lado. O número de algarismos escritos é igual a :
a) (-∞,0)∪(0,1/2)∪(1,3/2)∪(3/2,+∞)
a) 1999 c) 2001 e) 3999
b) (-∞,1/2)∪(1,5/2)∪(5/2,+∞)
b) 2000 d) 3998
c) (-∞,1/2)∪(1/2,3/2)∪(1,3/2)∪(3/2,+∞)
16. (IME 2008) Determine os valores de K para os quais o sistema de d) (-∞,0)∪(1,+∞)
ex + ey = ex + y e) nda
equações dado por:  , admite solução real.
x + y =K
04. (ITA 1999) Seja a ∈ R, a > 1. Se b = log 2 a, então o valor de

17. (UFRJ 2000) A figura a seguir mostra os gráficos das funções f e g, a a2 − 1


log4 a3 + log2 4a + log2 + (log8 a)2 − log 1 é:
definidas no intervalo ]0, 4] por: a+1 2
a −1
x x a) 2b – 3
f(x) = − ln x e g(x) = − (ln x)2 2b2 + 63b + 36
2 2 d)
18
onde ln expressa o logaritmo na base neperiana e (e ≅ 2,7). 65
b) b+2 b2 + 9b + 7
18 e)
9
2b2 − 3b + 1
c)
2
1 1
05. (ITA 1989) Sobre a expressão
= M + , onde 2 < x < 3,
log2 x log5 x
qual das afirmações abaixo está correta?
a) 1 ≤ M ≤ 2 c) 4≤M≤5 e) 7 ≤ M ≤ 10
b) 2 < M < 4 d) 5 < M < 7
Sejam M, N os pontos de interseção dos dois gráficos e P, Q suas
respectivas projeções sobre o eixo x. Determine a área do trapézio MNQP. 06. (IME 2012) Se log10 2 = x e log10 3 = y, então log518 vale:
x + 2y c) 2x + y 3x + 2y
a) e)
3x − 3− x 1− x 1+ x 1− x
18. (ITA 2010) Analise se a função f :  → , f(x) = é
2 x+y x + 2y
b) d)
bijetora e, em caso afirmativo, determine a função inversa f–1. 1− x 1+ x

19. (ITA 2008) Para x ∈ , Determine o conjunto solução de 07. (ITA 1987) Considere u = x ⋅ ln(3), v = x ⋅ ln(2) e eu ⋅ ev = 36. Nessas
condições, temos:
53x − 52x +1 + 4 . 5x =5x − 1
a) x = −4 c) x = −3 e) x = 2
b) x = 12 d) x = 9
20. Considere =  1  1 
b log  x + − 1 , com x  >  1.
a log  x −  e =
 x  x  08. (IME 2016) Sabendo-se que os números reais positivos a, b e c
Determine log  x 2 − x + 1 − 1  em função de a e b. formam uma progressão geométrica e log  5c  , log  3b  e log  a 
 x x2   a  5c   3b 
formam uma progressão aritmética, ambas nessa ordem, então se

EXERCÍCIOS DE pode afirmar que a, b e c

COMBATE a) formam os lados de um triângulo obtusângulo.


b) formam os lados de um triângulo acutângulo não equilátero.
c) formam os lados de um triângulo equilátero.
d) formam os lados de um triângulo retângulo.
01. (ITA 2007) Sejam x, y e z números reais positivos tais que seus
logaritmos numa dada base k são números primos satisfazendo e) não podem formar os lados de um triângulo.

logk ( xy ) = 49
09. (IME 2012) Sendo =α log12
= 18 e β log24 54 , o valor de
 x α ⋅ β + 5 ⋅ ( α − β ) é:
logk   = 44
 z
a) 0 c) 2 e) 5
Então, logk ( xyz) é igual a: b) 1 d) 3
a) 52 b) 61 c) 67 d) 80 e) 97
10. (ITA 2018) Se=
log2 π a e=
log5 π b, então
02. (ITA 1988) Considere = ( )
A(x) log 1 2x 2 + 4x + 3 , ∀x ∈ R. Então:
a) 1 1 1 3 1 1
2 + ≤ . d) < + ≤ 2.
a) A(x) > 1, para algum x ∈ R, x > 1. a b 2 2 a b
b) A(x) = 1, para algum x ∈ R. 1 1 1 1 1
b) + ≤ . e) 2< + .
c) A(x) < 1, apenas para x ∈ R tal que 0 < x < 1. a b 2 a b
d) A(x) > 1, para cada x ∈ R tal que 0 < x < 1. 1 1 3
c) 1< + ≤ .
e) A(x) < 1, para cada x ∈ R. a b 2

90 PROMILITARES.COM.BR
FUNÇÃO EXPONENCIAL E LOGARÍTMO

11. (IME 2005) Sejam a, b, c e d números reais positivos e diferentes 03. D


de 1. Sabendo que logad, logbd e logcd são termos consecutivos de  x
2
 
uma progressão aritmética, demonstre que: c2 = (ac)loga b . Façamos f ( x )= ( cos x )2018 e g ( x )= 2 − 2 π  .
Note que:
f(x) logx +1(x 2 − 2x − 8).
12. (ITA 2016) Seja f a função definida por =
f ( 0) = ( cos 0)
2018
Determine:
a) O domínio Df da função f. f ( 0) = 1
b) O conjunto de todos os valores de x ∈ Df tais que f(x) = 2. e
c) O conjunto de todos os valores de x ∈ Df tais que f(x) > 1.
f ( 0) = ( cos 0)
2018

4 f ( 0) = 1
log1/ 2 n 32
13. (ITA 2014) Determine o valor da soma ∑ log 8n+ 2  x
 
2

Logo, é uma solução da equação ( cos x )


2018
= 2 − 2 π  .
1 1/ 2

14. (ITA 2014) Determine as soluções reais da equação em x,

( ) log 16x 2
(log4 x )3 − log4 x 4 − 3 10 =
0.
 x
 
log100 16 Seja h :  → , h ( x ) =
f ( x ) − g ( x ) , ou seja,
= h( x) ( cos x ) 2018
− 2 + 2 π  .
É possível mostrar que h é uma função contínua em todo seu domínio.
15. (ITA 2012) Determine os valores de θ ∈ [ 0,2π ] tais que Note que:
( )
logtg(θ) esen θ ≥ 0 .  −x
2

( cos ( − x ))
 
)
h ( − x=
2018
− 2 + 2 π

 1
log
5
(
1 2
x – x +19 )  x
 
2

16. (ITA 2011) Resolva a inequação em  : 16 <  


 4
. h ( −x)
= ( cos x )2018 − 2 + 2 π 
17. (EN 2011) Uma progressão geométrica infinita tem o 4º termo h(–x) = h(x) ou seja, h é uma função par.
igual a 5. O logaritmo na base 5 do produto de seus 10 primeiros Note que:
termos vale 10 – 15log52. Se S é a soma desta progressão, determine  π
2

o valor de log2S.  
)
h ( π= ( cos π )2018 − 2 + 2 π 
h ( π) = 1 > 0
18. (IME 2011) Sejam a e b números reais e f(x) = a ⋅ sen x + b ⋅ 3 x + 4. Se
2
f(loglog310) = 5, qual o valor de f(loglog3)? 2018  π 1
 π  π  ⋅ 
=
h    cos  − 2 + 2 2 π 
 2  2
19. (IME 1995) Considerando log2 = a e log3 = b, encontre, em 1
função de a e b o logaritmo do número 5 11,25 no sistema de base  π
h   =−2 + 2 4 < 0
15.  2

π
ax − a− x Pelo Teorema de Bolzano, existe < α < π, tal que h(α) = 0.
20. (ITA 2006) Considere a equação x = m , na variável real x, 2
a + a− x Como h é uma função par, h(– α) = 0.
como 0 < a ≠ 1 . Determine o conjunto de todos os valores de m para  x
2
 
os quais esta equação admite solução real é? É possível mostrar que a equação ( cos x ) 2018
= 2−2 π admite
somente essas três raízes.
GABARITO
EXERCÍCIOS DE TREINAMENTO 04. A

01. E ylog
=3 3y
ylog3 3y − 6

(y ) ( )2 →=
2
log3 log3 a =a ylog3 3y
a2
→= log3 3y
= y 2log3 =
3y
y
log3 3y
a2 ylog3 3y
log30 3 = a ⇔ a = ⇔a= ⇔ log3 =
log30 1+ log3 1− a
a=3
log5 1 − log2 a = a2 − 6 → a2 − a − 6 = 0 →
log30 5 =b ⇔ b = ⇔b= ⇔ log2 =1 − b − b ⋅ log3 a=−2 (não comvém, pois y, a > 0)
log30 1+ log3
log3 3y
=a y= 3
a 1 − a − b + ab − ab 1 − a − b
⇒ log2 = 1 − b − b ⋅ = = fazendo y = 3x
1− a 1− a 1− a
log3 3y = 12 ⋅ (log3 3y ) = 12 ⋅ (1+ log3 y ) = 12 ⋅ (1+ x )

(3 )
1 ⋅(1+ x )
log3 3y
02. B =31 y= x 2

f(0) = 3 ⇒ a0 + b = 3 ⇒ b = 3 − 1 ⇒ b = 2. x =1 → y = 3
1 x ⋅ (1+ x ) = 1 → x 2 + x − 2= 0 → ou
f( −2) = 6 ⇒ a−2 + 2 = 6 ⇒ a−2 = 4 ⇒ a = 2
2 x =−2 → y =19
x
 1
∴ f(x) =   + 2 e : 1 1
 2 Produto → 3 ⋅ =
9 3
−2
 1
2
  1  1 15
f(2) − f( −2) =  + 2 −    + 2  = − 4 =−
 2   2  4 4
05. D
Se x é um número natural com 2015 dígitos, então:

PROMILITARES.COM.BR 91
FUNÇÃO EXPONENCIAL E LOGARÍTMO

102014 ≤ x < 102015 ⇒ 7 102014 ≤ 7 x < 7 102015 De 24033 Z2 − 22017 Z + 1 =0,


Sabendo que: 2 ⋅ 24033 Z2 − 2 ⋅ 22017 Z + 2 =0

(2 )
2014 2
2017
7
=
102014
10 7 ≅ 10287,7 Z − 2 ⋅ 22017 Z ⋅ 1+ 12 + 1 =0

(2 )
2015 2017 2
7 2015 Z −1 =
−1
=
10 10 7 ≅ 10287,8
22017 Z − 1 =i ou 22017 Z − 1 =−i
10287 < 7 x < 10288
De 22017 Z − 1 = i,
7
Logo , x terá 288 algarismos. 1 1
=Z + i (não convém, pois Im{Z} < 0.
22017 22017
06. D De 22017 Z − 1 = i,
a1 = 1000 1 1
=Z − i
a2 = log10 (1+ 1000) = 3, 22017 22017
a3 = log10 (1+ 3,) = 0, Então,
a4 = log10 (1+ 0,) = 0,
1 1
⋅ (loga 256) =
66
 − − 2017
22149 2
an = log10 (1+ 0,) = 0,
(loga 256)66 = 2132
Portanto, a alternativa [D] é a correta.
Como 0 < a < 1, loga256 < 0.

07. B Daí,
66
[I] Verdadeira. Para x1 e x2 pertencentes ao intervalo ]1, + ∞ [ e x1 > x2. loga 256 = − 2132
1 1 1 1 1 1 loga 256 = −4
x1 > x 2 ⇒ < ⇒− >− ⇒ 1− > 1− ⇒
x1 x 2 x1 x2 x1 x2 a−4 = 256
x1 − 1 x 2 − 1  x − 1  x − 1 1
⇒ > ⇒ log10  1  > log10  2  = 256
x1 x2  x1   x2  a4
1
Portanto a função é crescente para todo x real maior que 1. a4 =
256
[II] Verdadeira. 1
a=
3x  3
x 4
2x ⋅ 22 = ⇒   = 12 ⇒ x = log3 12,
3  2
2

Portanto, a equação tem apenas uma solução real. 10. A


1
[III] Falsa. log6 e loge 6 log6
1 =
log10 e = =
Se x =1 ⇒ 1+ 1 =21 =2, portanto 1 não é raiz da equação. log6 10 1 ln6
 1 log6
x x x
Se x > 1 ⇒ x + 1 > x ⇒ (x + 1) > x > x ⇒ (x + 1) > x 0,778
⇒ loge ≅ ≅ 0, 434
Se 0 < x < 1 ⇒ x + 1 > 1 ⇒ x + 1 > 1 ⇒ (x + 1)x > 1 > x 1,791
Portanto, a equação não admite nenhuma raiz real positiva.
11. B
08. C Sabendo que ln e = 1, temos:
M = a ⋅ L3 ⇒ logM =log(a ⋅ L3 ) 
  3 1

⇒ log M = log a + 3 ⋅ log L ln  y ⋅ z 2 ⋅ x 4  =
1 1 3
Como log a é constante, o gráfico de log M em função de log L é uma
    4 ⋅ ln x + ln y + 2 ⋅ ln z =1
 3 2 
reta crescente, que aparece no gráfico III.  ln(x ⋅ y ⋅ z ) = 1 ⇒  ln x + 3ln y + 2ln z = 1 ⇒
  1 3
  x   ln x − ln y − ln z = 1
09. A  ln  1 3  = 1  2 2
co log 65 256   2 2 
log 4 256
(a(2 ) )   y ⋅ z 
De Im {Z} = 1 loga 256
colog 2 256 (a )
(a )
, ln x + 4 ln y + 6ln
= z 4  ln x + 4 ln y + 6ln
= z 4
16  
⇒  ln x + 3ln y + 2ln z =1 ⇒  0 − ln y − 4 ln z = −3 ⇒
1
Im { Z} = ⋅ colog 265  256 ⋅ log 24 256 ⋅ colog 263  256 ⋅ ... ⋅ loga 256  2ln x − ln y − 3ln z = 0 − 9ln y − 15ln z = −6
16  a  (a )  a   2 
1  1  1  1  1 ln x + 4 ln y + 6ln z = 4
 1 1
Im { Z}= ⋅ ... ⋅  − 1  ⋅ 0 ⋅ (loga 256) 
66
⋅− ⋅ ⋅− ⋅ ⇒  0 − ln y − 4 ln z = −3
16  265  264  263  262  2 2
1 −1  0 + 0 + 21ln z = 21
Im { Z} = ⋅ ⋅ (loga 256)
66 
16 265+ 64 + 63+...+1
−1 Portanto,
Im { Z} =⋅ (loga 256)66 < 0, pois (loga 256)66 > 0.
22149

92 PROMILITARES.COM.BR
FUNÇÃO EXPONENCIAL E LOGARÍTMO

ln z = 1 ⇒ z = e 16.

ln y =−1 ⇒ y =e −1 1a SOLUÇÃO:

ln x = 2 ⇒ x = e2 ex + ey = ex + y (1)

2 −1 x + y =K (2)
Logo, x + y + z = e + e + e.
Pela equação (2): y = K – x, substituindo-se na (1) tem-se:
12. C ⇔ e2x − eK ⋅ ex + eK =
0 (4)

Sabendo que logb a = logc a , com a, b e c reais positivos e b, c ≠ 1 vem x ∈  ⇔ ex ∈ *+


logc b
2 K K
3 Logo, fazendo z = ex e substituindo em (4): z − e ⋅ z + e =0
3 log3
2
log3x + (log3 x) = 1⇔ x + (log x)2 = 1. S= P= eK > 0
3
x log3 3x
eK > 0

 m
( )
∆ ≥ 0 ⇔ e2K − 4 ⋅ eK ≥ 0 ⇔ eK ⋅ eK − 4 ≥ 0 ⇔
Daí, como logp (m ⋅=n) logp m + logp n e logp= logp m − logp n,
 n  eK − 4 ≥ 0 ⇔ K ≥  n 4
sendo m, n e p reais positivos e p ≠ 1, temos 2a SOLUÇÃO:
log3 x − 1 ex + ey
− + (log3 x)2 =
1. Usando M.A. ≥ M.G.: ≥ ex ⋅ ey
log3 x + 1 2
ex + y
Fazendo y = log3x, segue que ⇔ ≥ ex + y ⇔ ex + y ≥ 4 ⇔ eK ≥ 4 ⇔ K ≥ n4
2
y −1  1 
(y − 1)(y + 1) − = 0 ⇔ (y − 1)  y + 1 − = 0
y +1  y + 1
2
⇔ y(y − 1)(y + 2) = 0 17. (e − 1)
⇔ y =0 ou y = 1 ou y =−2. 4
x x ln x = 0 ⇔ x = 1
f(x) = g(x) ⇔ − ln x = − (ln x)2 ⇔ (lnx)2 = lnx ⇔ 
Desse modo, as raízes reais da equação dada são x = 1, x = 3 e x = 1 2 2 ln x =1 ⇔ x = e
2 9
 1 1 e
e, portanto, o resultado pedido é 12 + 32 +   = 10 + ∈ [10, 15[. Como f(1) = 1/2 e f(e) = −1
 9 81 2
e
⇒ P(1, 0), Q(e, 0), M(1, 1/2) e N(e, − 1)
2
13. A
área do trapézio = 1 ⋅  1 + e − 1 ⋅ (e − 1) =(e − 1)2
SBCDE = 3SABE ⇒ loga x + loga (x + 1) ⋅ 1 = 3 ⋅ 1⋅ loga x 2 2 2  4
2 2
1± 5
⇒ 2⋅logax = loga (x +1) ⇒ x = x +1 ⇒ x =
2

2 18. Para provar que f é bijetora, vamos provar que é injetora e


1+ 5 sobrejetora.
x>0⇒ x=
2 f é injetora ⇔ (f(x1) = f(x2) ⇒ x1 = x2, ∀ x1, x2 ∈ ).

14. D Suponha por absurdo que

Temos ( ∃ x1, x2 ∈ , x1 ≠ x2 e f ( x1) = f ( x2 )) ≡


f(x) 2f(x) 4f(x) 2n − 3 f(x) 9  3x1 – 3− x1 3x2 – 3− x2 
+ + +…+ +…≤ ⇔ ≡  ∃ x1, x 2 ∈ , x1 ≠ x 2 e = .
4 12 36 3n −1 4  2 2 
.
−x −x
 2 4  x x
Sem perda de generalidade x1 > x2, então 3 1 > 3 2 e – 3 1 > −3 2 ,
| f(x) | 1+ + +  ≤ 9 ⇔
 3 9  logo 3x1 – 3− x1 > 3x2 – 3− x2 é uma contradição, assim x1 = x2, e f é
1 injetora.
| f(x) | ≤9⇔
2 f é sobrejetora ⇔ ∀y ∈ , ∃x ∈ ; f(x) = y.
1−
3 Do passo anterior f é crescente, além disso é contínua, pois é a soma
| f(x) | ≤ 3 ⇔ de duas funções exponenciais. Como lim f(x) = – ∞ e lim f(x) = ∞,
| 3 − log(x) | ≤ 3 ⇔ x→–∞ x →∞
pelo Teorema do Valor Intermediário ∀y ∈ , ∃x ∈ ; f(x) = y, logo f
−3 ≤ 3 − log(x) ≤ 3 ⇔
é sobrejetora.
0 ≤ log(x) ≤ 6 ⇔
Usando f(f–1(x)) = x, vem:
100 ≤ x ≤ 106. −1 −1
3f ( x ) – 3− f ( x ) 2
x ⇒ 3f ( x )  – 2x 3f ( x ) – 1 =
−1 −1
= 0.
2  
15. B
2x ± 4x 2 + 4 −1
3f ( x ) > 0,
−1
Vamos utilizar log 2 = 0,3010 e log 5 = 1 – log 2 = 0,6990 Daí, 3f ( x ) = =±
x x 2 + 1, como
2
nº alg. 21999 = [log 21999] +1 = [1999 ⋅ log 2] + 1 = 602 −1
3f ( x ) =+
x x 2 + 1, daí f −1 ( x )= log3 (x + x 2 + 1) .
nº alg. 51999 = [log51999] + 1 = [1999 ⋅l og 5] + 1 = 1398
total de alg. = 602 + 1398 = 2000

PROMILITARES.COM.BR 93
FUNÇÃO EXPONENCIAL E LOGARÍTMO

19.
ANOTAÇÕES
( ) ( ) ( )
3 2
53x − 52x +1 + 4 ⋅ 5x = 5x − 1 ⇔ 5x − 5 5x + 4 5x = 5x − 1

Adotando y = 5x, temos:


y 3 − 5y 2 + 4y = y − 1 ⇔ y ( y − 1)( y − 4 ) = y − 1 ⇔
⇔ y ⋅ y −1 ⋅ y − 4 = y −1 ⇔
⇔ y − 1 ( y ⋅ y − 4 − 1) = 0 ⇔ y − 1 = 0 ou y ⋅ y − 4 = 1

⇔ y = 1ou y 2 − 4y = ±1 ⇔ y = 1ou y = 2 ± 5 ou y = 2 ± 3

Como y = 5x > 0, isto equivale a y =


1ou y =
2 + 5 ou y =
2± 3 .

=
Retornando a substituição, temos x = 0 ou x log5 2 + 5 ( ) ou
=x log5 2 ± 3 .( )
20. a + b
1 1 1  1
x2 − x + − = x2 − 2 −  x −  =
x x2 x  x
 1  1  1  1  1 
=  x +  ⋅  x −  −  x −  =  x −  ⋅  x + − 1
 x  x  x  x  x 
 1 1  1  1 
log  x 2 − x + − =  log  x −  ⋅  x + −=
1
 x x2   x x  
 1  1 
= log  x −  + log  x + − 1 = a + b
 x  x 

EXERCÍCIOS DE COMBATE
01. A 06. A
02. E 07. E
03. A 08. E
04. D 09. B
05. B 10. E

11. c2 = acloga b
12.
a) =
D ] 4, + ∞[.
b) S = φ.
 
=c) S  3 + 3 5 , +∞ 
 2 

13. 17
18
14. 2 + log4x

 π π   5π 
=
15. S  ,  ∪  π, 
4 2  4 

16. S = ]−∞, −2[ ∪ ]3, +∞[


a1 40
=
17. S = = 80 e =
log2 S log = 2 80
1− q 1− 1
2 = log 2
( 24
=⋅ 5)
= 4 + log2 5
18. 3

19. 1 ⋅ 2b + 1 − 3a
5 b + 1− a
20. m ∈ (–1, 1)

94 PROMILITARES.COM.BR
FUNÇÃO MODULAR
E DESIGUALDADES

DEFINIÇÃO DE MÓDULO Gráfico de f(x) = |x + 1|

 x, se x ≥ 0 y
x =
 − x, se x < 0

1
+
x
=
x)
g(
Exemplos: |1| = 1, |−1| = 1 e |0| = 0.
g(x) > 0

PROPRIEDADES DO MÓDULO x

a) |x| ≥ 0 g(x) < 0

b) |–x| = |x|
c) –|x| ≤ x ≤ |x| y

d) |x| . |y| = |x . y||

1
+
x
f(x

)=
x2 = x

)=
e)

f(x
–x

f) |x|² = x²

1
g) |x + y| ≤ |x| + |y| (desigualdade triangular) x

h) |x| – |y| ≤ |x + y|
i) |x| – |y| ≤ |x – y|

FUNÇÃO MODULAR Outra maneira de traçar o gráfico de uma função modular


composta é escrevendo-a sob a forma de uma função definida por
É a função de  em + que associa a cada x o elemento seu
várias sentenças abertas
módulo ou valor absoluto |x|.
f(x) = |x| x + 1, se x + 1 ≥ 0 ⇔ x ≥ −1
f ( x) = x + 1 = 
=
f(x) x se x ≥ 0  − x − 1, se x + 1 < 0 ⇔ x < −1

f(x) =
− x se x < 0
Ou seja, no intervalo (–∞,–1) o gráfico de f(x) é representado pelo
Im = + gráfico de f1(x) = –x – 1 e no intervalo [–1, + ∞ ), pelo gráfico de
Na reta real, o módulo de x é a distância de x até a origem 0. f2(x) = x + 1.
O gráfico da função modular é a união de duas semi-retas com
origem na origem do plano cartesiano e que são bissetrizes do 1º e EQUAÇÕES MODULARES
2º quadrantes.
As equações modulares podem ser resolvidas utilizando as
A imagem da função é Im = +, isto é, a função assume somente seguintes propriedades.
valores reais não negativos.
a ≥ 0 ⇒ (|x| = a ⇔ x = a ou x = −a)
y
|x| =|y| ⇔ x = y ou x = −y
Exemplos:
1) Resolva a equação |x − 2| = 6
x − 2 = 6 ⇔ x = 8

x − 2 = 6 ⇔ ou S = {8, −4}
x x − 2 =−6 ⇔ x =−4

2) Resolva a equação |x − 3| = |4x −1|


 2
O gráfico das funções modulares compostas pode ser obtido x − 3 =4x − 1 ⇔ x =− 3
traçando-se o gráfico da função original e espelhando-se a parte 
negativa em relação ao eixo x. x − 3= 4x − 1 ⇔ ou
 4
x − 3 =−(4x − 1) ⇔ x =
 5
S = {−2/3, 4/5}

PROMILITARES.COM.BR 95
FUNÇÃO MODULAR E DESIGUALDADES

3) Resolva a equação |3x + 9| = 1 − x A solução S2 é obtida da interseção entre x < −1 e x ≥ 8, logo


Para que a equação tenha solução é necessário que 1 −x ≥ 0, isto S2 = ∅.
é, x ≤ 1. Supondo está condição válida, tem-se: A solução da inequação é S = S1 ∪ S2 = [2, +∞)
3x + 9 =1 − x ⇔ x =−2 Assim como para as equações, as inequações modulares que
 apresentam soma ou subtração de módulos normalmente necessitam
3x + 9 = 1 − x ⇔ ou
3x + 9 =−(1 − x) ⇔ x =−5 que seja feito um estudo de sinal para a sua solução.

Exemplo:
Como os valores obtidos satisfazem a condição inicial ambos são Resolva a inequação |3x − 12| + |5 − x| < 12.
soluções: S = {−2, −5}
Para x ≤ 4, temos |3x −12| = −3x +12 e |5 − x| = 5 − x, então
As equações modulares que apresentam soma ou subtração de −3x +12 +5 −x < 12 ⇔ x > 5/4
módulos necessitam que seja realizado um estudo de sinal para a sua S1 = ]5/4, 4]
solução.
Para 4 ≤ x ≤ 5, temos |3x − 12| = 3x − 12 e |5 − x| = 5 − x, então
Exemplo:
3x −12 +5 −x < 12 ⇔ x < 19/2
Resolva a equação |x − 1| + |x + 6| = 13.
S2 = [4, 5]
Em primeiro lugar deve ser feito os estudos de sinais de x −1 e x + 6
Para x ≥ 5, |3x −12| = 3x −12 e |5 − x| = −5 + x, então
x−1 x+6
3x −12 −5 +x < 12 ⇔ x < 29/4
1 –6
S3 = [5, 29/4[
– + – +
S = S1 ∪ S2 ∪ S3 = ]5/4, 29/4[
Para x ≤ −6 temos x −1 ≤ 0 e x +6 ≤ 0 então
−x +1 −x −6 = 13 ⇔ x = −9
Como x = −9 satisfaz a condição x ≤ −6, é solução
PRINCIPAIS PROPRIEDADES DAS
Para −6 ≤ x ≤ 1 temos x + 6 ≥ 0 e x −1 < 0 então
DESIGUALDADES EM 
−x +1 +x +6 = 13 ⇔ 7 = 13 (absurdo) (1) a < b ⇔ a + c < b + c
Logo, não há solução no intervalo [−6, 1] 1 1
 a > b
se ab > 0
Para x ≥ 1 temos x + 6 ≥ 0 e x −1 ≥ 0 então e a<b⇔ 
1 < 1
se ab < 0
x −1 + x +6 = 13 ⇔ x = 4  a b
Como x = 4 satisfaz a condição x ≥ 1, é solução.
ac < bc se c > 0
( 2) a < b ⇔ 
S = {−9, 4} ac > bc se c < 0

( 3) m ∈ • e a < b ⇔
INEQUAÇÕES MODULARES  am < bm se m impar
Para a solução de inequações modulares é necessária a utilização  m m *
das seguintes propriedades dos módulos, onde a ≥ 0. a < b , ∀a,b ∈  + ou
 m m *
|x| < a ⇔ −a < x < a a > b , ∀a,b ∈  − se m par

-a a x ( 4) 0 < a < 1 ⇒ am < a e a > 1 ⇒ am > a


|x| > a ⇔ x < −a ou x > a ( 5) a < b ⇔ a < b, ∀a,b ∈ *+
-a a x ( 6) 0 < a < 1 ⇔ a < a < 1 e a > 1⇔ a < a
Note ainda que se a < 0, então |x| ≥ a, ∀x e |x| < a é sempre falso. a < a'
Exemplos: b < b'
1) Resolva a inequação |x + 3| ≤ 5. ( 7)  ⇒ a + b +  + l < a'+ b'+  + l'
 
|x + 3| ≤ 5 ⇔ −5 ≤ x +3 ≤ 5 ⇔ −8 ≤ x ≤ 2 l < l' 
S = [−8, 2]

2) Resolva a inequação |4x − 3| > 5. SINAL DO BINÔMIO DO PRIMEIRO


|4x − 3| > 5 ⇔ 4x −3 < −5 ou 4x −3 > 5 ⇔ GRAU
⇔ x < −1/2 ou x > 2
f(x) = ax + b, (a ≠ 0)
S = {x ∈ R | x < −1/2 ou x > 2}
Temos sucessivamente
3) Resolver em R a inequação 2x −7 + |x + 1| ≥ 0.  b
f ( x ) = ax + b = a  x +  = a ( x − x 0 )
Se x ≥ −1, |x + 1| = x + 1 então  a
2x − 7 + x + 1 ≥ 0 ⇔ x ≥ 2
b
A solução S1 é obtida da interseção entre x ≥ −1 e x ≥ 2, logo onde x 0 = − é a raiz do binômio, o que dá origem ao seguinte
a
S1 = [2, +∞)
teorema:
Se x < −1, |x + 1| = −x −1 então
“O binômio do primeiro grau ax + b possui o sinal de seu primeiro
2x − 7 − x − 1 ≥ 0 ⇔ x ≥ 8 coeficiente, a, para valores de x superiores à sua raiz e possui o sinal
de –a para valores de x inferiores à sua raiz.”

96 PROMILITARES.COM.BR
FUNÇÃO MODULAR E DESIGUALDADES

SINAL DO TRINÔMIO DO SEGUNDO 03. Assinale a alternativa que apresenta o valor mínimo da função
ϕ(x) = x − a + x − b + x − c + x − d , onde a < b < c < d são números
GRAU reais fixo e x assume valores reais arbitrários.
f(x) = ax² + bx + c, (a ≠ 0) a) a + b + c + d d) d + c −a −b
Consideremos a função quadrática f(x) = ax² + bx + c, (a ≠ 0) b) –a −b −c −d e) a −b −c −d
então, podemos escrever sucessivamente:
c) a + b −c −d
 b c
f ( x )= ax 2 + bx + c ⇔ f ( x )= a  x 2 + x +  ⇔
 a a 04. Se a, b, c são números reais menores que 1 e a + b + c = 2 o valor
 mínimo de
4ac − b2 
2
b
⇔ f ( x ) = a  x +  + ⇔ abc
 2a  4a2 

(1− a)(1− b)(1− c )
 b
2
b2 − 4ac 
⇔ f ( x ) = a  x +  −  é igual a:
 2a  4a2 

a) 3 b) 4 c) 5 d) 6 e) 8
Fazendo D = b² – 4ac temos
 05. Se x, y, z são números reais positivos, cuja soma vale 1, o valor
∆ 
2
b
f ( x ) = a  x +  − 2  mínimo de 1 + 4 + 9 é igual a:
 2a  4a 
 x y z
chamada a forma canônica da função quadrática a) 14 b) 15 c) 16 d) 24 e) 36

06. O número de soluções da equação


Daí tem-se que:
x 2 − x − 1 − 2 − 3 − 4 − 5 = x 2 + x − 30 é:
I. Quando ∆ < 0, a expressão entre colchetes é sempre positiva
e, neste caso, como a função quadrática é igual ao produto a) 0 b) 1 c) 2 d) 8 e) 32
do coeficiente “a” por um fator positivo,seu sinal será sempre
igual ao de “a” do que podemos concluir que: 07. A solução da desigualdade (logx 2) ⋅ (log2x 2) ⋅ (log2 4x ) > 1 é:
“Quando uma função quadrática não possui raízes reais, ela
a) − 2<x< 2
conserva sempre o sinal de coeficiente do termo do 2º grau, qualquer
que seja o valor atribuído à variável”. b) 2− 2
<x<2 2

c) 2–2 < x < 2–1 ou 1 < x < 2²


II.Quando ∆ = 0, a expressão entre colchetes é maior ou igual
d) 2− 2
< x < 2−1 ou 1 < x < 2 2

a zero quando fizermos x = − b , e então podemos concluir:


2a e) 0 < x < 2− 2
ou 1 < x < 2 2

“Quando uma função quadrática possui duas raízes reais e iguais,


ela conserva sempre o sinal do coeficiente do termo do 2º grau, a 08. (ITA 1971) Determinando-se a condição necessária e suficiente
não ser que se dê à variável o valor da raiz dupla, no qual a função sobre t para que a equação 4x – (int + 3)2x – int = 0 admita duas raízes
quadrática se anula”. reais e distintas, obtemos: (int representa o logaritmo neperiano de t)
a) t > e–3 d) 0 < t < e–9 ou e–1 < t < 1
III. Quando ∆ > 0, a expressão entre colchetes pode ser colocada b) e < t < 1
–3
e) e–1 < t < 1
sob a forma f(x) = a(x – x1)(x – x2) onde x1 e x2 são as raízes da c) 0 < t < e ou t > e
–9 –1
função quadrática e conclui-se que:
“Quando uma função quadrática possui duas raízes reais e 09. (ITA 1997) Dado um número real a com a > 1, seja S o conjunto
desiguais, ela conserva o sinal do coeficiente do termo do 2º grau, solução da inequação
para os valores da variável exteriores ao intervalo das raízes, e possui x −7
sinal contrário ao do coeficiente do termo do 2º grau para os valores  1
log 1 loga   ≤ log 1 (x − 1) .
da variável interiores ao intervalo das raízes”. a  a a

Então S é o intervalo
EXERCÍCIOS DE
a) [4 , +∞[ c) ]1 , 5] e) [1 , 4[

TREINAMENTO b) [4 , 7[ d) ]1 , 4]

10. (CHINESE MATHEMATICS COMPETITIONS) Sejam a e b números


reais e f(x) = a ⋅ sen x + b ⋅ 3 x + 4 . Se f(loglog310) = 5, qual o valor de
01. (UFCE 2004) A soma dos inteiros que satisfazem a desigualdade f(loglog3)?
|x – 7| > |x + 2| + |x – 2| é:
a) −5 d) 5
a) 14 d) –15
b) −3 e) depende de a e b
b) 0 e) –18
c) 3
c) –2
11. (CHINESE MATHEMATICS COMPETITIONS) Sabendo que [a]
02. Dados os conjuntos A = { x ∈  | x − 5 < 3} e B = { x ∈  | x − 4 ≥ 1} . representa o maior número inteiro que não é maior que a. Quantas
A soma dos elementos de A ∩ B é igual a: são as raízes reais da equação (log x)² – [logx] – 2 = 0?
a) 19 c) 21 a) 0 c) 2 e) 4
b) 20 d) 22 b) 1 d) 3

PROMILITARES.COM.BR 97
FUNÇÃO MODULAR E DESIGUALDADES

12. A solução da inequação ax² – 2x + 4 > 0 é: 18. (ITA 1986) Sejam a, b, c números reais dados com a < 0. Suponha
a) x > 2, se a = 0 que x1 e x2 sejam as raízes da função y = ax² + bx + c e x1 < x2. Sejam
b 2b + b2 − 4ac
b) +∞ < x < −∞, se a ≤ 14 x3 = − e x4 = − . Sobre o sinal de y podemos
2a 4a
c) 1+ 1 − 4a 1 − 1 − 4a , se a < 0 afirmar que:
<x<
a a a) y < 0, ∀x ∈ , x1 < x < x3
1+ 1 − 4a b) y < 0, ∀x ∈ , x4 < x < x2
d) x> , se 0 < a ≤ 14
a c) y > 0, ∀x ∈ , x1 < x < x4
1 − 1 − 4a d) y > 0, ∀x ∈ , x > x4
e) x< , se 0 < a ≤ 14
a e) y < 0, ∀x ∈ , x < x3

13. Supondo que x é um número real, x ≠ 0 e x ≠ 3, não é um possível 19. (FUVEST 2017) Considere a função fa : [0, 1] → [0, 1] que depende
valor da expressão: de um parâmetro a ∈ ]1,2], dada por
x −x x − 3 − ( x − 3)  1
+ se 0 ≤ x ≤ ,
x x−3 ax, 2
fa (x) = 
a) –4 a(1 − x), se 1 ≤ x ≤ 1.
 2
b) −2
c) 0 1 
Sabe-se que existe um único ponto pa ∈  , 1  tal que fa(pa) = Pa. Na
d) 2 2 
figura a seguir, estão esboçados o gráfico de fa e a reta de equação
e) todos os valores acima são valores possíveis da expressão
y = x.

14. A solução da inequação


( x + 2)( x − 1) ≥ 2 ( x + 2)
é dada por:
a) (–∞,2] d) [2,+ ∞)
b) (–2,2] e) (– ∞,–2]
c) [–2,+∞)

15. A solução de inequação


x2 + 1 1 x
− >
a2x − 2a 2 − ax a
é dada por:
1
a) x<− , se a = –2
3
b) –∞ < x < +∞ se a = 0
c) x ∈ (–1,2) se a = 1 a) Encontre uma expressão para o ponto pa em função de a.
 2  a + 1 
d) x ∈  −1,    , +∞ , se a < –2   1  1
 a  a − 1  b) Mostre que fa  fa    < para todo a ∈ ]1,2].
  2  2
e)  2 a + 1 , se a > 1 c) Utilizando a desigualdade do item b), encontre a ∈ ]1,2] tal que
x ∈ ,
 a a − 1    1  
fa  fa  fa     = pa , em que pa é o ponto encontrado no item a).
   2  
16. Se a é o menor valor que satisfaz a inequação |1 – 8x| ≤ 3 e
sen(y) = a, então o valor da constante k, que satisfaz a igualdade
sen(2y) = kcotg(y), é: 20. A evolução mensal do número de sócios de uma revista de
Matemática durante o ano de 2015 está expressa pela função:
1 1 e) 1
a) c) 100 − x(x − 4) se 1 ≤ x ≤ 4
8 4 
1 1 =
f(x) 100 se 4 < x ≤ 9
b) d) 100 + (x − 9) ⋅ (x − 12) se 9 < x ≤ 12
2 16 
em que x = 1 representa janeiro de 2015, x = 2 representa fevereiro
17. (EN 2017) Sejam g e f funções reais, determine a área da de 2015, e assim por diante.
região limitada pelo eixo y, por g(x) = –|x – 3| + 4 e pela assíntota de
3
a) Faça um esboço do gráfico da função. Qual foi o maior número
=
f(x) x 3 − x 2 e assinale a opção correta. de sócios nesse período?
13 b) Qual foi a média aritmética do número de sócios nos doze meses
a) c) 7 e) 9
4 de 2015?
40 81
b) d)
9 16

98 PROMILITARES.COM.BR
FUNÇÃO MODULAR E DESIGUALDADES

EXERCÍCIOS DE 08. (IME 2017) O sistema de inequações abaixo admite k soluções

COMBATE inteiras.
 x 2 − 2x − 14
 >3
 x
x ≤ 12
2x + 1 
01. (EN 1994) O conjunto-solução de > 3 é:
x−3 Pode-se afirmar que:
a) (8/5,3) ∪ (3,+∞)
b) (3,10) ∪ (10,+∞) a) 0 ≤ k < 2 d) 6 ≤ k < 8
c) (–∞,8/5) ∪ (3,10) b) 2 ≤ k < 4 e) k ≥ 8
d) (8/5,3) ∪ (3,10) c) 4≤k<6
e) (8/5,3) ∪ (10,+∞)
09. (IME 2010) Seja f(x) = |3 – log(x)|, x ∈ . Sendo n um número inteiro
02. (ITA 2002) Os valores de x ∈ R, para os quais a função real dada n−3
positivo, a desigualdade f(x) + 2f(x) + 4f(x) + … + 2 f(x) + … ≤ 9
por f(x) = 5 − 2x − 1 − 6 está definida, formam o conjunto 4 12 36 3n −1 4
a) [0, 1] d) (−∞, 0] ∪ [1, 6] somente é possível se:
b) [−5, 6] e) [−5, 0] ∪ [1, 6] Obs.: log representa a função logarítmica na base 10.
c) [−5, 0] ∪ [1, ∞) a) 0 ≤ x ≤ 106 d) 100 ≤ x ≤ 106
b) 10 ≤ x ≤ 10
–6 8
e) 10–6 ≤ x ≤ 106
03. (ITA 2007) Sobre a equação na variável real x, x −1 − 3 − 2 =0, c) 10 ≤ x ≤ 10
3 6

podemos afirmar que


a) ela não admite solução real. 10. (IME 2017) Seja f(x) = | x − 1| + | x − 2 | + | x − 3 | +  + | x − 2.017 |.
b) a soma de todas as soluções é 6. O valor mínimo de f(x) está no intervalo:
c) ela admite apenas soluções positivas. a) (– ∞,1008] d) (1010,1011]
d) a soma de todas as soluções é 4. b) (1008,1009] e) (1011,+ ∞)
e) ela admite apenas duas soluções reais. c) (1009,1010]

04. (ITA 1988) Sabendo-se que as soluções da equação |x|² – |x| – 6 = 11. (ITA 2018) Encontre o conjunto solução S ∈  da inequação
0 são raízes da equação x² – ax + b = 0, podemos afirmar que: 4

∑3
1081
a) a = 1 e b = 6 exponencial: 3x − 2 + x +k
≤ .
k =1
18
b) a = 0 e b = −6
c) a = 1 e b = −6 12. (ITA 2017) Determine número de soluções inteiras da inequação
d) a = 0 e b = −9 0 ≤ x 2 − | 3x 2 + 8x | ≤ 2 é
e) não existem a e b tais que x² – ax + b = 0 contenha todas as raízes a) 1. d) 4.
da equação dada. b) 2. e) 5.
c) 3.
05. (ITA 2017) O número de soluções inteiras da inequação
0 ≤ x 2 − | 3x 2 + 8x | ≤ 2 é 13. (ITA 2008) Dado o conjunto A = {x ∈ ; (3x 2 + 2x) < x 2 },
a) 1. c) 3. e) 5. expresse-o como união de intervalos da reta real.
b) 2. d) 4.
14. (ITA 2004) Determine os valores reais do parâmetro a para os
06. (EN 2013) A reta no ² de equação 2y – 3x = 0 intercepta o quais existe um número real x satisfazendo 1 − x 2 ≥ a - x.
x2 − 1
gráfico da função f(x) = x nos pontos P e Q. Qual a distância
x 15. (ITA 2017) Esboce o gráfico da função f:  →  dada por
entre P e Q? 1
=
f(x) 2−|x | − .
a) 2 15 d) 7 2
b) 2 13 e) 5
2 16. (ITA 2015) Considere as funções f1, f2, f:  → , sendo
c) 2 7
1 3
f1=
(x) x + 3, f2 =
(x) x + 1 e f(x) igual ao maior valor entre f1(x) e
x y 2 2
07. (EN 1996) O gráfico da relação + < 1 é a região do plano xy: f2(x), para cada x ∈ . Determine:
4 2
a) Todos os x ∈  tais que f1(x) = f2(x).
1
a) compreendida entre as retas y = − ( x + 4) . b) O menor valor assumido pela função f.
2
b) interior ao losango de vértices (0,2), (0,–2), (–4,0) e (4,0). c) Todas as soluções da equação f(x) = 5.
c) interior ao retângulo de vértices (–4,2), (–4,–2), (4,2) e (4,–2).
17. (IME 2018) Resolva a inequação abaixo, onde x é uma variável
d) interior à elipse de centro (0,0) com eixo maior AB sendo A(–4,0) real.
e b(4,0) e eixo menor CD onde C(0,2) e D(0,2).
2 | x 3 | −6x 2 + 3 | x | +2 < 0
e) interior à circunferência centrada em (0,0) e raio 4.

PROMILITARES.COM.BR 99
FUNÇÃO MODULAR E DESIGUALDADES

18. (ITA 2017) Determine todos os valores reais de x que satisfazem a 4) c ≤ x < d; a função ϕ (x) adquire seu valor mínimo para x = c, este
inequação 43x – 1 > 34x. valor é também igual a
AB + 2BC + CD.
19. (IME 2017) Resolva a inequação, onde x ∈ .
5) x ≥ d; o valor mínimo da função ϕ(x) é igual a
9x 2
>4 AB + 2BC + 3CD.
(1 − 3x + 1)2
Comparando os resultados obtidos vemos que o valor mínimo da
função ϕ (x) é igual a AB + 2BC + CD, assim
20. (IME 2016) Quantos inteiros k satisfazem à desigualdade b – a + 2 (c – b) + d – c = d + c – b – a.
2 log10 k − 1 + 10log10−1 k1 4 + 3 > 0?
A função ϕ(x) adquire este valor neste caso quando b ≤ x ≤ c.

GABARITO 04. E
EXERCÍCIOS DE TREINAMENTO Seja x = 1 – a, y = 1 – b, z = 1 – c. Então
01. E a + b + c = 2 a = 2 – (1 – y) – (1 – z) = y + z
1º caso: x < −2 Analogamente temos b = z + x e c = x + y. Daí, a expressão original se
x + y )( y + z)( z + x )
|x − 7| > |x + 2| + |x − 2| ⇔ −x + 7 > x − 2 − x + 2 ⇔ x > −7 torna (
xyz
Logo, −7 < x < −2, e as soluções inteiras são −6, −5, −4 e −3. x+y
Aplicando MA ≥ MG temos ≥ xy isto é, x + y ≥ 2 xy .
2º caso: 2 ≤ x < 2 2

|x − 7| > |x + 2| + |x − 2| ⇔ −x + 7 > x + 2 − x + 2 ⇔ x < 3 Analogamente, y + z ≥ 2 yz e z + x ≥ 2 zx , consequentemente


Logo, −2 ≤ x < 2, e as soluções inteiras são 2, −1, 0 e 1. ( x + y )( y + z)( z + x ) ≥ ( 2 )(
xy 2 yz 2 zx )( )=
8
3º caso: 2 ≤ x < 7 xyz xyz
|x − 7| > |x + 2| + |x − 2| ⇔ −x + 7 > x + 2 + x − 2 ⇔ x < 7/3
Logo, 2 ≤ x < 7/3, e a solução inteira é 2. 05. E
4º caso: 7 ≤ x Aplicando a desigualdade de Cauchy-Schwartz
|x − 7| > |x + 2| + |x − 2| ⇔ x − 7 > x + 2 +x − 2 ⇔ x < −7 (a2
1 )(
+ a22 + a23 ⋅ b12 + b22 + b23 )
Logo, não há soluções nesse caso.
≥ ( a1 ⋅ b1 + a2 ⋅ b2 + a3 ⋅ b3 )
2

A soma de todas as soluções inteiras é −6 −5 −4 −3 −2 −1 +0 +1 +2 =


−18 onde
=a1 =
x ; a2 =
y ; a3 z;
02. C 1 1 1
=b1 = ; b2 = ;b3
|x − 5| < 3 ⇔ −3 < x −5 < 3 ⇔ 2 < x < 8 ⇔ A = {3, 4, 5, 6, 7} x y z
|x − 4| ≥ 1 ⇔ x − 4 ≤ −1 ou x − 4 ≥ 1 ⇔ x ≤ 3 ou x ≥ 5 ⇔
obtemos o resultado desejado.
⇔ B = {x ∈ Z | x ≤ 3 ou x ≥ 5}
A ∩ B = {3, 5, 6, 7}
06. B
A soma dos elementos de A ∩ B é 3 +5 +6 +7 = 21

03. D
Tomemos esse número e marquemos os pontos A, B C e D
correspondentes aos números a, b, c e d. Designamos o ponto com a
abscissa variável x por M, veja a figura abaixo.

Examinemos os cinco casos seguintes.


1) x ≤ a: então
ϕ (x) = MA + MB + MC + MD = AB + 2MB + 2BC + CD.
Está claro que ϕ (x) adquirá seu valor mínimo nocaso em que o ponto
M coincida com o ponto A e que este valor será igual a
3AB + 2BC + CD. x 2 + x − 30 ≥ 0 ⇔ x ≤ −6 ou x ≥ 5
2) a < x ≤ b; no caso ⇒ x 2 − x ≥ 20
ϕ (x) = AM + MB + MC + MD = AB + 2MB + 2BC + CD.
x 2 − x − 1 − 2 − 3 − 4 − 5 = x 2 + x − 30
A função ϕ (x) adquire seu valor mínimo quando o ponto M coincide
com o ponto B; este valor é igual a 15
⇔ x 2 − x − 15 = x 2 + x − 30 ⇔ x =
AB + 2BC + CD 2

3) 5 ≤ x ≤ c; para estes valores de x a função ϕ (x) é constante e igual a


AB + 2BC + CD.

100 PROMILITARES.COM.BR
FUNÇÃO MODULAR E DESIGUALDADES

07. D 3º caso: 1 ≤ log x < 2 ⇒ log x  =


1
(logx 2) ⋅ (log2x 2) ⋅ (log2 4x ) > 1 (log x )2 − log x  − 2 = 0 ⇔ (log x ) − (1) − 2 = 0 ⇔ log x = ± 3
2

log2 4x
>1 ⇒ log x = 3 ⇔ x = 10 3
(log2 x ) ⋅ (log2 2x )
2 + log2 x 4º caso: log x = 2 valores que satisfazem a equação ⇒
2 ⇒ log x  =
>1 x = 100
(log2 x ) ⋅ (1+ log2 x )
log2 x = y
12. C
2+ y y2 − 2 De acordo com o valores particulares do parâmetro a, a saber, a = 0 e
> 1⇔ <0
y ⋅ (1+ y ) y(y + 1) a = 14 resolveremos a inequação em cada um dos seguintes casos:
( +) − 2 ( −) −1 ( +) 0 ( −) 2 ( +)  a> 1
 4
− 2 < y < −1ou 0 < y < 2 0 < a ≤ 1
 4
− 2 < log2 x < −1ou 0 < log2 x < 2
 a=0
2− 2
< x < 2−1 ou 1 < x < 2 2 
 a<0

1) Se a > 14 então o trinômio ax² – 2x + 4 possui discriminante


08. E negativo e o coeficiente do termo dominante(coeficiente do termo
Fazendo y = 2x de maior grau) positivo daí, o trinômio é positivo para todo valor
A equação do 2º grau y² – (int + 3) y – int = 0 deve possuir duas raízes de x isto é, a solução da inequação proposta nesta caso é dada por
distintas e positivas, pois y = 2x > 0. Assim, o discriminante, a soma e x ∈ ( −∞, + ∞ ) .
o produto das raízes devem ser positivos.
=
∆ (ln t + 3)2 + 4 ln > 0 ⇔ ln2 t + 10ln t + 9 > 0 2) Se 0 < a ≤ 1 então, o trinômio ax² – 2x + 4 possui as raízes:
4
−9 −1
⇔ ln t < −9 ou ln t > −1 ⇔ t < e ou t > e 1± 1 − 4a
x1,2 = , onde 1 − 1 − 4a ≤ 1+ 1 − 4a
S ln t + 3 > 0 ⇔ ln t > −3 ⇔ t > e −3
= a a a
P =− ln t > 0 ⇔ ln t < 0 ⇔ 0 < t < 1 Daí, a solução da inequação proposta é dada por
A interseção das três condições resulta e–1 < t < 1.  1 − 1 − 4a   1+ 1 − 4a 
 −∞, a   a
, +∞
   
.
09. D
x −7
 1 3) Se a = 0 então a inequação proposta toma a forma –2x + 4 > 0 e
log 1 loga   ≤ log 1 (x − 1)
a  a a daí obtemos x < 2.
⇒ log 1 loga a7 − x ≤ log 1 (x − 1)
a a 4) Se a < 0 então 1+ 1 − 4a < 1 − 1 − 4a e daí, neste caso, a solução
⇒ log 1 (7 − x) ≤ log 1 (x − 1) a a
a a
1 da inequação proposta é dada por 1+ 1 − 4a < x < 1 − 1 − 4a .
a > 1 ⇒ 0 < < 1 ⇒ 7 −x ≥ x −1 > 0 ⇔ 1 < x ≤ 4 a a
a
13. D
10. C
x − 3 − ( x − 3) (x − 3) − ( x − 3)
A função g(x) = f(x) – 4 é ímpar. x > 3⇒ = = 0
x−3 x−3
 1  x − 3 − ( x − 3) ( − x + 3) − ( x − 3)
loglog3 10 = log  = − loglog3
 log3 x < 3⇒ = =
x −3 x −3
f(loglog3) = g(loglog3) + 4 = g( − loglog3 10) + 4 = 2 − x + 3 2( − x + 3)
= = = −2
=−g(loglog3 10) + 4 =−[f(loglog3 10) − 4] + 4 = x−3 x−3
=− f(loglog3 10) + 8 =−5 + 8 =3 x −x x−x
x>0⇒ = = 0
x x
11. D x −x − x − x 2 − x 2 ⋅ ( − x)
x<0⇒ = = = =−2
x x x x
log x  ≤ log x ⇒ (log x ) − log x − 2 ≤ 0 ⇔ −1 ≤ log x ≤ 2
2
x −x x − 3 − ( x − 3)
x<0⇒ + = −2 + ( −2) = −4
1º caso: −1 ≤ log x < 0 ⇒ log x  =−1 x x−3
(log x )2 − log x  − 2 = 0 ⇔ (log x )2 − ( −1) − 2 = 0 ⇔ log x = ±1 x −x x − 3 − ( x − 3)
0< x < 3⇒ + = 0 + ( −2) = −2
1 x x −3
⇒ log x =−1 ⇔ x = x −x x − 3 − ( x − 3)
10 x > 3⇒ + = 0+0 = 0
x x−3
2º caso: 0 ≤ log x < 1 ⇒ log x  =0
(log x )2 − log x  − 2 = 0 ⇔ (log x ) − ( 0) − 2 = 0 ⇔ log x = ± 2
2
14. C
⇒ não há solução
A inequação proposta é equivalente aos seguintes sistemas de
inequações:

PROMILITARES.COM.BR 101
FUNÇÃO MODULAR E DESIGUALDADES

 2x − 1 ≥ 0

x≥
1 Analisando a diferença A 2 = 2 + a obtemos que se a < –2 então
 2 a
 
 x+2> 0 ou  x > −2 A2 > 0; se –2 < a < 0 então A2 < 0 ; se 0 < a < 1 ou a > 1, então A2 > 0.
  2 Finalmente, se a = –2 então A2 = 0. Consideremos agora, a diferença
 2x − 1 < ( x + 2) 2
 x + 2x + 5 > 0
 a2 − a + 2 . Uma vez que o discriminante do trinômio a² – a + 2
A3 =
a ( a − 1)
Como o trinômio x² + 2x + 5 possui discriminante negativo e coeficiente
é negativo e o coeficiente de a² é positivo então a² – a + 2 > 0 para
do termo dominante positivo, ele é positivo para qualquer valor de x
quaisquer valores de a e, o sinal da diferença A3 depende somente
e portanto, a solução deste último sistema e consequentemente da
do sinal do denominador a(a – 1) e temos então se a < 0, então A3
inequação dada é  1 , +∞ . > 0; se 0 < a < 1, então A3 < 0 e se a > 1, A3 > 0. Estamos agora em
 2 
condições de resolver a nossa inequação de acordo com os valores de
a par os diversos casos a saber, a < –2, –2 < a <0, 0 < a < 1 ou a = –2.
15. D
Nos primeiros três casos, obtemos respectivamente:
A inequação proposta pode ser colocada sob a forma
2
x +1 1 x (1− a) x 2 + 2x + 1+ a > 0 2 a+1 2 a+1 a +1 2
− − > 0 ou −1 < < ; < −1 < e < −1 <
a ( ax − 2) 2 − ax a a2 x − 2 a ( ) a a −1 a a −1 a −1 a
e assim, obtemos:
De acordo com o valores particulares do parâmetro a a saber, a = 1
e a = 0 resolveremos a inequação em cada um dos seguintes casos: 2 a +1
se a < –2 então −1 < x < ou x > ;
a=1 a a −1


 a=0 se –2 < a < 0 então 2 < x < −1 ou x > a + 1 ;
 a a −1
a ≠ 0 e a ≠ 1
a+1 2
se 0 < a < 1 então < x < −1 ou x > .
2x + 2 a −1 a
1) Para a = 1, a inequação proposta toma a forma > 0 e daí
encontramos que x < –1 ou x > 2. x−2
( x + 1)  x +
1

3
Finalmente, para a = –2, a inequação toma a forma >0
2) Para a = 0, a inequação proposta não possui solução. 1 x +1
e daí, encontramos x > − . Na resolução da inequação
a + 1 3
3) Se a ≠ 0 e a ≠ 1 então, fatorando o numerador da inequação ( x + 1)  x − 
a −1
obtemos < 0 , estamos interessados nos sinais das diferenças
2
a + 1
x −
A1, A2, A3 somente
a no intervalo a > 1 e neste caso, temos −1 < 2 < a + 1
(1− a)( x + 1)  x −  a a −1
a − 1 e aplicando o método dos intervalos encontramos a solução da
>0
2
a + 1
x−
a ( x + 1)  x − 
inequação a − 1
<0:
Inequação esta que deve ser considerada em dois casos, a saber, 2
x−
a
a ≠ 0 e a ≠ 0 . No primeiro caso, temos 1 – a > 0 e a inequação
  se a > 1 então x < –1 ou 2 < x < a + 1 . Escrevamos agora a resposta
a < 1 a > 1
a a −1
acima se torna
final da inequação proposta:
a + 1
( x + 1)  x −  (1) se a < –2 então, −1 < x <
2
ou x >
a +1
;
a − 1
>0 a a −1
2
x−
a (2) se a = –2, então x > − 1 ;
3
No segundo caso, 1 – a < 0 a inequação se torna
(3) se –2 < a < 0 então 2 < x < −1 ou x > a + 1 ;
a + 1 a −1
( x + 1)  x − 
a
a − 1 (4) se a = 0 então a inequação não possui solução;
<0
2
x−
a (5) se 0 < a < 1 então a + 1 < x < −1 ou x > 2 ;
a −1 a
Agora, para resolver as duas inequações recém formadas, pelo (6) se a = 1, então x < –1 ou x > 2;
método dos intervalos, torna-se necessário arrumarmos os pontos
(7) se a > 1 então x < –1 ou 2 < x < a + 1 .
–1, a + 1 e 2 em ordem crescente na reta real numérica. Para tanto, a a −1
a −1 a
façamos as seguintes diferenças: 16. A
a+1 2 a+1 2 Sabendo que | w | ≤ α ⇔ −α ≤ w ≤ α, para todo a ≥ 0, temos
=
A1 − ( −1) , A
= 2 − ( −1) e A
= 3 −
a −1 a a −1 a | 1 − 8x | ≤ 3 ⇔ −3 ≤ 1 − 8x ≤ 3
e analisemos o sinal de cada uma delas. ⇔ −3 ≤ 8x − 1 ≤ 3
2a ⇔ −2 ≤ 8x ≤ 4
Consideremos primeiramente a diferença A1 = e vemos então
a −1 1 1
que se a < 0, então A1 > 0; se 0 < a < 1 então A1 < 0 e se a > 1 então ⇔− ≤x≤ .
4 2
A1 > 0.
1 1
Logo, a = − e, portanto, sen y = − .
4 4

102 PROMILITARES.COM.BR
FUNÇÃO MODULAR E DESIGUALDADES

Desse modo, vem 3


1− u − 1
cos y lim ( f ( x ) − ax ) =
lim+
sen2y = k ⋅ cotg y ⇔ 2 ⋅ sen y ⋅ cos y = k ⋅ x →+∞ u→ 0 u
sen y 1
⋅ (1 − u) ⋅ ( −1)
−2 3
1
⇒ k = 2 ⋅ sen2 y lim ( f ( x ) − ax ) =lim+ 3 =

2 x →+∞ u→ 0 1 3
 1
⇒ k = 2⋅  −  1
⋅ (1 − u) ⋅ ( −1)
−2 3
 4 1
lim ( f ( x ) − ax ) =lim+ 3 =

1 x →−∞ u→ 0 1 3
⇒ k =.
8
1
Portanto, b = −
3
1
17. B ( x)
Dessa forma, a assíntota de f=
3
x 3 − x 2 ,é y= x − .
3
De g(x) = –|x – 3| + 4,

Então,
x + 1 se x < 3
g( x) = 
 − x + 7 se x ≥ 3

( x)
De f= 3
x 3 − x 2 , temos que sua assíntota é dada por y = ax + b,
de tal modo que:
f ( x) f ( x ) ou
a =
lim ou a lim
x →+∞ x x →−∞ x
b = lim ( f ( x ) − ax ) ou b = lim ( f ( x ) − ax ) .
x →+∞ x →−∞

( x)
De f= 3
x3 − x2 ,
As retas y = –x + 7 e y= x −
1
são perpendiculares, assim como as
3
 1 1
3 x 3 ⋅ 1 −  x ⋅ 3 1− retas y = –x + 7 e y = x + 1.
f ( x) 3
x3 − x2  x x= 1
= = = 3 1−
1
x x x x x B é ponto de intersecção das retas y= x − e y = –x + 7, logo, é
3
f ( x) 1  1
lim = solução do sistema y= x − 3 .
lim 3 1=
− 1
x →+∞ x x →+∞ x
y =− x + 7
f ( x) 1
lim = lim 3 1=
− 1
x →−∞ x x →−∞ x  11 10 
Então, B  ,  .
3 3
Assim, a = 1.
A área do trapézio ABCD é dada por:
Com a = 1, temos:
 2 2
 1 1 1  11   10  1 
f ( x ) − ax = x 3 − x 2 − x = 3 x 3 ⋅ 1 −  − x = x ⋅ 3 1 − − x
3
⋅ ( 3 − 0)2 + ( 4 − 1)2 +  3
− 0 +  −  −    ⋅
  
 x x 2   3 3  
 1 
f ( x ) − ax = x ⋅  3 1 − − 1
2 2
 11   10  40
 x  .  − 3 +  − 4 =
3   3  9
1 3 1−
−1
f ( x ) − ax = x
1 18. C
x
Como a < 0, b2 − 4ac > 0 e x1 < x2, então
1
3 1− −1 −b + b2 − 4ac −b − b2 − 4ac
0 =x1 = e x2
lim (=
f ( x ) − ax ) lim = x
2a 2a
x →+∞ x →+∞ 1 0
x x1 < x3 < x2, pois x3 é a média das raízes

Como houve uma indeterminação do tipo 0 , vamos usar a Primeira 2b + b2 − 4ac b b2 − 4ac
0 x4 =− =− −
Regra de L’ Hospital. 4a 2a 4a
1
Fazendo = u, 2 2
x Como 0 < − b − 4ac < − b − 4ac , então x3 < x4 < x2
4a 2a
Como a parábola possui concavidade voltada para baixo, então y > 0
quando x1 < x < x4.

PROMILITARES.COM.BR 103
FUNÇÃO MODULAR E DESIGUALDADES

19. EXERCÍCIOS DE COMBATE


1 01. D 03. D 05. C 07. B 09. D
a) Sendo < pa < 1 e (pa, pa) o ponto de interseção dos gráficos das
2 a 02. E 04. D 06. B 08. D 10. B
funções y = x e fa(x) = a(1 – x), temos pa = a(1 − pa ) ⇔ pa = .
a+1  1
11. S = x ∈  : x ≤ log3 
 1 a  2
b) De acordo com o gráfico, sabemos que fa   = . Logo, como
 2 2
1 a 12. C
1 < a ≤ 2 é equivalente a < ≤ 1, pela definição de fa, vem
2 2 1) x 2 − 3x 2 + 8x ≤ 0 → x =0 é solução
  1   a
fa  fa    = fa   2) 3x 2 + 8x ⋅ | x | ≤ | x | ⋅ | x |→ 9x 2 + 48x + 64 ≤ x 2 → x 2 + 6x + 8 ≤ 0
  2   2
 a soluções → −4 ≤ x ≤ −2
= a 1 −  soluções inteiras possíveis : −2, − 3, − 4
 2
1 1 Porém, substituindo-se o valor -3 na inequação original os valores não
= − (a − 1)2.
2 2 conferem. Logo, as soluções possíveis serão apenas 0, -2 e -4.
  1  1
Assim, para a = 1 temos o valor máximo de fa  fa    , ou seja, .
  2  2 13. A = ]-∞, -1[ ⋃ ]-1, -2/3] ⋃ ]2, +∞[
  1  1
Mas, como a ≠ 1 segue que fa  fa    < , para todo a ∈ ]1,2]. x
  2  2 1
14. a ≤ 2 = 2− x
=
g(x)
2
  1  1
c) Sabendo de (b) que fa  fa    < ,e que pa = a , pela definição
x
1
  2  2 a+1 = = 2−| x |
h(x)
15. Dividindo a função em “partes” para2esboçar:
de fa, vem
x
1
x
1 1
   1    a2  a = = 2− x
g(x) =
m(x) −
fa  fa  fa     =pa ⇔ fa  a −  = 2 2 2
   2    2  a+1
1
x 1
= = 2−| x |
h(x) f(x) 2−| x | −
=
 a2  a 2
⇔ aa −  = 2
 2  a+1 x
1 1
a =
m(x) −
⇔− (a2 (a − 1) − 2(a − 1)) =0 2 2
2(a + 1)
1
⇒ a2 − 2 =0 f(x) 2−| x | −
=
2
⇒ a =2.

20.
a) Calculando:
100 − x ⋅ (x − 4) se 1 ≤ x ≤ 4

=
f(x) 100 se 4 < x ≤ 9
100 + (x − 9) ⋅ (x − 12) se 9 < x ≤ 12

f(1) = 100 − x ⋅ (x − 4) = 100 − 1⋅ (1 − 4) = 103
f(2)= 100 − 2 ⋅ (2 − 4)= 104
f(3)= 100 − 3 ⋅ (3 − 4)= 103
f(4)= 100 − 4 ⋅ (4 − 4)= 100
⇒ fmáx (x) =
104
= f(6)
f(5) = f(7)
= f(8)
= f(9)
= 100
f(10)= 100 + (x − 9) ⋅ (x − 12) = 100 + (10 − 9) ⋅ (10 − 12) = 98
f(11)= 100 + (11 − 9) ⋅ (11 − 12)= 98
f(12)= 100 + (12 − 9) ⋅ (12 − 12)= 100

16.
1 3
a) f1(x) = f2 (x) ⇒ ⋅ | x | +3 = | x + 1|⇒| x | +6 = 3⋅ | x + 1|
2 2

b) Calculando:  9 3
Daí a solução da equação será dada por S = − , .
103 + 104 + 103 + 100 ⋅ 6 + 98 + 98 + 100  2 2
= 100,5
12

104 PROMILITARES.COM.BR
FUNÇÃO MODULAR E DESIGUALDADES

b) Construindo os gráficos das funções f1 e f2 num mesmo sistema, Logo,


temos: 1− 3 1+ 3
t< ou <t<2
2 2
Assim,
1− 3 1− 3
(i) x < (sem solução, pois
2
< 0)
2
1+ 3
(ii) < x <2
2
1+ 3
De x > ,
2
1+ 3 1+ 3
x> ou x < −
2 2
De x < 2,
−2 < x < 2
Dessa forma,
De acordo com o gráfico o menor valor assumido por f(x) é f(0) = 3. 1+ 3 1+ 3
−2 < x < − ou <x<2
9 3 2 2
c) Se x ≤ − ou x ≥ , temos:
2 2
 1+ 3 1+ 3 
3 10 7 13 S = x ∈  : −2 < x < − ou < x < 2 .
⋅ | x + 1|=5 ⇒| x + 1|= ⇒ x = ou x =− (não convém)  2 2 
2 3 3 3
7
Portanto, x =
3 18. Calculando:
9 2
Se − ≤ x ≤ , temos: 43x
2 3 43x −1 > 34x ⇒ > 34x ⇒ 43x > 4 ⋅ 34x
4
1 x x
⋅ | x | +3 =5 ⇒| x |=4 ⇒ x =4 (não convém) ou x =−4 64 x  64   64 
2 > 4 ⇒   > 4 ⇒ log64   < log64 4
81x  81  81  81  81
Portanto, x = -4.
x < log 8 22 ⇒ x < log8 2
( 9)
2

E o conjunto solução da equação f(x) = 5 será dado por: S = −4, .


7 9

 3

17. 2 ⋅ x − 6x + 3 ⋅ x + 2 < 0
3 2 19. Calculando:
Fazendo |x| = t 9x 2
> 4 → 1 − 3x + 1 ≠ 0 → 3x + 1 ≥ 0
(1 − 3x + 1)2
2 ⋅ t 3 − 6t 2 + 3 ⋅ t + 2 < 0
1 + 3x + 1 > 2
Por inspeção, verifica-se que 2 é raiz da equação 2 ⋅ t 3 − 6t 2 + 3 ⋅ t + 2 =0. 
Então, 9x 2 (1 − 3x + 1)2 ou
⋅ > 4 → (1 − 3x + 1)2 > 4 
(1 − 3x + 1) (1 − 3x + 1)
2 2
1 + 3x + 1 < −2
(não convém)

1 + 3x + 1 > 2 → 3x + 1 > 1 → x > 0 → S =*+
Então, as raízes da equação 2t − 2t − 1 =
2
0 também são raízes da
equação 2t3 − 6t2 + 3t + 2 =0. 20. C
De 2t − 2t − 1 =
2
0, Resolvendo a inequação:
1+ 3 1− 3 2 log10 k − 1 + 10log10−1 k1 4 + 3 > 0
t= ou t =
2 2 5
Portanto, 2 logk − 1 − logk + 3 > 0
2
  1+ 3     1− 3   logk = x
2t3 − 6t2 + 3t + 2 =2 ⋅ ( t − 2) ⋅  t −    ⋅  t −    .
 5x 5x
  2    2  2 x −1− + 3 > 0 → 2 x −1 > −3
2 2
Voltando à inequação 2t − 6t + 3t + 2 < 0, temos:
3 2
25x 2
4 ( x − 1) > − 15x − 9 → 25x 2 − 76x + 52 < 0
  1+ 3     1− 3   4
2 ⋅ ( t − 2) ⋅  t −    ⋅  t −    < 0 ∆ = ( −76)2 − 4 ⋅ 25 ⋅ 52 → ∆ = 576

  2    2 
76 ± 576 = x 52 50 > 1
=x →
2 ⋅ 50 x=2
1< x ≤ 2
1 < log10 k ≤ 2 → 10 < k ≤ 100 → 90 números inteiros

PROMILITARES.COM.BR 105
FUNÇÃO MODULAR E DESIGUALDADES

ANOTAÇÕES

106 PROMILITARES.COM.BR
ANÁLISE COMBINATÓRIA E
PROBABILIDADE

PERMUTAÇÃO COM REPETIÇÃO Resolução:


Discutiremos agora arrumações de uma coleção de objetos com Para resolver problemas de escolhas com repetição, precisamos
objetos repetidos, tais com a coleção {b, a, n, a, n, a} e escolhas de fazer uma correspondência um a um com um problema relacionado a
objetos de um conjunto onde um objeto pode ser selecionado mais uma escolha sem repetição. Suponhamos que as três variedades sejam
de uma vez, tais como pedir seis cachorros quentes a serem escolhidos sem molho, com molho e completo e que a atendente tenha anotado
dentre três variedades. Determinaremos as fórmulas para esses o seguinte pedido:
problemas de contagem através de dois exemplos: sem molho com molho completo
Exemplo: x xxxx x
Quantas arrumações podem ser feitas com as seis letras b, a, n,
a, n, a? Se cada s representa um cachorro quente, então o pedido acima
Resolução: significa um sem molho, quatro com molho e um completo. Uma vez
que todos os atendentes saibam que esta é a sequência dos pedidos
Formaremos as arrumações escolhendo primeiro as três posições de cachorros quentes (sem molho, com molho, completo) podemos
()
onde os a’s ficarão, isto é 63 = 20 maneiras agora, escolhamos as omitir os nomes das variedades escrevendo apenas x | xxxx | x. Assim,
suas posições (entre as três remanescentes) onde os n’s ficarão, isto qualquer pedido de k cachorros quentes consiste numa sequência
()
é 32 = 3 maneiras e finalmente na última posição fica o b. Assim, de k x1s e dois |’s. Reciprocamente, toda sequência de k x’s e dois |’s
existem 20 ⋅ 3 ⋅ 1 = 60 arrumações. representa um pedido: os x’s antes do primeiro | representa o número
de cachorros sem molho: os x’s entre os dois |’s representa o número
TEOREMA de cachorros com molho e os x’s finais representam o número de
cachorros completos. Deste modo, existe uma correspondência um
Se existem n objetos dos quais k1 são do tipo 1, k2 são do tipo a um entre pedidos e tais sequências, mas, o número de sequências
2, ..., e km são do tipo m onde k1 + k2 + ... + kn = n então o número de seis x’s e dois |’s é simplesmente o número de escolhas de duas
de arrumações destes n objetos é denotado por P(n; k1, k2, ..., km) é
n!
()
posições na sequência para os |’s. Assim, a resposta é 82 = 28.
.
k1!k 2 !...km !
TEOREMA
Prova com efeito, além do argumento utilizado no exemplo acima
a saber, escolhendo as posições para um dos tipos dentre aquelas que O número de escolhas com repetição de k objetos dentre n tipos
vão restando podemos provar o teorema acima da seguinte forma:  k + n − 1
de objetos é  .
Suponhamos que para cada tipo dos ki objetos do tipo i sejam  k 
dados índices 1, 2, 3, ..., m tornando-os distintos. Existem então Prova:
n! arrumações destes n objetos distintos. Enumeremos agora Com efeito, faremos um “pedido” para uma escolha como e
estas n! arrumações de objetos distintos enumerando todas as fizemos no exemplo anterior com um x para cada objeto escolhido.
P(n; k1, k2, ..., km) disposições (sem índices) dos objetos e então, para Como fizemos anteriormente, os x’s antes do primeiro | conta
cada disposição colocando os índices de todos os modos possíveis. Por o número de objetos do primeiro tipo, os x’s entre o primeiro e o
exemplo, da disposição baanna os índices podem ser colocados nos segundo |’s conta o número de objetos do segundo tipo, ..., e os x’s
a’s de 3! maneiras: após o (n - 1)-ésimo | conta o número de objetos do n-ésimo tipo (n - 1
b a1 a2 n n a3 b a2 a1 n n a3 b a3 a1 n n a2 traços são necessários para separar n tipos). O número de sequências
b a3 a2 n n a1 b a1 a3 n n a2 b a2 a3 n n a1  k + (n − 1) 
com k x’s e (n - 1) |’s é  .
 k 
Para cada uma dessas 3! maneiras para indexar os a’s, existem 2!
maneiras para indexar os n!. Assim, em geral, uma disposição qualquer
terá k1! maneiras de indexar os k1 objetos do tipo 1, k2 maneiras para DISTRIBUIÇÕES
o tipo 2, ..., km maneiras para o tipo n. Então Geralmente um problema de distribuição é equivalente a um
n! = P(n; k1, k 2 , ..., km ) . k1!k 2 !...km ! problema de arrumação ou de escolha com repetição. Problemas
especializados de distribuição devem ser divididos em subcasos que
ou
possam ser contados por intermédio de permutações e combinações
n! simples. Um roteiro geral para modelar problemas de distribuição
P(n; k1, k 2 , ..., km ) =
k1! k 2 ! ... km ! é, distribuições de objetos distintos correspondem a arrumações e
distribuições de objetos idênticos correspondem a escolhas.
COMBINAÇÕES COM REPETIÇÃO Assim, distribuir k objetos distintos em n urnas diferentes é
Exemplo: equivalente a colocar os objetos em linha e atribuir o nome de
cada uma das n diferentes urnas em cada objeto. Assim, existem
De quantos modos distintos podemos comprar seis cachorros
quentes podendo escolher entre 3 variedades distintas? n . n . n . ... n = nk
  
k vezes

PROMILITARES.COM.BR 109
ANÁLISE COMBINATÓRIA E PROBABILIDADE

distribuições. Se ki objetos devem ir para a urna i existem Observamos então que:


P(n; k1, k2, ..., kn) distribuições. • A 1ª coluna do quadro foi obtida fixando-se o objeto A e
Por lado, o processo de distribuir k objetos idênticos em n urnas permutando-se os objetos B, C, D de todos os modos
distintas é equivalente a escolher um subconjunto (não ordenado) de possíveis, isto é, 3! = 6 modos. Em cada linha uma disposição
k nomes de urnas, com repetição, entre as n escolhas de urnas. Assim, pode ser obtida de outra por uma rotação conveniente e
 k + n − 1 (k + n − 1)! dadas duas disposições em linhas diferentes, nenhuma pode
existem  = distribuições. ser obtida da outra por qualquer rotação.
 k  k!(n − 1)!
• Assim, chama-se permutação circular de n objetos distintos
qualquer disposição desses objetos em torno de um círculo e
Observação duas permutações circulares são indistinguíveis se, e somente
se, uma pode ser obtida a partir da outra por uma rotação
Os problemas de escolhas com repetição podem ser formulados de conveniente como por exemplo duas permutações quaisquer
três formas equivalentes a saber: de uma mesma linha do quadro acima. Diremos ainda que
1. O número de maneiras de escolhermos k! objetos com duas permutações circulares são distinguíveis se, e somente
repetição dentre n tipos de objetos distintos. se, uma não pode ser obtida da outra por qualquer rotação
como por exemplo duas permutações quaisquer em linhas
2. O número de maneiras de distribuir k objetos idênticos em n
diferentes do quadro acima.
urnas distintas.
• Portanto, no cálculo das permutações circulares interessa
3. O número de soluções inteiras não negativas da equação
apenas a posição relativa dos objetos entre si, isto é, o número
x1 + x 2 + ... + xn = k.
de permutações circulares distinguíveis.
É importante que sejamos capazes de reescrever um dado
Mostraremos a seguir, que o número de permutações circulares
problema enunciado em uma das formas acima sob as outras
de n objetos, denotado por (PCn), é igual a n!/n, isto é
duas. Muitos acham a versão 2 o meio conveniente de olhar para
tais problemas em virtude de uma distribuição ser mais fácil de n!
(PC)= = (n − 1)!
visualizar (na cabeça de alguém). Além disso, o argumento original n
n
com pedido de cachorros quentes que utilizamos para deduzir
vossa fórmula para escolhas com repetição foi na realidade um Como vimos nas figuras anteriores, casos n = 3 e n = 4,
modelo de distribuição. A versão 3 é a mais geral (e mais abstrata) permutações simples distintas podem gerar uma mesma permutação
do problema. circular. Se soubermos quantas permutações simples distintas geram
permutações circulares equivalentes teremos resolvido o problema.
É fácil ver que este número é n, pois, se não considerássemos
equivalentes figuras que podem coincidir por rotação, teríamos o total
PERMUTAÇÕES CIRCULARES de n!. Logo n(PC)n = n!, o que implica que (PC)=n
n!
= (n − 1)! .
n
Consideremos n objetos distintos e disponhamos esses n objetos
em torno de um círculo.
Se n > 3, podemos imaginar esses objetos situados nos vértices de PERMUTAÇÕES CAÓTICAS
um polígono, por exemplo um polígono regular.
Definiremos [n] = {1, 2, ..., n}. Uma permutação de [n] é dita
O quadro abaixo apresenta as disposições dos objetos A, B, C, D caótica quando não possui pontos fixos. O que é um ponto fixo
em torno de um círculo. de uma permutação? A definição é análoga ao ponto fixo de uma
A D C B função. Seja a1, a2, ..., an uma permutação de (1, 2, ..., n). Dizemos
que a permutação admite um ponto fixo quando existe i tal que ai = i.
B D A C D B C A Assim, uma permutação caótica é uma permutação sem pontos fixos.
Em 1708, um francês chamado Pierre Rémond de Montmor (1678-
C B A D 1719) propôs o seguinte problema. Suponha que dois conjuntos de
A D B C cartas A e B são dados. As cartas de A são primeiramente dispostas
em uma fileira e as de B são então postas aleatoriamente sobre as
C D A B D C B A cartas de A de modo que 52 pares de cartas são formadas. Encontre
a probabilidade de que não haja duas cartas iguais sobrepostas. Se
B C A D denotarmos o número de permutações caóticas de n elementos por
A B D C Dn , é imediato que o problema se resume a calcular D52. Vamos
então calcular Dn . Para isso, recorreremos à técnica do Princípio da
C B A D B C D A Inclusão-Exclusão. Defina por Ai o conjunto das permutações de (1, 2,
..., n) em que o número i é ponto fixo. Queremos calcular o número
D C A B
de permutações que pertencem a exatamente zero dos conjuntos
A C D B A1, A2, ..., An.Temos: |Ai| = n - 1!, |Ai ∩ Aj| = n - 2! (note que i e j
estão fixos e devemos permutar os n - 2 elementos restantes) e assim
B C A D C B D A
por diante. Logo, Dn = n! - |A1 ∪ A2 ... ∪ An| . Fica como exercício
D B A C
demonstrar que, usando o Princípio da Inclusão-Exclusão, Dn = n! [
1 1 ( −1)n
A C B D − + ... + ].
( 0!) (1!) (n!)
D C A B C D B A Vamos calcular, por exemplo,
1 1 1 1 1 1
B D A C D5 = 5! [ − + − + − ]=
44
A B C D
( 0!) (1!) ( 2!) ( 3!) ( 4!) (5!)

D B A C B D C A

C D A B

110 PROMILITARES.COM.BR
ANÁLISE COMBINATÓRIA E PROBABILIDADE

EXERCÍCIOS DE 09. (EN 2017) A é um conjunto com n elementos e B é seu subconjunto

TREINAMENTO
com p elementos, com n > p e n, p ∈  . Determine o número de
conjuntos X tais que B ⊂ X ⊂ A e assinale a opção correta.
a) 2n-p c) 2n+p e) 2n-p-1
b) 2 n-p+1
d) 2 n+p-1

01. Considere o conjunto C = {2, 8, 18, 20, 53, 124, 157, 224, 286,
345, 419, 527}. O número de subconjuntos de três elementos de C 10. (EN 2017) Calcule o número de soluções inteiras não negativas de
que possuem a propriedade “soma dos três elementos é um número x1 + x 2 + x 3 + x4 + x5 + x6 = 20, nas quais pelo menos 3 incógnitas são
ímpar” é nulas, e assinale a opção correta.
a) 94 b) 108 c) 115 d) 132 e) 146 a) 3.332 c) 3.543 e) 3.711
b) 3.420 d) 3.678
02. (AFA 2010) Numa sala de aula, estão presentes 5 alunos e 6
alunas. Para uma determinada atividade, o professor deverá escolher
11. (EN 2014) Qual a quantidade de números inteiros de 4 algarismos
um grupo de 3 dessas alunas e 3 dos alunos. Em seguida, os escolhidos
distintos, sendo dois algarismos pares e dois ímpares que podemos
serão dispostos em círculo de tal forma que alunos do mesmo sexo não
formar, usando algarismos de 1 a 9?
fiquem lado a lado. Isso poderá ocorrer de n. O número n é igual a:
a) 2400 d) 1440
a) 24000 b) 2400 c) 400 d) 200
b) 2000 e) 1200
03. (ITA 2018) Sobre duas retas paralelas r e s são tomados 13 pontos, c) 1840
m pontos em r e n pontos em s, sendo m > n. Com os pontos são
formados todos os triângulos e quadriláteros convexos possíveis. Sabe- 12. (EN 2014) Há 10 postos de gasolina em uma cidade. Desses
se que o quociente entre o número de quadriláteros e o número de 10, exatamente dois vendem gasolina adulterada. Foram sorteados
triângulos é 15/11. Então, os valores de n e m são, respectivamente, aleatoriamente dois desses 10 postos para serem fiscalizados. Qual
a) 2 e 11 c) 4e9 e) 6e7 é a probabilidade de que os dois postos infratores sejam sorteados?
b) 3 e 10 d) 5 e 8 1 1 1
a) c) e)
45 15 30
04. (ITA 2016) Pintam-se N cubos iguais utilizando-se 6 cores 1 2
b) d)
diferentes, uma para cada face. Considerando que cada cubo pode 90 45
ser perfeitamente distinguido dos demais, o maior valor possível de
N é igual a 13. (ITA 2006) Seja U um conjunto não vazio com n elementos, n ≥ 1.
a) 10 b) 15 c) 20 d) 25 e) 30 Seja S um subconjunto de P(U) com a seguinte propriedade:
Se A, B ∈ S, então A ⊂ B ou B ⊂ A.
05. (IME 2016) O valor da soma abaixo é:
Então, o número máximo de elementos que S pode ter é
 2016   2017   2018   2019   2020   2016  a) 2n-1
 + + + + + 
 5   5   5   5   5   6  n (n + 1)
b) , se n for par, e se n for ímpar.
 2020   2021  2022  2 2
a)   c)   e)  
 6   5   5  c) n + 1
 2021 d) 2n - 1
 2020 
b)   d)  
 7   6  e) 2n-1 + 1

14. (ITA 2003) O número de divisores “positivos” de 17640 que, por


06. (IME 2014) Em uma festa de aniversário estão presentes n famílias
sua vez, são divisíveis por 3 é:
com pai, mãe e 2 filhos, além de 2 famílias com pai, mãe e 1 filho.
Organiza-se uma brincadeira que envolve esforço físico, na qual uma a) 24 d) 54
equipe azul enfrentará uma equipe amarela. Para equilibrar a disputa, b) 36 e) 72
uma das equipes terá apenas o pai de uma das famílias, enquanto a
c) 48
outra equipe terá 2 pessoas de uma mesma família, não podendo
incluir o pai. É permitido que o pai enfrente 2 pessoas de sua própria
família. Para que se tenha exatamente 2014 formas distintas de se 15. (EN 2013) Um aspirante da Escola Naval tem, em urna prateleira
organizar a brincadeira, o valor de n deverá ser de sua estante, 2 livros de Cálculo, 3 livros de História e 4 livros de
Eletricidade. De quantas maneiras ele pode dispor estes livros na
a) 17 b) 18 c) 19 d) 20 e) 21 prateleira de forma que os livros de cada disciplina estejam sempre
juntos?
07. (ITA 2012) Deseja-se trocar uma moeda de 25 centavos, usando-se
a) 1728 d) 864
apenas moedas de 1, 5 e 10 centavos. Então, o número de diferentes
maneiras em que a moeda de 25 centavos pode ser trocada é igual a b) 1280 e) 288
a) 6 b) 8 c) 10 d) 12 e) 14 c) 960

08. (ITA 2007) Determine quantos números de 3 algarismos podem 16. (IME 2009) A figura abaixo é composta de 16 quadrados
ser formados com 1, 2, 3, 4, 5, 6 e 7, satisfazendo à seguinte regra: menores. De quantas formas é possível preencher estes quadrados
O número não pode ter algarismos repetidos, exceto quando iniciar com os números 1, 2, 3 e 4, de modo que um número não pode
com 1 ou 2, caso em que o 7 (e apenas o 7) pode aparecer mais de aparecer 2 vezes em:
uma vez. Assinale o resultado obtido. • uma mesma linha.
a) 204 c) 208 e) 212 • uma mesma coluna.
b) 206 d) 210 • cada um dos quatro quadrados demarcados pelas linhas contínuas.

PROMILITARES.COM.BR 111
ANÁLISE COMBINATÓRIA E PROBABILIDADE

04. (ITA 2004) Considere 12 pontos distintos dispostos no plano,


5 dos quais estão numa mesma reta. Qualquer outra reta do plano
contém, no máximo, 2 desses pontos. Quantos triângulos podemos
formar com os vértices nesses pontos?
a) 210 c) 410 e) 521
b) 315 d) 415

05. (FIVE HUNDRED MATHEMATICAL CHALLENGES) Um calendário


de mesa consiste de um dodecaedro regular com um mês diferente
em cada uma das suas 12 faces pentagonais. O número de formas
essencialmente diferentes de dispor os meses nessas faces é:
17. (IME 2008) Cinco equipes concorrem numa competição
a) 12! 12!
automobilística, em que cada equipe possui dois carros. Para a d)
largada são formadas duas colunas de carros lado a lado, de tal forma b) 11! 5
que cada carro da coluna da direita tenha ao seu lado, na coluna 11! 11!
da esquerda, um carro de outra equipe. Determine o número de c) e)
5 6
formações possíveis para a largada.
06. (ITA 2017) Com os elementos 1, 2, ..., 10 são formadas todas as
18. (IME 2010) Três dados iguais, honestos e com seis faces sequências (a1, a2, ..., a7). Escolhendo-se aleatoriamente uma dessas
numeradas de um a seis são lançados simultaneamente. Determine a sequências, a probabilidade de a sequência escolhida não conter
probabilidade de que a soma dos resultados de dois quaisquer deles elementos repetidos é
ser igual ao resultado do terceiro dado.
7! 10!
a) . d) .
107 ⋅ 3! 103 ⋅ 7!
19. (IME 2014) Um professor dá um teste surpresa para uma turma
de 9 alunos, e diz que o teste pode ser feito sozinho ou em grupos 10! 10!
b) . e) .
de 2 alunos. De quantas formas a turma pode se organizar para fazer 107 ⋅ 3! 107
o teste? (Por exemplo, uma turma de 3 alunos pode se organizar de 3!
4 formas e uma turma de 4 alunos pode se organizar de 10 formas). c) .
107 ⋅ 7!

20. (ITA 2013) Quantos tetraedros regulares de mesma dimensão 07. (ITA 2017) Um atirador dispõe de três alvos para acertar. O
podemos distinguir usando 4 cores distintas para pintar todas as suas primeiro deste encontra-se a 30 cm de distância; o segundo, a 40 m;
faces? Cada face só pode ser pintada com uma única cor. o terceiro alvo, a 60 cm. Sabendo que a probabilidade de o atirador
acertar o alvo é inversamente proporcional ao quadrado da distância
EXERCÍCIOS DE 2

COMBATE
e que a probabilidade de ele acertar o primeiro alvo é de , então a
3
probabilidade de acertar ao menos um dos alvos é
120 105
a) . d) .
160 135

01. (IME 1979) Uma rua possui um estacionamento em fila com n 119 119
b) . e) .
vagas demarcadas junto ao meio-fio de um dos lados. n automóveis, 154 144
numerados de 1 a n, devem ser acomodados, sucessivamente, pela 110
ordem numérica no estacionamento. Cada carro deve justapor-se a c) .
144
um carro já estacionado, ou seja, uma vez estacionado o carro 1 em
qualquer uma das vagas, os seguintes se vão colocando imediatamente
à frente do carro mais avançado ou atrás do carro mais recuado. 08. (ITA 2010) Um palco possui 6 refletores de iluminação. Num
Quantas configurações distintas podem ser obtidas desta maneira? certo instante de um espetáculo moderno os refletores são acionados
2
A figura abaixo mostra uma das disposições possíveis com 11 carros. aleatoriamente de modo que, para cada um dos refletores, seja de
3
a probabilidade de ser aceso.
11 10 8 7 6 2 1 3 4 5 9 Então, a probabilidade de que, neste instante, 4 ou 5 refletores sejam
acesos simultaneamente, é igual a
a) 2n c) nn e) 2n-1
16 479
b) n! d) 2n a) . d) .
27 729
49 24 25
02. (IME 2008) De quantas maneiras n bolas idênticas podem ser b) . e) + .
distribuídas em três cestos de cores verde, amarelo e azul? 81 34 35
151
n + 2 c) .
a)   n! e) 3n 243
 2 
c)
3!
n d) (n - 3)!
b)   09. (IME 2016) Os inteiros n e m são sorteados do conjunto
 3 {1, 2, 3, ..., 2016} podendo haver repetição. Qual a probabilidade do
produto n x m ser múltiplo de 12?
03. (IME 2007) Um grupo de nove pessoas, sendo duas delas irmãos,
deverá formar três equipes, com respectivamente dois, três e quatro 5 5 5
a) 12
c) 24
e)
integrantes. Sabendo que os dois irmãos não podem ficar na mesma 144
equipe, o número de equipes que podem ser organizadas é: 5 5
b) d)
a) 288 c) 480 e) 960 18 36
b) 455 d) 910

112 PROMILITARES.COM.BR
ANÁLISE COMBINATÓRIA E PROBABILIDADE

10. (ITA 1991) Uma escola possui 18 professores sendo 7 de 18. (IME 2001) Um comandante de companhia convocou voluntários
Matemática, 3 de Física e 4 de Química. De quantas maneiras para a constituição de 11 patrulhas. Todas elas são formadas pelo
podemos formar comissões de 12 professores de modo que cada uma mesmo número de homens. Cada homem participa de exatamente
contenha exatamente 5 professores de Matemática, no mínimo 2 de duas patrulhas. Cada duas patrulhas têm somente um homem em
Física e no máximo 2 de Química? comum. Determine o número de voluntários e o de integrantes de
a) 875 c) 1995 e) n.d.a. uma patrulha.
b) 1877 d) 2877
19. (IME 2000) Seja o conjunto:
11. (ITA 2018) De uma caixa que contém 10 bolas brancas e 6bolas =D {(k1,k2 ) | 1 ≤ k1 ≤ 13; 1 ≤ k2 ≤ 4; k1,k2 ∈ } .
pretas, são selecionadas ao acaso k bolas. Determine quantos subconjuntos
a) Qual a probabilidade de que exatamente r bolas sejam brancas, L = {( x1,x 2 ) ; ( y1,y 2 ) ; ( z1,z2 ) ; ( t1,t2 ) ; (r1,r2 )} , L ⊂ D, existem com 5 (cinco)
nas condições 0 ≤ k − r ≤ 6 e 0 ≤ k ≤ 10.
elementos distintos, que satisfazem simultaneamente seguintes
6

∑  r   6 − r .
10   6  condições:
b) Use o item (a) para calcular a soma
r =0 a) x1 = y1 = z1; e
b) x1 ≠ t1, x1 ≠ r1, t1 ≠ r1.
12. (ITA 2016) Numa certa brincadeira, um menino dispõe de uma
caixa contendo quatro bolas, cada qual marcada apenas com apenas 20. (IME 1989) Em cada uma das faces de um cubo constrói-se um
uma destas letras: N, S, L e O. Ao retirar aleatoriamente uma bola, ele círculo e, em cada círculo, marcam-se n pontos. Unindo-se estes
vê a letra correspondente e devolve a bola à caixa. Se essa letra for pontos:
N, ele dá um passo na direção Norte, se S, em direção Sul, se L, na
direção Leste e se O, na direção Oeste. a) quantas retas, não contidas numa mesma face do cubo, podem
ser formadas;
Qual a probabilidade de ele voltar para a posição inicial no sexto
passo? b) quantos triângulos, não contidos numa mesma face do cubo,
podem ser formados;
13. (ITA 2014) Seja Ω o espaço amostral que representa todos os c) quantos tetraedros, com base numa das faces do cubo, podem
resultados possíveis do lançamento simultâneo de três dados. Se A ⊂ ser formados;
Ω é o evento para o qual a soma dos resultados dos três dados é igual d) quantos tetraedros com todos os vértices em faces diferentes
a 9 e B ⊂ Ω o evento cuja soma dos resultados é igual a 10, calcule: podem ser formados.
a) n(Ω); b) n(A) e n(B); c) P(A) e P(B). Obs.: Suponha que, se 4 pontos não pertencem a uma mesma face,
então não são coplanares.
14. (IME 2018) Um ônibus escolar transporta n crianças. Sejam A o
evento em que dentro do ônibus tenham crianças de ambos os sexos
GABARITO
e B o evento em que há no máximo uma menina dentro do ônibus.
Determine o valor de n para que os eventos A e B sejam independentes. EXERCÍCIOS DE TREINAMENTO
01. C
15. (IME 2017) Seja A = {1, 2, 3, 4}. O conjunto possui 5 elementos ímpares e 7 pares.
a) Quantas funções de A para A têm exatamente 2 elementos em Para a soma dos elementos de um subconjunto de três elementos ser
seu conjunto imagem? ímpar, devemos ter 3 números ímpares, ou 2 pares e 1 ímpar.
b) Entre as 256 funções de A para A, sorteiam-se as funções f e 5⋅ 4 7⋅6
C53 + C72 ⋅ C15 = + ⋅ 5 = 10 + 105 = 115
g, podendo haver repetição. Qual a probabilidade da função 2 2
composta f  g ser uma função constante?

16. (ARML COMPETITION 2009) Alguns estudantes na aula de 02. B


educação física estão usando moletons azuis, e o resto está usando 5.4 6.5.4
moletons vermelhos. Há exatamente 25 maneiras de escolher um time 1ª) Escolha dos alunos: C53 ×C63 = × = 200
2 3.2.1
com três jogadores que inclua pelo menos um jogador vestido de cada 2ª) Disposição circular:
cor. Determine número de estudantes na aula.

17. (IME 2005) O sistema de segurança de uma casa utiliza um


teclado numérico, conforme ilustrado na figura. Um ladrão observa
de longe e percebe que:
• A senha utilizada possui 4 dígitos.
• O primeiro e o último dígitos encontram-se numa mesma linha.
• O segundo e o terceiro dígitos encontram-se na linha
imediatamente superior.
Calcule o número de senhas que deverão ser experimentadas pelo
ladrão para que com certeza ele consiga entrar na casa.

1 2 3 Disposição dos homens = 2!

4 5 6 Disposição das mulheres = 2!


Para cada arrumação dos homens e cada arrumação das mulheres
7 8 9
existem 3 posições relativas ⇒ 3
0
Total = C5 ×C6 × 2!× 2!× 3 =
3 3
2400
Teclado numérico

PROMILITARES.COM.BR 113
ANÁLISE COMBINATÓRIA E PROBABILIDADE

03. E 06. A
Sejam, respectivamente, x e y, o total de quadriláteros convexos e de (n + 2) ⋅ (3n + 2) = 1007
triângulos que podem ser formados com os pontos dados. (n + 2) ⋅ (3n + 2) = 19 ⋅ 53
Temos: n + 2 = 19 ⇒ n = 17
m n m! n! m ⋅ (m − 1) n ⋅ (n − 1)
x =  ⋅  = ⋅ = ⋅
2 2
    2!⋅ ( m − 2) ! 2!⋅ ( n − 2) ! 2 2 07. D
Sejam x, y e z a quantidade de moedas de 1, 5 e 10 centavos,
n m n! m! respectivamente, então 1⋅ x + 5 ⋅ y + 10 ⋅ z =25 .
y = m⋅  + n⋅  = m⋅ + n⋅ =
 2  2 2!⋅ (n − 2)! 2!⋅ (m − 2)! z= 2 ⇒ ( x, y ) ∈ {( 0,1) ; (5,0 )}
m ⋅ n ⋅ (n − 1) n ⋅ m ⋅ (m − 1) mn
+ = ⋅ (m + n − 2) z =1 ⇒ x + 5y =15 ⇒ ( x, y ) ∈ {( 0,3) ; (5,2) ; (10,1) ; (15,0 )}
2 2 2
m ⋅ (m − 1) n ⋅ (n − 1)
⋅ ( 0,5) ; (5, 4 ) ; (10,3) ; (15,2) ;
x 2 2 z = 0 ⇒ x + 5y = 25 ⇒ ( x, y ) ∈  
=
y mn ( 20,1) ; ( 25,0 ) 
⋅ (m + n − 2)
2 Logo, há um total de 2 + 4 + 6 = 12 possibilidades.
x (m − 1) ⋅ (n − 1)
=
y 2 ⋅ (m + n − 2) 08. E
Números sem algarismos repetidos: 7 ⋅ 6 ⋅ 5 = 210
x 15 m + n = Números que podem possuir algarismos repetidos: 177 e 277
Mas, = e 13, logo,
y 11 Total: 210 + 2 = 212

15 (m − 1) ⋅ (n − 1) 09. A
=
11 2 ⋅ 11 Do enunciado, temos:
30= mn − m − n + 1 {
A = x1, x 2, x 3 , ..., xp−1, xp , ..., xn }
30 = mn − (m + n) + 1
30 = mn − 13 + 1
{
B = x1, x 2, x 3 , ..., xp }
 
mn = 42  
=X x1, x 2, x 3 , ..., xp , −, −, −, ..., − 
Como m > n, n = 6 e m = 7.  

 (n−p) elementos 
04. E Cada um dos (n – p) elementos podem pertencer ou não ao conjunto
X, assim, pelo princípio da multiplicação, há 2n–p possibilidades para
Face superior do cubo: 6 possibilidades de cor
montar o conjunto X.
Face interior do cubo: 5 possibilidades de cor.
Faces Laterais do cubo: (4 – 1)! = 6 (permutação circular) 10. E
Considerando que cada uma das faces pode ser a face superior, o Do enunciado, devemos ter as seguintes situações:
3 incógnitas nulas ou 4 incógnitas nulas ou 5 incógnitas nulas.
número de cubos possíveis é:
Com 3 incógnitas nulas
6 ⋅5 ⋅ 6
=N = 30 6!
6 =
C6,3 = 20 é o total de maneiras de escolher as três incógnitas
3!⋅ 3!
nulas.
05. D Analisemos o caso em que x1 = x2 = x3 = 0. Assim, queremos encontrar
Utilizando a Relação de Stifel, pode-se escrever: o total de soluções inteiras não negativas e não nulas da equação x4
+ x5 + x6 = 20.
 n   n   n + 1
Relação de Stifel →   +  =  Assim, podemos escrever: x 4 = a + 1, x5 =
b + 1e x6 =c + 1.
 p   p + 1  p + 1
 2016   2017   2018   2019   2020   2016  Então,
 + + + + + = a+ 1+ b+ 1+ c + 1 =20
 5   5   5   5   5   6 
a+ b+ c = 17
 2016   2016   2017   2018   2019   2020 
 + + + + + = O total de soluções inteiras não negativas da equação
 5   6   5   5   5   5 
a + b + c = 17, é:
 2017   2017   2018   2019   2020 
 + + + + = 2,17 19! 19 ⋅ 18 ⋅ 17!
 6   5   5   5   5  =
P19 = = 171
2!⋅ 17! 2 ⋅ 17!
 2018   2018   2019   2020   2019   2019  Logo, pelo princípio da multiplicação, há 20 ⋅ 171 = 3420 soluções
 + + + = + +
 6   5   5   5   6   5  para a equação x1 + x2 + x3 + x4 + x5 + x6 = 20 na qual 3 incógnitas
 2020   2020   2020   2021 são nulas.
+ = + = 
 5   6   5   6  Com 4 incógnitas nulas
(n + 2) ⋅ (3n + 2) = 1007 6!
(n + 2) ⋅ (3n + 2) = 19 ⋅ 53 =
C6,4 = 15 é o total de maneiras de escolher as quatro
4!⋅ 2!
n + 2 = 19 ⇒ n = 17 incógnitas nulas.

114 PROMILITARES.COM.BR
ANÁLISE COMBINATÓRIA E PROBABILIDADE

Analisemos o caso em que x1 = x2 = x3 = x4 = 0. Assim, queremos Portanto, o número de maneiras distintas para dispor estes livros na
encontrar o total de soluções inteiras não negativas e não nulas da prateleira será dado por:
equação x5 + x6 = 20. 3!⋅ 2!⋅ 3!⋅ 4! = 6 ⋅ 2 ⋅ 6 ⋅ 24 = 1728
Assim, podemos escrever:
x5 = d + 1 e x6 = e + 1.
16. Há 4! = 24 maneiras de preencher o quadrado A com os números
Então, 1, 2, 3, 4. Feito isso, vamos agora preencher o quadrado D. Colocamos
d + 1+ e + 1 =20 o número 1 em qualquer posição do quadrado D (4 maneiras). Depois
d+e = 18 disso, teremos uma situação como a seguinte:

O total de soluções inteiras não negativas da equação d + e = 18, é:


18 19! 19 ⋅ 18!
=
P19 = = 19
18! 18!

Logo, pelo princípio da multiplicação, há 15 ⋅ 19 = 285 soluções para a


equação x1 + x2 + x3 + x4 + x5 + x6 = 20 na qual 4 incógnitas são nulas.
Agora, o número a não pode ser 2, pois ficaríamos sem opção para
Com 5 incógnitas nulas x, e o número b não pode ser 3, pois ficaríamos sem opção para y.
6! assim, nosso próximo passo é escolher o valor de c entre os números
C=
6,5 = 6 é o total de maneiras de escolher as quatro incógnitas 2, 3 e 4, o que determina imediatamente a e b devido às restrições
5!⋅ 1!
nulas. que acabamos de observar. É fácil ver que, para as demais posições do
Analisemos o caso em que x1 = x2 = x3 = x4 = x5 = 0. Assim, queremos número 1 em D, obtemos uma situaççao análoga. Logo há 4 × 3 = 12
encontrar o total de soluções inteiras não negativas e não nulas da maneiras de completar o quadrado D depois de preenchido o A.
equação x6 = 20. Feito isso, todos os espações que sobraram têm uma única maneira de
Só há uma solução para esse caso. serem preenchidos. Assim, o número total de maneiras de preencher
Logo, pelo princípio da multiplicação, há 6 ⋅ 1 = 6 soluções para a os quadrados é 24 × 12 = 288.
equação x1 + x2 + x3 + x4 + x5 + x6 = 20 na qual 5 incógnitas são nulas.
17. Seja Ai o conjunto das formações de largada em que dois carros
Portanto, o total de soluções inteiras não negativas de da equipe i estão lado a lado. Vamos calcular |A1∪A2∪A3∪A4∪A5| .
x1 + x2 + x3 + x4 + x5 + x6 = 20, nas quais pelo menos 3 incógnitas são Temos |Ai| = 5.2.8!, |A1∩Aj| = 20.22.6! |Ai∩Aj∩Ak|= 60.23.4 !,
nulas é 3420 + 285 + 6 = 3711. |Ai∩Aj∩Ak∩Al| = 120.24.2! e |Ai∩Aj∩Ak∩Al∩Am| = 120.25 .
Logo, |A1∪A2∪A3∪A4∪A5| = 5.5.2.8!-C (5,2).20.22.6!+C (5,3).60.23.4!-
11. D
C (5,4).120.24.2!+C (5,5) .120.25, o que nos dá |A1∪A2∪A3A4A5| =
Nos algarismos de 1 a 9 tem-se 4 algarismos pares e 5 algarismos
1539840. Mas esse último resultado conta o número de configurações
ímpares. Deve-se escolher 2 algarismos ímpares e 2 pares, permutando-
em que há pelo menos dois carros da mesma equipe lado a lado.
os. Assim, pode-se escrever:
Logo, o número de formações sem dois carros da mesma equipe lado
5 ⋅ 4 ⋅ 3! 4 ⋅ 3 ⋅ 2! a lado é 3628800 – 1539840 = 2088960. Note que consideramos dois
C52 ⋅ C24 =
⋅ 4! ⋅ =
⋅ 4! 1440 carros da mesma equipe como distintos .Se considerá-los como iguais,
3!⋅ 2! 2!⋅ 2!
devemos dividir a resposta por 32, o que nos dá 65280.
12. A
Pode-se escrever: 18. Temos os seguintes resultados possíveis:
2 10 ⋅ 9 ⋅ 8! 2 3!
Possibilidades de escolha de 2 postos → C=
10 = 45 (1,1 2) ------------------------------------------------ P3= = 3
8!⋅ 2! 2!
(1,2,3) ------------------------------------------------P3 = 3! = 6
Possibilidade de escolha dos 2 postos infratores → 1
1 (1,3,4)-------------------------------------------------P3 = 3! = 6
P(A) =
45 2 3!
(2,2,4)------------------------------------------------- P3= = 3
2!
13. C (1,4,5)-------------------------------------------------P3 = 3! = 6
Seja U = {u1, u2, ...un}. Então S = {{}, {u1},{u1, u2, ..., um}} tem n + 1 (2,3,5)-------------------------------------------------P3 = 3! = 6
elementos e satisfaz a propriedade do enunciado.
Tome agora S ⊂ P(U) com n(S) > n + 1. Então, como todo subconjunto (1,5,6)-------------------------------------------------P3 = 3! = 6
de U tem no máximo n elementos, pelo princípio da Casa dos Pombos, (2,4,6)-------------------------------------------------P3 = 3! = 6
existem X, Y ∈ S com X ≠ Y e n(X) = n(Y). Mas então é impossível que 3!
2
X ⊂ Y ou Y ⊂ X. Logo S não possui a propriedade pedida. (3,3,6) ------------------------------------------------ P3= = 3
2!
Assim, o maior número de elementos que um conjunto com a Total = 3.3 + 6.6 = 45.
propriedade pedida pode ter é n + 1.
45 5
Logo, a probabilidade será P = = .
14. C 6.6.6 24
Basta dividir 17640 por 3, isto é, calcular os divisores positivos de
5880 = 2³ ⋅ 3 ⋅ 5 ⋅ 7² que tem 48 divisores naturais. 19. Com nenhum grupo de dois alunos: 1
9
15. A Com um grupo de dois alunos:   = 36
Número de permutações das três disciplinas: 3!  2
Número de permutações dos livros de Cálculo: 2!  9 7
Número de permutações dos livros de História: 3!  ⋅ 
2 2
Número de permutações dos livros de Eletricidade: 4! Com dois grupos de dois alunos:     = 378
2

PROMILITARES.COM.BR 115
ANÁLISE COMBINATÓRIA E PROBABILIDADE

 9   7 5 Caso 4: três elementos


 ⋅ ⋅  nº funções g = 4 4 − 4 − 84 − 24 = 144
2 2 2 Total= 144 ⋅ 4 ⋅ 4= 144 ⋅ 16
Com três grupos de dois alunos:       = 1260
3! nº funções f= 4 ⋅ 4
 9   7 5   3 Assim, a probabilidade da função composta f  g ser uma função
 ⋅ ⋅ ⋅  constante será:
Com 4 grupos de dois alunos:  2   2   2   2  (6 + 64 + 336=
+ 144 ) ⋅ 16
= 945 =P(X)
Total casos favoráveis
= =
550 ⋅ 4 2 275
4! Total casospossíveis 44 ⋅ 44 256 ⋅ 16 ⋅ 4 2 256 ⋅ 8
Portanto, o número de formas para se organizar o trabalho é:
275
1+ 36 + 378 + 1260 + 945 =
2620. P(X) =
2048

20. Considerando as 4 cores distintas, iremos dividir o problema em 16. 7


quatro casos.
Primeiro caso (apenas uma cor): Todas as faces da mesma cor – 4 17. 171
possibilidades. 18. 10
Segundo caso (duas cores): 3 faces da mesma cor e uma diferente: 19. 54912
4.3 = 12; 2 cores diferentes, cada uma delas pintando duas faces:
20.
C4,2 = 6. 2
Terceiro caso (três cores): 3 cores; uma delas pintando duas faces, e as a) 15n ;
outras duas pintando uma face cada: 4.C3,2 = 12. b) 5n ( 7n − 3) ;
2

c) 5n (n − 1)(n − 2) ;
2
Quarto caso (4 cores distintas; fixando duas cores, teremos apenas
4
duas possibilidades para as outras): 2. d) 15n

Logo, o total de tetraedros será dado por: 4 + 12 + 6 + 12 + 2 = 36. ANOTAÇÕES

EXERCÍCIOS DE COMBATE
01. E 03. D 05. C 07. E 09. B
02. A 04. A 06. B 08. A 10. D

11. 10   6 
 ⋅ 
r k −r
a) A probabilidade pedida é    ;
16
 
6
10   6   
b) ∑   ⋅  =8008. k
r =0  r   6 − r 

25
12.
256
13.
a) 216
b) n (A) = 25
n (B) = 27
25 1
c) P(A) = e P(B) =
216 8

14. n = 3

15.
a) Considerando o conjunto dado, há 6 formas distintas de se escolher
dois elementos ( C4 ) . Existem 24 funções do conjunto destes dois
2

elementos. Em 2 casos o conjunto imagem tem apenas um elemento.


Assim, o total de funções será:
6 ⋅ ( 24 − 2) =
84

b) A função g poderá ter imagem com:


Caso 1: quatro elementos
nº funções =
g 4! = 24
Total = 24 ⋅ 4 = 6 ⋅ 16
nº funções f = 4
Caso 2: um elemento
nº funções g = 4
Total = 4 ⋅ 4 ⋅ 4 3 = 45 = 64 ⋅ 16
nº funções f= 4 ⋅ 4 3
Caso 3: dois elementos
nº funções g =6 ⋅ ( 24 − 2) =84
Total = 84 ⋅ 4 ⋅ 4 2 = 84 ⋅ 4 3 = 336 ⋅ 16
nº funções f= 4 ⋅ 4 2

116 PROMILITARES.COM.BR
SEQUÊNCIAS

SEQUÊNCIAS É necessário então dispor de um método com base lógica que


permita decidir sobre a validade ou não de uma indução vulgar.
Uma sequência de números reais é uma função a:  → , que
para cada n ∈  associa um número an pertencente aos números reais, … + ( 2n − 1) n2
Por exemplo 1 + 3 + 5 += (n ∈  )
*

chamado n-ésimo termo. Vamos verificar se é verdadeira para os 3 primeiros números


É denominada finita, a sequência que possui um número limitado naturais.
de termos. Do contrário a sequência é chamada infinita. Usualmente Para n = 1 ⇒ 1 = 12
representamos estes casos respectivamente como ( a1, a2 , a3 ,…, an ) e Para n = 2 ⇒ 1 + 3 = 4 = 22
( a1, a2 , a3 ,…, an ,…) . Para n = 3 ⇒ 1 + 3 + 5 = 9 = 32
Nem sempre uma dada sequência apresenta uma regra ou lei Para verificarmos se a relação é válida para qualquer n ∈ *
de formação definida ou conhecida. Nos casos em que tal regra é deveremos utilizar o princípio da indução finita, que consiste no
definida, ela pode ser apresentada principalmente das seguintes seguinte:
maneiras:
Uma proposição P(n), aplicável aos números naturais n, é
• Por meio de uma propriedade exclusiva dos termos da verdadeira para todo n ∈  , n > n0 , quando
sequência.
I. P (n0 ) é verdadeira, isto é, a propriedade é válida para n = n0
Exemplo: a sequência dos números primos, (an) = (2, 3, 5, 7, 11, …).
II. Se k ∈  , k > n0 e P(k) é verdadeira, então P(k + 1) também
• Por meio de uma expressão matemática que associa a cada n
é verdadeira.
um determinado valor de an.
Exemplo: an = 2n − 3, (an) = (−1, 1, 3, 5, 7, …).
Para 1 + 3 + 5 + … + ( 2n=
− 1) n2 (n ∈  )
*
provamos que é
verdadeira para n = 1, n = 2 e n = 3.
• Por meio de uma relação de recorrência que, a partir de um
Vamos admitir que é válida para P(k), com k ∈ * , ou seja
certo termo, determina cada termo seguinte em função dos
anteriores. 1+ 3 + 5 + … + ( 2k − 1) = k 2 (I)
Exemplo: a sequência em que o primeiro termo é a1 = 3 e cada termo Que será a nossa hipótese de indução.
a partir do segundo é dado por an = 2 an-1 + 1, (an) = (3, 7, 15, 31, 63, …). Agora provaremos a validade para P(k + 1).
1 + 3 + 5 + … + ( 2 (k − 1) − 1) + ( 2k − 1) + ( 2 (k + 1) − 1) = (k + 1)
2

RECORRÊNCIAS
Porém decorre de (I) que 1 + 3 + 5 + … + ( 2k − 1) = k 2 , assim
Uma relação de recorrência ou, como também é chamada, uma
1 + 3 + 5 + … + ( 2 (k − 1) − 1) + ( 2k − 1) + ( 2 (k + 1) − 1) = (k + 1)
2
equação de recorrência, é uma relação que determina cada termo de  
uma dada sequência, a partir de certo termo, em função dos termos k2
anteriores. k 2 + ( 2k + 2 − 1) = (k + 1)
2

Resolver uma relação ou equação de recorrência, significa k + ( 2k + 1) = (k + 1)


2 2

encontrar uma fórmula fechada para a recorrência, ou seja, uma


Ou seja, a igualdade é válida (k + 1) =(k + 1) .
2 2
expressão que forneça cada termo an da sequência em função apenas
de n e não dos termos anteriores. Tal expressão é chamada solução Vamos ver mais um exemplo.
da recorrência.
 1 1  1
Verificar se (1+ 1) 1+  1+  .…. 1+  =n + 1 é verdade.
 2  4   n
3
PRINCÍPIO DA INDUÇÃO FINITA Verificamos para n = 1 teremos 2, para n = 2 teremos 2. = 3 e
vamos supor que é válido para k. 2
A indução vulgar (generalização de propriedade após verificação
de que a propriedade é válida em alguns casos particulares) pode
(1+ 1) 1+
1 1  1
conduzir a enganos.  1 +  .…. 1+  =k + 1
 2  3   k
n3 3n2 7n
A relação y =− + − + 3 , definida para todo n ∈ *, temos E agora vamos verificar para (k+1)
6 2 3
Para n = 1 temos y = 2
(1 + 1) 1 +
1 1  1 1 
 1 +  .…. 1 +  1 +  = (k + 1) + 1
Para n = 2 temos y = 3  2 
 4
   k  k + 1
k +1
Para n = 3 temos y = 5
k + 1 + 1
Para n = 4 temos y = 7 (k + 1)  =k+2
 k +1 
Podemos ter a conclusão precipitada que para qualquer n teremos k +2=k +2
como resultado y sempre um número primo. Porém isso falha para
n = 5, pois teremos Onde verificamos que é verdadeira.
Exemplo:
53 3.52 7.5 −125 + 225 − 70 + 18
y =− + − +3= =8 1 1 1 1 n
+ + +…+ = ,∀ n∈ 
*
6 2 3 6 Verificar que
1.2 2.3 3.4 n. (n + 1) n + 1

PROMILITARES.COM.BR 119
SEQUÊNCIAS

n = 1 temos
1 SEQUÊNCIAS E SÉRIES NUMÉRICAS
2
1 1 4 2
n = 2 temos + = =
DEFINIÇÃO
2 6 6 3 Uma sequência numérica (an)n∈ é uma função, cujo domínio é
Supondo que é válido para k o conjunto dos números naturais, tal que a cada número natural n
1 1 1 1 k é associado um número real, chamado n-ésimo termo da sequência
+ + +…+ =
1.2 2.3 3.4 k. (k + 1) k + 1 numérica.
F: → 
E verificar para (k + 1)
n  F(n) = an
1 1 1 1 1 k +1
+ + +…+ + =
1.2 2.3 3.4 k. (k + 1) (k + 1) . ( (k + 1) + 1) (k + 1) + 1 Exemplo:
   F : → 
k
2
k +1 n  F(n) =
3n
k 1 k +1
+ =
k + 1 (k + 1) . (k + 2) k + 2 Geralmente representa-se uma sequência pelo seu termo geral,
2
k 1 k +1 =
no caso anterior, podemos escrever an , n ∈ .
+ = 3n
k +1 (k + 1) .(k + 2) k+2
( k + 2) 1 Assim teremos a0 = 2
k ( k + 2) + 1 k + 1 a1 =
2
=
(k + 1)(k + 2) k + 2 3
2
k 2 + 2k + 1 k +1 a2 =
= 9
(k + 1)(k + 2) k + 2 ..........
(k + 1)
2
k +1
=
(k + 1)(k + 2) k + 2 DEFINIÇÃO
k +1 k +1 Uma sequência numérica (an)n∈ é convergente se e somente se e
=
k+2 k+2 existir um número real a, tal que lim an = a e chamamos o número
n →+∞
real a de limite da sequência.
Onde verificamos que a igualdade é verdadeira e a relação válida.
Exemplo:
Exemplo:
Verificar se é válida a relação 2 2
=
A sequência an , n ∈  é convergente, pois, lim n = 0 .
n (n + 1)(n + 2) , 3n n →+∞ 3
1.2 + 2.3 + 3.4 + … + n (n + 1) = ∀ n∈ * .
3
Para n = 1 temos como resultado 2. DEFINIÇÃO
Seja (an)n∈ uma sequência numérica e (sn)n∈ a sequência numérica
2. ( 2 + 1)( 2 + 2) n

∑ a , n ∈ .
Para =
n = 2 temos 8 = 8.
3 definida=
por sn k
k =1
Supondo que é válido para k teremos n

A sequência (s ) , ou ∑ a , n ∈  , chamamos de série numérica.


k (k + 1)(k + 2)
n n∈ k

1.2 + 2.3 + 3.4 + … + k (k + 1) = k =1

3 Exemplo:
Assim para (k + 1) teremos Seja an = 2n + 1, n ∈ , a sequência dos números naturais
1.2 + 2.3 + 3.4 + … + k (k + 1)+ (k + 1) (k + 1) + 1 = ímpares, define-se a série dada pela soma dos n primeiros naturais
 ímpares, ou seja,
n

∑ (2k + 1) , n ∈ 
k (k +1)(k + 2)
3 sn= *

(k + 1) (k + 1) + 1 (k + 1) + 2 k =1


=
3 Assim teremos
k (k + 1)(k + 2) (k + 1)(k + 2)(k + 3) s1= 1= 1
+ (k + 1)(k + 2) =
3 3 s2 =1 + 3 = 4
k (k + 1)(k + 2) + 3 (k + 1)(k + 2) (k + 1)(k + 2)(k + 3) s3 =1 + 3 + 5 = 9
=
3 3
......
(k + 1)(k + 2)  (k + 3) = (k + 1)(k + 2)(k + 3) sn = 1+ 3 + 5 + ... + (2n + 1) = n2
3 3
Donde verificamos que a relação é válida.
DEFINIÇÃO
Uma série numérica (sn)n∈ é convergente se e somente se e existir
um número real s, tal que lim sn = s e chamamos o número real s
n →+∞
de limite da série.

120 PROMILITARES.COM.BR
SEQUÊNCIAS

SEQUÊNCIAS E SÉRIES DE FUNÇÕES DEFINIÇÃO


Uma série de funções
DEFINIÇÃO sn:  → 
Uma sequência de funções é convergente se e somente se e existir uma função real s, tal que
fn :  →  = s (x) , ∀ x ∈ 
lim sn (x)
n →+∞
x  fn (x)
e chamamos a função real s de limite da série.
é uma função, cujo domínio é o conjunto dos números naturais, tal Exemplo:
que a cada número natural n é associado uma função real, chamado Seja
n-ésimo termo da sequência de funções.
F : →  fn :  + → 
n  F(n) = fn x  =
fn (x) xn , n ∈  *

Exemplo: e
F : →  sn :  + → 
n
n  F(n) = fn

Onde fn :  → 
x  sn (x) = ∑xk =1
k

x  fn (x) = xn Então

Geralmente representa-se uma sequência de funções s : + → 


pelo seu termo geral, no caso anterior, podemos escrever  x
 ,0 < x <1
fn :  →  x  s(x) =  1 − x
 + ∞ , x ≥ 1
x  =
fn (x) xn , n ∈ .

Repare que fixando o valor do número real x obtemos uma Neste caso dizemos que o raio de convergência da série é R = 1,
sequência numérica. pois, a série é convergente para 0 < x < 1.

DEFINIÇÃO SOMATÓRIO
Uma sequência de funções O somatório é uma notação abreviada para indicar uma soma.
n
fn:  → 
é convergente se e somente se ∀ x ∈  a sequência numérica
O símbolo ∑ a , que se lê, somatório de a , variando i de 1 até n,
i =1
i i

(fn(x))n∈ for convergente, neste caso escrevemos lim fn (x) = f(x) . representa a soma de todos os valores de ai, quando se atribuem a i os
n →+∞
valores inteiros 1, 2, 3, ..., n.
E chamamos a função f:  →  n

O limite da sequência de funções.


Exemplo:
∑a = a + a + a +  + a
i =1
i 1 2 3 n

Seja o índice i é o índice do somatório (índice mudo) e assume todos os


+ valores inteiros de 1 a n chamados, respectivamente, limite inferior e
fn :  →  limite superior do índice i.
x  f=
n (x) x n , n ∈ * O número de parcelas do somatório é igual à diferença entre os
limites superior e inferior do índice mais uma unidade..
 0, 0≤ x <1
Então lim =  O somatório não precisa necessariamente ter 1 como limite
fn (x) =1, x 1
n →+∞
 +∞ ,x >1 inferior. Dessa fora, sendo n ≥ m dois inteiros, temos:
 n

DEFINIÇÃO ∑a = a
i=m
i m + am +1 + am + 2 +  + an

Seja n

fn:  →  O somatório ∑ (2i + 3) =5 + 7 + 9 +  + (2n + 3) =n ⋅ (n + 4)


i =1
indica
uma sequência de funções e a soma dos n primeiros termos de uma progressão aritmética de
sn:  →  primeiro termo 3 e razão 2.
a sequência de funções definida por
sn :  →  PROPRIEDADES
n n n n

x → s=
n (x) ∑ f (x) , n ∈ 
k =1
k
(i)
=i 1
∑ (a + b )= ∑ a + ∑ b
i i i
=i 1 =i 1
i

n
n
(ii) ∑ a= n⋅a
A sequência de funções (sn)n∈ , ou
série de funções.
∑ f , n ∈  , chamamos de
k =1
k i=1

n n
(iii)
i
=i 1 =i 1
∑ (k ⋅ a ) =k ⋅ ∑ a i

PROMILITARES.COM.BR 121
SEQUÊNCIAS

PRODUTÓRIO SOMA DOS TERMOS DE UMA


O produtório é uma notação abreviada para indicar um produto.
n
PROGRESSÃO ARITMÉTICO-
O símbolo
∏ a , que se lê, produtório de a , variando i de 1 até n,
i i
GEOMÉTRICA INFINITA
i =1
a (1 − q ) rq (1 − nq + (n − 1) q )
1
n n −1 n
representa o produto de todos os valores de ai , quando se atribuem a Vimos=
que S = . Se |q| < 1 e
(1 − q)
n
i os valores inteiros 1, 2, 3, ... , n. 1− q 2

n a1 rq
n → ∞, a soma Sn tenderá para o número 1 − q +
∏a = a ⋅ a ⋅ a ⋅ ⋅ a
i =1
i 1 2 3 n
a1 rq
(1 − q)
2 e escrevemos

=S lim = Sn + . Diremos então que S é a “soma” da série


n →∞ 1 − q (1 − q)2
o índice i é o índice do produtório (índice mudo) e assume todos os

valores inteiros de 1 a n chamados, respectivamente, limite inferior e aritmético-geométrica a1 + a2 + a3 +  + an +  ou S = an.


n =1
limite superior do índice i. Onde Sn é a soma dos n primeiros termos de uma progressão
O número de fatores do produtório é igual à diferença entre os aritmético-geométrica com parâmetro q tal que |q| < 1.
limites superior e inferior do índice mais uma unidade..
O produtório não precisa necessariamente ter 1 como limite PROGRESSÕES ARITMÉTICAS DE
inferior. Dessa fora, sendo n ≥ m dois inteiros, temos:
n ORDEM SUPERIOR
∏a = a i m ⋅ am +1 ⋅ am + 2 ⋅  ⋅ an
i=m
PA DE 2ª ORDEM
n
É uma sequência numérica na qual a diferença entre os termos
∏ 3 =3 ⋅ 3 ⋅ 3 ⋅  ⋅ 3 =3
n(n +1)
O produtório i 2 3 n 2
indica o produto dos consecutivos forma uma PA não estacionária (r ≠ 0).
i =1
n primeiros termos de uma progressão geométrica de primeiro termo Exemplo:
3 e razão 3.
1 4 9 16 25 (2ª ordem)
PROPRIEDADES 3 5 7 9 (PA)
n n n

(i)
=i 1
∏ (a ⋅ b=) ∏ a ⋅ ∏b
i i
=i 1=i 1
i i
O termo geral de uma PA de 2ª ordem é um polinômio do 2º grau
em n.
n
p(n) = an2 +bn +c
(ii) ∏a = a
i =1
n

A sequência formada pela soma dos n primeiros termos de uma


n n PA é uma PA de 2ª ordem. Isso pode ser verificado na fórmula da soma

=i 1=i 1
(iii) ∏ (k ⋅ a ) = k ⋅ ∏ ai
n
i dos termos da PA que é de 2º grau em n.

PA DE ORDEM K
SOMA DOS TERMOS DE UMA É uma sequência numérica na qual a diferença entre os termos
PROGRESSÃO ARITMÉTICO consecutivos forma uma PA de ordem k – 1.

GEOMÉTRICA FINITA Exemplo:

Uma PA-PG é uma sequência da forma (an × bn), onde (an) é uma 1 8 27 64 125 216 (3ª ordem)
PA e (bn) é uma PG.
O termo geral é da forma (a1 + (n - 1)r) ⋅ qn-1 7 19 37 61 91 (2ª ordem)

Teorema: 12 18 24 30 (PA)

A soma dos n primeiros termos de uma progressão aritmético O termo geral de uma PA de ordem k é um polinômio de grau k
geométrica é dada por em n.

a1 ⋅ (1 − qn ) rq(1 − nqn −1 + (n − 1)qn ) p(n) = αknk + αk−1nk−1 + ... +α1n + α0


=Sn +
1− q (1 − q)2 A sequência formada pelas somas nos n primeiros termos de uma
PA de ordem k é uma PA de ordem k + 1.
Demonstração:
Sn =a1 + (a1 + r) ⋅ q + (a1 + 2r) ⋅ q2 + ... + (a1 + (n − 2)r) ⋅ qn − 2 + (a1 + (n − 1)r) ⋅ qn −1(1) SOMA DAS POTÊNCIAS DE GRAU K DE UMA
q ⋅ Sn =a1 ⋅ q + (a1 + r) ⋅ q2 + (a1 + 2r) ⋅ q3 + ... + (a1 + (n − 2)r) ⋅ qn −1 + (a1 + (n − 1)r) ⋅ qn (2) PA
(1) − (2) A soma das potências de grau k de n termos de uma PA é um
(1 − q) ⋅ Sn = a1 + (rq + rq2 + ... + rqn − 2 + rqn −1) − (a1 + (n − 1)r) ⋅ qn polinômio de grau (k +1) em n.
rq(1 − qn −1)
Como rq + rq2 + ... + rqn − 2 + rqn −1 = ,temos
Sn = a1k + ak2 +  + akn−1 + akn = αk +1nk +1 + αknk +  + α1n + α 0
1− q
Assim, para calcular a soma dos quadrados dos n primeiros
rq(1 − qn −1) números naturais positivos, basta fazer
(1 − q) ⋅ Sn = a1 + − (a1 + (n − 1)r) ⋅ qn
1− q
Sn =12 + 22 + 32 +  + n2 = an3 + bn2 + cn + d
a1 ⋅ (1 − qn ) rq(1 − nqn −1 + (n − 1)qn )
=Sn + n=1⇒a+b+c+d=1
1− q (1 − q)2
n = 2 ⇒ 8a + 4b + 2c + d = 5
n = 3 ⇒ 27a + 9b + 3c + d = 14
n = 4 ⇒ 64a + 16b + 4c + d = 30

122 PROMILITARES.COM.BR
SEQUÊNCIAS

1 1 1 05. (EN 2016) Seja q = (cos 5°) ⋅ (cos 20°) ⋅ (cos 40º) ⋅ (cos 85) a razão de
⇒ a= , b= , c= ed=0 1
3 2 6 uma progressão geométrica infinita com termo inicial a0 = . Sendo
4
1 3 1 2 1
⇒ Sn =1 + 2 + 3 +  + n = n + n + n
2 2 2 2
assim, é correto afirmar que a soma dos termos dessa progressão vale:
3 2 6
1 3 7
a) c) e)
15 15 15
EXERCÍCIOS DE 2 4

TREINAMENTO
b) d)
15 15

06. (IME 2015) A soma dos termos de uma progressão aritmética é


244. O primeiro termo, a razão e o número de termos formam, nessa
01. (IME 2018) Sejam x1, x2, x3 e x4 os quatro primeiros termos ordem, outra progressão aritmética de razão 1. Determine a razão da
de uma PA com x1 = x e razão r, com x, r ∈ . O determinante de primeira progressão aritmética.
 x1 x1 x1 x1  a) 7 b) 8 c) 9 d) 10 e) 11
x x x x 
 1 2 2 2
é
 x1 x 2 x 3 x 3 
  07. (ITA 2015) Seja (a1,a2 ,a3 ,...) a sequência definida da seguinte
 x1 x 2 x 3 x 4 
forma: a1 = 1, a2 = 1 e =
an an−1 + an−2 para n ≥ 3. Considere as
a) 0 c) x4 ⋅ r3 e) x ⋅ r3
b) x ⋅ r4
d) x ⋅ r 4 afirmações a seguir:
I. Existem três termos consecutivos, ap, ap+1, ap+2, que, nesta ordem,
02. (ITA 2018) Uma progressão aritmética (a1, a2, ..., an) satisfaz a formam uma progressão geométrica.
propriedade: para cada n ∈ , a soma da progressão é igual a 2n + II. a7 é um número primo.
 a1 a2 a3  III. Se n é múltiplo de 3, então an é par.
 a a5 a6  é É (são) verdadeira(s)
5n. Nessas condições, o determinante da matriz  4
a7 + 2 a8 a9  a) apenas II. d) apenas II e III.
a) –96. d) 99.
b) –85. e) 115. b) apenas I e II. e) I, II e III.
c) 63. c) apenas I e III.

03. (ITA 2017) Considere dois círculos no primeiro quadrante: 08. (EN 2015) A soma dos três primeiros termos de uma PG crescente
vale 13 e a soma dos seus quadrados 91. Justapondo-se esses termos,
π
• C1 com centro (x1, y1), raio r1 e área . obtém-se um número de três algarismos. Pode-se afirmar que o resto
16 da divisão desse número pelo inteiro 23 vale:
• C2 com centro (x2, y2), raio r2 e área 144π. a) 1 b) 4 c) 8 d) 9 e) 11

Sabendo que (x1, y1, r1) e (x2, y2, r2) são duas progressões geométricas
7 1 1+ 2 1+ 2 + 3
com somas dos termos iguais a e 21, respectivamente, então a 09. (EN 2014) Considere a sequência x1 = ; x2 = ; x3 = ;
4 2 1+ 2 1+ 2 + 4
distância entre os centros de C1 e C2 é igual a
1+ 2 + 3 + 4
123 131 137 x4 = ; . O valor de xn é
a) . c) . e) . 1+ 2 + 4 + 8
2 2 2
n +1 n(n + 1) n(n + 1)
129 135 a) c) e)
b) . d) . 2 2n − 1 2(2n − 1)
2 2 n(n − 1) n(n + 1)
b) d)
2n 2n
04. (ITA 2017) Das afirmações:
I. Todo número inteiro positivo pode ser escrito, de maneira
10. (IME 2014) Em uma progressão aritmética crescente, a soma
única, na forma 2k −1(2m − 1), em que k e m são inteiros de três termos consecutivos é S1 e a soma de seus quadrados é S2.
positivos. Sabe-se que os dois maiores desses três termos são raízes da equação
 1
x2 − S1 x +  S2 −  =
0. A razão desta PA é
 π  2
II. Existe um número x ∈ 0,  de tal modo que os números
 2 a) 1 c) 6 e) 1
6
 π  π  3π 
a1 = senx,
= a2 sen  x + = e a4 sen  x +
, a3 sen  x +  =  6 6
 4  2  4  b) d)
6 3
estejam, nesta ordem, em progressão geométrica.
III. Existe um número inteiro primo p tal que p é um número
11. (EN 2013) Sabendo que= π π π 
racional. b cos + + + ...  então o valor
 3 6 12 
é (são) verdadeira(s) de log2 b é
a) apenas I. d) apenas I e II. a) 1 d) –2
b) apenas II. e) todas. b) 0 e) 3
c) apenas III. c) –1

PROMILITARES.COM.BR 123
SEQUÊNCIAS

12. (IME 2013) Entre os números 3 e 192 insere-se igual número de  k −3
termos de uma progressão aritmética e de uma progressão geométrica 16. (IME 2018) Seja a matriz A =   , com k real.
com razão r e q, respectivamente, onde r e q são números inteiros. O 4 2 
número 3 e o número 192 participam destas duas progressões. Sabe- Determine a faixa de valores de k para que exista uma matriz
8
 1 1 de números reais P tal que as condições abaixo sejam atendidas
se que o terceiro termo de 1+  , em potências crescentes de , simultaneamente:
 q q
r a) A TP + PA =I em que A T é a transposta da matriz A e I é a matriz
é . O segundo termo da progressão aritmética é
9q identidade;
a) 12 d) 99 b) P seja simétrica;
b) 48 e) 129 c) p11 > 0, em que p11 é o elemento da linha 1 e coluna 1 de P; e

c) 66 d) |P|> 0, em que |P| é o determinante da matriz P.

π π π  17. (ITA 2018) Um poliedro convexo tem faces triangulares e


=
13. (EN 2013) Sabendo que b sec3  + + + ...  então, o valor
 3 6 12  quadrangulares. Sabe-se que o número de arestas, o número de faces
de log2 b é triangulares e o número de faces quadrangulares formam, nessa
a) 8 d) 1 ordem, uma progressão aritmética de razão –5. Determine o número
de vértices do poliedro.
b) 4 e) 0
c) 3 18. (IME 2018) Seja um cubo regular, onde os centros de suas
faces são vértices de um octaedro. Por sua vez, os centros das
14. (EN 2012) Os números reais a, b, c, d, f, g, h constituem, nesta faces deste octaedro formado são vértices de outro cubo. Obtendo
consecutivamente octaedros e cubos infinitamente, determine a razão
y
da soma do volume de todos os poliedros inscritos pelo volume do
 2 9 det A
ordem, uma progressão aritmética. Se = e lim 1+  , onde A cubo inicial.
y→+∞
 y
1 a a2  n 19. (IME 2018) Sejam a, b, c e d números reais positivos diferentes
  +∞
 1
é a matriz 1 b b2  e h = ∑   , então o valor de
n= 3  4 
de 1. Temos que loga d, logb d e logc d são termos consecutivos de
1 d d2 
  uma progressão geométrica e que a, b e c formam uma progressão
1 15
a) − d)
3 16 aritmética em que a < b < c.

21 31 =
Sabendo-se que b bloga b − a, determine:
b) − e)
16 48 a) Os valores de a, b e c;
b) As razões das progressões aritmética e geométrica, r e q,
49
c) − respectivamente.
48
π π
=
20. (ITA 2018) Seja z cos + isen . Pedem-se:
15. (ITA 2012) 7 7
Notações: kπ kπ
a) Use a propriedade zk =
cos + isen , k ∈ , para expressar
 : Conjunto dos números naturais; 7 7
 : Conjunto dos números reais; π 3π 5π
cos , cos e cos em função de z.
 + : Conjunto dos números reais não negativos; 7 7 7
a π 3π 5π
i: unidade imaginária; i2 = –1; b) Determine inteiros a e b tais que =cos + cos + cos .
P (A): conjunto de todos os subconjuntos do conjunto A; b 7 7 7
n (A): número de elementos do conjunto finito A;
EXERCÍCIOS DE

COMBATE
AB : segmento de reta unindo os pontos A e B;
arg z: argumento do número complexo z;
[a,b] = {x ∈  : a ≤ x ≤ b}
A \ B ={x : x ∈ A e x ∉ B}
Ac: complementar do conjunto A; 01. (ITA 2014) Considere os polinômios em x ∈  da forma
n

∑a x
k =0
k
k
= a0 + a1x + a2x2 + ... + anxn ,n ∈  . p(x) =x5 + a3x3 + a2x2 + a1x. As raízes de p(x) = 0 constituem uma
1
Observação: Os sistemas de coordenadas considerados são progressão aritmética de razão quando ( a1, a2 , a3 ) é igual a
cartesianos retangulares. 2

Sabe-se que (x + 2y, 3x − 5y, 8x − 2y, 11x − 7y + 2z) é uma progressão 1 5 5 1


a)  , 0,  . d)  , 0,  .
aritmética com o último termo igual a −127. Então, o produto xyz é 4 4 4 4
igual a:
1 5 1 1
a) −60. d) 30. b)  , 1,  . e)  , − 1, −  .
4 4 4 4
b) −30. e) 60.
c) 0. 1 5
c)  , 0, −  .
4 4

124 PROMILITARES.COM.BR
SEQUÊNCIAS

3 08. (ITA 2005) Uma esfera de raio r é seccionada por n planos


02. (ITA 2011) Considere a equação algébrica ∑ ( x − ak )4 −k = 0.
k =1 meridianos. Os volumes das respectivas cunhas esféricas contidas em
Sabendo que x = 0 é uma das raízes e que (a1, a2, a3) é uma progressão uma semi-esfera formam uma progressão aritmética de razão πr3/45.
geométrica com a1 = 2 e soma 6, pode-se afirmar que Se o volume da menor cunha for igual a πr3/18, então n é igual a
a) a soma de todas as raízes é 5. a) 4. c) 6. e) 7.
b) o produto de todas as raízes é 21. b) 3. d) 5.
c) a única raiz real é maior que zero.
d) a soma das raízes não reais é 10. 09. (ITA 1999) O conjunto de todos os números reais q > 1, para os
e) todas as raízes são reais. quais a1, a2 e a3 formam, nesta ordem, uma progressão geométrica
de razão q e representam as medidas dos lados de um triângulo, é:
03. (ITA 2010) Considere a matriz
a) ]1, 1+ 5 [
2
a1 a2 a3 
=A  0 a4 a5  ∈ M3x3 ( ), 1+ 5
b) ]1, ]
 0 0 a6  2
em que a4 = 10, det A = – 1000 e a1, a2, a3, a4, a5 e a6 formam, nesta 1+ 5
ordem, uma progressão aritmética de razão d > 0. Pode-se afirmar c) ]1, ]
5
a
que 1 é igual a
d d) ]1, 1+ 5 [
4
a) – 4. d) – 1.
e) ]1, 1+ 5 [
b) – 3. e) 1.
c) – 2.
10. (ITA 1997) Sejam a1, a2, a3 e a4 números reais formando,
nesta ordem, uma progressão geométrica crescente com a1 ≠ 0.
04. (IME 2010) Seja S = 12 + 32 + 52 + 72 + ... + 792. O valor de S satisfaz: Sejam x1, x2 e x3 as raízes da equação a1x3 + a2x2 + a3x + a4 = 0.
Se x1 = 2i, então
a) S < 7 × 104
a) x1 + x2 + x3 = -2
b) 7 × 104 ≤ S < 8 × 104
b) x1 + x2 + x3 = 1
c) 8 × 104 ≤ S < 9 × 104
c) x12 + x22 + x32 = 4
d) 9 × 104 ≤ S < 105
d) x1 . x2 . x3 = 8
e) S ≥ 105
e) x1 . x2 + x1 . x3 + x2 . x3 = 5
05. (ITA 2010) Considere a progressão aritmética (a1, a2, ..., a50) de
razão d. 11. (IME 2016) Os inteiros a1, a2 , a3 , ..., a25 estão em PA com razão
10 50
Se ∑
n=1
an = 10 + 25d e ∑
n=1
an = 4550, então d – a1 é igual a não nula. Os termos a1, a2 e a10 estão em PG, assim como a6 , aj e a25.
Determine j.
a) 3. d) 11.
12. (ITA 2015) Sabe-se que 1, B, C, D e E são cinco números reais que
b) 6. e) 14. satisfazem às propriedades:
c) 9. I. B, C, D, E são dois a dois distintos;
II. os números 1, B, C, e os números 1, C, E, estão, nesta ordem, em
06. (IME 2010) Seja f(x)
= | 3 − log(x) |, x ∈ . Sendo n um número inteiro progressão aritmética;
f(x) 2f(x) 4f(x) 2n−3 f(x) 9 III. os números B, C, D, E, estão, nesta ordem, em progressão
positivo, a desigualdade + + +…+ +…≤
4 12 36 3n−1 4 geométrica.
somente é possível se: Determine B, C, D, E.
Obs.: log representa a função logarítmica na base 10.
a) 0 ≤ x ≤ 106 d) 100 ≤ x ≤ 106 13. (ITA 2010) A progressão geométrica infinita (a1, a2, ..., an, ...)
−6 8
tem razão r < 0. Sabe-se que a progressão infinita (aI, a6, ..., a5n+1, ...)
b) 10 ≤ x ≤ 10 e) −6
10 ≤ x ≤ 10 6
tem soma 8 e a progressão infinita (a5, a10, ..., a5n, ...) tem soma 2.
c) 3
10 ≤ x ≤ 10 6 Determine a soma da progressão infinita (a1, a2, ..., an, ...).

07. (ITA 2007) Se A, B e C forem conjuntos tais que 14. (ITA 2007) Considere um triângulo isósceles ABC, retângulo em
n (A ∪ B) = 23, n (B - A) = 12, n (C - A) = 10, n (B ∩ C) = 6 e n (A B. Sobre o lado BC, considere, a partir de B, os pontos D e E, tais
∩ B ∩ C) = 4, então n (A), n (A ∪ C), n (A ∪ B ∪ C), nesta ordem. que os comprimentos dos segmentos BC, BD, DE, EC, nesta ordem,
formem uma progressão geométrica decrescente. Se â for o ângulo
a) formam uma progressão aritmética de razão 6.
EÂD, determine tg â em função da razão r da progressão.
b) formam uma progressão aritmética de razão 2.
c) formam uma progressão aritmética de razão 8, cujo primeiro 15. (ITA 2007) Seja k um número inteiro positivo e
termo é 11. Ak = {j ∈ | N: j ≤ k e mdc (j, k) = 1}.
d) formam uma progressão aritmética de razão 10, cujo último Verifique se n(A3), n(A9), n(A27) e n(A81), estão ou não, nesta ordem,
termo é 31. numa progressão aritmética ou geométrica. Se for o caso, especifique
e) não formam uma progressão aritmética. a razão.

PROMILITARES.COM.BR 125
SEQUÊNCIAS

16. (ITA 2006) Seja (a1, a2, a3, ... ,an, ...) uma progressão geométrica 1 0 0 0
infinita de razão positiva r, em que a1= a é um número real não nulo.
1 1 1 1
Sabendo que a soma de todos os termos de índices pares desta det A = x ⋅ r3 ⋅
progressão geométrica é igual a 4 e que a soma de todos os termos 1 1 2 2
de índices múltiplos de 3 é 16/13, determine o valor de a + r. 1 1 2 3
1 0 0 0
17. (ITA 2005) Seja a1, a2, ... uma progressão aritmética infinita tal que 1 1 1
1 1 1 1 1+1
n
= ( −1) ⋅ 1 2 2 = 1⋅ 1 = 1
∑ a=
k =1
3k n 2 + πn2 , para n ∈ * 1 1 2 2
1 2 3
1 1 2 3
Determine o primeiro termo e a razão da progressão.
Então,
18. (ITA 2003) Considere a seguinte situação baseada num dos det A = x ⋅ r3 ⋅ 1
paradoxos de Zenão de Eléia, filósofo grego do século V a.C. Suponha
que o atleta Aquiles e uma tartaruga apostam uma corrida em linha reta, det A = xr3
correndo com velocidades constantes v(A) e v(T), com 0 < v(T) < v(A).
Como a tartaruga é mais lenta, é-lhe dada uma vantagem inicial, de
02. A
modo a começar a corrida no instante t = 0 a uma distância d1 > 0 na
frente de Aquiles. Calcule os tempos t1, t2, t3,... que Aquiles precisa Do enunciado, temos:
para percorrer as distâncias d1, d2, d3,..., respectivamente, sendo que, Sn 2n2 + 5n
=
para todo n ≥ 2, dn denota a distância entre a tartaruga e Aquiles no
a1 = S1 = 2 ⋅ 12 + 5 ⋅ 1 = 7
n−1
instante ∑t
k =1
k da corrida. S2= a1 + a2

Verifique que os termos t(k), k = 1, 2, 3,..., formam uma progressão Mas, S2 = 2 ⋅ 22 + 5 ⋅ 2 = 18


geométrica infinita, determine sua soma e dê o significado desta Então, 18= 7 + a2
soma.
a2 = 11
Daí, sendo r a razão da progressão aritmética, r = a2 − a1 = 11− 7 = 4.
19. Calcule a soma dos vinte primeiros termos da sequência
{1, 0, 3, 10, 21, . . .}. Dessa forma,
a3 = 11+ 4 = 15
20. Devemos colocar 500 bolas formando um triângulo, com uma a4 = a3 + 4 = 15 + 4 = 19
bola na primeira linha, duas na segunda, três na terceira, etc...
a5 = a4 + 4 = 19 + 4 = 23
a) quantas linhas haverá?
a6 = a5 + 4 = 23 + 4 = 27
b) quantas bolas sobrarão? a7 = a6 + 4 = 27 + 4 = 31
a8 = a7 + 4 = 31+ 4 = 35
GABARITO
a9 = a8 + 4 = 35 + 4 = 39
EXERCÍCIOS DE TREINAMENTO
Assim, o determinante da matriz é:
01. E
7 11 15
 x1 x1 x1 x1 
x x = 19 23 27
x 2 x 2 x 2 
Seja A = 
1
. 33 35 39
 x1 x2 x3 x3 
 
 x1 x2 x3 x 4  Multiplicando a coluna 1 por (-1) e somando às colunas 2 e 3,
7 11 15 7 4 8 7 2 4
Do enunciado, temos:
19 23 27 = 19 4 8 = 2 ⋅ 2 ⋅ 19 2 4
x x x x 33 35 39 33 2 6 33 1 3
x x +r x +r x +r
det A = 7 2 4
x x + r x + 2r x + 2r
x x + r x + 2r x + 3r No determinante 19 2 4 , multiplicando a coluna 2 por (-2) e
x x x x x 0 x x 33 1 3
somando à coluna 3,
x x x +r x +r x r x +r x +r
det A + 7 2 4 7 2 0
x x x + 2r x + 2r x r x + 2r x + 2r
19 2 4 = 19 2 0
x x x + 2r x + 3r x r x + 2r x + 3r
33 1 3 33 1 1
x 0 x x x 0 0 x
x r x x +r x r r x +r 7 2 0
det A =
0+ + 3+ 3 7 2 7 2
x r x x + 2r x r 2r x + 2r 19 2 0 =1⋅ ( −1) ⋅ = =7 ⋅ 2 − 2 ⋅ 19 =−24
19 2 19 2
x r x x + 3r x r 2r x + 3r 33 1 1
x 0 0 x x 0 0 0 Portanto,
x r r x x r r r x = 2 ⋅ 2 ⋅ ( −24 )
det A = 0 + 0 + +
x r 2r x x r 2r 2r x = −96
x r 2r x x r 2r 3r

126 PROMILITARES.COM.BR
SEQUÊNCIAS

03. E PA 2 → b1 , b2 , b3 → a1 , r , n → a1 , (a1 + 1) , (a1 + 2)


Calculando os raios das circunferências, tem-se:
Conforme o enunciado, a PA 2 é formada por elementos da PA1.
π 1
- C1 tem centro (x1, y1) e área , logo r1 = . Assim, é fácil deduzir também as seguintes relações em função da
16 4 razão r:
- C2 tem centro (x 2 , y 2 ) e área 144 π, logo r2 = 12.
r = a1 + 1 → a1 = r − 1
Sabendo que (x1,y1,r1) e (x2,y2,r2) são duas progressões geométricas n= r + 1
7
com somas dos termos iguais a e 21, pode-se escrever: Pelas fórmulas de um termo qualquer de uma PA e de soma de todos
4
7 3 x1 os termos de uma PA, pode-se deduzir:
x1 + y1 + r1 = ⇒ x1 + y1 = ⇒ y1 =
4 2 2 (a + a ) ⋅ n
S= 1 n e an = a1 + (n − 1) ⋅ r
3 x 3 x1 = 1 2
1
x1 + y1 = ⇒ x1 + = ⇒ 2x1 + x1 = 3 ⇒ [a1 + a1 + (n − 1) ⋅ r] ⋅ n (2a1 + nr − r) ⋅ n
2 2 2 y1 = 1 = S =
2 2 2
x 2 + y 2 + r2 = 21 ⇒ x 2 + y 2 = 9 ⇒ y 2 = 2 3x 2
x2 = 3 Substituindo a1, n, r e S pelas relações deduzidas anteriormente (todas
x 2 + y 2 =9 ⇒ x 2 + 2 3x 2 =9 ⇒ em função de r) e dados do enunciado, tem-se:
y2 = 6
[2 ⋅ (r − 1) + (r + 1) ⋅ r − r ] ⋅ (r + 1) → 488 =
244 = [2 ⋅ (r − 1) + (r + 1) ⋅ r − r ] ⋅ (r + 1)
2
Por fim, calculando a distância pedida: 488 = (2r − 2 + r + r − r) ⋅ (r + 1) → 488 = (2r − 2 + r2 ) ⋅ (r + 1) → 488 = r3 + 3r2 − 2
2

2 490 = r3 + 3r2 → 490 = r2 ⋅ (r + 3)


(1− 3)2 +  
1 137
D= − 6 ⇒ D =
2  2 Nota-se que 490 é múltiplo de 2, 5, 7 e 10. Fazendo as devidas
substituições, percebe-se que o único valor capaz de satisfazer a
equação do terceiro grau acima é 7:
04. A 490 = r2 ⋅ (r + 3) = 72 ⋅ (7 + 3) = 49 ⋅ 10 = 490
1−1
[I] VERDADEIRA. Se o número for ímpar k = 1, ou seja, 2 (2m − 1).
Logo, a raiz da primeira progressão aritmética apresentada no
Logo o número é o produto de um por ele próprio. Se o número for
enunciado, cuja soma dos termos é 244, é igual a 7.
par ele é o produto de um ímpar por uma potência de 2, ou seja,
2n−1(2m − 1).
07. D
[I] Falsa. Considerando a existência dos termos da sequência na
[II] FALSA. Calculando: condição dada, temos:
( a2 )2 ⇒ sen x ⋅ sen ( x + π 2=) ( )
2
a1 ⋅ a= sen x + π 4 1± 5
3
x ⋅ q2 = x ⋅ q + x ⇒ q2 − q − 1 = 0 ⇒ q = (irracional)
2 2
 2  1
sen x ⋅ cos x =
 ⋅ ( sen x + cos x )  ⇒ sen x ⋅ cos x =⋅ (1+ 2 ⋅ sen x ⋅ cos x ) Portanto, não existem termos desta sequência que formam uma PG,
 2  2 pois os termos são todos positivos e não nulos.
2 ⋅ sen x ⋅ cos x =
1+ 2 ⋅ sen x ⋅ cos x ⇒ 0 ≠ 1
[II] Verdadeira. Determinando o sétimo termo da sequência, temos
a7 = 13, que é um número primo.
[III] FALSA. Considerando a e b como inteiros com MDC igual a 1
(fração irredutível) e sendo b diferente de zero, pode-se escrever: [III] Verdadeira. Analisando a paridade da sequência (ímpar, ímpar, par,
a a2 ímpar, ímpar, par,...) percebemos que os termos de ordem 3, 6, 9,
p = ⇒ p = 2 ⇒ a2 = pb2. Mas um quadrado perfeito não pode
b b 12,... são pares.
ser igual a um não quadrado perfeito, assim p não pode ser racional.
Portanto, apenas as afirmações [II] e [III] são verdadeiras.

05. D
=q (cos 5°) ⋅ (cos 20°) ⋅ (cos 40°) ⋅ (cos 85°) 08. A
cos 90° + cos 80° cos 80° Seja (a, b, c) a progressão geométrica crescente cujos termos
=
(cos 5°) ⋅ (cos 85°) =
2 2
cos 60° + cos 20° 1 cos 20° queremos determinar. Tem-se que a + b + c =13 e a2 + b2 + c2 =
91.
(cos 20°) ⋅ (cos 40°) = = +
2 4 2 Além disso, sabemos que b2 = ac. Logo, vem
cos 80°  1 cos 20°  cos 80° cos 80° ⋅ cos 20° cos 80° cos100° + cos 60°
=q ⋅ + =  + = + = (a + b + c)2= 132 ⇔ a2 + b2 + c2 + 2(ab + ac + bc)= 169
2 4 2  8 4 8 8
cos100° + cos 80° 1 2 ⋅ cos 90° ⋅ cos10° 1 1 ⇔ 91+ 2b(a + b + c) = 169
= = + = + →q
8 16 8 16 16 ⇔ 26b = 78
1
Sinfinita = 4 →S 4 ⇔b= 3.
infinita =
1− 116 15
Em consequência, de a + c =
10 e ac = 9, segue que a = 1 e c = 9.

06. A Portanto, como 139 ≡ 1mod23, podemos concluir que a resposta é 1.


Considerando as duas progressões aritméticas dadas, tem-se as duas
PAs com os seguintes termos: 09. E
PA1 → a1 , a2 , a3 , a4  an , da qual não sabe-se a razão nem o número Analisando os termos dados pode-se observar que o numerador é
de termos.

PROMILITARES.COM.BR 127
SEQUÊNCIAS

soma dos termos de uma PA com a1 = 1 e razão 1. Já o denominador Portanto, log2 −=


8 log2=
8 3.

é a soma dos termos de uma PG com a1 = 1 e razão 2.


14. C
Logo, pode-se escrever sobre xn :
Fazendo y = 2z, vem
n ⋅ (1+ n)
2= n ⋅ (1+ n) ⋅ n ⋅ (n + 1) y 2
1   1 9
z
=
xn →
= xn  2 9
(
1⋅ 2n − 1 2 ) 1⋅ 2n − 1 ( )
2 ⋅ 2n − 1 ( ) edet A= lim 1+  ⇔ edet A=  lim 1+  
y →+∞  y 
z →+∞  z  
2 −1 2
⇔ edet A =
e9
2
10. B ⇔ det A = .
9
Considerando os três números me PA (a − r), a e (a + r), temos: Sendo r a razão da progressão aritmética (a,b,c,d,f,g,h), podemos
S1 = (a − r) + a + (a + r) = 3a escrever b= a + r, d= a + 3r, g = a + 5r e h= a + 6r. Logo,
S2 = (a − r)2 + a2 + (a + r)2 = 3a2 + 2r2 1 a a2 1 1 1
2 2
Logo, 1 b b2 = ⇔ a b d =
9 9
 1  1 1 d d2 a2 b2 d2
x 2 − S1 x +  S2 −  =0 ⇒ x 2 − 3ax +  3a2 + 2r2 −  =0
 2  2 2
⇔ (d − a)(d − b)(b − a) =
9
As raízes da equação são a e (a + r). Logo: 2
⇔ 3r ⋅ 2r ⋅ r =
a + a + r = 3a ⇒ a = r 9
1
1 ⇔r=
a ⋅ (a + r)= 3a2 + 2 ⋅ r2 − 3
2
Como a = r, temos: e, portanto,
3
2 1 2  1
r ⋅ (r + r)= 3r + 2r − n
1  4 
+∞

1
2 =
a + 6r ∑ 1
 
4
⇔ a =
+ 6 ⋅
3 1− 1
3 ⋅ r2 = n=3
2 4
6 1
r= ± ⇔ a= −2
6 48
95
6 ⇔ a =− .
Como r ≥ 0, temos: r = . 48
6
Finalmente, segue que
b − 2g = a + r − 2(a + 5r)
11. C =−a − 9r
π π π 1 95
A sequência  , , 
,  é uma PG de razão , portanto =
48
−3
 3 6 12  2
 π  49
  2π 1 = − .
b = cos  3  = cos = − . 48
 1− 1  3 2
 2
1 15. A
Logo, log2 − =−1.
2
Progressão Aritmética ⇒ ( a1,a2 ,a3 ,) . Logo, a2 − a1 = a3 − a2.
12. C 3
Portanto, ( 3x − 5y ) − ( x − 2y ) =( 8x − 2y ) − ( 3x − 5y ) ⇒ y =−
x
8 10
O terceiro termo do desenvolvimento de 1+ 
1
será dado por  2x 9x 43x 
2 q   A PA ficará representada em função de x por:  , , , −127 
8 6  1  28 r 5 2 5 
 1 .  = 2 = ⇒ q.r = 32.22.7 Aplicando propriedades de PA, temos:
 2  q  q 9q
Considerando a PG, temos 192 = 3.q → 64 = q ; como p q e r
n-1 n-1 2x 9x 43x
− 127 = + ⇒ x =−10 e y = 3
são inteiros, concluímos n-1 = 3 ou n-1 = 6, n-1= 2 ou n-1 = 1, pois 5 2 5
(4)3 = (± 2)6 = (± 8)2 = 64. Como 11x − 7y + 2z =−127 ⇒ z =2
Considerando a PA, temos 192 = 3 + (n-1).r → r = 189/(n-1), logo Portanto, x ⋅ y ⋅ z =( −10 )( 3)( 2) =−60
n-1 = 3 e r = 63.
Logo, o segundo termo da PA será dado por 3 + 63 = 66. 16.
x y
Seja P =  .
13. C z w 
π π π Como P é simétrica,
Considerando que + + ... é a soma dos termos de um PG infinita
1
3 6 12 x y  x z 
de razão , podemos escrever: z =   =⇒ y z.
2  w  y w 
π π  
2π  1  1 Dessa forma,
b=sec3 3 ⇒ b = sec3 3 =sec3 = = =−8
1 1 3  cos3 2π   1 3 x y 
1− − P= 
2 2  3   2  y w 

128 PROMILITARES.COM.BR
x
1 −6 4
SEQUÊNCIAS
2k 1 0
Dy =−3 0 4 =4 ⋅ ( 3 − 2k )
0 1 4
Como A TP + PA =
I, 2k 8 1
 k 4   x y   x y   k −3  1 0 Dw =−3 2 + k 0 =⋅ 2 k 2 + 2k + 21 ( )
 −3 2  ⋅  y w  +  y w  ⋅  4 2  = 0 1 0 −6 1
         
 kx + 4y ky + 4w   kx + 4y −3x + 2y   1 0 Dessa forma,
 −3x + 2y −3y + 2w  ⋅ ky + 4w −3y + 2w  =  
    0 1 4 (k + 16 ) k + 16
 2 ⋅ (kx + 4y ) −3x + 4w + y ⋅ ( 2 + k )   1 0 =x =
 = 
8 ( k + 6 ) ⋅ (k + 2) 2 (k + 6 ) ⋅ (k + 2)
 −3x + 4w + y ⋅ ( 2 + k ) −6y + 4w  0 1 4 ( 3 − 2k ) 3 − 2k
=y =
2 (kx + 4y ) = 1 8 (k + 6 ) ⋅ (k + 2) 2 (k + 6 ) ⋅ (k + 2)

−3x + 4w + y ( 2 + k ) = 0
(
2 k2 + 2k + 21 )
−6y + 4w = k2 + 2k + 21
1 =w =
 8 (k + 6 ) ⋅ (k + 2) 4 (k + 6 ) ⋅ (k + 2)
2kx + 8y = 1
 De p11 > 0, x > 0.
−3x + ( 2 + k ) y + 4w = 0
−6y + 4w = Então,
 1
k + 16
2k 8 0 >0
2 (k + 6 ) ⋅ (k + 2)
D =−3 2 + k 4 =8 ⋅ (k + 6 ) ⋅ (k + 2)
0 −6 4

Se k = −6, temos o seguinte sistema, −16 < k < −6 ou k > – 2 (11)


−12x + 8y = 1 −12x + 8y = 1 (1)
  De P > 0, xw − y 2 > 0, ou seja, xw > y 2.
−3x − 4y + 4w = 0 ⇔ −12x − 16y + 16w = 0 ( 2)
−6y + 4w =   
2
 0 −6y + 4w = 0 ( 3) k + 16

k 2 + 2k + 21
> 
3 − 2k

2 (k + 6 ) ⋅ (k + 2) 4 (k + 6 ) ⋅ (k + 2)  2 (k + 6 ) ⋅ (k + 2) 
Das equações (1) e ( 2) , (k + 16 ) ⋅ (k2 + 2k + 21) ( 3 − 2k )2
>
( −12x + 8y ) − ( −12x − 16y + 16w ) =
1− 0 8 (k + 6 ) ⋅ (k + 2)
2 2
4 (k + 6 ) ⋅ (k + 2)
2 2

−12x + 8y + 12x + 16y − 16w =


1
(k + 16 ) ⋅ (k2 + 2k + 21) ( 3 − 2k )2
1 (4)
24y − 16w = − >0
8 (k + 6 ) ⋅ (k + 2) 4 (k + 6 ) ⋅ (k + 2)
2 2 2 2

Da equação ( 3) , (k + 16 ) ⋅ (k2 + 2k + 21) − ( 3 − 2k )2


>0
0 (5 )
24y − 16w = 8 (k + 6 ) ⋅ (k + 2)
2 2

Das equações (4) e (5) conclui-se que o sistema é impossível para (


k 3 + 18k 2 + 21k + 16k 2 + 32k + 336 − 2 ⋅ 9 − 12k + 4k 2 ) >0
k = −6. 8 ⋅ (k + 6 ) ⋅ (k + 2)
2 2

Se k = −2, temos o seguinte sistema: k 3 + 10k 2 + 77k + 318


>0
8 ⋅ (k + 6 ) ⋅ (k + 2)
2 2
−4x + 8y =1 (6)

 −3x + 4w =0 ( 7) A inequação acima é equivalente à seguinte inequação:

−6y + 4w = 1 ( 8)
k 3 + 10k 2 + 77k + 318 > 0 (k ≠ −6 e k ≠ −2)
Das equações (7) e (8), Daí,
( −3x + 4w ) − ( −6y + 4w ) = 0 −1
k 3 + 6k 2 + 4k 2 + 77k + 318 + 144 − 144 > 0
−3x + 4w + 6y − 4w =−1
k 3 + 6k 2 + 77k + 462 + 4k 2 − 144 > 0
−3x + 6y =−1
−12x + 24y =
−4 ( 9) (
k 2 ⋅ (k + 6 ) + 77 ⋅ (k + 6 ) + 4 ⋅ k 2 − 36 > 0)
k 2 ⋅ (k + 6 ) + 77 ⋅ (k + 6 ) + 4 ⋅ (k + 6 ) ⋅ (k − 6 ) > 0
Da equação (6),
3 (10 )
−12x + 24y =
(k + 6 ) ⋅ (k2 + 77 + 4 ⋅ (k − 6 ) ) > 0

Das equações (9) e (10) conclui-se que o sistema é impossível para


(k + 6 ) ⋅ (k2 + 4k + 53) > 0
k = −2. (k + 6 ) ⋅ (k2 + 4k + 4 + 49 ) > 0
1 8 0
(k + 6 ) ⋅ ( (k + 2)2 + 49 ) > 0
Dx = 0 2 + k 4 = 4 ⋅ (k + 16 )
1 −6 4 Note que a inequação (k + 6 ) ⋅ ( (k + 2)2 + 49 ) > 0 é equivalente à
2k 1 0
inequação k + 6 > 0, logo, k > −6 (12) .
Dy =−3 0 4 =4 ⋅ ( 3 − 2k )
Das inequações (11) e (12), temos: k > −2
0 1 4
Resposta: A faixa de valores de k para que exista uma matriz de
2k 8 1
números reais P satisfazendo as condições dadas é ]−2, + ∞[ .
(
2 k 2 + 2k + 21
Dw =−3 2 + k 0 =⋅ )
0 −6 1

PROMILITARES.COM.BR 129
SEQUÊNCIAS

17. Sejam n, n − 5 e n − 10, respectivamente, as quantidades de Voctaedro = 2 ⋅ VIJKLM


arestas, faces triangulares e quadrangulares. 1 a
Voctaedro = 2 ⋅ ⋅ x 2 ⋅
Então, 3 2
3 ⋅ (n − 5) + 4 ⋅ (n − 10 ) a ⋅ x2
n= Voctaedro =
2 3
2
2n = 3n − 15 + 4n − 40 a 2
a ⋅  
n = 11
Voctaedro =  2 
Logo, o poliedro possui 11 arestas, 6 faces triangulares e 1 face 3
quadrangular, ou seja, possui 7 faces. a a2 ⋅ 2
Voctaedro= ⋅
Dessa forma, sendo V o número de vértices do poliedro, temos: 3 4
V − 11 + 7 =2 1 3
Voctaedro= ⋅a
V=6 6
1
Resposta: Seis vértices Voctaedro = Vcubo
6
Agora, observemos o octaedro e o cubo inscrito nele.
18. Do enunciado, temos a figura abaixo:

A1 é baricentro do triângulo IJM.

D1 é baricentro do triângulo ILM.


Vcubo = a3 Dessa forma, temos a figura abaixo:

No triângulo PQ
1 1M,
2 2
x x
No triângulo JMP,
(PQ1)
1=
2
  + 
 2  2
2 2
a a x 2
x2   +  
= 1 1=
PQ
 2  2 2
a 2
x= Da semelhança entre os triângulos IPQ
1 1 e IA1D1,
2
VIJKLM : Volume da pirâmide de base quadrada JKLM e vértice I.

130 PROMILITARES.COM.BR
SEQUÊNCIAS

IA1 y 19.
=
IP1 PQ 1 1
a) Teremos:
2 y a<b<c
=
3 x 2 PG : (loga d, logb d, logc d)
2 PA : ( a, b, c )
2 x 2
y= ⋅ =b bloga b − a ( i)
3 2
x 2 Da PG,
y=
3
(logb=
d)
2
loga d ⋅ logc d

Assim, o volume do cubo A1B1C1D1E1FG logb d ⋅ logb d = loga d ⋅ logc d


1 1H1 é dado por:
logb d logb d
x 2
3
logb d ⋅ logb d = ⋅
3 logb a logb c
F G1H=
VA1B1C1D1E11 1
y=  
 3  logb a ⋅ logb c = 1
x3 ⋅ 2 2 1
F G1H1 =
VA1B1C1D1E11 logb c = (ii)
27 logb a
a 2 Da equação (i),
Mas, x = , logo,
2 =b bloga b − a
3
1 a 2
F G1H1 =⋅ 
VA1B1C1D1E11   ⋅2 2
 b+a =bloga b
27  2 
logb b
1 3 1 b+a =b logb a
F G1H1 =
VA1B1C1D1E11 ⋅a ⋅2 2 ⋅2 2 ⋅
27 8 1
1 3 b+a =blogb a
F G1H1 =
VA1B1C1D1E11 a
27
1
2 1 3
F G1H1=
VA1B1C1D1E11 ⋅ a  De b + a =blogb a e da equação (ii)
9 6 
b+a =blogb c

2
b+a =c (iii)
Então, o volume do cubo inscrito no octaedro equivale a do volume
do octaedro. 9 Da PA,
Dessa forma, sendo V o volume do primeiro cubo, temos: 2b= a + c (iv )
Das equações (iii) e (iv),
Volume do primeiro octaedro: 1 V 2b = a + b + a
6
b = 2a
Volume do segundo cubo: 2 ⋅ 1 V =
V
9 6 27 De b = 2a e b + a =c,
c = 3a
Volume do segundo octaedro: 1 ⋅ 2 ⋅ 1 V =V
6 9 6 162
e b bloga b − a,
De b = 2a =
2 1 2 1 V
Volume do terceiro cubo: ⋅ ⋅ ⋅ V= =2a ( 2a)loga (2a) − a
9 6 9 6 729 log 2+1
3a = ( 2a) a
1 2 1 2 1 V
Volume do terceiro octaedro: ⋅ ⋅ ⋅ ⋅ V= =3a ( 2a) a ⋅ 2a
log 2
6 9 6 9 6 4374
 3
= 2loga 2 ⋅ aloga 2
2
Sendo r a razão pedida, temos:
3
= 2loga 2 ⋅ 2
V V V   V V  2
 + + + ...  +  + + ... 
 6 162 4374   27 729  3
r= 2loga 2 =
V 4
V V 3
6 + 27 log2 2loga 2 = log2  
1 1 4
1− 1− 3
r= 27 27 loga 2 = log2  
V 4
11 1 3
r= = log2  
52 log2 a 4
11 1
Resposta: log2 a =
52 3
log2  
4
log2 2
log2 a =
3
log2  
4
log2 a = log 3  2
 
4
log 3 2 PROMILITARES.COM.BR 131
a=2 4
1 3
= log2  
log2 a 4
SEQUÊNCIAS
1
log2 a =
3
log2  
4
log2 2 20.
log2 a =
3 a) Note que:
log2  
4  kπ  kπ kπ π (14 − k )
log2 a = log 3  2 • cos  2π −=  cos ⇒ cos
= cos
 7  7 7 7
 
4
 k π  k π k π π (14 − k )
log 3 2 • sen  2π −  = − sen ⇒ sen = − sen
a=2 4  7 7 7 7

Então, kπ kπ
=
De zk cos + isen ,
log 3 2 log 3 2
7 7
b= 2 ⋅ 2 4
e c= 3 ⋅ 2 4 14 −k (14 − k ) π (14 − k ) π
=z cos + isen
7 7

b) Da PA, Por outro lado,


r= b − a (14 − k ) π (14 − k ) π
=zk cos − isen
=
r 2a − a 7 7
r=a Então,
log 3 2
(14 − k ) π
r=2 4 zk + z14 −k =
2cos
7
Da PG, (14 − k ) π
logb d
cos
7
=
1 k 14 −k
2
z +z ( )
q=
loga d k = 13,
π
=q
1

logad
loga d loga b
=
cos
7 2
(
1 13
z +z )
1 k = 11,
q=
loga ( 2a) 3π
=
cos
7 2
(
1 11 3
z +z )
1
q= k = 9,
loga 2 + 1

q=
1 cos=
7
1 9 5
2
(
z +z )
log2 2
+1
log2 a
1 b) Teremos:
q=
1 π π π 1
log2 a
+1 cos
7
1
( ) 1
+ cos + cos = ⋅ z + z13 + ⋅ z3 + z11 + ⋅ z5 + z9
7 7 2 2 2
( ) ( )
1 π π π 1
q=
1
+1 7 7 7 2
(
cos + cos + cos = ⋅ z + z3 + z5 + z9 + z11 + z13 )
log 3 2
π π π 1
log2 2 4
7 7 7 2
(
cos + cos + cos = ⋅ z + z3 + z5 + z7 + z9 + z11 + z13 − z7 )
1
q=
1
+1 π π
  2 7 
π 1  
z ⋅ z − 1
 7
( )


log 3 2 cos + cos + cos = ⋅  − z 
7 7 7 2  z2 − 1
4  
 
1
q=
3
log2   + log2 2 0 em z14 −k cos
k ==
(14 − k ) π (14 − k ) π
+ isen ,
4 7 7
z14 cos 2π + isen2π
=
1
q= z14 = 1
 3
log2  
 2
7 em z14 −k cos
k ==
(14 − k ) π (14 − k ) π
q = log3 2 + isen ,
7 7
7
2 z= cos π + isenπ
z7 = −1
Resposta:
Logo,
5π 1  z ⋅ (1− 1)
log 3 2 log 3 2 log 3 2
π 3π 
a) a= 2 4 , b= 2 ⋅ 2 4 e c= 3 ⋅ 2 4 ; cos + cos + cos = ⋅ − ( −1) 
7 7 7 2  z2 − 1 
log 3 2
π 3π 5π 1
b) r = 2 4
e q = log3 2. cos + cos + cos =
7 7 7 2
2

a π 3π 5π 1 π 3π 5π
De =cos + cos + cos e =cos + cos + cos , os
b 7 7 7 2 7 7 7
a π 3π 5π
inteiros a = 1 e b = 2 satisfazem =cos + cos + cos .
b 7 7 7

132 PROMILITARES.COM.BR
SEQUÊNCIAS

Resposta: a1 = 1 = α2 + α1 + α0
π 1 3π 1 3 11 5π 1 5 9
(
a) cos = ⋅ z + z13 , cos
7 2
)
= ⋅ z + z , cos
7 2
( )
= ⋅ z +z ;
7 2
( ) =
a0 =
0 α0 ⇒=
α2 0.5 =
α1 0.5 =
α0 0
a−1 = 0 = α2 − α1 + α0
a π 3π 5π
=cos + cos + cos . 1 2
b) Os inteiros a = 1 e b = 2 satisfazem
b 7 7 7 Portanto,=
ai
2
(i + i). Dispomos de 500 bolas. Então,
EXERCÍCIOS DE COMBATE ai ≤ 500 ⇒ i2 + i ≤ 1000 ⇒ 0 ≤ i ≤ 31.
O maior valor de i é 31, ou seja, faremos 31 linhas.
01. C 03. D 05. D 07. D 09. A
02. A 04. C 06. D 08. C 10. A 312 + 31
b) ao final da 31ª linha teremos colocado = 496 bolas. Logo,
4 bolas ficarão de fora. 2
11. r= 7 ⋅ a1

ANOTAÇÕES
12. D = (-2)⋅(-1/2) = 1 e E =( −2) ⋅ 1 =−2

a1 8+ 2 (8 + 2)
13. a1 + a2 + a3 + a4 + .... = = = 2. = 14 − 6 2
1− q 2 2+ 2
1+
2
14. tg β = r2/(2 - r)

15. A sequência (2, 6, 18, 54) é uma PG de razão 3

16. 11


17. A razão é: r =
3

18. É uma PG infinita de primeiro termo d1/v(A), razão v(A)/v(B) e soma


d1/(v(A) - v(B)), tempo necessário para Aquiles alcançar a tartaruga.

19. Seja {a1} a sequência {1,0,3,10,21,} . Constatamos facilmente


que ∆a1 = ai +1 − ai = bi forma a sequência {−1,3,7,11,} (PA de razão
4) e, portanto, {a1} é uma PA de ordem 2. Escrevemos:
ai = α2i2 + α1i + α 0

Como b0 = −1 − 4 = −5 e b−1 = b0 − 4 =−9 , obtermos rapidamente


que a0 = a1 − b0 = 1 − ( −5) = 6 e a−1 = a0 − b−1 = 6 − ( −9 ) = 15 .
Formamos o sistema:
a1 = 1 = α2 + α1 + α0
a0 = 6 = α0 ⇒ α2 = 2 α1 = −7 α0 = 6
a−1 = 15 = α2 − α1 − α0

Portanto, a1 = 2i2 − 7i + 6 e
n n n

=
Sn
=i 1
∑ (2i − 7i +=
2
6 ) 2∑ i − 7∑ i +=
=i 1=i 1
6n 2

2n (n + 1)( 2n + 1) 7n (n + 1) 2n (n + 1)( 2n + 1) − 3n ( 7n − 5)
= − = + 6n
6 2 6
S20 = 4390

20.
a) seja a1 o número total de bolas colocadas ao final da linha i. geramos
então a sequência {ai } = {1,3,6,10,15,}. Constatamos facilmente que
∆ai = ai +1 − ai = bi forma a sequência {2,3,4,5,} (PA de razão 1) e,
portanto, {ai } é uma PA de ordem 2. Escrevemos:
ai = α2i2 + α1i + α 0

Como b0 = 2 − 1 = 1e b−1 = b0 − 1 = 0, obtemos rapidamente


a0 = a1 − b0 = 1 − 1 = 0 e a−1 =a0 − b−1 =0. Formamos o sistema

PROMILITARES.COM.BR 133
SEQUÊNCIAS

ANOTAÇÕES

134 PROMILITARES.COM.BR
MATRIZES

MULTIPLICAÇÃO DE MATRIZES
Sejam duas matrizes A = (aij)m×n e B = (bjk)n×p, o produto de A por B, A . B, é a matriz m × p, C = (cik)m×p, onde o elemento cik, localizado na i-ésima
linha e k-ésima coluna, é obtido multiplicando-se os elementos da i-ésima linha de A pelos correspondentes elementos da k-ésima coluna de B e
somando os produtos parciais assim obtidos.
n
cik = ai1b1k + ai2b2k + ai3b3k +  + ainbnk = ∑ a ⋅b
j=1
ij jk

Considerando as linhas da matriz A e as colunas da matriz B como vetores no Rn, cada elemento cik é obtido pelo produto escalar do i-ésimo
vetor linha de A pelo k-ésimo vetor coluna de B.
O produto de duas matrizes AB somente existe quando A possui tantas colunas quantas são as linhas de B. Nesse caso, diz-se que as duas
matrizes A e B são conformáveis para a multiplicação.
O produto AB é uma matriz que possui o número de linhas de A e o número de colunas de B.

cik = ai1b1k + ai2b2k + ai3b3k +  + ainbnk

PROPRIEDADES
a) A multiplicação de matrizes não é comutativa, isto é, para duas matrizes quaisquer A e B é falso que AB = BA necessariamente.
1º caso: AB existe e BA não existe
m ≠ p ⇒ Am×n ⋅ Bn×p = ABm×p
Bn×p ⋅ Am×n ⇒ ∃ BA

2º caso: AB e BA existem, mas são de tipos diferentes


Am×n ⋅ Bn×m = ABm×m
Bn×m ⋅ Am×n = BAn×n

3º caso: AB e BA existe e são do mesmo tipo (A e B são matrizes quadradas de mesma ordem), mesmo assim em geral temos AB ≠ BA.

4º caso: Sejam A e B matrizes quadradas e de mesma ordem, quando ocorre AB = BA, diz-se que A e B comutam.
b) Associatividade: (A ⋅ B) ⋅ C = A ⋅ (B ⋅ C)

c) Distributividade em relação à adição: A⋅ (B + C) = A ⋅ B + A ⋅ C


A⋅ (B + C) = A ⋅ B + A ⋅ C

d) (k ⋅ A) ⋅ B = A ⋅ (k ⋅ B) = k ⋅ (A ⋅ B)
e) Elemento Neutro: Amxn ⋅ In = Im ⋅ Amxn = Amxn

PROMILITARES.COM.BR 131
MATRIZES

1 0  0 MATRIZ IDENTIDADE
= 0 1  0 É a matriz diagonal, na qual todos os elementos da diagonal
1, se i j
In = ( δij )nxn onde δij =  ⇒ In =  principal são iguais a 1.
0, se i ≠ j    
  0  0
0 0  1 1
0 1  0
Ιn = ( δij )nxn ⇒ Ιn = 
f) Multiplicação pela matriz nula: 0pxm ⋅ Amxn =
0pxn    
 
Amxn ⋅ 0nxp =
0mxp 0 0  1

1, se i = j
onde δij = é o chamado delta de Kronecker.
0, se i ≠ j
TIPOS DE MATRIZES
A matriz identidade é o elemento neutro da multiplicação de
matrizes, assim Amxn ⋅ Ιn = Ιm ⋅ Amxn = Amxn .
MATRIZ QUADRADA
A matriz constituída pelo mesmo número de linhas e colunas é MATRIZ TRIANGULAR SUPERIOR
chamada matriz quadrada.
Chama-se matriz triangular superior, à matriz quadrada que
Assim, uma matriz constituída por n linhas e n colunas é uma possui todos os elementos abaixo da diagonal principal nulos.
matriz quadrada de ordem n × n ou simplesmente uma matriz
quadrada de ordem n. A é triangular superior ⇒ aij = 0, se i > j

 a11 a12  a1n  a11 a12 a13  a1n 


a  0 a  a2n 
22 a23
= =
M (a  21 a22  a2n  
ij )nxn A=0 0 a33  a3n 
     
   
 an1 an2  ann        
 0 0 0  ann 

MATRIZ-LINHA
É toda matriz de ordem 1 × n, ou seja, que possui uma única linha.
MATRIZ TRANSPOSTA
A matriz transposta de A, At, é a matriz obtida a partir de A,
= =
M (aij )1xn [ a11 a12  a1n ] trocando-se ordenadamente suas linhas por colunas. Seja A = (aij)m×n,
então At = (bij)n×m, onde bij = aij para todo i, j.
A matriz transposta de A possui tantas linhas quantas são as
MATRIZ COLUNA colunas de A e tantas linhas quantas são as colunas de A.
É toda matriz de ordem m × 1, ou seja, que possui uma única
coluna.

 a11 
a 
= =
M (a  21 
ij )mx1
  
 
am1
A transposta de uma matriz quadrada pode ser obtida invertendo
os elementos em relação à diagonal principal. Os elementos da
MATRIZ NULA diagonal principal não mudam de posição.
É toda matriz que possui todos os seus elementos iguais a zero.
PROPRIEDADES
0 0  0 
0 0  0  a. (At)t = A
0mxn =   m linhas
       b. (A + B)t = At + Bt
  c. (A – B)t = At – Bt
0 0  0 

n colunas d. k ∈ R ⇒ (kA)t = k ⋅ At
e. (A ⋅ B)t = Bt ⋅ At
A matriz nula é o elemento neutro da adição de matrizes, assim
A + 0 = A e 0 + A = A. f. (A ⋅ B . C)t = Ct ⋅ Bt ⋅ At

MATRIZ DIAGONAL MATRIZ SIMÉTRICA


É toda matriz quadrada em que os elementos não pertencentes Uma matriz quadrada diz-se simétrica quando aij = aji para todo
à diagonal principal são iguais a zero, ou seja, aij = 0 sempre que i ≠ j. 1 ≤ i, j ≤ n, ou seja, quando é igual à sua matriz transposta.
Daí resulta que os elementos simétricos em relação à diagonal
a11 0  0 
0 a principal são iguais.
22  0 
An =  A é simétrica ⇒ A = At
     
   1 4 5
 0 0  ann 
Exemplo: A =  4 2 6 é simétrica.
 
 5 6 3
Se A é quadrada, a matriz A + At é sempre simétrica.

132 PROMILITARES.COM.BR
MATRIZES

MATRIZ ANTI-SIMÉTRICA MATRIZ INVERSÍVEL


Uma matriz quadrada diz-se anti-simétrica quando aij = –aji para Uma matriz quadrada A de ordem n é inversível se existe uma
todo 1 ≤ i, j ≤ n, ou seja, quando é igual à oposta de sua matriz matriz A–1, chamada matriz inversa, tal que A–1A = AA–1 = In.
transposta. Uma matriz A é inversível se, e somente se, ela é não-singular
A é anti-simétrica ⇒ A = −At (det A ≠ 0).
Daí resulta que os elementos simétricos em relação à diagonal A é inversível ⇔ det A ≠ 0
principal são opostos e os elementos pertencentes à diagonal principal Assim, uma matriz A não é inversível se, e somente se, ela é
são nulos. singular (det A = 0).
 0 −4 −5
Exemplo:
= A  4 0 −6 é anti-simétrica. PROPRIEDADES
 5 6 0  a. (AB)–1 = B–1A–1
Se A é quadrada, a matriz A − At é sempre anti-simétrica. b. (ABC)–1 = C–1B–1A–1
c. (At)–1 = (A–1)t
MATRIZES COMUTATIVAS
Como já visto no anteriormente, em geral, AB ≠ BA. MATRIZ ORTOGONAL
Se A e B matrizes quadradas e de mesma ordem, diz-se que A e B Uma matriz é dita ortogonal quando sua transposta e sua inversa
comutam, quando ocorre AB = BA. coincidem.
A e B comutam ⇒ AB = BA A é ortogonal ⇒ At = A–1

MATRIZES ANTICOMUTATIVAS MATRIZ COFATORA


Se A e B matrizes quadradas e de mesma ordem, diz-se que A e B A matriz cofatora de uma matriz quadrada A, indicada comumente
são anticomutativas, quando ocorre AB = −BA. por A’, é outra matriz quadrada cujos elementos são os cofatores dos
A e B são anticomutativas ⇒ AB = −BA elementos correspondentes da matriz A.
No próximo módulo aprenderemos a calcular os cofatores de uma
MATRIZ INVOLUTIVA matriz.
Uma matriz quadrada A diz-se involutiva quando A² = I.
A é involutiva ⇒ A² = I MATRIZ ADJUNTA
A matriz adjunta de uma matriz quadrada A, comumente indicado
A matriz A = 
1 0  1 0
 é involutiva, pois A = 0 1 . por A, é a transposta da matriz dos cofatores.
2

0 −1  
MATRIZES SEMELHANTES
MATRIZ NILPOTENTE Duas matrizes A e B são ditas semelhantes, se existir uma matriz
Uma matriz quadrada A é dita nilpotente se A2 = 0. não singular P tal que
A é nilpotente ⇒ A2 = 0 B = P–1 ⋅ A ⋅ P
Uma matriz quadrada é dita nilpotente de ordem p se Ap = 0 e p Se A e B são semelhantes, então det A = det B.
é o menor inteiro positivo para o qual isso ocorre.
 p
A é nilpotente de ordem p ⇒ A = 0 CÁLCULO DA MATRIZ INVERSA
 k
A ≠ 0, k < p É possível obter a inversa de uma matriz a partir da sua definição
5 2 6 como no exemplo abaixo:
A matriz A =  5 2 6  é nilpotente de 3ª ordem, pois
Seja a matriz A = 
  3 7
 −2 −1 −3 
5 11
A2 ≠ 0 e A3 = 0.
Fazendo A −1 =   , temos:
a c

b d
MATRIZ IDEMPOTENTE  a c  3 7   1 0
Uma matriz quadrada A é dita idempotente se A2 = A. A −1A =Ι 2 ⇒  ⋅  = 
b d 5 11 0 1
A é idempotente ⇒ A2 = A
3a + 5b 7a + 11b  1 0
 2 −2 −4  ⇒ = 
A matriz A =  −1 3 4  é idempotente, pois A2 = A. 3c + 5d 7c + 11d 0 1
 
 1 −2 −3 Usando a igualdade das matrizes obtemos a = −11/2, b = 7/2,
c = 5/2 e d = −3/2.
MATRIZ SINGULAR  11 7 
− 2 2 
Matriz singular é a matriz quadrada cujo determinante é nulo. ⇒A −1
=
 
A é singular ⇒ det A = 0  5 3
− 
 2 2 
Matriz não-singular é a matriz quadrada cujo determinante é
diferente de zero. Esse método é muito trabalhoso, principalmente para matrizes
A é não-singular ⇒ det A ≠ 0 de maior ordem. Vamos então desenvolver outros métodos para a
obtenção da matriz inversa

PROMILITARES.COM.BR 133
MATRIZES

OBTENÇÃO DA MATRIZ INVERSA USANDO MATRIZES EQUIVALENTES


DETERMINANTES Duas matrizes A e B de mesma ordem são equivalentes, indicado
por A~B, quando uma é obtida da outra por meio de uma seqüência
COFATOR de operações elementares.
Seja uma matriz quadrada de ordem n ≥ 2 e aij um elemento
qualquer de A. O cofator do elemento aij é o número Aij =( −1)i+ j ⋅ Mij DETERMINAÇÃO DE MATRIZES INVERSAS
onde Mij é o menor complementar de aij. PELO MÉTODO DE ELIMINAÇÃO DE GAUSS
Calculando os cofatores a partir dos menores obtidos no exemplo Esse método consiste em colocar uma matriz identidade ao lado
anterior: da matriz considerada. Em seguida, efetuam-se as mesmas operações
A11 = ( −1)1+1M11 =
M11 A12 = ( −1)1+ 2M12 =
−M12 elementares nas filas de ambas as matrizes até que a matriz A tenha
sido reduzida à matriz identidade. A matriz identidade sobre a qual
( −1)2+ 2M22 =
A 22 = ( −1)2+ 3M23 =
M22 A 23 = −M23 foram efetuadas as operações elementares será convertida, então, na
matriz inversa de A.
MATRIZ DOS COFATORES Observemos a aplicação desse método no exemplo a seguir:
Seja M uma matriz quadrada de ordem n. A matriz dos cofatores 1 3 3
Seja a matriz A = 1 4 3
M’ é a matriz obtida a partir de M, substituindo cada elemento pelo
seu cofator.  
1 3 4 
 a11 a12  a1n   A11 A12  A1n  1 3 3 1 0 0  1 3 3 1 0 0
a  a2n  A   
=M 
21 a22
= ⇒ M'  21 A 22  A 2n  =[A Ι 3 ] 1 4 3 0 1 0 (1) 0 1 0 −1 1 0 (2)
            ~
1 3 4 0 0 1 0 0 1 −1 0 1
~
     
a a
 n1 n2  ann   An1 An2  Ann 
 1 0 3 4 −3 0  1 0 0 7 −3 −3
 
 1 0 2  −3 9 −1
0 1 0 −1 1 0 (3) 0 1 0 −1 1 0 
Se M = 2 1 3 , então M'
=  2 −6 −1 .
0 0 1 −1 0
~
     1 0 0 1 −1 0 1 
3 1 0  −2 1 1   7 −3 −3

⇒ A −1 =
 −1 1 0 
MATRIZ ADJUNTA  −1 0 1 

Seja M uma matriz quadrada de ordem n e M’ a matriz dos
cofatores de M, a matriz adjunta de M, indicada por M, é a transposta (1) 2ª linha menos 1ª linha e 3ª linha menos 1ª linha
da matriz M’. (2) 1ª linha menos o triplo da 2ª linha
M = (M') t (3) 1ª linha menos o triplo da 3ª linha

 −3 2 −2 EXERCÍCIOS DE
M  9 −6 1  .
TREINAMENTO
No exemplo acima
=
 −1 −1 1 

TEOREMA PARA O CÁLCULO DA MATRIZ INVERSA  a11 a12 a13 


Seja M uma matriz quadrada de ordem n com det M ≠ 0, e M a 01. Seja A uma matriz da forma: a  . Seja f: (3:3) → 
 21 a22 a23 
sua matriz adjunta, a matriz inversa de M, indicada por M–1, é dada por: a função dada por: a31 a32 a33 
1 I.  (3:3) é o conjunto das matrizes quadradas de ordem 3;
M−1
= ⋅ M , onde det M ≠ 0
detM 3

No mesmo exemplo como det M = −5, temos M= −1 1


−5
⋅ II. f(A) = c1c2c3, onde
= ci ∑
= a ,iij 1,2,3 .
 3 2 2  j=1
 −
 −3 2 −2  5 5 5  Assinale a alternativa falsa.

 9 −6 1  =  − 9 6 − 1 .   1 2 −3
   5 5 5 a) f   4 5 6  = 0
 −1 −1 1     
 1 2 1   7 8 0 
−   
 5 5 5 
 1 1 1
Note que, sendo M uma matriz quadrada, a inversa de M existe se,
e somente se, det M ≠ 0. b) f  1 1 1 = 27
 
 1 1 1
  1 2 3   1 3 2
MÉTODO DA ELIMINAÇÃO DE GAUSS
c) f   4 5 6 = f   4 6 5
   
  7 8 9   7 9 8
OPERAÇÕES ELEMENTARES COM FILAS    
Denominam-se operações elementares com filas em uma matriz   1 2 3   1 2 3
às três seguintes operações: d) f   7 8 9 = f   4 5 6
   
a. permutação de duas filas quaisquer;   4 5 6   7 8 9
   
b. multiplicação de uma fila por um escalar; e   1 2 3   1 2 3
c. substituição de uma linha pela sua soma com outra e) f   4 5 6 = f   4 5 9
multiplicada por um escalar não-nulo.    
  7 8 9   7 8 6
   

134 PROMILITARES.COM.BR
MATRIZES

02. (FUVEST 2004) Uma matriz real A é ortogonal se A . At = I, Um pesquisador decide representar as informações presentes no
onde I indica a matriz identidade e At indica a transposta de A. Se texto através do uso de incógnitas de acordo com a tabela a seguir.
1 
x
A =  2  é ortogonal, então x2 + y2 é igual a: INCÓGNITA SIGNIFICADO
 
 y z M Número de óbitos do sexo masculino
1 1 3
a) c) e) F Número de óbitos do sexo feminino
4 2 2

3 3 m Número de homicídios do sexo masculino


b) d)
4 2 f Número de homicídios do sexo feminino
03. (ITA 1995) Dizemos que duas matrizes n×n A e B são semelhantes Assinale a alternativa que apresenta, corretamente, a forma matricial
se existe uma matriz n×n inversível P tal que B = P–1AP. Se A e B são do sistema de equações lineares que representa as informações
matrizes semelhantes quaisquer, então contidas no texto.
a) B é sempre inversível.  0 0 1 1
b) se A é simétrica, então B também é simétrica.  49 49 
 3 0 0   M  59.627
c) B2 é semelhante a A.  10 103   F   59.627
a)
d) se C é semelhante a A, então BC é semelhante a A2.  79  ⋅  =  
 3 0 −1 0   m  0 
e) det (λI −B) = det (λI −A), onde λ é um real qualquer.  10   f   0 
 9 
 0 0 −1
103 
04. (ITA 1994) Sejam A e P matrizes reais quadradas de ordem n tais
que A é simétrica (isto é, A = At) e P é ortogonal (isto é, PPt = I = PtP),
 0 0 1 1
P diferente da matriz identidade. Se B = PtAP então:  49 
49
a) AB é simétrica  2 0 0  M  59.627
 10 102   F   59.627
b) BA é simétrica b)  79  ⋅  =  
 2 0 1 0  m  0 
c) det A = det B  10   f   0 
d) BA = AB  9 
 0 0 1

e) B é ortogonal 102
 1 1 0 0   M  59.627
05. Seja I a matriz identidade de ordem n e, S e U matrizes n x n, onde  0,049 0,049 0 0   F   59.627
c)   ⋅  =  
0 1 1  1  0,079 0 −1 0   m  0 
 1 0 1  1  0
  0,09 0 −1  f   0 
n ≥ 2. Determine a matriz inversa da matriz S =  1 1 0  1 em
   0 0 1 1 
1 1 1  1      
 1 1 1  0  49 49 
1 1 1  1  3 0 0   M  59.627
   10 103   F   59.627
função da matriz U = 1 1 1  1 . d)  79  ⋅   =  
   0 −1 1   m  0 
     103    
1 1 1  1
  f   0 
 9 
0 0 1 
a)
1
d)
1  103 
U−Ι U+ Ι
n n −1
 0 0 1 1   M  59.627
1  4,9 1
b)
1
U+ Ι e) U−Ι e) 0 4,90  F   59.627
n n −1   ⋅  =  
 0 0 1 −7,9  m  0 
c)
1
U  0 0,9 0 1   f   0 
n −1
 1 2 08. Analise as proposições abaixo.
06. Sejam a e b números reais tais que a matriz A =   satisfaz
0 1 I. O produto de uma matriz linha por uma matriz linha é uma matriz
a equação A² = aA + bI, em que I é a matriz identidade de ordem 2. linha.
Logo, o produto ab é igual a II. Uma matriz identidade elevada ao quadrado é uma matriz
a) –2. b) –1. c) 1. d) 2. identidade.
III. O produto de uma matriz por sua transposta é a matriz identidade.
07. Leia o texto a seguir. Assinale a alternativa correta.
Segundo o Sistema de Informações sobre Mortalidade (SIM), do a) Somente as afirmativas I e III são verdadeiras.
Ministério da Saúde, em 2014 houve 59627 homicídios no Brasil, o b) Somente as afirmativas I e II são verdadeiras.
que representa 4,9% do total de óbitos do mesmo ano. Restringindo
esses dados ao sexo masculino, obtemos que 7,9% desse novo total c) Somente a afirmativa II é verdadeira.
de óbitos são homicídios. De forma análoga, se restringirmos os dados d) Somente as afirmativas II e III são verdadeiras.
ao sexo feminino, observamos que aqueles causados por homicídio e) Todas as afirmativas são verdadeiras.
representam 0,9% desse total.
(Adaptado de: Instituto de Pesquisa Econômica Aplicada e Fórum
Brasileiro de Segurança Pública. Atlas da Violência 2016. p. 6).

PROMILITARES.COM.BR 135
MATRIZES

09. Considere uma matriz quadrada de ordem n tal que Ak = 0, para 13. Os marcos A, B, C e D de uma cidade estão conectados por pistas
algum k ∈ N. Qual das opções abaixo representa (I + A)–1, onde I é a de rodagem, conforme mostra a malha viária indicada no diagrama da
matriz identidade de ordem n? figura 1. A figura 2 indica uma matriz que representa as quantidades
k −1 k −1
de caminhos possíveis de deslocamento entre os marcos (dois a dois).
a) I – n ⋅ A c) ∑A
j= 0
j e) I+ ∑ ( −1) .A
j =1
j j Considera-se um caminho entre dois marcos qualquer percurso que
não viole o sentido da pista, que não passe novamente pelo marco de
onde partiu e que termine quando se atinge o marco de destino final
k −1 k −1 pela primeira vez. As flechas da figura 1 indicam o sentido das pistas
b) I+ ∑A
j=1
j
d) I− ∑A j de rodagem.
j=1

1 a 
10. Sendo a um número real, considere a matriz  . Então,
 0 −1
A2017 é igual a
 1 0 c) 1 1
a)  0 1 . 1 1 .

1 a   1 a2017 
b)  0 −1 . d)  .
 0 −1 

11. Um ponto P, de coordenadas (x,y) do plano cartesiano ortogonal, Durante período de obras na malha viária descrita, a pista de rodagem
entre os marcos A e D passou a ser de mão simples (sentido de A
é representado pela matriz coluna   , assim como a matriz coluna
x
para D), e a pista do marco C para o marco D, ainda que tenha
y  permanecido com mão simples, teve seu sentido invertido, passando
 x  representa, no plano cartesiano ortogonal, o ponto P de a ser de D para C. Comparando os 16 elementos da matriz da figura 2
y 
  com seus correspondentes na matriz da nova configuração de malha
viária, a quantidade de elementos que mudarão de valor é igual a
coordenadas (x,y).
a) 5. b) 6. c) 7. d) 8. e) 9.
0 −1  x 
Sendo assim, o resultado da multiplicação matricial   ⋅   é uma
1 0  y 
14. Para cada inteiro positivo n, defina a matriz Mn = 
matriz coluna que, no plano cartesiano ortogonal, necessariamente 1 n
A soma
 0 1 .
representa um ponto que é
dos elementos da matriz produto P = M1 ⋅ M2 ⋅ M3 M21 é
a) uma rotação de P em 180º no sentido horário, e com centro em
a) 229. b) 231. c) 233. d) 235.
(0,0)
b) uma rotação de P em 90º no sentido anti-horário, e com centro
em (0,0). 1 1 1 1
1 2 3 4 
c) simétrico de P em relação ao eixo horizontal x 15. (ITA 2001) Considere a matriz A =  . A soma dos
d) simétrico de P em relação ao eixo vertical y. 1 4 9 16 
 
e) uma rotação de P em 90º no sentido horário, e com centro em 1 8 27 64 
(0,0). elementos da primeira coluna da matriz inversa de A é:
a) 1 b) 2 c) 3 d) 4 e) 5
12. Em um papel quadriculado n x n, com n par, pode-se escrever
todos os números inteiros de 1 a n² em sequência, como no exemplo
16. (IME 2003) Considere a matriz A, n × n, de coeficientes reais, e k
da figura 1, em que se escolheu n = 4. Em seguida, dobrando o papel
um número real diferente de 1. Sabendo-se que A³ = k ⋅ A, prove que
ao meio duas vezes, uma na direção vertical e outra na horizontal,
a matriz A + I é invertível, onde I é a matriz identidade nxn.
faz-se com que alguns dos números escritos se sobreponham. Observe
que, no caso em que n = 4 os números 1, 4, 13 e 16 iriam se sobrepor
no canto superior esquerdo da folha dobrada, como mostrado na 17. (ITA 2005) Sejam A e B matrizes 2 × 2 tais que AB = BA e que
figura 2. satisfazem à equação matricial A² + 2AB – B = 0. Se B é inversível,
mostre que
a) AB–1 = B–1A
b) A é inversível.

18. (ITA 1988) Seja A uma matriz quadrada inversível, de ordem


3. Seja B a matriz dos cofatores da matriz A. Sabendo-se que
det A = −2, calcule det B.

19. (OIMU 2003) Sejam A e B matrizes reais n × n tais que


AB + A + B = 0. Prove que AB = BA.

Repetindo o procedimento descrito acima para um papel quadriculado 20. Determine duas matrizes 2 × 2 B e C com elementos inteiros tais
50 × 50, um dos números que ficaria sobreposto ao número 2016 é  −1 1  3 3
que  =
 B +C .
a) 435. c) 484. e) 536.  0 −2
b) 436. d) 485.

136 PROMILITARES.COM.BR
MATRIZES

EXERCÍCIOS DE Destas, é(são) verdadeira(s)

COMBATE a) apenas II.


b) apenas I e II.
d) apenas II e III.
e) todas.
c) apenas I e III.

01. (ITA 2019) Considere as seguintes afirmações a respeito de 06. (ITA 2003) Sejam A e P matrizes n x n inversíveis e B = P– 1 AP.
matrizes A de ordem n × n inversíveis, tais que os seus elementos e os
de sua inversa sejam todos números inteiros: Das afirmações:

I. |det(A)| = 1. I. Bt é inversível e (Bt)-1 = (B-1)t.

II. AT = A–1. II. Se A é simétrica, então B também o é.

III. A + A é uma matriz diagonal.


–1 III. det(A - x) = det(B - x), ∀x ∈ .

É(são) sempre VERDADEIRA(S) é(são) verdadeira(s):


a) apenas I. c) apenas I e II. e) todas. a) todas. d) apenas I e III.
b) apenas III. d) apenas I e III. b) apenas I. e) apenas lI e III.
c) apenas I e lI.
02. (ITA 2018) Sejam A e B matrizes quadradas n × n tais que
A + B = A ⋅ B e In a matriz identidade n × n. Das afirmações: 07. (ITA 2002) Sejam A e B matrizes quadradas de ordem n tais que
I. In – B é inversível; AB = A e BA = B. Então [(A + B)t]² é igual a
II. In – A é inversível; a) (A + B)² d) At + Bt
III. A ⋅ B = B ⋅ A. b) 2(A ⋅ B )
t t
e) At ⋅ Bt
é (são) verdadeira(s) c) 2 (A + B )
t t

a) Somente I. c) Somente III. e) Todas.


08. (ITA 2011) Considere as afirmações abaixo:
b) Somente II. d) Somente I e II.
I. Se M é uma matriz quadrada de ordem n > 1, não–nula e não–
inversível, então existe matriz não–nula N, de mesma ordem, tal
1 −1
03. (ITA 2016) Se M =   2 1
 e N =  −1 3 , então M ⋅ N – M N
T –1
que M ⋅ N é matriz nula.
2 0    II. Se M é uma matriz quadrada inversível de ordem n tal que
é igual a
det(M² – M) = 0, então existe matriz não–nula X, de ordem n × 1,
3 5 3 11 3 11
tal que MX = X.
 2 − 2  2 − 2  2 − 2
a)   c)   e)    cos θ − sen θ 
5 − 3 13 − 5  13 − 3  π
III. A matriz  tg θ θ  é inversível, ∀θ ≠ + kπ , k ∈ Z.
 2 2   2 2   2 2   1 − 2sen2  2
 sec θ 2 
3 1 3 5
 2 − 2  2 − 2 Destas, é(são) verdadeira(s):
b)   d)  
7 − 5 13 − 3  a) apenas II. d) apenas II e III.
 2 2   2 2  b) apenas I e II. e) todas.
c) apenas I e III.
04. (ITA 2015) Seja A = (aij)5×5 a matriz tal que aij = 2i – 1(2j –1), 1 ≤ i,
j ≤ 5. Considere as afirmações a seguir: 09. (IME 2012) São dadas as matrizes quadradas inversíveis A, B e
I. Os elementos de cada linha i formam uma progressão aritmética C, de ordem 3. Sabe-se que o determinante de C vale (4 – x) onde
de razão 2i. 1
x é um número real, o determinante da matriz inversa de B vale −
II. Os elementos de cada coluna j formam uma progressão 3
geométrica de razão 2. e que (CAt)t = P–1BP, onde P é uma matriz inversível. Sabendo que
 0 0 1
III. tr A é um número primo.
A =  3 x 0 , determine os possíveis valores de x.
É (são) verdadeira(s)  
 1 0 0
a) apenas I. d) apenas I e III.
b) apenas I e II. e) I, II e III. Obs.: (M)t é a matriz transposta de M.
c) apenas II e III. a) –1 e 3 d) 1 e 3
b) 1 e –3 e) –2 e –3
05. (ITA 2011) Considere as afirmações abaixo: c) 2e3
I. Se M é uma matriz quadrada de ordem n > 1, não nula e não
inversível, então existe matriz não nula N, de mesma ordem, tal 10. Sejam A e S matrizes reais n × n, com S uma matriz antissimétrica.
que M N é matriz nula. Se A verifica a relação A = (I + S) ⋅ (I – S)–1, com (I – S) não singular,
II. Se M é uma matriz quadrada inversível de ordem n tal que det então podemos concluir que A é necessariamente:
(M2 – M) = 0, então existe matriz não nula X, de ordem n × 1, tal a) uma matriz nilpotente.
que MX = X.
b) uma matriz idempotente.
III. A matriz
c) uma matriz diagonal singular.
 cos θ −senθ 
d) uma matriz de Vandermonde não singular.
 tgθ π
 θ  é inversível, ∀θ ≠ + kπ, k ∈ .
1 − 2sen2  2 e) uma matriz ortogonal.
 sec θ 2

PROMILITARES.COM.BR 137
MATRIZES

11. (ITA 1986) Dizemos que duas matrizes reais, 2 × 1, A e B 16. A matriz quadrada M, de ordem n > 1, satisfaz a equação
quaisquer são linearmente dependentes se e somente se existem dois M² – M – I, onde I é a matriz identidade de ordem n > 1. Determine,
números reais x e y não ambos nulos tais que xA + yB = 0, onde 0 é em termos de M e I, a matriz M2003.
 1  k −n + 1
a matriz nula 2 × 1. Se A =  n  , B =   , onde k ∈ * e n ∈
k − 1  2   1 0
17. (ITA 2016) Seja A a matriz de ordem 3 × 2, dada por A = 0 1 .
 = {1, 2, 3, ...} podemos afirmar que, para cada n ∈ ,
 1 1
a) A e B são linearmente dependentes, ∀ k ∈ *. a) Determine todas as matrizes B tais que BA = I . 2

b) existe um único k ∈ * tal que A e B não são linearmente b) Existe uma matriz B com BA = I2 que satisfaça BBT = I2? Se sim, dê
dependentes. um exemplo de uma dessas matrizes.
c) existe um único k ∈ * tal que A e B são linearmente dependentes.
18. (ITA 2011) Determine todas as matrizes M ∈ M2×2() tais que
d) existem apenas dois valores de k ∈ * tais que A e B são
MN = NM, ∀N ∈ M2×2().
linearmente dependentes.
e) não existe valor de k ∈ * tal que A e B são linearmente
19. (ITA 2008) Uma matriz real quadrada A é ortogonal se A é inversível
dependentes.
e A-1 = At. Determine todas as matrizes 2 × 2 que são simétricas e
ortogonais, expressando-as, quando for o caso, em termos de seus
0 −3
elementos que estão fora da diagonal principal.
1 1 2 5
12. (EN 2013) Sejam A =  e B= e B’ a
 4 −3 0 1 −2 6 
20. (IME 2017) Seja M uma matriz real 2 × 2. Defina uma função f na
transposta de B. O produto da matriz A pela matriz B’ é qual cada elemento da matriz se desloca para a posição seguinte no
 a b  c a
 9 2 10   5 4  −1 10 sentido horário, ou seja, se M =  , implica que f(M) =  .
e)  c d  d b
a)  −8 6 0 c)  0 6  −2 1 
   
 21 −21 −6  −6 0 Encontre todas as matrizes simétricas 2 × 2 reais na qual M² = f(M).

 5 0 −6  −1 11
b)  4 6 0  d)  20 10 GABARITO
EXERCÍCIOS DE TREINAMENTO
01. E
 1 a −2
13. (IME 2017) Seja A = a − 2 1 1  com a ∈ . Sabe-se que   1 2 −3
 2 −3 1  f   4 5 6  = (1+ 2 − 3) ⋅ ( 4 + 5 + 6) ⋅ ( 7 + 8 + 0) = 0
 
  7 8 0 
det(A² – 2A + I) = 16. A soma dos valores de a que satisfazem essa  
 1 1 1
condição é: f  1 1 1 = (1+ 1+ 1) ⋅ (1+ 1+ 1) ⋅ (1+ 1+ 1) = 27
 
Obs.: det(X) denota o determinante da matriz X.  1 1 1
 
a) 0 b) 1 c) 2 d) 3 e) 4 Nas letras c) e d), basta observar que trocar a ordem de linhas ou
colunas não altera o resultado da operação.
 a b   1 2 3
14. (IME 2016) Seja A =   . O maior valor de a, com a ≠ 1, que
 −b a f   4 5 6 = (1+ 2 + 3) ⋅ ( 4 + 5 + 6) ⋅ ( 7 + 8 + 9) = 2160
 
satisfaz A24 = I é:   7 8 9
 
Obs.: I é a matriz identidade 2 x 2.   1 2 3
a)
1 2 f   4 5 9 = (1+ 2 + 3) ⋅ ( 4 + 5 + 9) ⋅ ( 7 + 8 + 6) = 2268 ⇒
d) ( 3 − 1)  
  7 8 6
2 4
2 2
b) e) ( 3 + 1)   1 2 3   1 2 3
2 4 ⇒ f   4 5 6 ≠ f   4 5 9
3    
c)   7 8 9   7 8 6
   
2

15. (FUVEST 2004) Uma matriz real A é ortogonal se A ⋅ At = I, onde I


02. E
1 
x
indica a matriz identidade e At indica a transposta de A. Se A =  2  A ⋅ At = Ι ⇔
 
é ortogonal, então x2 + y2 é igual a:  y z 1  1 
 2 x ⋅  2 y =  1 0
    0 1
1 3  y z  x z
a) d)
4 2  1 1 
2
 4+x 2
y + xz
 1 0
3 3 ⇒ =  
b) e)  1 y + xz y 2 + z2  0 1
4 2
 2 
1 1
c) 2
2 4 + x = 1

1
⇒  y + xz = 0
2
 y 2 + z2 =1

138 PROMILITARES.COM.BR 
3
⇒ x2 =
y z  x z
 1 2 1  MATRIZES
 4+x 2
y + xz
 1 0
⇒ =  
 1 y + xz y 2 + z2  0 1
 2 
1 2 1 0 0  0
4 + x = 1
0
  1 0  0
1 Ι = 0 0 1  0 e
⇒  y + xz = 0
2  
y + z =
2 2     
1 0
  0 0  1

3 1 1 1  1
⇒ x2 = 1 1 1  1
4 
U = 1 1 1  1 ,
3 3  
y = −2xz ⇒ y² = 4x²zz ⇒ y 2 = 4 ⋅ ⋅ (1 − y z ) ⇒ y 2 =     
4 4
3 1 1 1  1
⇒ x2 + y2 = 
2
tais que S = U – I e U² = n ⋅ U.
03. E Para qualquer número real c, temos
λI − B = −1
λI − P AP = −1
λ (P P) − P AP = −1 −1
P ( λP) − P AP = −1 (U − I) ( cU − I)= cU2 − ( c + 1) U + I
−1 −1 −1
= P ( λP − AP) = P [λ ( Ι ⋅ P) − AP] = P [( λΙ ) ⋅ P − AP] = = ( cn − ( c + 1) ) U + I
= P −1( λ ⋅ Ι − A)P 1
Assim, se escolhermos cn – (c + 1) = 0 ⇔ c = , teremos
⇒ det( λ = −1
⋅ Ι − B) det[P ( λ ⋅ Ι =
− A)P] n −1
−1 1
1 S−1 =(U − I) = U−I
−1
= detP ⋅ det( λ ⋅ Ι − A) ⋅ detP
= ⋅ det( λ ⋅ Ι − A) ⋅ detP n −1
detP 2 − n 1 1 
= det( λ ⋅ Ι − A)  n − 1 n − 1  n − 1
 
 1 2−n 1 

=  n −1 n −1 n − 1
04. C  
I. VERDADEIRA      
 1 1 2 − n
  
B = Ι − A ⇒ B2 = ( Ι − A ) = Ι 2 − ΙA − AΙ + A 2 =
2
 n −1 n −1 n − 1
= Ι − A −A +A = Ι − A = B

II. VERDADEIRA 06. A


Tem-se que
AB = A ( Ι − A ) = AΙ − A 2 = A − A = 0n 
 1 2  1 2  1 2  1 0
BA = ( Ι − A ) A = ΙA − A 2 = A − A = 0n  A 2 = aA + bI ⇔  ⋅ = a  + b 0 1
0 1 0 1 0 1  
⇒ AB = BA
1 4  a + b 2a 
⇔ =
III. FALSA 0 1  0 a + b
AB =0n ⇒ det ( AB) =det 0n a + b = 1
⇔
⇒ det
 A ⋅ det B =
0 ⇒ det B =0 2a = 4
≠0
a = 2
Logo, B não é inversível. ⇔ ⋅
b = −1
IV. VERDADEIRA
Por conseguinte, vem a . b = 2 ⋅ (–1) = –2.
B = Ι − A ⇔ A + B = Ι ⇒ ( A + B) = Ι 2
2

⇔ A 2 + AB + BA + B2 = Ι ⇔ 07. A
⇔ A 2 + 0n + 0n + B2 =Ι ⇔ A 2 + B2 =Ι Sabendo que:
V. VERDADEIRA, pois AB = 0n.
INCÓGNITA SIGNIFICADO
É possível identificar as matrizes A e B, e posteriormente tirar as
conclusões acima.
Número de óbitos do
M
A 2 = A ⇒ A −1 ⋅ A ⋅ A = A −1 ⋅ A sexo masculino
⇔ Ι⋅A = Ι ⇔ A = Ι 0,049(M + F) = 59627
B = Ι − A ⇒ B = Ι − Ι = 0n Número de óbitos do
F
sexo feminino

05. E Número de homicídios


m m = 0,079M
do sexo masculino
Sejam as matrizes n×n
Número de homicídios
f f = 0,009F
do sexo feminino

Pode-se escrever:

PROMILITARES.COM.BR 139
MATRIZES

 0 0 1
1
 49 49 
 3 0 0   M  m+ f   59.627
 10 103   F  0,049M + 0,049F  59.627
 79 =  ⋅   =   
 3 0 −1 0   m  0,079M − m   0 
 10   f   0,009F − f   0 
 9 
 0 0 −1
103 

08. C
[I] Falsa. Sejam A = [1 –1] e B = [1 0 1] duas matrizes linha. Como
as ordens de A e de B são, respectivamente, iguais a 1 × 2 e 1 × 3,
podemos concluir que a matriz produto A . B não existe, uma vez que
o número de colunas da matriz A é diferente do número de linhas da
matriz B.
Portanto:
[II] Verdadeira. Sabendo que In é matriz identidade de ordem n, e
x =9 ⋅ 50 + 16 =466.
sendo A uma matriz quadrada de ordem n, temos A ⋅ In = In ⋅ A = A.
Portanto, se A = In então I²n = In ⋅ In = In. y =9 ⋅ 50 + 35 =485.
z = 40 ⋅ 50 + 35 = 2035.
[III] Falsa. Considere a matriz A = 
1 0
 e a sua transposta
Portanto, a resposta correta é [D], 485.
0 0
 1 0 Desse modo, temos A ⋅ A t  1 0  1 0  1 0
At =  = 0 0 ⋅ 0 = .
 0 0 .
    0 0 0 13. B

Mas A ⋅ At ≠ I2. Pelas informações do enunciado, tem-se:

09. E
(I + A ) (I − A + A 2 − A 3 +  + ( − A )(k −1) ) = I − ( − A )k = I
k −1

∑ ( −1) ⋅ A
−1 (k −1)
⇒ (I + A ) = I − A + A2 − A3 +  + ( −A )
j j
= I+
j=1

10. B Assim, após as mudanças nas vias a nova matriz será:


Calculando:
 1 a   1 a   1 0
A2 =  ⋅ =
 0 −1  0 −1  0 1
A2 = I
A 4 = A2 ⋅ A2 = I ⋅ I = I
A6 = A 4 ⋅ A 2 = I ⋅ I = I

A 2016 = A 2014 ⋅ A 2 = I ⋅ I = I Portanto, 6 elementos da matriz terão seus valores alterados.
2017 2016 2017 1 a 
A =A ⋅ A =⋅
I A =A → A =
 0 −1 14. C
Escrevendo as matrizes e fazendo as multiplicações:
11. B  1 1  1 2  1 3
=
M1 ⋅ M2  = .
Fazendo a multiplicação proposta:  0 1  0 1  0 1
0 −1  x   − y   1 3  1 3  1 6
=
M1 ⋅ M2 ⋅ M3  0 = .
1 0  ⋅ y  =
  1  0 1  0 1
    x
 1 6  1 4  1 10
Assim, se substituirmos os valores de x e y por números e M1 ⋅ M
=2 ⋅ M3 ⋅ M4 = .
representarmos estes no plano cartesiano o resultado da multiplicação  0 1  0 1  0 1 
proposta representa um ponto que é uma rotação de P em 90º no
É possível perceber que a cada multiplicação, o resultado será sempre
sentido anti-horário, e com centro em (0,0).
o mesmo para os elementos a11, a21 e a22. Quanto ao elemento
a12, este será a soma dos elementos correspondentes nas matrizes
12. D multiplicadas. Assim, o elemento a12 da matriz P é igual a soma
a12 = 1 + 2 + 3 + 4...+ 21, ou seja, uma PA de 21 elementos e razão
O número 2016 encontra-se na linha 41, na coluna 16, pois
40 ⋅ 50 + 16 = 2016. 1. A soma de todos os elementos desta PA será  1+ 21 ⋅ 21 = 231.
 2 
Considerando que as dobras serão feitas entre as linhas 25 e 26 e  1 231
Logo, a matriz P será: P =  e a soma de seus elementos é
entre as colunas 25 e 26, temos os elementos que serão sobrepostos  0 1 
ao número 2016 indicados na tabela abaixo: 1 + 231 + 0 + 1 = 233.

140 PROMILITARES.COM.BR
 −2 0 
A 2 + 3A = 2
 =−2I ⇔ A + 3A + 2I =0
 0 −2 MATRIZES
3 2 3 2
⇒ A + 3A + 2A =0 ⇒ A + 3A + 3A + I =A + I ⇔
⇔ ( A + I) = A + I ⇔ A = ( A + I) + ( −I)
3 3 3

15. A 0 1   −1 0 
⇒ B =A +I =  e C = −I =  0 −1
A ⋅ A–1 = I  0 −1  
1ª linha de A: L1 = [1 1 1 1]
EXERCÍCIOS DE COMBATE
α 
  01. A 06. D 11. D
1ª coluna de A : C =  β 
−1
02. E 07. C 12. D
1
γ 
  03. C 08. E 13. D
φ
04. E 09. D 14. E
O elemento a11 = 1 da matriz identidade é a11 = L1 ⋅ C1 = 1
05. E 10. E 15. E
⇒α+β+γ+δ=1

16. 16. M2= M − Ι


M=3
M2 ⋅ M
= (M − Ι ) ⋅ M
= M2 − M
= (M − Ι ) − M
= −Ι
(A + Ι )(aA 2 + bA + cΙ=
) aA 3 + bA 2 + cA + aA 2 + bA + cΙ
(A + Ι )(aA 2 + bA + c=
Ι ) akA + bA 2 + cA + aA 2 + bA + cΙ
M4 =M3 ⋅ M =( −Ι ) ⋅ M =−M

(A + Ι )(aA 2 + bA + cΙ ) = (a + b)A 2 + (ak + b + c)A + cΙ M5 =M4 ⋅ M =( −M) ⋅ M =−M2 =−(M − Ι ) =Ι −M


Tomando a + b = ak + b + c = 0 e c = 1, temos M6 = M5 ⋅ M = ( Ι − M) ⋅ M = M − M2 = M − (M − Ι ) = Ι
1 1 ⇒ M2003 = (M6 )333 ⋅ M5 = Ι333 ⋅ ( Ι − M) = Ι − M
a= ,b = − e c= 1
1− k 1− k
1 2 1
⇒ (A + Ι )( A − A + Ι) = Ι 17.
1− k 1− k
1 2 1 a) A matriz B deverá ser do tipo 2 × 3 para que B ⋅ A = I2, assim:
⇒ (A= + Ι ) −1 A − A+Ι
1− k 1− k x y z
B= 
a b c
17. e
a) AB = BA ⇔ AB ⋅ B−1 = BA ⋅ B−1 ⇔ A = BA ⋅ B−1  1 0
x y z    1 0  x + z y + z  1 0
  0 1 = 
⋅ ⇒ =
−1 −1 −1 −1 −1
⇔ B ⋅ A = B ⋅ BA ⋅ B ⇔ B ⋅ A = A ⋅ B  
 a b c   1 1  0 1  a + c b + c   0 1
2
b) A + 2AB − B =0 ⇔ B= A × ( A + 2B)  
⇒ det B = det A ⋅ det (A +2B) Da equação acima temos dois sistemas lineares:
B é inversível ⇒ det B ≠ 0 ⇒ det A ≠ 0 ⇒ A é inversível. x + z =1
 ⇒ z =1 − x e y = x - 1
y + z =0
18.
a + c =0
−1 1 −1  1   ⇒ c =−a e b =+
1 a
A= ⋅ Bt ⇒ det A= det  ⋅ Bt
det A  det A  b + c =1
3
 1  Portanto, todas as matrizes B serão da forma:
⇒ det A −1 =
  det B
t
det A   x x − 1 1− x 
1 1 B=  , com x e a números reais.
⇒ = ⋅ det B ⇒ detB = ( detA ) = ( −2)2 = 4
2
 a 1+ a 1+ a 
det A (det A )3
b) Efetuando o produto da matriz x pela sua transposta, temos:
 x a 
19.  x x − 1 1− x     1 0
 ⋅
  x − 1 1 + a =
 
AB + A + B =0 ⇒ AB + A + B + I =I ⇒  a 1 + a −1  1 − x −a   0 1 
A(B + I) + (B + I) =I ⇒ (A + I)(B + I) =I  

Logo, A + I e B + I são inversas uma da outra, donde Efetuando o produto das matrizes, temos as seguintes equações:
(A + I)(B + I) = (B + I)(A + I) = I 1
x 2 + (x − 1)2 + (1 − x)2 = 1 ⇒ 3x 2 − 4x + 1 = 0 ⇒ x = 1 ou x =
Expandindo a última desigualdade vem BA + B + A + I = I e subtraindo 3
esta da igualdade dada no enunciado obtém-se AB = BA. ax + (a + 1) ⋅ (x − 1) − a ⋅ (1 − x) =0

Admitindo x = 1 na equação acima, temos a = 0.


20. Portanto, uma possível matriz B será:
 −1 1   1 −3  1 0 0
=
A  A2 
 ⇒= ⇒ B= 
 0 −2 0 4  0 1 0
 −2 0 
A 2 + 3A = 2
 =−2I ⇔ A + 3A + 2I =0
 0 −2
18.
⇒ A 3 + 3A 2 + 2A =0 ⇒ A 3 + 3A 2 + 3A + I =A + I ⇔
x y  a b
⇔ ( A + I) = A + I ⇔ A = ( A + I) + ( −I)
3 3 3 Considerando M =   e N= 
z w  c d
0 1   −1 0 
⇒ B =A +I =  e C = −I =  0 −1
0 −1  

PROMILITARES.COM.BR 141
MATRIZES

 ax + cy = ax + bz
 az + cw =cx + dz

Fazendo M ⋅ N = N ⋅ M temos: 
 bx + dy = ay + bw
bz + dw =cy + dw
Considerando b = 0 e c = 1, concluímos que y = 0. Tomando agora
b = q e c qualquer com y = 0, obtemos z = 0, então x = w.
 x 0
Logo,
= M   , ∀x
0 x 

 1 0   −1 0   1 − b   − 1 − b2 
2
b b
19.  ,  ,  e 
 0 1   0 −1  b − 1 − b2   b 1 − b2 

20. Como queremos M simétrica, pode-se escrever:


a b
M= 
b c
a b   a b   a2 + b2 ab + bc 
M2 =  ⋅ = 
b c   b c   ab + bc b2 + c2 
a2 + b2 =
b

 b a   a + b ab + bc  ab + bc =
2 2
a
=
f(M) =   2 
→
 c b   ab + bc b + c 
2
ab + bc =c
b2 + c2 =
b

Logo c = a, portanto:
b =1
a= 0 → ou
b=0
a + b =
2 2
b
 → ou
2ab = a
b =12 → a =± 12

Assim,
 0 0 0 1 1 1  − 1 1 
     2 2  ; 2 2 
S =   ; ; 
 0 0   1 0   12 1   1
2  2 − 1 
2 

ANOTAÇÕES

142 PROMILITARES.COM.BR
DETERMINANTES E
SISTEMAS LINEARES

PROPRIEDADES DO DETERMINANTES VII. Seja uma matriz quadrada A, de ordem n. Se A possui duas
linhas (ou colunas) proporcionais, então
I. Seja uma matriz quadrada A, de ordem n, então: det A = 0
det A = det At
ka kb kc
ou seja, o determinante da matriz é igual ao determinante da sua Assim, x y z = 0, pois a 1ª e a 3ª linha são proporcionais.
matriz transposta.
a b c
II. Seja uma matriz quadrada A, de ordem n ≥ 2. Se a matriz B é
obtida trocando-se as posições de duas linhas (ou colunas), então VIII. Seja A, B e C três matrizes quadradas de mesma ordem, tais
det B = −det A que B e C são idênticas exceto pela i-ésima linha, A é idêntica
a B e C exceto pela i-ésima linha que é obtida somando as
1 4 7 7 4 1 i-ésimas linhas de B e C, então
Assim, o determinante 2 5 8 = − 8 5 2 onde foi trocada a det A = det B + det C
3 6 9 9 6 3
Vamos aplicar essa propriedade ao cálculo do determinante a
posição da 1ª e da 3ª coluna. seguir:
a 1 3a + 2 a 1 3a a 1 2
III. Seja uma matriz quadrada A, de ordem n ≥ 2. Se A possui b 2 3b + 4 = b 2 3b + b 2 4 = 0 + 0 = 0
duas filas paralelas iguais, então
c 3 3c + 6 c 3 3c c 3 6
det A = 0
Note que os dois determinantes resultantes possuem colunas
a b c
proporcionais.
Logo, x y z = 0, pois a 1ª e a 3ª linha são iguais.
a b c
IX. Teorema de Jacobi: Adicionando-se a uma linha (ou coluna)
uma outra linha (ou coluna) multiplicada por um número, o
IV. Seja uma matriz quadrada A, de ordem n. Se A possui uma determinante não se altera.
fila (linha ou coluna) com todos seus elementos nulos, então
A utilidade desse teorema consiste em zerar alguns elementos de
det A = 0 determinada fila a fim de simplificar os cálculos após a aplicação do
0 0 0 Teorema de Laplace.
Assim, x y z = 0, pois a 1ª linha é nula. Vamos aplicar a propriedade para simplificar o cálculo do
determinante abaixo, subtraindo da 3ª linha o dobro da 1ª linha e
a b c
posteriormente utilizando Laplace na 3ª coluna.

V. Seja uma matriz quadrada A, de ordem n. Seja B a matriz 4 2 1 4 2 1


2 3
obtida multiplicando-se uma fila (linha ou coluna) de A pelo 2 3 0 = 2 3 0 = 1⋅ ( −1)1+ 3 ⋅ =
−3 3
número k, então 5 7 2 5 − 2 ⋅ 4 7 − 2 ⋅ 2 2 − 2 ⋅1
det B = k ⋅ det A 2 3
= 1⋅ ( −1)1+ 3 ⋅ = 2.3 − 3.( −3) = 15
Uma consequência dessa propriedade é que é possível colocar em −3 3
evidência algum fator comum a todos os elementos de uma fila (linha
ou coluna), antes de efetuar o cálculo do determinante.
X. Adicionando-se a uma linha (ou coluna) uma combinação linear
2 4 7 1 4 7 das outras linhas (ou colunas) o determinante não se altera.
Dessa forma, 4 5 8 = 2 ⋅ 2 5 8
6 6 9 3 6 9
XI. Seja uma matriz quadrada A, de ordem n. Se A possui uma
linha (ou coluna) que seja combinação linear das outras linhas
VI. Seja uma matriz quadrada A, de ordem n e k um número (ou colunas), então
real, então
det A = 0
det (k ⋅ A) = kn ⋅ det A
x m ax + bm
É importante observar a diferença entre essa propriedade e a O determinante y n ay + bn = 0, pois a 3ª coluna é uma
anterior. Nessa caso multiplica-se a matriz de ordem n por k, o que
equivale a multiplicar por k cada uma das n linhas do determinante, z p az + bp
resultando que o determinante fica multiplicado por kn. combinação linear da 1ª e 2ª colunas (a vezes a 1ª mais b vezes a 2ª).

PROMILITARES.COM.BR 143
DETERMINANTES E SISTEMAS LINEARES

XII. Teorema de Cauchy: Seja uma matriz quadrada A, de ordem MATRIZ DE VANDERMONDE
n ≥ 2. A soma dos produtos dos elementos de uma linha
(ou coluna), ordenadamente, pelos cofatores dos elementos Matriz de Vandermonde é a matriz quadrada na qual as colunas
correspondentes de outra linha (ou coluna), é igual a zero. são formadas por potências de mesma base, com expoente inteiro
n
det A, se p = q variando desde 0 a (n 1), ou seja, os elementos de cada coluna formam
∑a
j=1
pj ⋅ Aqj =

0, se p ≠ q
uma progressão geométrica de 1º elemento 1.
Os elementos da 2ª linha são chamados elementos característicos
n
det A, se p = q da matriz.
∑i=1
aip ⋅ Aiq =

0, se p ≠ q  1 1 1  1 
 a a a  an 
Note que o caso p = q é a expressão do Teorema de Laplace.  1 2 3

M =  a12 a22 a32  an2 


 
      
XIII. Determinante da Matriz Triangular: Seja uma matriz  n −1 n −1 n −1 
triangular (superior ou inferior), o seu determinante é o a1 a2 a3  ann −1
produto dos elementos da diagonal principal.
O determinante da matriz de Vandermonde de elementos
a11 a12 a13  a1n  característicos a1, a2, ..., an é indicado por V(a1, a2, ..., an).
0 a  a2n 
 22 a23 O determinante V(a1, a2, ..., an). é igual ao produto de todas as
A=0 0 a33  a3n  (matriz triangular superior) diferenças ai – aj, com i > j.
 
 
0


0

0


 
ann 
V(a1, a2 =
, ,an ) ∏ (a − a )
1≤ j<i≤n
i j

a11 a12 a13  a1n 


0 a a2n 
 22 a23  TEOREMA DE BINET
A=0 0 a33  a3n  (matriz triangular inferior)
  Sejam A e B matrizes quadradas de mesma ordem, então:
      
 0 0 0  ann  det (A ⋅ B) = det A ⋅ det B
n

∑a .
Em ambos os casos det A = a11 ⋅ a22 ⋅  ⋅ ann = Esse teorema permite que seja calculado o determinante de
ii
um produto de matrizes através do determinante dos fatores, sem
i=1
necessidade de efetuar o produto.
XIV. Determinante da Matriz Identidade: O determinante da Uma conseqüência imediata é que, se A é uma matriz quadrada
matriz identidade vale 1. e n ∈ N*, temos:

det In = 1 det(An) = (detA)n

Por exemplo, se A = 
1 3 0 2
 e B =  1 3 , então
REGRA DE CHIÓ  2 4   
det(A ⋅ B) = det A ⋅ det B = (1 ⋅ 4 − 2 ⋅ 3) ⋅ (0 ⋅ 3 −1 ⋅ 2) = (−2) ⋅ (−2) = 4
A Regra de Chió permite abaixar a ordem do determinante no Considerando a expressão A–1A = AA–1 = In e o teorema de Binet,
qual a11 = 1, o que pode ser obtido pela aplicação das propriedades temos:
anteriores.
Regra Prática: 1
det(A −1) =
1º) Seja uma matriz de ordem n com a11 = 1, suprimem-se a 1ª det A
linha e a 1ª coluna.
2º) De cada elemento restante da matriz subtraímos o produto dos EXERCÍCIOS DE

TREINAMENTO
elementos que se encontram nas extremidades das perpendiculares,
traçadas do elemento considerado à 1ª linha e à 1ª coluna.
3º) Com as diferenças obtidas, constrói-se uma matriz de ordem
(n −1) cujo determinante é igual ao determinante original.

a b 
Vamos aplicar esse método para o cálculo do determinante a 01. (UECE 2019) Os elementos a, b, c, d da matriz M =   são
c d
seguir, onde inicialmente foram trocadas as posições da 1ª e 2ª linhas
distintos entre si e escolhidos aleatoriamente no conjunto {1, 3, 5, 7}.
e depois da 1ª e 3ª colunas para que tivéssemos a11 = 1:
Considerando-se, para cada escolha destes elementos, d o determinante
6 2 3 5 −2 3 1 4 1 3 −2 4 de M, o número de valores distintos que d pode assumir é
−2 3 1 4 6 2 3 5 3 2 6 5 a) 6. b) 8. c) 16. d) 24.
=
− = =
3 2 7 9 3 2 7 9 7 2 3 9
02. (UNICAMP 2019) Sabendo que a e b são números reais, considere
0 15 −2 3 0 15 −2 3 −2 15 0 3
 1 a 1
2 − 3⋅ 3 6 − 3 ⋅ ( −2) 5 − 3⋅ 4 −4 12 −7 a matriz quadrada de ordem 3, A =  b 1 a .
 
= 2 − 7⋅3 3 − 7 ⋅ ( −2) 9 − 7⋅ 4 = −19 17 −19 =  2 b 2
15 − ( −2) ⋅ 3 0 − ( −2) ⋅ ( −2) 3 − ( −2) ⋅ 4 21 −4 11 Se a soma dos elementos em cada linha da matriz A tem sempre o
= −757 mesmo valor, então o determinante de A é igual a
a) 0. b) 2. c) 5. d) 10.

144 PROMILITARES.COM.BR
DETERMINANTES E SISTEMAS LINEARES

03. (ITA 2018) Sejam A e B matrizes quadradas n × n tais que  1 0 0 7 0 2


A + B = A ⋅ B e In a matriz identidade n × n. Das afirmações: 09. (ITA 2017) Sejam D = 0 2 0 e P = 0 1 0 .
I. In – B é inversível;    
0 0 3 2 0 5
II. In – A é inversível;
Considere A = P–1DP. O valor de det(A² + A) é
III. A ⋅ B = B ⋅ A.
a) 144. c) 240. e) 360.
é (são) verdadeira(s)
b) 180. d) 324.
a) Somente I. d) Somente I e II.
b) Somente II. e) Todas.  1 a −2
c) Somente III. 10. (IME 2017) Seja A = a − 2 1 1  com a ∈ . Sabe-se que
 2 −3 1 
04. (ESPCEX (AMAN) 2018) Uma matriz quadrada A, de ordem 3, é
det(A² – 2A + I) = 16. A soma dos valores de a que satisfazem essa
i − j, se i > j
definida por aij =  . condição é:
i+ j
( −1) , se i ≤ j Obs.: det(X) denota o determinante da matriz X
Então det(A–1) é igual a a) 0 b) 1 c) 2 d) 3 e) 4
a) 4. 1
d) .
b) 1. 4 11. (UECE 2016) Se V é uma matriz quadrada e n é um número
1 natural maior do que um, define-se Vn = V ⋅ Vn–1. Com essa definição,
c) 0. e) .
2  1 2
para a matriz V =  , pode-se afirmar corretamente que o valor
05. (ITA 2018) Sejam x1, ..., x5 e y1, ..., y5 números reais arbitrários e  0 1
A = (aij) uma matriz 5 × 5 definida por aij = xi + y1, 1 ≤ i, j ≤ 5. Se r é a
do determinante da matriz Y =V + V 2 + V 3 +  + V 2016 é igual a
característica da matriz A, então o maior valor possível de r é
a) 2 × 2016.
a) 1. b) 2. c) 3. d) 4. e) 5.
b) 2 × 2017.
06. (ITA 2018) Considere a matriz c) 2016 × 2016.
1 x x 2
x 
3 d) 2016 × 2017.
 
1 2 3 4
, x ∈ . 12. (EN 2016) A equação
 −1 3 4 5
  sen2x 1 sec2 x
 −2 2 1 1  31
1 cos2 x 0 = − ,
Se o polinômio p(x) é dado por p(x) = det A, então o produto das 16
1 0 1
raízes de p(x) é
1 1 1  π
a) . c) . e) . com x ∈  0,  , possui como solução o volume de uma pirâmide com
2 5 11  2
1 1 base hexagonal de lado  e altura h = 3. Sendo assim, é correto
b) . d) .
3 7 afirmar que o valor de  é igual a:
2π 2 8π π
07. (EPCAR (AFA) 2018) Sejam a e b números positivos tais que o a) c) e)
9 9 4
1 0 0 −1
 0 1  vale 24. π 32π
determinante da matriz 2 a b) d)
 1 −1 b 1 18 9
 
0 0 0 1
13. (EN 2015) Uma função y = f(x) é definida pelo determinante da
 b 2
Dessa forma o determinante da matriz   é igual a  x 2 x − 1 x −2 
 3 a   3 
x 1− x 
matriz A = 
x x
em cada x ∈  tal que A é invertível.
a) 0 b) 6 c) –6 d) 6
1 0 0 0 
 x 1 0 −1 
 1 cos x sen x É correto afirmar que o conjunto imagem de f é igual a
08. (EPCAR (AFA) 2017) Seja a matriz A =  cos x 1 0 . a) (– ∞,4] d) (– ∞,4)
 
 sen x 2 1  b)  – {0,4} e) [4, + ∞)
Considere a função f:  →  definida por f(x) = det A. c) (– ∞,4] – {0}
1
Sobre a função g:  →  definida por g(x) = 1 − ⋅ | f(x) |, em que |f(x)|
é o módulo de f(x), é correto afirmar que 2 14. (ITA 2015) Considere a matriz M = (mij)2×2 tal que mij = j – i + 1,
i, j = 1,2.
a) possui período π.
 n 1 0
b) seu conjunto imagem é  − 1 , 0 .
 2 
Sabendo-se que det 
 k =1

Mk − n   =
1 1
252, então o valor de n é

c) é par. igual a
 π π a) 4. c) 6. e) 8.
d) é crescente no intervalo  − ,  .
 4 4 b) 5. d) 7.

PROMILITARES.COM.BR 145
DETERMINANTES E SISTEMAS LINEARES

15. (Ime 2015) Dada a matriz A, a soma do módulo dos valores de x I. |det(A)| = 1.
que tornam o determinante da matriz A nulo é: II. AT = A–1.
1 2x 00  III. A + A–1 é uma matriz diagonal.
 2 
x 1 x −1 2  É(são) sempre VERDADEIRA(S)
A=
1 x+4 0 0  a) apenas I. c) apenas I e II. e) todas.
 
 x −1 1 x − 2 b) apenas III. d) apenas I e III.
a) 7 c) 9 e) 11
03. (IME 2018) Sejam x1, x2, x3 e x4 os quatro primeiros termos de
b) 8 d) 10 uma P.A. com x1 = x e razão r, com x, r ∈ . O determinante de
 x1 x1 x1 x1 
16. (IME 2016) Define-se A como a matriz 2016 x 2016, cujos x x
 1 2 x 2 x 2  é
elementos satisfazem à igualdade:
 x1 x 2 x3 x3 
 i + j − 2 para i, j {1, 2, ..., 2016}.  
ai,j =  ,  x1 x 2 x3 x 4 
 j − 1
a) 0 c) x4 ⋅ r3 e) x ⋅ r3
Calcule o determinante de A
b) x4 ⋅ r d) x ⋅ r4
17. (IME 2015) Sejam S = a + b + c e P = a ⋅ b ⋅ c. Calcule o determinante
abaixo unicamente em função de S e P. 04. (ITA 2018) Uma progressão aritmética (a1, a2,...,an) satisfaz a
propriedade: para cada n ∈ , a soma da progressão é igual a 2n2 + 5n.
a2 + (b + c)2 2b2 (a + b)2 + c2
 a1 a2 a3 
2a2 (a + c)2 + b2 (a + b)2 + c2 Nessas condições, o determinante da matriz  a
 4 a5 a6  é
2
a b2 (a + b)2 a7 + 2 a8 a9 
a) –96. c) 63. e) 115.
18. (UNB 2012) Dada uma matriz quadrada A, define-se o traço de b) –85. d) 99.
A, simbolizado por tr(A), como a soma dos elementos de sua diagonal
principal. A partir dessas informações e considerando as matrizes 05. (ITA 1987) Quaisquer que sejam os números reais a, b e c, o
 0,7 0,2  2 −1 determinante da matriz
=P  = , Q = e R 100Q−1 PQ, determine o valor do
 0,3 0,8  3 1  1 1 1 1 
1 1+ a 1 1 
quociente det(R) , em que det(R) é o determinante da matriz R.  
tr(R) 1 1 1+ b 1 
Despreze, caso exista, a parte fracionária do resultado final obtido,  
1 1 1 1+ c 
após ter efetuado todos os cálculos solicitados.
é dado por
19. (IME 2012) Calcule as raízes de f(x) em função de a,b e c, sendo a) ab + ac + bc c) zero e) 1
x a b c b) abc d) abc + 1
a x c b
a,b,c e x ∈  (real) e f ( x ) = .
b c x a 06. (ITA 1990) Sejam A, B e C matrizes quadradas n × n tais que A e B
c b a x são inversíveis e ABCA = At, onde At é a transposta da matriz A. Então
podemos afirmar que:
20. (ITA 2012) Considere a matriz quadrada A em que os termos Nota: det X denota o determinante da matriz quadrada X.
da diagonal principal são 1,1 + x1,1 + x2,..., 1 + xn e todos os outros
termos são iguais a 1. Sabe-se que (x1, x2, ..., xn)é uma progressão a) C é inversível e detC = det(AB)–1
1 b) C não é inversível pois det C = 0
geométrica cujo primeiro termo é e a razão é 4. Determine a ordem
2 c) C é inversível e det C = det B
da matriz A para que o seu determinante seja igual a 256.
d) C é inversível e detC = (detA)² ⋅ detB

e) C é inversível e det C = det A


EXERCÍCIOS DE det B

COMBATE 07. (ITA 1992) Considere a equação


 2 2 2 
 
det  G(x) 2x F(x)  = 0
01. (IME 2019) Calcule o valor do determinante: [G(x)]2
 4x 2 [F(x)]2 
4 2 1
4 3
x2 − 1
log 81 log 900 log 300 onde F(x) = x + x − x + 1 e G(x) = , com x ∈ R, x ≠0.
2
x x
(log 9)2 2 + 4 log 3 + 2 (log 3)2 (log 3 + 2)2
Sobre as raízes reais dessa equação, temos:
a) 1 c) 4 e) 16 a) Duas delas são negativas.
b) 2 d) 8 b) Uma delas é um número irracional.
c) Uma delas é um número par.
02. (ITA 2019) Considere as seguintes afirmações a respeito de
matrizes A de ordem n × n inversíveis, tais que os seus elementos e os d) Uma delas é positiva e outra negativa.
de sua inversa sejam todos números inteiros: e) n.d.a.

146 PROMILITARES.COM.BR
DETERMINANTES E SISTEMAS LINEARES

08. (ITA 1993) Sabendo-se que a soma das raízes da equação 15. (IME 2000) Calcule o determinante:
1 −1 0 2 1 1 1 1 1 1 1
x 0 x 0 1 3 1 1 1 1 1
=0
0 b x x 1 1 5 1 1 1 1
b x 2 b D= 1 1 1 7 1 1 1
1 1 1 1 9 1 1
é −8/3 e que S é o conjunto destas raízes, podemos afirmar que:
1 1 1 1 1 11 1
a) S ⊂ [−17, −1] d) S ⊂ [−10, 0]
1 1 1 1 1 1 13
b) S ⊂ [1, 5] e) S ⊂ [0, 3]
c) S ⊂ [−1, 3]
16. (ITA 2003) Sejam a, b, c e d números reais não-nulos. Exprima o
09. (ITA 1998) Sejam A e B matrizes reais quadradas de ordem 2 valor do determinante da matriz
que satisfazem a seguinte propriedade: existe uma matriz inversível bcd 1 a a2 
tal que:  
 acd 1 b b2 
A = M–1BM  
abd 1 c c2 
Então:
 abc 1 d d2 
a) det(–At) = det B 
b) det A = −det B na forma de um produto de números reais.
c) det (2A) = 2det B
17. (IME 2017) Classifique o sistema abaixo como determinado,
d) Se det B ≠ 0, então det (−AB) < 0 possível indeterminado e impossível de acordo com os valores reais
e) det (A −Ι) = −det (Ι −B) de m.

10. (ITA 87/88) Seja A uma matriz real que possui inversa. Seja n um (m − 2)x + 2y − z = m + 1
número inteiro positivo e An o produto da matriz A por ela mesma n  2
2x + my + 2z = m + 2
vezes. Das afirmações abaixo a verdadeira é:  3
a) An possui inversa, qualquer que seja o valor de n. 2mx + 2(m + 1)y + (m + 1)z = m + 3
b) An possui inversa apenas quando n = 1 ou n = 2.
c) An possui inversa e seu determinante independe de n. 18. (ITA 2014) Considere o sistema linear nas incógnitas x, y e z
d) An possui inversa para valor algum de n, n > 1. x + y + 2z = 0

e) Dependendo da matriz A, a matriz An poderá ou não ter inversa. − x + ( sen=
θ ) y + 4z 0, θ ∈ [0,2π] .

11. (IME 92/93) Determine o valor de x para que:  2x + (1 − cos 2θ ) y + 16z =
0

x 2 4 6 a) Determine θ tal que o sistema tenha infinitas soluções.


x x+2 0 10 b) Para θ encontrado em (a), determine o conjunto-solução do
=0 sistema.
x2 0 4x 4
x 4 10 x + 2 19. Verificar a identidade seguinte, aplicando as propriedades dos
determinantes
12. (IME 88/89) Calcule o determinante da matriz cos 2a cos2 a sen2 a
 a2 (a + 1)2 (a + 2)2 (a + 3)2  cos 2b cos2 b sen2 b = 0.
 2 
2 2
b (b + 1) (b + 2) (b + 3)  .
2 cos 2c cos2 c sen2 c
 2 2 2 2
 c (c + 1) (c + 2) (c + 3) 
d2 (d + 1)2 (d + 2)2 (d + 3)2  1+ a1 1 1  1
 
1 1+ a2 1  1
13. (ITA 88/89) Sabendo-se que x e y são reais, tais que x ÷ y = 3π/4, 20. Calcule o determinante: 1 1 1+ a3  1
 2tan x 1+ tan x     
verifique se a matriz  é ou não inversível
1+ tan y tan y  1 1 1  1+ an

14. (IME 82/83) Seja um determinante definido por ∆1 = |1| e


GABARITO
1 1  1
1 1 1
−1 2 0 0  0 0 EXERCÍCIOS DE TREINAMENTO
0 −1 2 0  0 0 01. A
∆n = Escolhendo os elementos da diagonal principal, os elementos da
0 0 −1 2  0 0
diagonal secundária ficam determinados univocamente. Logo, a
      
 4
resposta é= 4!
0 0 0 0  −1 2 = 6.
 2 2! ⋅ 2!
a) pede-se a fórmula de recorrência (isto é, a relação entre ∆n e ∆n-1).
02. D
b) Calcule a expressão de ∆n em função de n.
Desde que 2 + a = a + b + 1 = b + 4, temos a = 3 e b = 1 Logo, vem

PROMILITARES.COM.BR 147
DETERMINANTES E SISTEMAS LINEARES

1 3 1  1 −1 1 
det A = 1 1 3 =A  1 1 −1
2 1 2 2 1 1 
Chió −2 2 1 −1 1
=
−5 0 =
det A 1 1= −1 4
= 10. 2 1 1

( )
A −1
det = =
1 1
det A 4
03. E
Notemos que:
(In − A ) ⋅ (In − B) = In ⋅ In − In ⋅ B − A ⋅ In + AB 05. B

(In − A ) ⋅ (In − B) = In − B − A + AB Do enunciado,


 x1 + y1 x1 + y 2 x1 + y 3 x1 + y 4 x1 + y5 
Mas, A + B = AB, logo, x + y
 2 1 x2 + y2 x2 + y3 x2 + y 4 x 2 + y5 
(In − A ) ⋅ (In − B) = In − B − A + A + B A =  x 3 + y1 x3 + y2 x3 + y3 x3 + y 4 x 3 + y5 
 
(In − A ) ⋅ (In − B) = In  x 4 + y1 x 4 + y2 x 4 + y3 x4 + y4 x 4 + y5 
det ((In − A ) ⋅ (In − B) ) = det In  x5 + y1
 x5 + y 2 x5 + y 3 x5 + y 4 x5 + y5 
det (In − A ) ⋅ det (In − B) = 1 Vamos escrever a matriz A na forma escalonada.
Assim, Na matriz A, multiplicando a linha 1 por (–1) e somando às linhas 2,
3, 4 e 5, temos:
det (In − A ) ≠ 0 e det (In − B) ≠ 0
 x1 + y1 x1 + y 2 x1 + y 3 x1 + y 4 x1 + y5 
Portanto, In – B e In – A são inversíveis, o que garante que as afirmações x − x x 2 − x1 x 2 − x1 x 2 − x1 x 2 − x1 
[I] e [II] são verdadeiras.  2 1
B =  x 3 − x1 x 3 − x1 x 3 − x1 x 3 − x1 x 3 − x1 
Notemos que:  
 x 4 − x1 x 4 − x1 x 4 − x1 x 4 − x1 x 4 − x1
(In − B) ⋅ (In − A ) = In ⋅ In − In ⋅ A − B ⋅ In + BA  x5 − x1
 x5 − x1 x5 − x1 x5 − x1 x5 − x1 
(In − B) ⋅ (In − A ) = In − A − B + BA Na matriz B, multiplicando a coluna 1 por (–1) e somando às colunas
Como (In − A ) ⋅ (In − B) = (In − B) ⋅ (In − A ) 2, 3, 4 e 5, temos:
 x1 + y1 y 2 − y1 y 3 − y1 y 4 − y1 y5 − y1
Então, (In − B) ⋅ (In − A ) = 
In
 x 2 − x1 0 0 0 0 
Daí, 
C= 3 1
x − x 0 0 0 0 
 
In = In − A − B + BA x
 4 − x 1 0 0 0 0 
In = In − ( A + B) + BA  x5 − x1 0 0 0 0 

Mais uma vez, A + B = AB, logo, Note que é possível obter matrizes de ordem 3, 4 e 5, com determinante
In =In − AB + BA nulo, assim, a maior ordem possível de uma matriz com determinante
não nulo é 2, ou seja, o maior valor para a característica da matriz é 2.
AB = BA
Portanto, o maior valor possível de r é 2.
Dessa forma, a afirmação [III] é verdadeira.
Portanto, todas as afirmações são verdadeiras.
06. D
Do enunciado, temos:
04. D
1 x x2 x3 1 2 3 4
 a11 a12 a13  2
1 2 3 4 1 x x x3
A = a21 a22 a23  p ( x) = = −
−1 3 4 5 −1 3 4 5
a31 a32 a33 
−2 2 1 1 −2 2 1 1
( −1)
a11 =
1+1
=
1 1 2 3 4
( −1)
a12 =
1+ 2
=
−1 0 x − 2 x2 − 3 x3 − 4
p ( x) = −
( −1)
a13 =
1+ 3
=
1 0 5 7 9
a21 = 2 − 1 = 1 0 6 7 9

( −1)
a22 =
2+ 2
=
1 x − 2 x2 − 3 x3 − 4
p ( x ) = −1⋅ ( −1)
1+1
⋅ 5 7 9
( −1)
a23 =
2+ 3
=
−1
6 7 9
a31 = 3 − 1 = 2
a32 = 3 − 2 = 1  x − 2 x2 − 3 x3 − 4 x − 2 x2 − 3 x3 − 4 
 
( −1)
a33 =
3+ 3
=
1 p ( x ) = −1⋅  5 7 9 + 5 7 9 
 5 7 9 1 0 0 
Então,
 3+1 x − 3 x − 4 
2 3
p ( x ) = −1⋅  0 + 1⋅ ( −1) ⋅ 
 7 9 

( ( )
p ( x ) = −1⋅ 9 ⋅ x 2 − 3 − 7 ⋅ x 3 − 4 ( ))
p ( x) =
−7x 3 + 9x 2 + 1
148 PROMILITARES.COM.BR
6 7 9
 x − 2 x − 3 x − 4 x − 2 x2 − 3 x3 − 4 
2 3
⇒ A 2 + A = P −1 ⋅ D2 + D ⋅ P ( ) DETERMINANTES E SISTEMAS LINEARES

p ( x ) = −1⋅  5 7 9 + 5 7 9 
 (
det A += 2
A det P )
( ) ⋅ det (D −1 2
)
+ D ⋅ det (P ) ⇒
 5 7 9 1 0 0  ⇒ det ( A + =
A ) det (D + D)
2 2

 3+1 x − 3 x − 4 
2 3
 1 0 0  1 0 0  1 0 0
p ( x ) = −1⋅  0 + 1⋅ ( −1) ⋅
 7 9 
 D2 + D 0 2 0 ⋅ 0 2 0 + 0 2 0 ⇒
=
0 0 3 0 0 3 0 0 3
( (
p ( x ) = −1⋅ 9 ⋅ x 2 − 3 − 7 ⋅ x 3 − 4 ) ( )) 2 0 0 
p ( x) =
−7x + 9x + 1 3 2
⇒ D2 + D = 0 6 0 
 
0 0 12
O produto das raízes de p(x) é −1 = 1
( −7) 7 ( )
det A 2 + A = 2 ⋅ 6 ⋅ 12 = 144
Observação: é possível mostrar que p(x) admite somente uma raiz real,
portanto o produto das raízes de p(x) envolve as raízes complexas, o
que faz com que a questão não admita resposta correta, uma vez que 10. D
o enunciado diz que x é real.
det(A 2 − 2A + I) =16
Desconsiderando que x é real, a resposta correta é a que está na
A 2 − 2A + I = X → ( A − I) = X
2
alternativa [D].
det ( A − I) =±4
07. D  1 a −2
Tem-se que A − I= a − 2 1 1 
1 0 0 −1  2 −3 1 
a 0 3
2 a 0 1 8a − 12 = 4 → a = 2 
= 24 ⇔ −1 b 2= 24 
1 −1 b 1 det ( A − I) = 8a − 12 ⇒ ou  2 + 1= 3
0 0 1
0 0 0 1 8a − 12 =−4 → a =1
⇔ ab =
24.

Portanto, a resposta é 11. C


b 2
De acordo com a definição, é fácil ver que Vn = 
1 2n
= a⋅ b − 3⋅ 2  0 1  ,
para todo n
3 a inteiro positivo. Logo, temos
= ab − 6  1 2  1 4  1 6  1 4032
=
Y  + + ++ 
= 24 − 6  0 1  0 1  0 1 0 1 
= 2 6− 6  2016 2016 ⋅ 2017
= .
= 6.  0 2016 

A resposta é det Y = 2016 ⋅ 2016 – 0 ⋅ 2017 ⋅ 2016 = 2016 ⋅ 2016.


08. C
Calculando: 12. B
1 cos x sen x sen2x 1 sec2 x
31
1 cos2 x 0 =sen2x ⋅ cos2 x ⋅ 1 − sec2 x ⋅ cos2 x ⋅ 1 − 1 =−
det A = cos x 1 0 = 1+ 2 ⋅ sen x ⋅ cos x − cos2 x → 1 0 1
16

sen x 2 1 2
31 16  sen 2x  31 16
→ det A =
sen 2x ( sen x ⋅ cos x )2 − ( sec x ⋅ cos x )2 =− + →
 2 
−1=− +
16 16 16 16
2
 sen 2x  31 16 16 4 2 1
→ ( sen 2x ) =
2
Analisando as alternativas uma a uma:   = − + + → sen 2x = → sen 2x =
2  16 16 16 16 4 2
[A] FALSO. O período é igual a T = π . 2x= 30° → x= 15°=
π
2 12

[B] FALSO. O conjunto imagem é  2 ;1 .


1 B⋅h 1 6 ⋅ 2 ⋅ 3 π 6 ⋅ 2 π 4π π
V= → V= ⋅ ⋅ 3= → = → 2 = → =
3 3 4 12 4 12 72 18
[C] VERDADEIRO. A função g é par: g(–x) = g(x).
Observação: Seria possível uma segunda solução atendendo a
[D] FALSO. A função não é sempre crescente no intervalo mencionado: condição de x no primeiro quadrante, que seria x = 75º, porém não há
g( − π 4)
= 1 2, g(0)
= 1 e g( π 4)
= 1 2. x 2 xvalor
alternativa de resposta para esse − 1 de
x x.−2
x3 x x 1− x
f(x) =
09. A 13. C 1 0 0 0
Calculando: Sendo f : D ⊂  → , temos x 1 0 −1
A = P −1 ⋅ D ⋅ P ⇒ A 2 = P −1 ⋅ D ⋅ P ⋅ P −1 ⋅ D ⋅ P ⇒ x2 x −1 x −2 1 0 0 0
3
2
⇒ A = P ⋅D ⋅P −1 2
x 3
x x 1− x = − x x x 1− x
f(x) =
2 −1
A + A= P ⋅ D ⋅ P + P ⋅ D ⋅ P ⇒ 2 −1 1 0 0 0 x2 x −1 x −2
2
⇒ A + A = P ⋅ D + D ⋅P −1
( 2
) x 1 0 −1 x 1 0 −1
1 0Chió x0 x 1− x
0
(
det A +=
A det P2
)
( ) ⋅ det (D −1 2
)
+ D ⋅ det (P ) ⇒
x x 3
x 1− x=− x − 1 x −2
⇒ det ( A + =
A ) det (D + D)
2 2 = − 2
x x − 1 x −2 1 0 −1
 1 0 0  1 0 0  1 0 0 x 1 0 −1 1 0 −1
D2 + D 0 2 0 ⋅ 0 2 0 + 0 2 0 ⇒
= x x 1− x = x − 1 x −2
Chió
0 0 3 0 0 3 0 0 3 =− x − 1 x −2 x x 1− x
Chió x x − 3
2 0 0  1 0 −1 =
0 6 0  PROMILITARES.COM.BR 149
⇒ D2 + D = 1 0 −1 x 1
 
1 0 0 0
DETERMINANTES
x 3 E SISTEMAS LINEARES
x x 1− x
= − 2
x x − 1 x −2
x 1 0 −1
x x 1− x  i + j − 2
=
16. Considere An = {aij}, onde aij  ∀ 1 ≤ i, j ≤ n. Subtraindo-
Chió
=− x − 1 x −2  j − 1
1 0 −1 se a linha (n – 1) da linha n, a linha (n – 2) da linha (n – 1) e assim
sucessivamente, pode-se escrever, utilizando a Relação de Stifel:
1 0 −1
 m  m − 1  m − 1
=
x − 1 x −2  k  −  k  =
 x − 1
x x 1− x
1 1 1  1
Chió x x−3
=  0 1  n − 2
x 1 0   1   n − 2
 0
1 x−3
=x  1  2  n − 1
1 1 =det An 0=
 0  1   n − 2
= 4 − (x − 2)2.
   
Como A é invertível, segue que D =  – {4}. Logo, o conjunto imagem  n − 2  n − 1  2(n − 2)
de f é (– ∞,4] – {0}. 0   
 0   0   n − 2 
 0 1  n − 2
14. C  0 1   n − 2

Escrevendo a matriz M = 
1 2
 0 1 ,
temos:  1  2  n − 1
 0  1   n − 2
 1 4  = = det An −1
M2 = 
 0 1   
 1 6  n − 2  n − 1  2(n − 2)
M3 =   0   0    n − 2 
 0 1
 1 8 Assim, pode-se concluir que o det An = det A1 - |1| = 1. Logo,
M4 = 
 0 1 det A2016 = det A1 = 1.

 1 2 ⋅ n 17. Fazendo multiplicações de linhas e colunas e desenvolvendo os
Mn = 
 0 1  termos, tem-se:
a2 + (b + c)2 2b2 (a + b)2 + c2
Daí podemos escrever que: 2 2 2
=D 2a (a + c) + b (a + b)2 + c2 →
2 + 4 + 6 + 9 +  + 2n = n2 + n 2 2
a b (a + b)2
 n n2 + n
M + M2 + M3 + M4 + ... + Mn =
  (b + c)2 − a2 b2 − (a + c)2 0
0 n 
→ a2 (a + c)2 c2
e a2 b2 (a + b)2
 n
1 0 0 n2 + n (b + c − a) ⋅ (b + c + a) (b − a − c) ⋅ (b + c + a) 0
det 

∑ M − n 1
k =1
k
 =
1
252 ⇒
n 0
=
252 ⇒ =D a2 (a + c)2 c2

3
0 b2 − (a + c)2 (a + b)2 − c2
⇒ n= + n2 252 ⇒ 63 + 62 ⇒ n =6
(a + b + c) ⋅ (b + c − a) (a + b + c) ⋅ (b − a − c) 0
=D a2 (a + c)2 c2
15. A 0 (a + b + c) ⋅ (b − c − a) (a + b + c) ⋅ (a + b − c)
Pela regra de Chió: (b + c − a) (b − a − c) 0
1 − 2x ⋅ x 2 x −1 2 1 − 2x 3 x −1 2 D = (a + b + c)2 ⋅ a2 (a + c)2 c2 =
det A = (x + 4) − 2x 0 0 = 4−x 0 0 0 (b − c − a) (a + b − c)
−1 − 2x ⋅ x 1 x − 2 −1 − 2x 2 1 x −2 (b + c − a) −2c 0
= (a + b + c)2 ⋅ a2 2ac c2
Aplicando o Teorema de Laplace, utilizando a segunda linha da matriz, 0 −2a (a + b − c)
tem-se:
x −1 2 Substituindo os valores de S e P e calculando o determinante, tem-se:
det A = (4 − x) ⋅ Cof21 = (4 − x) ⋅ ( −1)2+1 ⋅ =
1 x−2 S − 2a −2c 0
S2 ⋅ a2
D= 2ac c2 =
= (x − 4) ⋅ [ (x − 1) ⋅ (x − 2) − 2] = (x − 4) ⋅ (x − 3x)
2
0 −2a S − 2c
det A = (x − 4) ⋅ (x 2 − 3x)
= S2 ⋅ (S − 2a) ⋅ (2ac) ⋅ (S − 2c) − ( −2a) ⋅ (c2 ) ⋅ (S − 2a) − (a2 ) ⋅ ( −2c) ⋅ (S − 2c)
Para que det A = 0, tem-se:
D = S2 ⋅ 2ac ⋅ [ (S − 2a) ⋅ (S − 2c) + c ⋅ (S − 2a) + a ⋅ (S − 2c)]
0 = (x − 4) ⋅ (x 2 − 3x) D = S2 ⋅ 2ac ⋅ S2 − 2Sc − 2Sa + 4ac + Sc − 2ac + Sa − 2ac  =
=x1 4; = x 2 0; = x3 3
= S2 ⋅ 2ac ⋅ S2 − 2Sc − 2Sa + Sc + Sa
Logo, a soma do módulo dos valores de x que tornam o determinante D =S3 ⋅ 2ac ⋅ [S − c − a] =S3 ⋅ 2ac ⋅ [b] =S3 ⋅ 2acb =S3 ⋅ 2P → D =2PS3
da matriz A nulo é 7 (|4| + 0 + |3| = 7).

150 PROMILITARES.COM.BR
x −b−c +b a−c −b+c
= (x + a + b + c)(x − a − b + c)
DETERMINANTES
a−b x −Ec SISTEMAS LINEARES
x −c a−b
= (x + a + b + c)(x − a − b + c)
a−b x −c
18. 033. = (x + a + b + c)(x − a − b + c)[(x − c)2 − (a − b)2 ]
1 −3
A matriz cofatora de Q é a matriz cof Q =  Desse modo, = (x + a + b + c)(x − a − b + c)(x − a + b − c)(x + a − b − c).
1 2 .
Portanto, os zeros de f são –a – b – c, a + b – c, a – b + c e –a + b + c.
a adjunta de Q = é Q (cof  1 1 e, portanto, como
= Q)t 
 −3 2
detQ = 2 ⋅ 1 – 3 ⋅ (–1) = 5, vem 20. A matriz será da seguinte forma:
 1 1 1 1 1 1 1 1 1 . . .
−1 1 1  1 1  5 5   1 
Q = ⋅Q = ⋅ = . 1 1+ 
det Q 5  −3 2  3 2 2
1 1 1 1 1
−  
 5 5 1 1 1+ 2 1 1 1 1 
 
Assim, obtemos  1 1 1 1 + 8 1 1 1 
1 1 1 1 1+ 32 1 1 
R= 100 ⋅ Q−1 ⋅ P ⋅ Q  
1 1 1 1 1 1+ 128 1 
 1 1 1 1 
 5 5   0,7 0,2  2 −1 1 1 1 1 1+ 512
=100 ⋅   
 ⋅  ⋅  . 
 − 3 2  0,3 0,8  3 1 . 
 5 5  
 . 
 2 2  7 2  2 −1
=  ⋅ ⋅
 −6 4  3 8  3 1 Multiplicando a primeira linha por -1 e somando com as demais linhas,
 20 20  2 −1 obteremos a seguinte matriz:
=  ⋅
 −30 20  3 1 1 0 0 0 0 0 0 . . .
100 0   1 
= . 0 0 0 0 0 0 
 0 50  2 
0 0 2 0 0 0 0 
Daí, det(R) = 100 ⋅ 50 – 0 ⋅ 0 = 5000 e tr(R) = 100 + 50 = 150.  
0 0 0 8 0 0 0 
det(R) 5000 0 0 0 0 32 0 0 
Por conseguinte, = ≅ 33.  
tr(R) 150
0 0 0 0 0 128 0 
0 0 0 0 0 0 512 
19. Temos  
x a b c . 
. 
a x c b  
f(x) =  . 
b c x a
c b a x Temos uma matriz diagonal que o determinante é obtido multiplicando
x +a+b+c a b c os elementos da diagonal principal. Para que p determinante seja 256
deveremos efetuar o produto de 5 elementos da diagonal principal
a+ x +c +b x c b
=  1
b+c+x+a c x a 1⋅   ⋅ 2 ⋅ 8 ⋅ 32 =256 . Portanto, a matriz A deverá ser de ordem 5.
 2
c +b+a+ x b a x
1 a b c EXERCÍCIOS DE COMBATE
1 x c b 01. E 03. E 05. B 07. E 09. A
= (x + a + b + c)
1 c x a 02. A 04. A 06. A 08. D 10. A
1 b a x 11. x = -2 ou x = 4/7
x −a c −b b−c
=(x + a + b + c) c − a x − b a − c 12. 0
b−a a−b x −c
x −a+c −b c −b b−c 13. a matriz não é inversível
= (x + a + b + c) c − a + x − b x − b a − c
b−a+a−b a−b x −c 14. a) ∆
= 2n −1 + ∆n −1
n
x −a+c −b c −b b−c
b) ∆n = 2n − 1 ;
= (x + a + b + c) c − a + x − b x − b a − c
0 a−b x −c
15. Subtraindo a 1ª linha de todas as outras linhas.
1 c −b b−c
1 1 1 1 1 1 1 1 1 1 1 1 1 1
= (x + a + b + c)(x − a − b + c) 1 x − b a − c
1 3 1 1 1 1 1 0 2 0 0 0 0 0
0 a−b x −c
1 1 5 1 1 1 1 0 0 4 0 0 0 0
x −b−c +b a−c −b+c
= (x + a + b + c)(x − a − b + c) D= 1 1 1 7 1 1 1 = 0 0 0 6 0 0 0
a−b x −c
1 1 1 1 9 1 1 0 0 0 0 8 0 0
x −c a−b 0 0 0 0 0 10 0
= (x + a + b + c)(x − a − b + c) 1 1 1 1 1 11 1
a−b x −c 0 0 0 0 0 0 12
1 1 1 1 1 1 13
= (x + a + b + c)(x − a − b + c)[(x − c)2 − (a − b)2 ]
= (x + a + b + c)(x − a − b + c)(x − a + b − c)(x + a − b − c).

PROMILITARES.COM.BR 151
DETERMINANTES E SISTEMAS LINEARES

O último determinante é o determinante de uma matriz triangular Daí, lembrando que cos 2θ = 1 − 2sen2 θ, obtemos
superior e pode ser obtido pelo produto dos elementos da diagonal
principal. sen2 θ − sen θ − 2= 0 ⇔ (sen θ − 2)(sen θ + 1)= 0.
⇒ D = 1 ⋅ 2 ⋅ 4 ⋅ 6 ⋅ 8 ⋅ 10 ⋅ 12 = 4680 Assim, convém apenas sen θ = −1. Sendo θ ∈ [0, 2π], concluímos que

θ = rad.
16. Inicialmente, vamos multiplicar a 1ª linha por a, a 2ª por b, a 3ª 2
por c e a 4ª por d e, conseqüentemente, dividir todo o determinante 3π
por abcd. b) Para θ = rad temos
2
bcd 1 a a2 abcd a a2 a3  1 1 2  1 1 2 
acd 1 b b2 1 abcd b b2 b3    
= ⋅ =  −1 −1 4    0 0 6 
abd 1 c c2 abcd abcd c c2 c3  2 2 16   0 0 12 
   
abc 1 d d2 abcd d d2 d3 L'2 → 1⋅ L1 + L2
1 a a2 a3 1 1 1 1 L'3 → ( −2) ⋅ L1 + L3
2 3
abcd 1 b b b a b c d  1 1 2
= ⋅ = 2 =  
abcd 1 c c2 c3 a b2 c2 d2  0 0 6
1 d d2 d3 a3 b3 c3 d3 0 0 0
 
= (b −a)(c −a)(d −a)(c −b)(d −b)(d −c) L''3 → ( −2) ⋅ L'2 + L'3.
O último determinante calculado é um determinante de Vandermonde.
x + y + z =0
O sistema equivalente é  .
6z = 0
17. Calculando:
Portanto, temos z = 0, x = -y e o conjunto solução do sistema é
(m − 2) 2 −1 S = {(-α, α, 0); α ∈ }.
det A = 2 m 2
2m 2(m + 1) (m + 1)
19. cos2x = cos²x – sen²x
det A = (m − 2) ⋅ m ⋅ (m + 1) + 2 ⋅ 2 ⋅ 2m + 2 ⋅ 2(m + 1) ⋅ ( −1) − [2m ⋅ m ⋅ ( −1)] −
c1 = c2 – c3
[2 ⋅ 2 ⋅ (m + 1)] − [2 ⋅ 2(m + 1) ⋅ (m − 2)] n n
det A = m ⋅ (m − 2) ⋅ (m + 1) + 8m − 4(m + 1) + 2m2 − 4(m + 1) − 4(m + 1) ⋅ (m − 2)
det A = m3 − m2 − 2m + 8m − 4m − 4 + 2m2 − 4m − 4 − 4m2 + 4m + 8
20.

k =1
ak ⋅ (1 + ∑ a1 )
k =1 k

det A = m − 3m + 2m =m ⋅ (m − 1) ⋅ (m − 2)
3 2

Caso 1 → m ≠ 0 ou m ≠ 1 ou m ≠ 2 → det A ≠ 0 → sistema possível ANOTAÇÕES


e determinado
Caso 2 → m =0 → det A =0 → sistema possível e indeterminado, pois:
−2x + 2y − z =
1

2x + 2z= 2 → Dx= 0; Dy= 0; Dz= 0
2y + z =3

Caso 3 → m =1 → det A =0 → sistema impossível, pois:
− x + 2y − z =2
 (Linha1× 2) + Linha2 5y =7
2x + y + 2z = 3 → Resolvendo : →
2x + 4y + 2z = (Linha1× 2) + Linha3 8y =
8
 4

Caso 4 → m =2 → det A =0 → sistema impossível, pois:


2y − z = 3
 Linha1 2y − z = 3
2x + 2y + 2z = 6 → Resolvendo : →
4x + 6y + 3z = (Linha2 × −2) + Linha3 2y − z = −1
 11

Logo,
m ∈  − {0, 1, 2} → sistema possível e determinado
m= 0 → sistema possível e indeterminado
m ∈ {1, 2} → sistema impossível

18.
a) Como o sistema é homogêneo, basta que o determinante da
matriz dos coeficientes seja nulo para que o sistema seja possível e
indeterminado. Logo, vem
1 1 2
−1 sen θ 4 = 0 ⇔ cos 2θ + 2sen θ + 3= 0.
2 1 − cos 2θ 16

152 PROMILITARES.COM.BR
TRIGONOMETRIA I

TRIGONOMETRIA NO TRIÂNGULO Da mesma forma:


c b c b
RETÂNGULO ABC senCˆ
= cos Cˆ
;= = ; tgCˆ cotgCˆ
e= .
a a b c
Seja o triângulo retângulo em A, conforme a figura a seguir:
Comparando os resultados, concluímos que
=senBˆ cos
= ˆ cos Bˆ senC
C; = ˆ e tgBˆ cotgC.ˆ

Como B̂ + Cˆ =900, concluímos que:


cosθ = sen(90º – θ)
senθ = cos(90º – θ)
tgθ = cotg(90º – θ)

RELAÇÕES FUNDAMENTAIS
Definem-se: Pelo teorema de Pitágoras, sabemos que a² = b² + c².
cateto oposto b Dividindo-se ambos os membros da equação por a², temos:
Seno: senBˆ
= =
hipotenusa a b2 c2
+ =1 ⇔ sen2 Bˆ + cos2 Bˆ =1.
cateto adjacente c a2 a2
Cosseno: cos Bˆ
= =
hipotenusa a Assim, para um ângulo θ, podemos escrever:
ˆ cateto oposto b
Tangente: tgB =
= sen²θ + cos²θ = 1
cateto adjacente c
tgθ = cotg(90º – θ)
Lembre-se do macete SOHCAHTOA
(relação fundamental da trigonometria)
Podemos definir também as funções:
Dividindo a relação fundamental por cos²θ, temos:
ˆ c ateto adjacente= c= 1 sen2 θ 1 2
Cotangente: cotgB= =
+1 ⇔ 1+ tg= θ sec2 θ .
cateto oposto b tgBˆ cos2 θ cos2 θ
1 Dividindo a relação fundamental por sen²θ, temos:
Secante: sec Bˆ =
cos Bˆ cos2 θ 1 2
1+ = ⇔ 1+ cotg
= θ cossec2 θ .
1 sen2 θ sen2 θ
Cossecante: cossec Bˆ =
senBˆ 1 + tg²θ = sec²θ
1 + cotg²θ = cossec²θ

ÂNGULO SOMA
Na figura abaixo considere AD = 1.

 CD CD 
senb = e AD =⇒1 senb = ⇔ CD = senb (1)
 AD 1 
∆ACD :  
AC
 cos b = e AD =⇒ AC
1 cos b = ⇔ AC =cos b (2)
 AD 1 

PROMILITARES.COM.BR 153
TRIGONOMETRIA I

 BC BC 
sena= e AC= cos b(2) ⇒ sena= ⇔ BC= sena.cos b(3) 
 AC cos b
∆ABC :  
AB AB
 cos a= e AC= cos b(2) ⇒ cos a= ⇔ AB= cos a.cos b(2)
 AC cos b 

 DF DF 
sen ( a + b) = e AD =1 ⇒ sen ( a + b) = ⇔ DF =sen ( a + b) (5)
 AD 1 
∆AFD :  
 cos ( a + b) = AF AF
e AD =1 ⇒ cos ( a + b) = ⇔ AF =cos ( a + b) (6)
 AD 1 

ˆ = 90° - a
 AFG : AGF
 
ˆ =AGF
 CGD ˆ ˆ = 90° - ( 90° - a ) =a
ˆ =90° - AGF
 ⇒ CDG
 ˆ ˆ 
= 90° - CGD 
 CDG

 EC EC 
sena= e CD= senb (1) ⇒ sena= ⇔ EC= sena ⋅ senb (7)
 CD senb 
∆DCE :  
DE DE
 cos a= e CD= senb (1) ⇒ cos a= ⇔ DE= senb ⋅ cos a (8)
 CD senb 
 DF =EF + DE e EF =BC ⇒ DF =BC + DE
 DF sen ( a + b) (5)
= 
  ⇒ sen ( a + b)= sena ⋅ cos b + senb ⋅ cos a
=BC sena ⋅ cos b ( 3) 
 
 DE senb ⋅ cos a ( 8)
= 
 AF = AB − FB e FB = EC ⇒ AF = AB − EC
 AF cos ( a + b) (6)
= 
  ⇒ cos ( a + b)= cos a ⋅ cos b − sena ⋅ senb
=AB cos a ⋅ cos b ( 4 ) 
 
 EC sena ⋅ senb ( 7)
= 
sen ( a + b) sena ⋅ cos b + senb ⋅ cos a
( a + b)
tg= = ⇔
cos ( a + b) cos a ⋅ cos a − sena ⋅ senb

sena ⋅ cos b senb ⋅ cos a


+
cos a ⋅ cos b cos a ⋅ cos b tga + tgb
tg ( a + b) = ⇒ tg ( a + b) =
cos a ⋅ cos b sena ⋅ senb 1 − tga ⋅ tgb

cos a ⋅ cos b cos a ⋅ cos b

154 PROMILITARES.COM.BR
TRIGONOMETRIA I

ÂNGULO DIFERENÇA
Na figura abaixo considere AD = 1.

 CD CD 
senb = e AD =⇒1 senb = ⇔ CD = senb (1)
 AD 1 
ACD :  
AC AC
 cos b = e AD =⇒
1 cos b = ⇔ AC =cos b ( 2)
 AD 1 

 BC BC 
sena = e AC = cos b (2) ⇒ sena = ⇔ BC = sena ⋅ cos b (3)
 AC cos b 
∆ABC :  
 cos a = AB AB
e AC = cos b (2) ⇒ cos a = ⇔ AB = cos a ⋅ cos b (4)
 AC cos b 

 DF DF 
sen ( a − b) = e AD =1 ⇒ sen ( a − b) = ⇔ DF =sen ( a − b) (5)
 AD 1 
∆AFD :  
 cos ( a − b) = AF AF
e AD =1 ⇒ cos ( a − b) = ⇔ AF =cos ( a − b) (6)
 AD 1 

ˆ = 90° - a
 ABC : ACB
  ⇒ DCE ˆ =90° - ( 90° - a) =a
ˆ =90° - ACB
ˆ
 DCE ˆ
= 90° - ACB 
 ED ED 
sena= e CD= senb (1) ⇒ sena= ⇔ ED= sena ⋅ senb (7)
 CD senb 
∆DCE :  
CE CE
 cos a= e CD= senb (1) ⇒ cos a= ⇔ CE = senb ⋅ cos a (8)
 CD senb 

PROMILITARES.COM.BR 155
TRIGONOMETRIA I

 EB = BC − CE e EB = DF ⇒ DF = BC − CE
 DF sen ( a − b) (5)
= 
  ⇒ sen ( a − b)= sena ⋅ cos b − senb ⋅ cos a
=BC sena ⋅ cos b ( 3) 
 
 CE senb ⋅ cos a ( 8)
= 
 AF = AB + BF e BF = ED ⇒ AF = AB + ED
 AF cos ( a − b) (6)
= 
  ⇒ cos ( a − b)= cos a ⋅ cos b + sena ⋅ senb
=AB cos a ⋅ cos b ( 4 ) 
 
 ED sena ⋅ senb ( 7)
= 
sen ( a − b) sena ⋅ cos b − senb ⋅ cos a
( a − b)
tg= = ⇔
cos ( a − b) cos a ⋅ cos a + sena ⋅ senb
sena ⋅ cos b senb ⋅ cos a

cos a ⋅ cos b cos a ⋅ cos b tga − tgb
tg ( a − b) = ⇒ tg ( a − b) =
cos a ⋅ cos b sena ⋅ senb 1+ tga ⋅ tgb
+
cos a ⋅ cos b cos a ⋅ cos b

ARCO DOBRO
sen ( a + a) =sena ⋅ cos a + cos a ⋅ sena ⇔ sen2a =2 ⋅ sena ⋅ cos a

cos ( a + a) = cos a ⋅ cos a − sena ⋅ sena ⇔ cos 2a = cos2 a − sen2 a ⇔

 cos =

(
2a cos2 a − 1 − cos2 a ⇔ cos = )
2a 2cos2 a − 1 

 cos 2a =
 1 −(sen2
a − sen)
2
a ⇔ cos 2a =
1 − 2sen2 
a 
2 ⋅ sena ⋅ cos a
sen2a 2 ⋅ sena ⋅ cos a cos a ⋅ cos a 2tga
=
tg2a = = ⇔ tg2a =2
cos 2a cos2 a − sen2 a cos2 a sen2 a 1 − tg a
− 2
cos a cos a
2

ÂNGULOS NOTÁVEIS
ÂNGULOS DE 30° E 60°
Seja o triângulo equilátero ABC, conforme a figura a seguir:

No ∆ABM, temos:
x 3
 AM 2 3
= = =
sen60
AB x 2
x
 BM 2 1
cos 60= = =
AB x 2
 sen60 3 2
=
tg60 = = 3
cos 60 12

156 PROMILITARES.COM.BR
TRIGONOMETRIA I

 1 ÂNGULOS DE 22° 30’ E 67° 30’


=
sen30 =
cos 60
2 2 2 2
45° 1 − cos 45° 1− − 2− 2
  3 2 2 2
=
cos =
30 sen60 =
sen = = = ⇔
2 2 2 2 2 4
 sen30 12 3 2− 2
=
tg30 = = sen22°30' =
cos 30 3 2 3 2
2 2 2
45° 1+ cos 45° 1+ + 2+ 2
ÂNGULO DE 45° =
cos = = 2 2 2
= ⇔
Seja o quadrado ABCD, conforme a figura a seguir: 2 2 2 2 4
2+ 2
cos 22°30' =
2

2− 2
45° sen 45° 2 2 2− 2 2 2− 2
=
tg = = ⋅ = ⇔
2 cos 45° 2 2+ 2 2 2+ 2 2+ 2
2

tg22°30'
=
2− 2

2− 2 2− 2
= =
2 ( 2 −1 )⇔
2+ 2 2− 2 4−2 2
tg22°30' = 2 −1

2+ 2
) cos 22°30'=
= sen ( 90° − 22°30'=
sen67°30'
2
2− 2
) sen22°30'=
= cos ( 90° − 22°30'=
cos 67°30'
2
1
No ∆ABC, temos: tg67°30'= tg ( 90° − 22°30')= ctg22°30'= ⇔
tg22°30'
 ˆ BC x 2
=
sen 45 =
senBAC = = 1 1 2 +1 2 +1
AC x 2 2 tg67°30'
= = ⋅ = ⇔
2 −1 2 −1 2 +1 2 −1
) sen 45= 2

cos 45= sen ( 90 − 45= tg67°30' = 2 +1
2
sen 45 22
45
tg= = = 1 22°30’ 67°30’
cos 45 22
2− 2 2+ 2
SENO
2 2
ÂNGULOS DE 15° E 75°
=
sen15 45°) sen60° ⋅ cos 45° − sen 45° ⋅ cos 60° ⇔
° sen ( 60° − = 2− 2
CO-SENO 2+ 2
3 2 2 1 6− 2 2 2
sen15=° ⋅ − ⋅=
2 2 2 2 4
=
cos15 45°) cos 60° ⋅ cos 45° + sen60° ⋅ sen 45° ⇔
° cos ( 60° − = TANGENTE 2 −1 2 +1
1 2 3 2 6+ 2
cos15°= ⋅ + ⋅ =
2 2 2 2 4
ÂNGULOS DE 18° E 72°
6− 2
sen15° 6 − 2 6 − 2 6 − 2⋅ 6 ⋅ 2 + 2 Seja um triângulo ABC isósceles de base BC e seja um ponto P
=
tg15° = 4
= ⋅ = ⇔
cos15° 6−2 pertencente ao lado AC tal que AP = PB = BC.
6+ 2 6+ 2 6− 2
4
8 − 2⋅ 2 ⋅ 2 ⋅ 3 8 − 4 3
tg15° = = = 2− 3
4 4
6+ 2
=
sen75 ° sen ( 90° − 15
= °) cos15
= °
4
6− 2
75° cos ( 90° − 15
cos= = °) sen15
= °
4
1 1 2+ 3
=
tg75 ° tg ( 90° − 15
= °) c tg15
= ° = ⋅ ⇔
tg15° 2 − 3 2 + 3
2+ 3
tg75° = = 2+ 3
4−3

PROMILITARES.COM.BR 157
TRIGONOMETRIA I

AP = ˆ =
PB ⇔APB é isósceles de base AB ⇔ PAB ˆ =
PBA α 1 1 1 5 −1 5 −1 5 −1
=
sen18 ° = = ⋅ = =
ˆ = PBA
 PAB ˆ = α  x+2 5 − 1+ 2 5 +1 5 −1 5 −1 4
 ˆ ˆ =
⇒ PBC 2α
ˆ 
2
= PAB
 PBC ˆ + PBA  5 − 1 5 − 2 5 +1
cos2 18° = 1 − sen2 18° = 1 −   = 1− ⇔
BP = ˆ =
BC ⇔BPC é isósceles de base PC ⇔ BPC ˆ =
BCP 2α  4  16

ˆ
ABC é isósceles de base AB ⇔ ABC = ˆ
ACB =2α 16 6 − 2 5 10 + 2 5 10 + 2 5 10 + 2 5
cos2 18=
° − = ⇒ cos18=
° =
ˆ + ABP
ˆ = ABC
ˆ ⇔ PBC
ˆ = ABC
ˆ − ABP
ˆ = 2α − α = α 16 16 16 16 4
PBC
6−2 5
2 sen2 18° 16 6−2 5 16 6−2 5
=
tg 18° = = ⋅ = ⇔
cos2 18° 10 + 2 5 16 10 + 2 5 10 + 2 5
16

6 − 2 5 10 − 2 5 60 − 12 5 − 20 5 + 20
tg2 18°
= ⋅ = ⇔
10 + 2 5 10 − 2 5 100 − 20
80 − 32 5 5 − 2 5
tg2 18°
= =
80 5
10 + 2 5
=
sen72 ° sen ( 90° − 18
= °) cos18
= °
4
5 −1
cos=72° cos ( 90° − 18
= °) sen18
= °
4
1 5 5+2 5
2
tg= 72° tg ( 90° −=
2
18°) ctg=2
18° = ⋅ ⇔
tg2 18° 5 − 2 5 5 + 2 5

tg2 72° =
(
5⋅ 5 + 2 5 )= 5+2 5
ˆ + ABC
ABC : BAC ˆ= 180° ⇔ α + 2α + 2=
ˆ + ACB α 180° ⇔ 25 − 20
180° 36° e 54°.
5α= 180° ⇔ α= = 36°
5
sen2 ( 2 ⋅ 18°)= ( 2 ⋅ sen18° ⋅ cos18°)2= 4 ⋅ sen2 18° ⋅ cos2 18° ⇔
Sem perda de generalidade, considere AP = PB = PC = 2 u.c.
6 − 2 5 10 + 2 5
(unidades de comprimento). sen2 36° = 4 ⋅
16

16
=
4
162
(
⋅ 60 + 12 5 − 20 5 − 20 ⇔ )
Os triângulos ABC e BPC são semelhantes pelo caso A.A.A.
10 − 2 5
(ângulo, ângulo, ângulo), portanto: sen2 36
= °
4
162
(
⋅ 40 − 8 =
5
16
162
) (
⋅ 10 − 2 =5
16
⇔ )
x+2 2 −2 ± 22 − 4 ⋅ 1⋅ ( −4 )
= ⇔ x 2 + 2x =4 ⇔ x 2 + 2x − 4 =0 ⇔ x = ⇔ 10 − 2 5
2 x 2 ⋅1 sen36° =
4
−2 ± 20 −2 ± 2 5
x= = =−1± 5 ⇒ x = 5 − 1
2 2
cos2 36° = 1 − sen2 36° = 1 −
10 − 2 5 16 − 10 − 2 5
= ⇔
( )
A partir do vértice A, traçe a mediana AM. 16 16
( )
2
A mediana relativa à base principal de um triângulo isósceles é 6 + 2 5 5 + 2 5 +1 5 +1 5 +1
2
ˆ = 90° e MÂB = 90º – 2.36º = 90º – 72º = 18º. =
cos 36° = = ⇔ cos
= 36°
altura, portanto AMB 16 16 16 4
10 − 2 5
sen2 36° 16 10 − 2 5 16 10 − 2 5
tg2 36°
= = = ⋅= ⇔
cos2 36° 6+2 5 16 6+2 5 6+2 5
16
2 10 − 2 5 6 − 2 5 60 − 20 5 − 12 5 + 20
=
tg 36° ⋅= ⇔
6+2 5 6−2 5 36 − 20

80 − 32 5
tg2 36° = = 5−2 5
16
5 +1
=
sen54 ° sen ( 90° − 36
= °) cos=
36°
4
10 − 2 5
54° cos ( 90° − 36
cos= = °) sen36
= °
4
1 1 5+2 5
2
tg= 2
36°) ctg=
54° tg ( 90° −= 2
36° = ⋅ ⇔
tg2 36° 5 − 2 5 5 + 2 5

tg2 54°
=
(5=
+ 2 5) 5+2 5
25 − 20 5

158 PROMILITARES.COM.BR
TRIGONOMETRIA I

18° 36° 54° 72°

5 −1 10 − 2 5 5 +1 10 + 2 5
SENO
4 4 4 4

10 + 2 5 5 +1 10 − 2 5 5 −1
COSSENO
4 4 4 4

5−2 5 5+2 5
TANGENTE² 5−2 5 5+2 5
5 5

FORMULAS DE DUPLICAÇÃO USANDO TANGENTE


x x x
2tg 1 − tg2 2tg
=senx = 2 cosx = 2 tgx 2
x x x
1+ tg2 1+ tg2 1+ tg2
2 2 2
Demonstração:
 x x
sen cos
x x 2 x  2 2  2 x
(I) senx 2sen
= = cos 2cos = tg
2 2 2  cos2 x  sec2 x 2
 
2  2
x
2 tg
= 2
2 x
1+ tg
2
x 2 2
(II) cosx
= 2cos2 = −1
x
= −1
x
−1
2 sec2 1+ tg2
2 2
 2 x x
2 − 1+ tg  1 − tg 2
 2 2
= =
2 x x
1+ tg 1+ tg2
2 2

TRANSFORMAÇÃO DE PRODUTO EM SOMA


sen ( a + b) (1) e senacos b − senb cos a =
senacos b + senb cos a = sen ( a − b) (2).

1
b sen ( a + b) + sen ( a − b) ⇔ senacos=
Somando-se as equações (1) e (2) tem-se: 2senacos= b ( sen ( a + b) + sen ( a − b))
2
1
Subtraindo-se as equações (1) e (2) tem-se: 2senb cos=
a sen ( a + b) − sen ( a − b) ⇔ senb cos=
a ( sen ( a + b) − sen ( a − b))
2
cos ( a − b) (3) e cos acos b − senasenb =
cos acos b + senasenb = cos ( a + b) (4).

Somando-se as equações (3) e (4) tem-se:


1
b cos ( a + b) + cos ( a − b) ⇔ cos acos=
2cos acos= b
2
( cos ( a + b) + cos ( a − b))
Subtraindo-se as equações (3) e (4) tem-se:
1
= cos ( a − b) − cos ( a + b) ⇔ senasenb
2senasenb =
2
( cos ( a − b) − cos ( a + b))

TRANSFORMAÇÃO DE SOMA EM PRODUTO


a + b = p 2a = p + q a+b a−b
 ⇔  = ⇔p = eq .
a − b = q 2b = p − q 2 2
p+q p−q
sen ( a + b) + sen ( a − b) =2senacos b ⇔ senp + senq =2 ⋅ sen ⋅ cos
2 2
p−q p+q
sen ( a + b) − sen ( a − b) =2senb cos a ⇔ senp − senq =2 ⋅ sen ⋅ cos
2 2

PROMILITARES.COM.BR 159
TRIGONOMETRIA I

p+q p−q
cos ( a + b) + cos ( a − b) =2cos acos b ⇔ cos p + cos q =2 ⋅ cos ⋅ cos
2 2
p+q p−q
cos ( a − b) − cos ( a + b) =2senasenb ⇔ cos q − cos q =2 ⋅ sen ⋅ sen
2 2

OUTRAS FÓRMULAS
sena cos b senb cos a sen ( a ± b) sen ( a ± b)
a sen ( a ± b) ⇔
senacos b ± senb cos= ± = ⇔ tga ±=
tgb
cos a cos b cos a cos b cos acos b cos acos b

sena cos b senb cos a sen ( a ± b) sen ( a ± b)


a sen ( a ± b) ⇔
senacos b ± senb cos= ± = ⇔ ctgb ± ctga
=
sena senb sena senb cos acos b cos acos b
cos a cos b sena senb cos ( a  b) cos ( a  b)
cos acos b ± senasenb = cos ( a  b) ⇔ ± = ⇔ ctga ±=
tgb
sena cos b sena cos b senacos b senacos b

FUNÇÕES TRIGONOMÉTRICAS INVERSAS


Todas as funções trigonométricas não são bijetoras. Para podermos definir suas funções inversas, vamos restringir o domínio das funções de
maneira conveniente a fim de obter uma função bijetora. O gráfico das funções trigonométricas inversas pode ser obtido refletindo-se o gráfico da
função trigonométrica em relação à reta y = x (bissetriz dos quadrantes ímpares).

FUNÇÃO ARCO SENO

 π π  π π
Seja f :  − ,  → [ −1,1] tal que f(x) = senx uma função bijetora, então a sua inversa é f : [ −1,1] →  − ,  tal que f–1(x) = arcsenx. Assim,
−1

 2 2  2 2

temos: y = f(x) = senx ⇔ x = f–1(y) = arcseny.


Propriedade fundamental:
 π π
θ= arcsenk, k ∈ [ −1,1] ⇔ sen θ= k ∧ θ ∈  − , 
 2 2
Propriedades:
arcsen ( − x ) = − arcsen x, ∀x ∈ [ −1,1]
) x; ∀x ∈ [ −1,1]
sen ( arcsen x=
 π π
arcsen ( sen y )= y; ∀y ∈  − , 
 2 2
A figura seguinte mostra o gráfico da função arco seno.

FUNÇÃO ARCO COSSENO


Seja f: [0,π] → [–1,1] tal que f(x) = cosx uma função bijetora, então a sua inversa é f–1:[–1,1] → [0,π] tal que f–1(x) = arccosx. Assim, temos:
y = f(x) = cosx ⇔ x = f-1(v) = arccosy.

160 PROMILITARES.COM.BR
TRIGONOMETRIA I

Propriedade fundamental: EXERCÍCIOS DE


θ= arccos k, k ∈ [ −1,1] ⇔ cos θ= k ∧ θ ∈ [ 0, π ]

Propriedades:
TREINAMENTO
arccos ( − x ) = π − arccos x, ∀x ∈ [ −1,1]
) x; ∀x ∈ [ −1,1]
cos ( arccos x= 01. O valor da expressão tg1º⋅tg2º⋅ ... ⋅tg88º⋅tg89º é:
arccos ( cos y )= y; ∀y ∈ [ 0, π ] a) 1 b) 2 c) 3 d) 4

A figura seguinte mostra o gráfico da função arco cosseno. 02. (ITA 2000) Sabe-se que x é um número real pertencente ao
intervalo ]0,2π[ e que o triplo da sua secante, somado ao dobro da
sua tangente, é igual a 3. Então, o cosseno de x é igual a:

3 15
a) d)
4 26
2 13
b) e)
7 49
5
c)
13
03. (CN 2009) Um triângulo retângulo, de lados expressos por números
inteiros consecutivos, está inscrito em um triângulo equilátero T de
lado x. Se o maior cateto é paralelo a um dos lados de T, pode-se
concluir que x é aproximadamente igual a
a) 6,5 c) 7,5 e) 8,5
b) 7,0 d) 8,0

04. (EFOMM 2013) Dois observadores que estão em posições


coincidentes com os pontos A e B, afastados 3 km entre si, medem
simultaneamente o ângulo de elevação de um balão, a partir do chão,
FUNÇÃO ARCO TANGENTE como sendo 30º e 75º, respectivamente. Se o balão está diretamente
π π acima de um ponto no segmento de reta entre A e B, então a altura
Seja f :  − ,  →  tal que f(x) = tgx uma função bijetora, do balão, a partir do chão, em km, é:
 2 2 
1 2 3
π π a) c) e)
então a sua inversa é f −1 :  →  − ,  tal que f–1(x) = arctgx. 3 5 2
 2 2 
b) 5 2
d)
Assim, temos: y= f ( x )= tg x ⇔ x= f −1 ( y )= arctg y . 2 3
Propriedade fundamental:
 π π 05. (ITA 1995) Considere C uma circunferência centrada em O e raio
θ= arctgk, k ∈  ⇔ tg θ= k ∧ θ ∈  − ,  2r, e t a reta tangente a C num ponto T. Considere também A um
 2 2
ponto de C tal que AÔT = θ é um ângulo agudo. Sendo B o ponto de
Propriedades: t tal que o segmento AB é paralelo ao segmento OT, então a área do
trapézio OABT é igual a:
arctg ( − x ) =− arctg x, ∀x ∈ 
a) r2(2 cos θ - cos 2θ) d) r2(2 sen θ + cos θ)
tg ( arctg x )= x; ∀x ∈ 
b) 2r2(4 cos θ - sen 2θ) e) 2r2(2 sen 2θ - cos 2θ)
 π π
arctg ( tg y )= y; ∀y ∈  − ,  ∞ c) r2(4 sen θ - sen 2θ)
 2 2
A figura seguinte mostra o gráfico da função arco tangente. 06. (ITA 2008) A soma de todas as soluções distintas da equação
π
cos3x + 2cos6x + cos9x = 0, que estão no intervalo 0 ≤ x ≤ , é
2
igual a:
a) 2π 7π
d)
23π 6
b)
12 13π
e)
9π 12
c)
6

07. (EN 2003) O número de soluções reais da equação


 1
sen  = x − 2
 x

é igual a n; assim, pode-se concluir que:


a) n = 0 c) n=2 e) n > 3
b) n = 1 d) n = 3

PROMILITARES.COM.BR 161
TRIGONOMETRIA I

πx
08. (EN 2013) Qual o valor da expressão cossec2πx + cotg + 2, onde
2
 x  π
x é a solução da equação trigonométrica arctgx + arctg  =
 x + 1 4
definida no conjunto  – {–1}?
a) 3 d) 2
b) –1 4+ 2
e)
6+ 2 2
c)
2

09. (ITA 1998) O valor de


tg10 x − 5 tg8 x sec2 x + 10 tg6 x sec 4 x −
1+ sen π / n 1+ cos 2π / n
−10 tg4 x sec6 x + 5 tg2 x sec8 x − sec10 x a) d)
1 − sen π / n 1 − cos 2π / n
 π 1+ cos π / n
para todo x ∈ 0,  , é: b) 1+ cos 2π / n
 2 1 − sen π / n e)
a) 1 c) –secx + tgx e) zero 1 − sen2π / n
d) –1 1+ sen2π / n
− sec2 x c)
b) 1 − sen2π / n
1+ sen2 x

10. (ITA 2007) Assinale a opção que indica a soma dos 14. (IME CG 2003) Seja α um ângulo do quarto quadrante
  k²π   1
elementos de A ∪ B, sendo = A = cujo cosseno é igual a . Determine o valor de y na expressão:
xk sen²  =: k 1,2 e 4
  24  
  (3k + 5)π   sec2 α − sec α ⋅ cos ecα . (Obs.: secα, cossecα e cotgα) representam
=
B = yk sen²  =: k 1,2 . y=
  24   1 − ctgα
respectivamente a secante, a cossecante e a cotangente do ângulo α.)
a) 0 2− 2+ 3 a) 4 d) 16 15
d)
3 b) 4 15 e) 15
b) 1
2+ 2 − 3 c) 16
e)
c) 2 3
15. (ITA 1997) Seja n ∈  com n > 1 fixado. Considere o
p 
11. (ITA 2009) Considere o triângulo ABC de lados a = BC, b = AC e conjunto= A  : p,q ∈  e 0 < q < n . Definimos f:  →  por
c = AB, e os ângulos internos α = CÂB, β = A B C e γ = B C
 A. Sabendo- q 
se que a equação x² – 2bx⋅cosα + b² - a² = 0 admite c como raiz dupla, f(x) = [cos(n!πx)]2π. Se f(A) denota a imagem do conjunto A pela
pode-se afirmar que função f, então

a) α = 90º a) f(A) = ]–1,1[ d) f(A) = {0}

b) β = 60º b) f(A) = [0,1] e) f(A) = {0,1}

c) γ = 90º c) f(A) = {1}

d) O triângulo é retângulo apenas se α = 45º.


16. (IME 1988) Demonstre a identidade
e) O triângulo é retângulo e b é hipotenusa.
 3 + cos4x 
tg2x + cotg2x =
2
 1 − cos4x 
12. (ITA 1993) Num triângulo ABC retângulo em A, seja D a projeção
de A sobre BC. Sabendo-se que o segmento BD mede  cm e que o 17. (IME 1991) Sejam A, B e C os ângulos de um triângulo. Mostre
ângulo DÂC mede θ graus então a área do triângulo ABC vale: que sen2A + sen2B + sen2C = 4senAsenBsenC.
2 2
a) secθtgθ d) cossecθcotgθ 18. (IME 2016) A circunferência C tem equação x² + y² = 16.
2 2
Seja C’ uma circunferência de raio 1 que se desloca tangenciando
2 2 internamente a circunferência C, sem escorregamento entre os pontos
b) sec²θtgθ e) cossec²θcotgθ
2 2 de contato, ou seja, C’ rola internamente sobre C.
2
c) secθtg²θ
2

13. Em uma coroa circular estão inscritas n circunferências, cada uma


tangente às duas vizinhas. Se o raio da circunferência interna da coroa
mede 1, então o raio da circunferência externa da coroa mede:

162 PROMILITARES.COM.BR
TRIGONOMETRIA I

Define-se o ponto P sobre C’ de forma que no início do movimento 05. (ITA 2015) Considere todos os triângulos retângulos com os
de C’ o ponto P coincide com o ponto de tangência (4,0), conforme lados medindo a , 2 a e a. Dentre esses triângulos, o de maior
figura a. Após certo deslocamento, o ângulo de entre o eixo x e a reta
que une o centro das circunferências é α, conforme figura b. hipotenusa tem seu menor ângulo, em radianos, igual a
- Determine as coordenadas do ponto P marcado sobre C’ em 3 1 4
a) arctg . c) arctg . e) arctg .
função do ângulo α. 4 2 5

- Determine a equação em coordenadas cartesianas do lugar 3 3


b) arctg . d) arctg .
geométrico do ponto P quando α varia no intervalo [0,2π). 3 5

19. (IME 2012) Os ângulos de um triângulo obtusângulo são 105°, 2ab


α e β. Sabendo que m ∈  (real), determine: =
06. (ITA 2014) Sabendo que sen x a ≠ 0 e b ≠ 0, um possível
a2 + b2 ,
1
a) as raízes da equação 3secx + m( 3 cosx – 3senx) = 3cosx + valor para cos sec 2x − tg x é
2
3 senx, em função de m;
b) o valor de m para que α e β sejam raízes dessa equação.
a−b a2 + b2
a) . d) .
ab 4ab
π π a+b a2 − b2
=
20. (ITA 2018) Seja z cos + isen . Pedem-se: b) . e) .
7 7 2ab 4ab
kπ kπ
a) Use a propriedade zk = cos + isen , k ∈ , para expressar
7 7 a2 − b2
π 3π 5π c) .
cos , cos e cos em função de Z. ab
7 7 7
a π 3π 5π 1
b) Determine inteiros a e b tais que =cos + cos + cos . 07. (ITA 2013) Se cos 2x = , então um possível valor de
b 7 7 7 2
cotg x − 1
é
cossec(x − π ) − sec( π − x)
EXERCÍCIOS DE

COMBATE a) 3
. c) 2. e) 2.
2
b) 1. d) 3.

01. (ITA 2019) Considere um retângulo ABCD em que o comprimento 10


 1 
do lado AB é o dobro do comprimento do lado BC. Sejam M o ponto 08. (IME 2017) No desenvolvimento de  x ⋅ sen 2β + cos 2β o
médio de BC e N o ponto médio de CM. A tangente do ângulo MÂN  x 
é igual a valor do termo independente de x é igual a 63/256 Considerando que
1 4 16 α é um número real, com 0 < β < π/8 e x ≠ 0, o valor de β é:
a) . c) . e) .
35 35 35 a) π/9 c) π/16 e) π/24
2 8 b) π/12 d) π/18
b) . d) .
35 35
sen4 α + cos 4 α
09. (IME 2017) Calcule o valor de , sabendo-se que
02. (ITA 2016) Um triângulo retângulo tem perímetro igual a  5, 1 sen6 α + cos6 α
em que a é comprimento da hipotenusa. Se α e β são seus ângulos sen α cos α = .
5
agudos, com α < β então sen(β – α) é igual a 22 25 26
a) c) e)
a) 5 − 2 5. 21 23 25
d) 20 5 − 44.
23 13
b) −6 + 3 5. e) 18 5 − 40. b) d)
22 12
c) 16 5 − 35.

10. (IME 2016) Seja a equação sen(2x) = 1 . As soluções dessa equação


 3π   π  tgx 2
03. (ITA 2016) Se tgx = 7 e x ∈  π,  , então sen3x é igual a para x ∈  − , π  formam um polígono no círculo trigonométrico de
 2  2 
a) − 14 . 14 14 área
c) . e) .
8 4 6 e) 1
3 5 3
a) c)
14 14 2 8
b) . d) − .
8 4 1
b) 3 d)
2
04. (ITA 2016) Seja ABC um triângulo equilátero e suponha que M e
N são pontos pertencentes ao lado BC tais que BM = MN = NC. Sendo π 2− 3
α a medida, em radianos, do ângulo MÂN então o valor de cosα é 11. (ITA 2015) Seja n um inteiro positivo tal que sen = .
2n 4
13 15 17 a) Determine n.
a) . c) . e) .
14 16 18 π
b) Determine sen .
14 16 24
b) . d) .
15 17

PROMILITARES.COM.BR 163
TRIGONOMETRIA I

12. (ITA 2012) Determine os valores reais de x de modo que


sen(2x) − 3 cos(2x) seja máximo.
GABARITO
Notações: EXERCÍCIOS DE TREINAMENTO
: Conjunto dos números naturais; 01.
: Conjunto dos números reais;
1
+: Conjunto dos números reais não negativos; = − x)
tg x ctg ( 90°= − x) 1
⇔ tg x ⋅ tg ( 90°=
i: unidade imaginária; i² = –1; tg ( 90° − x )
P(A): conjunto de todos os subconjuntos do conjunto A; =
P tg1° ⋅ tg2° ⋅ ... ⋅ tg88° ⋅ tg89° ⇔
n(A): número de elementos do conjunto finito A; =
P ( tg1° ⋅ tg89°) ⋅ ( tg2° ⋅ tg88°) ⋅ ( tg3° ⋅ tg87°) ⋅ ... ⋅ ( tg 44° ⋅ tg 46°) ⋅ tg 45° ⇔
AB: segmento de reta unindo os pontos A e B; P = 1⋅ 1⋅ 1⋅ ... ⋅ 1⋅ 1 = 1
arg z: argumento do número complexo z;
[a,b] = {x ∈ : a ≤ x ≤ b}
A\B = {x: x ∈ A e x ∉ B} 02.
AC: complementar do conjunto A; 3sec x + 2tg x =
3 ⇔ 2tg x =
3 − 3sec x ⇔
n

∑a x
k=0
k
k
= a0 + a1x + a2x 2 + ... + an xn ,n ∈ . ⇔ 4 tg2 x =
9 − 18 sec x + 9 sec2 x
⇔ 4 ( sec2 x − 1) = 9 − 18 sec x + 9 sec2 x ⇔
Observação: Os sistemas de coordenadas considerados são
cartesianos retangulares. ⇔ 5sec2 x − 18 sec x + 13 =
0
13
⇔ sec x =∨
1 sec x = ⇔
13. (ITA 2011) Num triângulo AOB o ângulo AÔB mede 135° e os 5
lados AB e OB medem 2. cm e 2 − 3cm , respectivamente. A 5
⇔ cos x =
1 ∨ cos x =
circunferência de centro em O e raio igual a medida de OB intercepta 13
AB no ponto C(≠ B). 5
x ∈ ]0,2π[ ⇒ −1 ≤ cos x < 1 ⇒ cos x =
a) Mostre que OÂC mede 15°. 13
b) Calcule o comprimento de AC.
03. Sejam k – 1, k e k + 1, k ∈ Z, os lados do triângulo retângulo.
14. (ITA 2011)
Pelo teorema de Pitágoras, temos:
π π π π π π
a) Calcule  cos2 − sen2  cos − 2sen cos sen . (k + 1)2 = k2 + (k − 1)2 ⇔
 5 5 10 5 5 10
π π ⇔ k 2 + 2k + 1= k 2 + k 2 − 2k + 1 ⇔
b) Usando o resultado do item anterior, calcule sen cos .
10 5 ⇔ k 2 − 4k =0 ⇔
15. (ITA 2004) Prove que, se os ângulos internos α, β e γ de um ⇔ k = 0 (não convém) ou k = 4
triângulo satisfazem a equação:
Logo, os lados do triângulo retângulo são 3, 4 e 5.
sen (3α) + sen (3β) + sen (3γ) = 0, então, pelo menos, um dos três
ângulos α, β ou γ é igual a 60º. A figura a seguir representa o triângulo equilátero T e o triângulo
retângulo inscrito.
16. (IME 2010) Considere a sequência

1 11 1 1 1 11 1 1 1 1 1 11
a1 = + , a2 = + + , a3 = + + + ,...
2 22 2 2 2 22 2 2 2 2 2 22

Determine o produto dos 20 primeiros termos dessa sequência.

17. (IME 2002) Resolva a equação tg α + tg (2α ) = 2 tg (3α), sabendo-


se que α ∈ [0, π/2).
Para α, 2α, 3α ≠ kp + p/2 (k ∈ Z), temos:

18. (IME 2006) Seja a matriz D dada por:


 1 1 1 
 
D=  p q r 
 ˆ ˆ ˆ 
sen (P) sen (Q) sen (R)
na qual p, q e r são lados de um triângulo cujos ângulos opostos são, DE  BC ⇒ ∆ADE é equilátero ⇒ AD
= DE
= 4
respectivamente, P,ˆ Qˆ e Rˆ . Determine o valor do determinante de D ˆ= EDF
DE  BC ⇒ CFD ˆ= 90º ⇒ ∆DFC é retângulo

ˆ = DF 3
senDCF ⇒ sen60º = ⇔
19. Resolva a equação 2 sen 11x + cos 3x + 3 sen 3x =
0. DC DC
3 3
⇔ DC= = = 2 3
( )(
20. Determine o valor de 1 − cotg 22o ⋅ 1 − cotg 23o . ) sen60º 3 2
x = AC = AD + DC = 4 + 2 3 ≈ 4 + 2 ⋅ 1,73 = 7, 46

164 PROMILITARES.COM.BR
TRIGONOMETRIA I

04. 2cos6xcos3x + 2cos6x =


0
⇒ 2cos6x ( cos3x + 1) =0
⇒ cos6x = 0 ou cos3x = −1
π π kπ
(i) cos6x = 0 ⇒ 6x = + kπ ⇒ x = + (k ∈  )
2 12 6
 π π 5π 
⇒ x ∈ , , 
12 4 12 
π 2kπ
(ii) cos3x = −1 ⇒ 3x = π + 2kπ ⇒ x = + (k ∈  )
3 3
π
⇒ x ∈ 
 3
Logo, a soma pedida é dada por
Seja P a posição do balão e P’ a projeção de P sobre o segmento AB, π π 5π π 13π
+ + + =
ˆ = 90 − 30 = 60
então ∆APP’ e ∆BPP’ são triângulos retângulos e APP' 12 4 12 3 12
ˆ  
e BPP' = 90 − 75 = 15 .

Consequentemente, o ângulo  1
07. Como −1 ≤ sen   ≤ 1 , temos que –1 ≤ x – 2 ≤ 1, ou seja, 1 ≤ x ≤ 3.
ˆ = APP'
APB ˆ + BPP'
ˆ = 60 + 15 = 75 = ABP
ˆ .  x
2  1
Portanto, o triângulo ABP é isósceles e AP = AB = 3. Sabemos que para x > a função y = sen   é estritamente
π  x
PP' PP' 1 3 decrescente, logo ela corta uma única vez a reta y = x – 2.
No ∆APP’, temos sen30 = = = ⇔ PP' = km , que é a
medida da altura do balão. AP 3 2 2

05.

O ponto D é a projeção ortogonal do ponto A sobre o raio OT.


No ∆ADO, retângulo em D,
OD = 2rcosθ
 x 
AD = 2rsenθ 08. arctgx = α e arctg  =
β
 x + 1
Cálculo de AB:
 x 
Então tg = α x, tg = β 
AB = DT = OT – OD = 2r – 2rcosθ  x + 1
Cálculo da área do trapézio OABT:  π π
E α, β ∈  − , 
AB + OT  2 2
A= .AD
2  x  π
arctgx + arctg  =
 x + 1 4
2r − 2rcosθ + 2r
.2rsenθ π tgα + tgβ
2 α + β= ⇒ tg ( α + β)= = 1
=( 4r − 2rcosθ) .rsenθ 4 1 − tgαtgβ
x
r2 ( 4senθ − 2cosθsenθ) x+ 2
⇒ x + 1 = 1⇔ x + 2x = 1⇔ 2x 2 + x − 1 = 0
r2 ( 4senθ − sen2θ) x x + 1− x2
1 − x.
x +1
1
x= −1 ou x =
06. Utilizando as fórmulas de transformação de soma em produto, 2
temos:
1
 3x + 9x   3x − 9x  Como x + 1 ≠ 0 ⇔ x ≠ −1 , então x =
=
cos3x + cos9x 2cos  = cos  2cos6xcos3x 2
 2   2 
πx π π
cossec2πx + cotg + 2= cossec2 + cotg + 2= 12 + 1+ 2= 2
Assim, a equação do enunciado pode ser escrita como: 2 2 4

PROMILITARES.COM.BR 165
TRIGONOMETRIA I

09. Como ( x − a) = x5 − 5x 4a + 10x 3a2 − 10x 2a3 + 5xa4 − a5


5
e π
π r π π sen
1 + tg²x = sex²x, temos: sen = ⇔ sen + r ⋅ sen = r ⇔ r = n
n 1+ r n n π
tg10 x − 5 tg8 x sec2 x + 10 tg6 x sec 4 x − 10 tg4 x sec6 x + 1 − sen
n
+5 tg2 x sec8 x − sec10 x = π π
sen 1+ sen
( tg2 x − sec2 x ) =
=
5
( −1) =−1
5
R = 1+ 2r = 1+ 2 ⋅ n
π
= n
π
1 − sen 1 − sen
n n
10.
14.
 π π π   11π 
x= sen2  = cos2  − = cos2 
1
 24   2 24   24  3π
< α < 2π e cos α =
1
π  π π  π 2 4
x= sen2 = cos2  − = cos2  
2
6  2 6   3  1 15
2
⇒ senα = − 1 −   = −
π  4 4
y1 = sen2
3 4 15 15
⇒ sec α = 4 e cos secα = −
e c tgα = −
2  11π  15 15
y 2 = sen 
 24   4 15 
42 − 4 ⋅  −
15 
2

∑ x + y =2
2
sec α − sec α ⋅ cos secα 
⇒ i i =y = =
i=1
1 − ctgα  15 
1−  − 
 15 
16 15
11.
2 2 2
16 +
= = 15
16 15 + 15
= 16
( )
x − 2bx ⋅ cos α + b − a = 0 15 15 + 15
1+
⇒ ( −2b cos α ) − 4 ⋅ 1⋅ (b2 − a2 ) = 0 ⇔
2
15
⇔ 4b2 ( cos2 α − 1) + 4a2 = 0 ⇔ b2 sen2 α= a2 ⇔
a2 15.
⇔ sen2 α =
b2 n!  n!p 
0<q<n⇒ ∈  ⇒ cos  π =±1

= c
2b cos α
= b cos α ⇔ cos =α
c q  q 
2 b 2n
 p   p 
= ( ±1) = 1 ⇒ f ( A ) = {1}
2n
a2 c2 f   = cos  n! π ⋅  
⇒ sen2 α + cos2 α = 2 + 2 = 1 ⇒ a2 + c2 = b2  q   q 
b b

Logo, o triângulo ABC é retângulo em B, então= ˆ


β ABC
= 90 .
16. Desenvolvendo o lado esquerdo da expressão, temos:

12. sen2x cos2x sen4 x + cos 4 x


tg2x + cotg2x = + =
cos2x sen2x sen2xcos2x
Considere as identidades:
3 + cos4x
sen4 x + cos 4 x =
4
2
 sen2x  sen2 2x 1 − cos4x
sen2=
xcos2x  = =
 2  4 8

Segue que:
3 + cos4x
2
tg x + cotg=
x2 4 = 2  3 + cos4x 
1 − cos4x  
1 − cos4x 
ˆ = θ ⇒ BAD
DAC ˆ = 90 − θ ⇒ ABD
ˆ =θ 8
AD AD
∆ABD : tg=
θ = ⇔ AD=  tg θ
BD  17.
CD CD sen2A + sen2B + sen2C
∆ACD : tg=
θ = =  tg2 θ
⇔ CD
AD  tg θ
 2B + 2C  2B − 2C
= 2senAcosA + 2sen  cos 
 2   2 
13. Ligando o centro de duas circunferências consecutivas orma-se um = 2senAcosA + 2sen (B + C) cos (B − C)

triângulo isósceles de lados iguais (1 + r), base 2r e ângulo central . Como A + B + C = 180º, temos que B + C = 180º, que implica
Traçando a altura do triângulo, temos: n
sen(B + C) = senA.
Substituindo: 2senA(cosA + cos(B – C))

166 PROMILITARES.COM.BR
TRIGONOMETRIA I

  A + B − C  A − B + C  3
= 2senA  2cos   cos    Portanto, tgx = m e tgx =
  2 2  3
π
=
Logo, x arctg(m) + k.π ou x = + k.π, com k ∈ Z
A + B − C 180o − 2C 6
Como A + B + C = 180º, temos que = = 90o − C ,
2 2
 A + B − C
ou seja, cos 
 2
=

cos 90o =
− C senC ( ) =
b) tgα m =
e tgβ
3
3

 A − B + C Logo, α = 30º, então α = 180º – 105º – 30º = 45º.


Analogamente, temos cos  = senB . Logo, sen2A + sen2B
 2  Logo, m = 1.
+ sen2C = 4senAsenBsenC
20.
18. a) Note que:
a) Se a circunferência C’ deslocou-se α, então ela percorreu uma kπ  kπ kπ π (14 − k )

distância d igual a: • cos  2π −=  cos ⇒ cos
= cos
 7 7 7 7
α d
 kπ  kπ kπ π (14 − k )
2π 2πR • sen  2π −  = − sen ⇒ sen = − sen
 7 7 7 7
α ⋅ 2π ⋅ 4
d= → d = 4α kπ kπ
2π =
De zk cos + isen ,
7 7

=z14 −k cos
(14 − k ) π + isen (14 − k ) π
7 7

Por outro lado,

=zk cos
(14 − k ) π − isen (14 − k ) π
7 7

Então,

zk + z14 −k =
2cos
(14 − k ) π
7
k) π
(14 − =
cos
7 2
(
1 k 14 −k
z +z )
k = 13,
π
=
cos
7
1 13
2
(
z +z )
k = 11,
Pode-se escrever:

= OO' + O'P
OP = ( 3cos α,3sen α ) + ( 3cos α, −3sen α=) =
cos
7 2
(
1 11 3
z +z )
= ( 3cos α + 4 cos 3
α − 3cosα,3sen α − 3sen α + 4sen3α ) k = 9,

Assim, cos=
7 2
(
1 9 5
z +z )
(
P =( x, y ) = 4 cos α, 4sen α
3 3
)
b) Teremos:
π π π 1
b) Da relação fundamental da trigonometria, tem-se: cos
7
1
( 1
+ cos + cos = ⋅ z + z13 + ⋅ z3 + z11 + ⋅ z5 + z9
7 7 2 2 2
) ( ) ( )
2 2
sen α + cos α =1 π π π 1
 x  y
2
2 2
2
7 7 7 2
(
cos + cos + cos = ⋅ z + z3 + z5 + z9 + z11 + z13 )
3
 4  +  4  =→
1 x 3+y 3 = 16
3 3
π π π 1
    (
cos + cos + cos = ⋅ z + z3 + z5 + z7 + z9 + z11 + z13 − z7
7 7 7 2
)
19. π π π 1 
cos + cos + cos = ⋅
 7
z ⋅ z2 − 1

7
−z 
(( ) )
7 7 7 2  z2 − 1
 
a) 3sec x + m ( 3 cos x – 3 sen x ) =  1 
3 cos x + 3 sen x  ⋅  
 cos x 

3 ⋅ sec2x + m⋅ 3 − 3m ⋅ tgx =3 + 3 ⋅ tgx


( )
3. 1+ tg2x – (3m − 3) ⋅ tgx + m ⋅ 3 − 3 =0
3 ⋅ tg2x – (3m − 3) ⋅ tgx + m 3 =
0

PROMILITARES.COM.BR 167
TRIGONOMETRIA I

= em z14 −k cos
k 0=
(14 − k ) π + isen (14 − k ) π ,  π
sen  2x −  máximo, logo
7 7  3
z14 cos 2π + isen2π
= π π
2x − = + k.2π onde k ∈ 
z14 = 1 3 2
= em z14 −k cos
k 7=
(14 − k ) π + isen (14 − k ) π , 2x =

+ k.2π onde k ∈ 
7 7 6
7
z= cos π + isenπ 5π
x= + k.π onde k ∈ 
z7 = −1 12

Logo,
π 3π 5π 1  z ⋅ (1 − 1)  13.
cos + cos + cos = ⋅ − ( −1)
7 7 7 2  z2 − 1  a) Utilizando o teorema dos senos, temos:
π 3π 5π 1 2− 3 2 2− 3
cos + cos + cos = = ⇔
= senα
7 7 7 2 senα sen135o 2
a π 3π 5π 1 π 3π 5π
De =cos + cos + cos e =cos + cos + cos ,  6 − 2
2
b 7 7 7 2 7 7 7 2− 3 2− 3
Sabendo que sen215o =   ⇔ sen15 =
o
= ,
 4  4 2
os inteiros a = 1 e b = 2 satisfazem concluímos então que:
a π 3π 5π OÂB = 15º
=cos + cos + cos .
b 7 7 7
b) O triângulo ACB é isósceles logo AC = AB = 2 − 3cm.
π 1 3π 1 3 11 5π 1 5 9
( 13
a) cos = ⋅ z + z , cos
7 2
)
7
= ⋅ z + z , cos
2
(
7
)
= ⋅ z +z ;
2
( )
a π 3π 5π
b) Os inteiros a = 1 e b = 2 satisfazem =cos + cos + cos .
b 7 7 7

EXERCÍCIOS DE COMBATE
01. C 03. B 05. C 07. A 09. B
02. D 04. A 06. E 08. E 10. A

11.
π π
a) Como n é um número inteiro devemos considerar que 0 < <
2⋅n 2
π π 2 3 π 14.
1 − cos 1 − cos − 1 − cos
π 2− 3
n ⇒ = n ⇒ 2= 2 n ⇒ π 3π
=
sen = cos  cos2= ⇒ n 2 6π  π π π π
2⋅n 2 4 2 4 2 a) n 25 − sen 5  cos 10 − 2sen 5 cos 5 sen 10 .
 
2
π 2 3 π 2π π 2π π  2π π  π
1 − cos − 1 − cos = cos .cos − sen .sen = cos  + =  cos  =
 0
n ⇒ 2= 2 n ⇒ π 3 5 10 5 10  5 10  2
= cos = ⇒n 6
2 4 2 n 2
2 b) da relação 1, temos:
 2 π 2 π π π π π
π  cos − sen  cos =
2.sen .cos .sen
b) Calculando, inicialmente, o valor do cos , temos:  5 5 10 5 5 10
12
 2 π π π 2π
π π π π π π π 1 2 3 2 2 +  cos
6 − sen2  cos 1.cos
cos =cos  −  =cos ⋅ cos − sen ⋅ sen = ⋅ + ⋅ =  5 5 10 π π 5 sen π .cos=
π 1 π
12 3 4 3 4 3 4 2 2 2 2 4 = cos .sen ⇔= ⇔ sen
π 5 10 π 10. 5 4 10.
2.sen 4.sen
2 3 2 2 + 6 cos2 π − sen2 π  cos π 2π 5 10
+ ⋅ =   π π
1.cos
π π 1 π π
2 2 2 4  5 5  10 5
= cos .sen ⇔ = sen .cos= ⇔ sen .cos
π 5 10 π 10. 5 4 10. 5
2.sen π 4.sen
Obtendo, agora, o valor5de sen . 10 2π π 2π π π
24 Pois, cos= sen ( = + )
5 10 5 10 2
 2+ 6
π 1 −   4− 2− 6
1 − cos
 π  12 = 4  4 4 − 2 − 6 15. Sejam α, β e γ as medidas dos ângulos internos de um triângulo
=sen   = = (α, β e γ ϵ ]0, π[).
 24  2 2 2 8
4− 2− 6 Temos que α + β + γ = π ⇔ γ = π - α - β.
4 4− 2− 6 sen(3α) + sen(3β) + sen(3γ) =
=
2 8
sen(3α) + sen(3β) + sen [3(π - α - β)] =
2 sen [(3α + 3β)/2] cos [(3α - 3β)/2] + sen [3π - (3α + 3β)] =
12. 2 sen [(3α + 3β)/2] cos [(3α - 3β)/2] + sen (3α + 3β) =
1 3   π 2 sen [(3α + 3β)/2] cos [(3α - 3β)/2] + 2sen [(3α + 3β)/2] cos [(3α + 3β)/2] =
sen(2x) − 3 cos(2x)
=  sen(2x) − =
cos(2x)  ⋅ 2 2.sen  2x − 
 2 2   3 2 sen [(3α + 3β)/2] { cos [(3α - 3β)/2] + cos [(3α + 3β)/2]} =
2 sen [3(α + β)/2] 2 cos (3α/2) cos (- 3β/2) =
Para que a expressão seja máxima deveremos ter
4 sen [3(π - γ)/2] cos (3α/2) cos (3β/2) =

168 PROMILITARES.COM.BR
TRIGONOMETRIA I

4 sen [(3π/2) - (3γ/2)] cos (3α/2) cos (3β/2) = p + q p − q


sabemos que: sen p + sen q = 2 . sen   . cos  
- 4 cos (3γ/2) cos (3α/2) cos (3β/2).  2   2 
Desse modo, - 4 cos (3γ/2) cos (3α/2) cos (3β/2) = 0 se, e somente se:  π  π
cos (3α/2) = 0 ou cos (3β/2) = 0 ou cos (3γ/2) = 0  11x + 3x + 6   11x − 3x − 6 
⇔ 2 . sen   . cos  =0
 2   2 
O que nos dá: 3α/2 = π/2 → α = π/3 = 60° ou 3β/2 = π/2 → β = π/3 =
   
60° ou 3γ/2 = π/2 → γ = π/3 = 60°
 π  π
c.q.d. sen  7x +  = 0 ou cos  4x −  = 0
 12   12 
π π π
7x + =kπ 4x − = + kπ, k ∈ 
1 11 π 12 12 2
16. a1 = + = cos 30°
2 22 6 kπ π kπ 7π
x= − ,k∈ x= + ,k∈
a 1 1 7 84 4 48 .
Lembrando que, cos= + cos a
2 2 2 O conjunto-solução é
1 1 1 11 π  kπ π kπ 7π 
Concluímos que a2 = + + =cos e que S =x ∈  | x = − ou x = + , k ∈ 
2 2 2 22 2⋅6  7 84 4 48 .
1 1 1 1 1 11 π
a3 = + + + =cos 2
2 2 2 2 2 22 2 ⋅6 20. 2
Percebemos, então que devemos calcular o seguinte produtório:
20

∏ cos =
k =1
π 1 
. 
6 2  k −1
cos
π
219 ⋅ 6
π
2 ⋅6
π
⋅ cos 18 ⋅ cos 17 ⋅ ⋅ ⋅ ⋅ cos 0
2 ⋅6
π
2 ⋅6
ANOTAÇÕES

π
Multiplicando e dividindo o produtório por sen e utilizando por
sen2α 6.219
várias vezes a relação senα.cos α = , temos que:
2
20

∏ k =1
π 1 
cos  . k −1  =
 6 2 21
3
 2 .sen π
6 ⋅ 219
17.
sen α sen(2α ) sen α cos(2α ) + cos α sen(2α )
+ = 2tg(3α ) ⇔ = tg(3α ) ⇔
cos α cos(2α ) cos α cos(2α )
sen(3α ) sen(3α )  1 2 
=
2 ⇔ sen(3α )  − =0⇔
cos α cos(2α ) cos(3α )  cos α cos(2α ) cos 3α 
 cos 3α − 2cos α cos(2α ) 
sen(3α )  =0⇔
 cos α cos(2α )cos(3α ) 
 cos α cos 2α − sen α sen2α − 2cos α cos 2α 
sen(3α )  =0⇔
 cos α cos(2α )(cos 3α ) 
cos α
sen(3α ) = 0 ⇔ sen(3α ) = 0 ⇔
cos α cos 2α cos 3α
3α = kπ ⇔

α=
3
 π π
Como α ∈ 0,  , o conjunto verdade da equação é V = {0, }
 2 3

18. A segunda linha é duas vezes o circunraio vezes a terceira linha,


pela Lei dos Senos. Logo as linhas 2 e 3 são proporcionais e, portanto,
o determinante é nulo.
19. 2sen 11x + cos 3x + 3 sen 3x = 0, dividimos ambos os termos
por 2
2 1 3
⇔ .sen 11x + . cos 3x + . sen 3x = 0
2 2 2
π π
⇔ sen 11x + sen . cos 3x + sen 3x . cos =0
6 6
π
⇔ sen 11x + sen( + 3x) = 0,
6

PROMILITARES.COM.BR 169
TRIGONOMETRIA I

ANOTAÇÕES

170 PROMILITARES.COM.BR
NÚMEROS COMPLEXOS

NÚMEROS COMPLEXOS – CIRCUNFERÊNCIA


FORMA TRIGONOMÉTRICA O conjunto dos pontos tais que:
|z - z1| = r é uma circunferência de centro em z1 e raio r, com r > 0.

Seja z= (a,b)= a + bi
MEDIATRIZ
O conjunto dos pontos tais que:
=r a2 + b2 é o módulo do complexo z.
|z - z1| - |z - z2| é a mediatriz do segmento z1z2.
a b
cosθ= ; senθ= ⇔ a= r cos θ e b=rsenθ
r r
=
z r(cos θ + isenθ= ) rcisθ (Forma Trigonométrica)
com θ ∈ [0,2π], θ é o argumento principal.

GEOMETRIA DOS NÚMEROS


COMPLEXOS
DISTÂNCIA ENTRE DOIS COMPLEXOS
Representando os complexos como vetores, é fácil representar o vetor
diferença e encontrar o módulo da diferença entre os afixos de dois vetores.
Sejam z1 = x1 + y1 ⋅ i e z2 = x2 + y2 ⋅ i
Observação
O módulo |z2 - z1| representa a distância do afixo de z2 ao afixo de z1.

ELIPSE
O conjunto dos pontos tais que:
|z - z1| + |z - z2| = 2a, com 2a > |z1 - z2|.

PROMILITARES.COM.BR 187
NÚMEROS COMPLEXOS

Então as formas de representar um número complexo


z= ( a,b ) = a + bi = r ( cos θ + i ⋅ sen θ ) = r ei θ
Propriedades:
i( α +β )
iα iβ
a) r e ⋅ s ⋅ e = rs ⋅ e eiθ − e−iθ
f) sen θ =
2i
r ⋅ eiα r i( α −β)
b) = ⋅e
s ⋅ eiβ s θ iθ
eiθ − 1 2.i.sen ⋅ e 2
g) =
2
c) (r ⋅ e )
iα n
rn einα
=⋅
iθ θ iθ
d) ln =
z ln r + iθ h) e= + 1 2.cos ⋅ e 2
2
eiθ + e−iθ
e) cos θ =
2

RAÍZES ENÉSIMAS DA UNIDADE


2kπ
É uma elipse com focos em z1 e z2 e eixo maior igual a 2a. wk = n
1= n cis 0 = cis
, 0 ≤ k ≤ n-1. ,
n
Esses números representam, no plano de Argand-Gauss, os
HIPÉRBOLE vértices de um polígono regular de n lados inscrito no círculo de raio
O conjunto dos pontos tais que: unitário centrado na origem.
|z - z1| - |z - z2| = 2a, com 2a < |z1 - z2|. As raízes wk com mdc (k, n) =1 são denominadas raízes primitivas
É um dos ramos da hipérbole com focos em z1 e z2. da unidade.
Exemplo:
Calcule as raízes quintas da unidade

zk = 5 1 = cis 2kπ
5
k = 0 , z1 = 1
2π 2π 2π
k = 1 , z2 = cis
= cos + isen
5 5 5
4π 4π 4π
k = 2, z3 = cis
= cos + isen
5 5 5
6π 6π 6π
k = 3, z4 = cis
= cos + isen
5 5 5
8π 8π 8π
k = 4, z5 = cis
= cos + isen
5 5 5

FORMA EXPONENCIAL
Um complexo de módulo unitário pode ser representado na forma
cos θ + i ⋅ sen θ = ei θ
Acompanhe a “demonstração” usando Moivre
n
 θ θ
cos θ + i ⋅ sen
= θ  cos + i ⋅ sen  ∀n ∈
= N*
 n n
n n
 θ θ  θ
= lim  cos + i ⋅ sen = lim 1+ i ⋅ =
x →∞
 n n x →∞
 n
n/i θ i θ
 1  
lim 1+
=  ei θ
 =
n / iθ
 
x →∞

Lembre-se que
n
 1
lim 1+ =
 e, cos0
= 1 e que se x → 0, x ≈ senx
x →∞
 n
Portanto, a forma exponencial de um complexo é (i) Qualquer valor de k > 4 dará uma solução repetida
=z r ( cos θ + i ⋅ sen
= θ ) r ei θ (ii) As cinco raízes são os vértices de um pentágono regular
inscrito em uma circunferência de raio unitário centrada na origem.

188 PROMILITARES.COM.BR
NÚMEROS COMPLEXOS

4
EXERCÍCIOS DE 07. (IME 2013) Considere a equação em , ( z − 5 + 3i) =1. Se z0 é a

TREINAMENTO solução que apresenta o menor argumento principal dentre as quatro


|z |
soluções, então o valor de 0 é
a) 29. c) 3 5. e) 3 6.
01. Os pontos que representam os números complexos z1 e z2 b) 41. d) 4 3.
encontram-se sobre uma circunferência no plano complexo, cujo
centro é o ponto associado ao número complexo i e o raio é 1. A parte 08. (ESPCEX 2018) Na figura abaixo, está representado o plano de
π Argand-Gauss com os afixos de 12 números complexos, identificados
real de z1 ⋅ z2 é 0 e o argumento de z1 é . O valor de z2 é:
6 de A a L. Sabe-se que esses afixos dividem a circunferência em 12
partes iguais e que A = (1, 0).
a) 1+ i 3 3
d) − + i
2 2
3 3
b) + i
2 2 3 1
e) − + i
2 2
c) 3 1
+ i
2 2

02. Colocando-se a soma dos 22 primeiros termos da progressão


geométrica cujo primeiro termo é igual a i e cuja razão é igual a (1+ i)
sob a forma c + di onde c e d são números reais podemos afirmar que
c + d é igual a :
a) 2049 d) −2048
b) 2048 e) −2049
c) 1023

03. (EN 2014) Desenha-se no plano complexo o triângulo T com O polígono regular cujos vértices são os afixos de 4

vértices nos pontos correspondentes aos números complexos a) BEHK. c) ADGJ. e) CEIK.
z1, z2 , z3 , que são raízes cúbicas da unidade. Desenha-se o triângulo b) CFIL. d) BDHJ.
S, com vértices nos pontos correspondentes aos números complexos
w 1, w 2 , w 3 , que são raízes cúbicas de 24 3. Se A é a área de T e 09. (IME 2018) Seja a função H:  →  definida por
B é a área de S, então
a3s3 + a2s2 + a1s + a0
a) B = 12A H(s) =
b2s2 + b1s + a0
b) B = 18A
com aj e bk reais, para j = 0, 1, 2, 3 e k = 0, 1, 2. Seja a função
c) B = 24A
f :  →  em que f(w ) é a parte real de H(iw ) em que i= −1 é a
d) B = 36A
unidade imaginária e w ∈ . A afirmação correta a respeito de f(w )
e) B = 42A é:
10 a) f(w ) é uma função impar.
 1+ 3i 
04. (IME 2015) z = 
Se  , então o valor de b) f(w ) é uma função par.
 1− 3i 
2arcsen(Re(z)) + 5arctg(2Im(z)) é igual a c) f(w ) é sempre negativa.
d) f(w ) é sempre positiva.

a) − . 4π e) f(w ) é uma função periódica.
3 d) .
3
π
b) − . 5π | z − 2 |=| z + 4 |
3 e) . 10. (EFOMM 2018) Resolvendo o sistema  , para
3 | z − 3 | + | z + 3 |=
10
2π z complexo, encontramos como solução
c) .
3
8 6 8 6
a) −1+ i; − 1− i
05. (IME 2013) A soma das raízes da equação em , z8 − 17 z4 + 16 =
0, 5 5
tais que z − | z |=
0, é 8 6 8 6
a) 1. c) 3. e) 5. b) +1+ i; + 1− i
5 5
b) 2. d) 4.
6 8 6 8
c) −1+ i; − 1− i
5 5
06. (IME 2013) Seja λ solução real da equação λ + 9 + 2λ + 17 =12.
6 8 6 8
Então a soma das soluções z, com Re z > 0, da equação z = λ − 32, é
4
d) +1+ i; + 1− i
5 5
a) 2. d) 4.
b) 2 2. e) 16. 8 6 8 6
e) +1− i; − 1− i
c) 4 2. 5 5

PROMILITARES.COM.BR 189
NÚMEROS COMPLEXOS

11. (EN 2017) Seja z um número complexo e i a unidade imaginária. 17. (IME 2015) Descreva o lugar geométrico do número complexo z
Determine z de forma que o triângulo de vértices i, z e iz seja equilátero que atende à equação
e assinale a opção correta. arg(z− z1) − arg(z− z2 ) − arg(z− z3 ) =
k π,
−5 πi πi em que z1 é real, z2 e z3 são complexos conjugados com parte
( 3 − 2)e 4
( 3 + 2)e 4
imaginária não nula e k é um número inteiro.
a) z= ou z = −
2 2
Obs.: arg(z) é o argumento do número complexo z.
πi −πi
( 5 + 3)e 6
( 5 − 3)e 6
18. Mostre que as imagens dos complexos se z, w e s são vértices de
b) z= ou z = −
2 2 um triângulo eqüilátero então z2 + w2 + r2 = zw + zr + wr.
−3 πi πi
( 6 + 3)e 4
( 6 − 3)e 4 19. (IME 2012) Seja o número complexo Z = a + bi, com a e b ∈  (real)
c) z= ou z =
2 2
a 3 (1+ ab )
= 3 2

πi 5 πi ei= −1 . Determine o módulo de Z sabendo que  .


b = 3 ( a b –1)
3 2
( 6 − 2)e 4
( 6 + 2)e 4
d) z= ou z =
2 2
11πi πi 20. (IME 2016) Sejam a1 = 1 – i, an = r + si e an + 1 = (r – s) + (r + s)
( 3 + 2)e 6
( 3 − 2)e 6 i (n > 1) termos de uma seqüência. Determine, em função de n, os
e) z= ou z = −
2 2 valores de r e s que tornam esta seqüência uma progressão aritmética,
sabendo que r e s são números reais e i= −1 .
12. (IME 2017) Sejam Z1 e Z2 números complexos tais que Z2 é
imaginário puro e | Z1 − Z2 |=
| Z2 | . Para quaisquer valores de Z1 e EXERCÍCIOS DE
Z2 que atendam a essas condições tem-se que:
a) Im(Z2 ) > 0 COMBATE
b) Im(Z2 ) ≤ 0
c) | Z1 |≤ 2 | Z2 | 01. (ITA 2018) As raízes do polinômio 1+ z + z2 + z3 + z4 + z5 + z6 + z7 ,
quando representadas no plano complexo, formam os vértices de um
d) Re(Z1) ≥ 0
polígono convexo cuja área é
e) Re(Z1) ≤ Im(Z2 )
2 −1 3 2 +1
a) . d) .
2 2
13. (IME 2015) O lugar geométrico no plano complexo de w= z + 1 z,
sendo z número complexo tal que | z |= k e k > 1, é um(a): 2 +1 e) 3 2
b) .
a) segmento de reta d) elipse 2
b) circunferência e) parábola c) 2.
c) hipérbole
02. (ITA 2018) O lugar geométrico das soluções da equação
x2 + bx + 1 =0, quando | b |< 2, b ∈ , é representado no plano
14. (IME 2008) Seja x um número real ou complexo para o qual complexo por
a) dois pontos.
 1  6 1
x+  =1 . O valor de x + 6  =
1 é: b) um segmento de reta.
 x  x 
a) 1 d) 4 c) uma circunferência menos dois pontos.
b) 2 e) 5 d) uma circunferência menos um ponto.
c) 3 e) uma circunferência.

15. (IME 2007) Sejam z e w números complexos tais que: 2(a + bi)
03. (ITA 2017) Considere a equação (a − bi)501 = .
2 2 (a2 + b2 )250 + 1
 w − z =4 + 12i
 O número de pares ordenados (a, b) ∈  2 que satisfazem a equação é
 z − w =2 + 4i
a) 500. d) 503.
onde z e w representam, respectivamente, os números complexos
conjugados de z e w. O valor de z + w é: b) 501. e) 504.
a) 1 – i d) 2 – 2i c) 502.
b) 2 + i e) –2 + 2i
04. (ITA 2016) Considere o polinômio p com coeficientes complexos
c) –1 + 2i
definido por p(z) = z4 + (2 + i)z3 + (2 + i)z2 + (2 + i)z + (1+ i).

1 3 Podemos afirmar que


16. (EN 2014) Sabendo que z é o número complexo z= + i, qual a) nenhuma das raízes de p é real.
2 2
2 3 n
o menor inteiro positivo n, para o qual o produto z ⋅ z ⋅ z  z é um b) não existem raízes de p que sejam complexas conjugadas.
real positivo? c) soma dos módulos de todas as raízes de p é igual a  2 + 2.
a) 1 c) 3 e) 5 d) o produto dos módulos de todas as raízes de p é igual a 2 2.
b) 2 d) 4 e) o módulo de uma das raízes de p é igual a 2.

190 PROMILITARES.COM.BR
NÚMEROS COMPLEXOS

05. (ITA 2016) Considere as afirmações a seguir: 11. (IME 2015) Seja M ⊂  dado por M = {| z2 + az − 1|: z ∈  e | z |= 1},
I. Se z e w são números complexos tais que z − iw =−
1 2i e com a ∈ . Determine o maior elemento de M em função de a.
w − z = 2 + 3i, então z2 + w 2 =−3 + 6i.

II. A soma de todos os números complexos z que satisfazem 12. (IME 2009) Seja z = ρ ⋅ e um número complexo onde ρ e θ
2 | z |2 + z2 = 4 + 2i é igual a zero. são, respectivamente, o módulo e o argumento de z e i é a unidade
imaginária. Sabe-se que= ρ 2a cos θ , onde a é uma constante real
III. Se z= 1− i, então z59= 229 (−1+ i). positiva. Determine a representação de z no plano complexo é
É (são) verdadeira(s)
a) apenas I. d) apenas II e III. 9z2
13. Resolva a equação z2 + –5, onde z pertence ao conjunto
=
( z + 3)
2
b) apenas I e II. e) I, II e III.
dos números complexos.
c) apenas I e III.

2Z
06. (IME 2007) Assinale a opção que indica o módulo do número 14. (IME 2016) Seja Z um número complexo tal que possui
Zi
1 3π
complexo , x ≠ kπ, k ∈ . argumento igual a e log3 (2Z + 2Z + 1) =
2. Determine o número
1+ icot g x 4
complexo Z.
a) | cos x |
b) (1 + sen x) / 2
z3
c) cos2 x 15. (IME 2009) Sabe-se que z1 z2 = e z3 + z4 – z3 – z4 =
0 , sendo
z4
d) | cos sec x |
z1, z2, z3 e z4 números complexos diferentes de zero. Prove que z1 e z2
e) | sen x | são ortogonais.
4
Obs.: números complexos ortogonais são aqueles cujas representações
3
 1− ix   1+ 1 1− i  gráficas são perpendiculares entre si e z é o número complexo
07. (IME 2007) Considere a equação 16  =   −  .
 1+ ix   1− i 1+ i  conjugado de z.
Sendo x um número real, a soma dos quadrados das soluções dessa
equação é
16. (IME 2017) Sejam os complexos z= a + bi e w = 47 + ci, tais que
a) 3 c) 9 e) 15
z3 + w = 0. Determine o valor de a, b e c, sabendo que esses números
b) 6 d) 12 são inteiros e positivos.

08. (IME 2010) Se z é uma solução da equação em  ,


17. (ITA 2013) Para z= 1+ iy, y > 0, determine todos os pares ( a, y ) ,
12
  2 −1 2 + 1  , pode-se afirmar que a > 1, tais que z10 = a. Escreva a e y em função de  Arg z.
2
z − z + z =−  ( )
2 + i  −i 
3  
  3
18. (IME) Considere os números complexos Z1 = senα + icos α e
a) ( )
i z−z < 0 . Z2 = cos a – isen a, onde α é um número real. Mostre que, se Z =
Z1Z2, então –1 ≤ Re (Z) ≤ 1 e –1 ≤ Im (Z) ≤ 1, onde Re (Z) e Im (Z) indicam,
b) i ( z − z) > 0 . respectivamente, as partes real e imaginária de Z.

c) z ∈ [5,6] .
19. (IME 2011) Sejam z=
1 10 + 6i e z2= 4 + 6i , onde i é a unidade
d) z ∈ [6,7] .
 z – z1  π
imaginária, e z um número complexo tal que arg  = ,
1  z – z2  4
e) z+ > 8 .
z
determine o módulo do número complexo ( z − 7 − 9i) .
Obs.: arg ( w ) é o argumento do número complexo w.
09. (EN 2016) O conjunto S formado por todos os números complexos
z que satisfazem a equação | z − 1| = 2 | z + 1| é representado
geometricamente por uma 20. (ITA 2014)
a) reta vertical. a) Determine o valor máximo de | z + i |, sabendo que
5  4 | z − 2 |= 1, z ∈ .
b) circunferência de centro  , 0  e raio .
3  3 b) Se zo ∈  satisfaz (a), determine zo .
c) parábola com vértice na origem e eixo de simetria 0x.
d) elipse de centro (−3, 0) e eixo maior horizontal.
5 4 GABARITO
e) circunferência de centro  − , 0  e raio .
 3  3 EXERCÍCIOS DE TREINAMENTO
01. No plano complexo seja O a origem e C o ponto associado ao
10. (EN 2015) Considere os números complexos da forma complexo i, então a circunferência no enunciado tem centro em C e
 π  passa por O.
=zn p cis (17 − n),  , com n ∈  * . O menor número natural n, tal
 50  π
arg z1 = ⇒ OZ  = π ⇒ OCZ =ˆ 1 COZ
= =ˆ 1O π ⇒ o triângulo
ˆ 1 CZ
que o produto z1 ⋅ z2 ⋅  ⋅ zn é um número real positivo, é igual a 6
1
3 3
a) 8 d) 33 COZ1 é equilátero ⇒ OZ1 = 1
b) 16 e) 50 3 1 3 1
⇒ z1 = + i ⇒ z1 = − i
c) 25 2 2 2 2

PROMILITARES.COM.BR 191
NÚMEROS COMPLEXOS

3 y 06. B
z2 =x + yi ⇒ Re ( z1 ⋅ z2 ) = x + = 0 ⇔ y =− 3x Reescrevendo a equação, obtemos
2 2
Se z2= x + yi está na circunferência de centro i e raio 1, temos: λ + 9 + 2λ + 17= 12 ⇔ λ + 9 + 2( λ + 9) − 1
= 12.
2
 y  Fazendo α = λ + 9, vem
z2 − i =1 ⇒ x + ( y − 1) i =1 ⇒ x 2 + ( y − 1) =1 ⇒  −  + ( y − 1) =1
2 2

 3
2α2 =
− 1 144 − 24α + α2
⇔ 4y 2 − 6y =0⇔y=
3
0 ou y = ⇒ z2 =0 ou z2 =−
3 3
+ i 2α2 − 1
= 12 − α ⇒
2 2 2 12 − α ≥ 0
α2 + 24α − 145 = 0

02. E α ≤ 12
α = 5 ou α = −29
i (1+ i) − 1 i ( 2i) − 1
22 11
    =−1− 2048i ⇔
S22 = = α ≤ 12
(1+ i) − 1 i
⇔ α =5.
c = −1 e d = −2048 ⇒ c + d = −2049
Daí,
03. A λ + 9 = 5 ⇒ λ + 9 = 25
O triângulo T é um triângulo equilátero inscrito numa circunferência ⇔ λ =16,
de raio 1 e o triângulo S é um triângulo equilátero inscrito numa
e a equação em z fica
circunferência de raio 2 3. Ou seja:
z 4 = 16 − 32 ⇒ z = 4 −16.
3  2kπ 
z =1 → z =1⋅ cis   , com k =0,1,2.
 3  Logo, se a + bi = −16, então ρ= | a + bi =| 16 e θ = arg(a + bi) = π.
Portanto, pela Segunda Fórmula de Moivre, segue que as raízes
π
( )  2k 
3
w 3 = 24 3 = 2 3 → w = 2 3 ⋅ cis   , com k = 0, 1, 2. quartas de -16 são
 3 
Sabendo que a razão entre as áreas de figuras semelhantes é igual ao=  π π  3π 3π 
z0 4 16  cos + isen=  , z1
4
16  cos + isen  ,
quadrado da razão da semelhança, pode-se escrever:  4 4  4 4 

A  1 
2
1 4  5π 5π   7π 7π 
= = → B = 12A =z2 16  cos + isen
=  e z3
4
16  cos + isen  .
B  2 3  12  4 4   4 4 
Destas, somente z0 e z3 satisfazem Re z > 0 e, portanto, a soma
pedida é
04. D
2 +i 2 + 2 −i 2 =2 2.
10
 1 3 
 2 + ⋅ i  
 1+ 3 ⋅ i     2 2  10
10 10
  =  cos 60 + isen60°  = cos( −240°) +07.
o
B −240° 10 =
 
 =
 2 
1
+
3
⋅ i     o   i ⋅ sen ( )
10  
 1+1−3 ⋅3i ⋅ i    22 1 −2 3 ⋅i    cos cos 300 + isen300
 60o + isen60°   °
10
10
  =    2 2   =   = cos( −240°) + i ⋅ sen ( −240° )  =
10  1 − 3 ⋅ i   1 3     cos 300o + isen300° 
en60°   2  − ⋅ i  
⋅ sen  10 =
 = =cos([cos120  2 ( −2240
−240°°)++i ⋅i sen120 °]  °=) cos1200
10 
° + i ⋅ sen1200 = − +
1 3
⋅i
en300°  2 2
1 3
= [cos120° + i ⋅ sen120°] = cos1200° + i ⋅ sen1200 = − +
10 
⋅i
2 2
1 3
00 = − + Logo, ⋅ i 2arcsen(Re(z)) + 5arctg(2Im(z)) = 2 ⋅  − π  + 5 ⋅ π = 4 π .
2 2  
 6 3 3

05. C
Seja z= a + bi, com a, b ∈ .
Sabendo que z− | z | =
0, vem

z= | z | ⇔ a + bi= a2 + b2
⇔ (a + bi) = a2 + b2
2

⇔ (a2 − b2 ) + 2abi =a2 + b2


⇒=
b 0 e a ∈ +,

ou seja, queremos calcular a soma das raízes reais e não negativas da


As raízes quartas de 1 são –1, 1, –i e i.
equação. Logo, fazendo z 4 = x, obtemos
Logo
x 2 − 17x + 16 = 0 ⇔ x = 1ou x = 16.
z – 5 + 3i= -1z1= 4 – 3i
Daí, z – 5 + 3i = 1z2 = 6 – 3i
z 4 =1 ⇒ z = 4 1 =1 z – 5 + 3i= -iz3 = 5 – 4i
e z – 5 + 3i= iz 4= 5 – 2i
z 4 = 16 ⇒ z = 4
16 = 2. Localizando os afixos desses complexos, temos que:
Portanto, a soma pedida é 1 + 2 = 3. 42 + ( −5)2 = 41
z3 = 5 – 4i possui o menor argumento e I5 – 4iI =

192 PROMILITARES.COM.BR
NÚMEROS COMPLEXOS

08. A
( −a3b1 + a2b2 ) w 4 + ( −a0a2 + a1b1 − a2b2 ) w2 + a02
=
Sendo O o centro da circunferência, temos: H (iw ) +
( a0 − b2w2 ) + (b1w )2
2

ˆ= 2=π π
AOB
12 6 (a b w
3 2
5
+ ( −a0a3 + a2b1 − a1b2 ) w 3 + ( a0a1 − a0b1 ) w )i
ˆ = 4 ⋅ π = 2π
AOE (a 0 − b2w 2 2
) + (b1w )
2

6 3
( −a3b1 + a2b2 ) w 4
+ ( −a0a2 + a1b1 − a2b2 ) w 2 + a02
f (w) =
Sendo z 4 o número complexo cujo afixo é o ponto E,
(a )
2
+ (b1w )
2
0 − b2w 2
 2π 2π 
z4 =
1⋅  cos + isen  ( −a b + a b )( − w ) + ( −a0a2 + a1b1 − a2b2 )( − w ) + a20
4 2
 3 3  f ( −w ) = 3 1 2 2
Fazendo z = ρ ( cos θ + isenθ ) , (
2 2
a0 − b2 ( − w ) + (b1 ( − w ) )
2
)
4
z =ρ 4
( cos 4θ + isen4θ ) ( −a3b1 + a2b2 ) w 4 + ( −a0a2 + a1b1 − a2b2 ) w2 + a02
f ( − w )= f ( w )=
( a0 − b2w2 ) + (b1w )2
2

Daí,
f ( −w ) = f (w)
 2π 2π 
ρ4 ( cos 4θ + isen4θ ) = 1⋅  cos + isen 
 3 3  Assim, f ( w ) é uma função par.

=ρ 1e cos= 4θ cos
3 10. A
2π  z − 2 = z + 4
De cos 4θ =cos ,
3 
2π  z − 3 + z + 3 =
10
4=θ + 2kπ, =k 0, 1, 2, 3
3 Fazendo z= x + yi, com x e y reais,
π
=
θ (1+ 3k ) ,=
k 0, 1, 2, 3 
( x − 2)2 + y 2 = ( x + 4 )2 + y 2 ( i)
6 

Para k = 0 , 
 ( x − 3) 2
+y +2
( x + 3) 2
+y =2
10 (ii)
π
θ= Da equação (i) ,
6
Para k = 1, ( x − 2)2 + y 2 = ( x + 4 )2 + y 2
2π ( x − 2)2 − ( x + 4 )2 = 0
θ=
3 ( x − 2 + x + 4 ) ⋅ ( x − 2 − x − 4 ) =0
Para k = 2, 2x + 2 =0
7π x = −1
θ=
6 Substituindo x = −1 na equação (ii) ,
Para k = 3,
( −4 )2 + y 2 + 22 + y 2 =
10

θ=
3 Fazendo y 2 + 4 =
a,

Assim, os afixos de 4 E são os pontos B,E, H e K, portanto, o polígono 12 + a + a =


10
regular é o polígono BEHK. 12 + a = 10 − a

( ) = (10 − a )
2 2
12 + a
09. B
12 + a= 100 − 20 a + a
a3s3 + a2s2 + a1s + a0
H (s) = 20 a = 88
b2s2 + b1s + a0
22
a3 (iw ) + a2 (iw ) + a1 ⋅ (iw ) + a0
3 2
a=
H (iw ) = 5
b2 (iw ) + b1 ⋅ (iw ) + a0
2
2
 22 
a= 
a ⋅ i3 ⋅ w 3 + a2 ⋅ i2 ⋅ w 2 + a1 ⋅ w ⋅ i + a0  5 
H (iw ) = 3
b2 ⋅ i2 ⋅ w 2 + b1 ⋅ w ⋅ i + a0
Verificando,
−a3w 3i − a2w 2 + a1wi + a0
H (iw ) =
(a )
2 2
− b2w 2 + b1wi  22   22 
0 12 +   +   = 10
 5   5 
H (iw )
−a3w 3i − a2w 2 + a1wi + a0

(a
0 )
− b2w 2 − b1wi
784 22
(a0 − b2w 2
) + b wi1 (a
0 − b2w 2
) − b wi
1 25
+
5
= 10

28 22
+ = 10
5 5
10 = 10

PROMILITARES.COM.BR 193
NÚMEROS COMPLEXOS

2
 22  | Z1 | → corda da circunferência de diâmetro = 2 | Z2 |
Logo, a =   é solução da equação 12 + a + a =
10
 5  | Z1 |≤ 2 | Z2 |
2
Como =
a y + 4,
784 12. A
y2
= −4
25
Desenvolvendo o somatório:
384
y2 = 15 2k −1 15
π π π π 3π 29π
25
8 6 = k 1=k 1
∑ Img  cos 36 + i ⋅ sen 36=



∑ sen(2k − 1)=
36
sen
36
+ sen
36
+ ... + sen
36
y= ± 15 2k −1 15
5 π π  π π 3π 29π
Então,
= k
∑ 1=

Img  cos
 36
+ i ⋅ sen = 
36  k 1

sen(2k − 1)
= sen
36 36
+ sen
36
+ ... + sen
36

8 6 Porém, pode-se escrever:


8 6
z =−1+ i ou z =−1− i
5 5
sen x + sen(x + r) + ... + sen(x + (n − 1) ⋅ r) =
( 2 )
sen x + (n − 1) ⋅ r ⋅ sen nr
2 ( )
11. D
sen r
2 ( )
Assim, substituindo-se na expressão anterior e utilizando as relações
Sendo z = ( x, y ) , iz = ( −y, x ) . trigonométricas, tem-se:
Os vértices do triângulo equilátero são:
 π π   π 
=A (= 0, 1) , B ( x, y ) e C ( − y, x ) . sen  + 14 ⋅  ⋅ sen 15 ⋅ 
 36 36   36  sen75° ⋅ sen75° sen2 75°
= =
Então,
  
sen π 36 ( ) sen π 36 (
sen π 36 ) ( )
= AC
AB = BC
1− cos150° 1+ 3 3+2
 2  2  2 =
sen2
75° = 2 ⇒ sen
= 2
75°
= AC
AB = BC 2 2 4
3+2 1 3+2
( x − 0 )2 + ( y − 1)2 =−
( y − 0 )2 + ( x − 1)2 =−( y − x ) 2 + ( x − y )2 ⋅ =
4 sen π 36 ( )
4 ⋅ sen π 36 ( )
x 2 + ( y − 1) = y 2 + ( x − 1) = ( x + y ) + ( y − x )
2 2 2 2

Das igualdades acima,


13. D
x 2 + ( y − 1) = y 2 + ( x − 1)
2 2
Sendo z um número complexo de forma z = k ⋅ (cos θ + i ⋅ senθ). Se W
x 2 + y 2 − 2y + 1 = y 2 + x 2 − 2x + 1 também é complexo de forma x + yi, então pode-se escrever:
x=y ( i) 1 1  1  1
x + yi = z + = k ⋅ (cos θ + i ⋅ senθ) + ⋅ (cos θ − i ⋅ senθ) =  k +  ⋅ cos θ + i ⋅  k −  ⋅ senθ
z k  k  k
Mais uma vez das igualdades acima,
1 1  1  1
x + yi =2 z + = k ⋅ (cos θ + i ⋅ senθ) + ⋅ (cos θ − i ⋅ senθ) =  k +  ⋅ cos θ + i ⋅  k −  ⋅ senθ
y 2 + ( x − 1) = ( x + y ) + ( y − x )
2 2
z k  k  k
−2x + 1 = y 2 + x 2 Considerando isso como representação de um par ordenado, então:
x 2 + 2x − 1+ y 2 =0  1  1 
(x, y) →   k +  cos θ ; i  k −  senθ 
Como x = y,  k  k 
x 2 + 2x − 1+ x 2 =0 Considerando k > 1, tais coordenadas identificam uma elipse de
2
2x + 2x − 1 =0  1  1
centro na origem, eixo menor  k −  e eixo maior  k +  .
 k  k
−1± 3
x=
2
14. B
Dessa maneira, 1a solução:
5 πi

z=
−1− 3
+
−1− 3
i=
(
6 + 2  5π 
cis   =
)6+ 2 ( ) e4
x2 − x + 1 = 0 ⇔ x =
1
±
3  π
i ⇔ x = cis  ± 
2 2 2  4  2 2 2  3
ou 1
Como: cisθ + = cisθ + cis( −θ=
) 2cos θ
πi cisθ

z=
−1+ 3
+
−1+ 3
i=
(
6 − 2 π
cis   =
)
6− 2 ( ) e4
x6 =
1  π
+ 6 2cos  6.
=  2 cos= ( 2π ) 2
2 2 2 4 2 x  3
2a solução:
2
12. C  1 1 1
12 ⇔ x 2 + 2 + 2 =
x +  = 1 ⇔ x2 + 2 =
−1
Calculando:  x x x
3
=
Z2 ai, a ∈   2 1
( −1)3 ⇔ x6 + 6 + 3x 2 ⋅ 2 ⋅  x 2 + 2  =
1 1 1
| Z1 − ai | =
|a| x + 2  = −1
 x  x x  x 
1 1
distância de Z1 até ai = | a | x 6 + 6 + 3 ⋅ ( −1) =−1 ⇔ x 6 + 6 =2
x x
Z1 → circunferência do centro em ai e raio | a |

194 PROMILITARES.COM.BR
NÚMEROS COMPLEXOS

15. D Dessa concluir que z está no eixo real (y = 0 → z = x), ou que z é


O sistema é equivalente a uma circunferência (b2 + a2 − x2 − y2 − 2wx + 2wa =
0 ) com centro z1
(w − z) (w + z) = 4 + 12i (w − z) (w + z) = 4 + 12i e raio igual a:
 ⇔ 2 2
 z − w =2 + 4i z − w = 2 − 4i   z2 + z3    z2 − z3 
( w − a)
2
R= + b2 =
 z1 −   +  
4 + 12i   2    2i 
Logo ( −2 + 4i) ( z + w ) = 4 + 12i ⇔ z + w = = 2 − 2i Portanto, o lugar geométrico de z que atende a equação dada é a
−2 + 4i
união da reta real com a circunferência de centro z1, que passa por
z2 e z3 , excluindo-se esses três pontos (z1, z2 e z3 ), pois não existe
16. C arg(0).
Pode-se escrever:
1 3 π 18. z, w e r vértices de um triângulo eqüilátero
z =+ i=
1⋅ cis
2 2 3 ⇓
 n ⋅ (n + 1) π   n ⋅ (n + 1)z⋅2π+ w2 + r2 = zw + zr + wr
n⋅(n +1)
2 3 n 1+ 2+ 3+...+n
z ⋅ z ⋅ z  z =z =z 2 =1⋅ cis  ⋅  → z ⋅ z2 ⋅ z3  zn =1⋅ cis  
 2 3  6 

n ⋅ (n + 1) π   n ⋅ (n + 1) ⋅ π 
⋅  → z ⋅ z2 ⋅ z3  zn =1⋅ cis  
2 3  6  z

Assim, para que o produto seja um número real positivo seu w 120°
α
argumento deve ser um arco congruente a 2π. Assim, o menor valor
positivo de n será:
120°
n ⋅ (n + 1) ⋅ π
= 2π → n ⋅ (n + 1) = 12 → n = 3
6

17. Considerando: r
z1 = w
Pelo desenho, podemos escrever:
z2= a + bi
z = ρ cis α
z3= a − bi
w = ρ cis (α + 120°) = ρ cis α cis 120° = z cis 120°
z= x + yi
r = ρ cis (α + 240°) = ρ cis α cis 240° = z cis 240°
Pode-se escrever, a partir da equação do enunciado:
• z2 + w2 + r2 = z2 + z2 cis2 120° + z2 cis2 240° = z2 (1 + cis 240° + cis 480°)
z − z1 = (x − w ) + yi
• zw + zr + wr = z (z cis 120°) + z (z cis 240°) + (z cis 240°) =
z − z2 = (x − a) + (y − b)i
z2 cis
= 120 2 2
° + z cis 240° + z cis

360 z2 + z2 cis 240° + z2 cis 480°.
° =
z − z3 = (x − a) + (y + b)i cis 480 ° 1

cis(360°+120° )
Fazendo:
Logo, z2 + w2 + r2 = zw + zr + wr.
y
z− z1 = α → t g α =
x−w
19.
y−b
z− z2 = β → t g β = 3 2
x−a a − 3ab = 3
 3
y+b 2
b − 3a b =−3
z− z3 = θ → t g θ =
x−a Z =a + bi ⇒ Z3 =a3 + 3a2bi + 3ab2i2 + b3i 3 =(a3 − 3ab2 ) + (3a2b − b3 )i
3
Substituindo na equação dada, tem-se: ⇒ Z3 =3 + 3i ⇒ Z = 32 + 32 = 18 ⇒ Z =6 18
α − β − θ = kπ, sendo k um número inteiro.
α − β = θ + kπ → t g θ = tg (α − β)
20. Como ak são os termos de uma PA então an +1 − an =
q onde q é a
tg α − tg β razão desta PA. Sendo assim,
t g θ= → t g θ + t g θ ⋅ tg α ⋅ tg β= tg α − tg β
1+ tg α ⋅ tg β
q = (r – s) + (r + s)i – (r + si) = -s + ri e ainda
t g θ ⋅ tg α ⋅ tg β= tg α − tg β − t g θ
an =a1 + (n − 1) . q ⇔ r + si =1− i + (n − 1)(ri − s) ⇔
Substituindo e desenvolvendo:
r =−1 s(n − 1)
t g θ ⋅ tg α ⋅ tg β= tg α − tg β − t g θ ⇔ r + si =1− s(n − 1) + i(r(n − 1) − 1) ⇔ 
s = r(n − 1) − 1
y+b y y−b y y−b y+b
⋅ ⋅ = − − n−2 n
x−a x−w x−a x−w x−a x−a =
O que dá como solução s = e r .
n2 − 2n + 2 n2 − 2n + 2
y ⋅ (y − b )
2 2
y −2y y ⋅ ( y2 − b2 ) y ⋅ ( x − a) − 2y ⋅ ( x − w )
= + → =
( x − w ) ⋅ ( x − a) x − w x − a ( x − w ) ⋅ ( x − a)
2 2
( x − w ) ⋅ ( x − a) EXERCÍCIOS DE COMBATE
y ⋅ ( y2 − b2 ) ⋅ ( x − w ) ⋅ ( x − a) =  y ⋅ ( x − a) − 2y ⋅ ( x − w )  ⋅ ( x − w ) ⋅ ( x − a)
2
01. D 06. E
y ⋅ ( y2 − b2 ) =y ⋅ ( x − a) − 2y ⋅ ( x − w ) ⋅ ( x − a) 02. C 07. B
2

3 2 2 2
y − yb − yx + 2yxa − ya + 2yx − 2yxa − 2ywx + 2ywa =
02 03. D 08. E
y ⋅ (b + a − x − y − 2wx + 2wa) =
2 2 2 2
0 04. E 09. E
05. B 10. A

PROMILITARES.COM.BR 195
NÚMEROS COMPLEXOS
 2Z   2b2 − 8 
arg   =arctg   =−1 → 2b − 8 =−8b ⇒ 2b + 8b − 8 =0 ⇒ b + 4b − 4 =
2 2 2

 Zi   8b 
11. Vamos considerar o complexo z= x + y i. ∆= 42 − 4 ⋅ 1⋅ −4 ⇒ ∆= 32= 22 ⋅ 22 ⋅ 2
Como:
1 −4 ± 4 2 b =−2 + 2 2 (não convém, pois | b |> 2)
z =1⇒ z ⋅ z =1⇒ z = = b ⇒
z 2 b =−2 − 2 2
 1 
z2 + az − 1 = z ⋅  z + a −  = z ⋅ z + a − z =1⋅ x + yi + a − x + yi = a + 2yi = aPortanto
+ ( 2y ) Z será igual a:
2 2

 z Z = 2 + −2 − 2 2 i ( )
+ a − x + yi = a + 2yi = a2 + ( 2y )
2

Dos números complexos da forma z= x + y i de módulo 1, podemos z − ( −z 4 )= z3 − z 4


15. A equação 3 indica que:
admitir z = i para que a + ( 2y ) seja máximo, portanto o maior
2 2
z3 z e de − z ;
1º) equidista de 4 4
elemento de m será dado por:
z
a2 + ( 2 ⋅ 1) =
2
m2 + 4 2º) 3 pertence à mediatriz do segmento de extremos z 4 e − z 4 ;
z
3º) 3 é ortogonal a z 4 ;
z3  π
= r cis  ±  (r ∈  ).
12. Como z = ρ ⋅ eiθ , e=

(cos θ + isenθ) z  2
4º) 4
⇔ z = ρ(cos θ + isenθ) Portanto
=
⇔ z 2acos2 θ + 2asenθ cos θi z z z z 2  π z r  π
z1 z2= 1 2 2= 1 z2 = r cis  ±  ⇔ 1= 2
cis  ±  ⇒
z2 z2  2 z2 z2  2
 1 + cos 2θ 
⇔ z = 2a   + asen2θi ⇔ z = (a + acos 2θ) + (asen2θ)i z1 é ortogonal a z2 .
 2 
Sendo x a parte real e y a parte imaginária de z,
(x − =
a)2 a2 cos2 2θ 16. Calculando:
= y 2
a sen 2θ +
2 2 z3 + w = 0
( a + bi)
3
(x − a)2=
+ y 2 a2 (cos

2
θ
+ sen
2
θ) + 47 + ci = 0 → a3 + 3a2bi − 3ab2 − b3i + 47 + ci = 0
(a − 3ab2 + 47) + ( 3a2b − b3 + c ) i =
1
3
0
(x – a)2 + y2 = a2: circunferência de centro (a,0) e raio a.
Logo,
13. 3a2b − b3 + c = 0
2 2
9z2  3z  3z  z2   z2 
z +
2
+ 5 = 0 ⇔  z −  + 2 ⋅ z ⋅ + 5 = 0 ⇔   + 6.   + 5 = 0 a =1 → b = 4
( z + 3)
2
 z + 3 z+3  z + 3  z + 3a3 − 3ab2 + 47 = 0 → a 3b2 − a2 = 47 → a ∈ , logo : ou
2
( )
z  z2   z2  a = 47 → b ∉ 
+5 = 0 ⇔   + 6.  +5 = 0
3  z + 3  z + 3 3a b − b + c = 0 → 3 ⋅ 1 ⋅ 4 − 1 + c = 0 → c = 52
2 3 2 3

Portanto,
z2 −1 ± i 11 17. Sabendo que z = 1 + iy, com y > 0, segue que z pertence
=−1 ⇔ z2 + z + 3 =0 ⇔ z =
z+3 2 ao primeiro quadrante do plano complexo. Além disso, temos
,
ou  π
y = tg Argz, com Argz ∈  0,  . Daí,
z 2
−5 ± i 35  2
=−5 ⇔ z2 + 5z + 15 =0 ⇔ z = | z | = 1 + y 2
= 1 + tg2
Argz = sec Argz.
z+3 2 .
Logo, o conjunto solução da equação é Como a > 1 e a = z , da Primeira Fórmula de Moivre, vem
10

 −1 + i 11 −1 − i 11 −5 + i 35 −5 − i 35  = | a | |= z10 | sec10 Argz e


S= , , , 
 2 2 2 2  . Arga = Argz10 = 0 ⇔ 10 Argz = 2kπ

⇔ Argz = , k ∈ .
14. Considerando Z como um número complexo qualquer de forma 5
a + bi, pode-se escrever: π 2π
Assim, Argz = ou Argz = e, desse modo,
log3 (2Z + 2Z + 1) =2 → 2Z + 2Z + 1 =32 ⇒ 2 ⋅ (a + bi) + 2 ⋅ (a − bi) =8 ⇒ 2a =8 ⇒ a =4 5 5
a − bi) =8 ⇒ 2a =8 ⇒ a =4  π π 2π 2π 
(a, y) =  sec10 , tg  ou (a, y) =  sec10 , tg  .
Substituindo:  5 5  5 5 
2Z 2 ⋅ (2 + bi) 4 + 2bi b − 2i 4b − 8i + 2b2i + 4b 8b − 8i + 2b2i  8b Portanto,   −8 + 2b 
2
= = ⋅ = = =  2 +
  2 10  ⋅ i
Zi (2 − bi) ⋅ i b + 2i b − 2i b2 + 4 b2 + 4  b + 4 (a, y)  =b(sec + 4 Argz,
 tg Argz).
b − 8i + 2b2i  8b   −8 + 2b2 
=  2 +
  2  ⋅ i
b2 + 4 b + 4   b + 4  18.

Sabendo que o argumento de Z é igual a , conclui-se que Z= Z1 ⋅ Z2= ( senα + icos α )( cos α − isenα ) ⇔
4
−8 + 2b2
> 0, portanto, | b |> 2. = Z ( )
2senα cos α + i cos2 α − sen2α ⇔
b2 + 4 ⇔= Z sen2α + icos 2α ⇔
Substituindo:
Re (= Z ) sen2α ∈ [ −1, 1]
 2Z   2b2 − 8   =0
arg   =arctg   =−1 → 2b − 8 =−8b ⇒ 2b + 8b − 8 =0 ⇒ b + 4b−Im4(=
2 2 2
Z ) cos 2α ∈ [ −1, 1]
 Zi   8b 
8b − 8 =0 ⇒ b
∆=2 +44b
2
−−44⋅ 1=
⋅ 0−4 ⇒ ∆= 32= 22 ⋅ 22 ⋅ 2 ABCD é um quadrado. Dados A = (1, 2) e B = (3, 5) determine C e D
(duas soluções).
−4 ± 4 2 b =−2 + 2 2 (não convém, pois | b |> 2)
=b ⇒
2 b =−2 − 2 2
)

196 PROMILITARES.COM.BR
NÚMEROS COMPLEXOS

20.
Solução: (4 + 2 2, 2 + 1) ou ( −2 2, − 2 − 1).
1, com z= x + yi e x, y ∈ , vem
a) Desde que | z − 2 | =
19. 3 2 | z − 2 | = 1 ⇔ | x − 2 + yi | = 1
1ª SOLUÇÃO: ⇔ (x − 2)2 + y 2 =
1
z − z1 ( x + yi) − (10 + 6i) ( x − 10 ) + ( y − 6 ) i
z = x + yi ⇒ = = = ⇒ (x − 2)2 + y 2 =
12 ,
z − z2 ( x + yi) − ( 4 + 6i) ( x − 4 ) + ( y − 6 ) i
( x − 10 ) + ( y − 6 ) i ( x − 4 ) − ( y − 6 ) i ou seja, os números complexos z que satisfazem | z − 2 | =
1, pertencem
= à circunferência de centro em (2,0) e raio 1.
( x − 4 ) + ( y − 6)
2 2

Lembrando que | z + i | denota a distância do complexo z= x + yi ao


( x − 10 )( x − 4 ) + ( y − 6 ) 6 ( y − 6)
2
complexo w = −i, considere a figura.
+ i
( x − 4 ) + ( y − 6) (x − 4) + ( y − 6)
2 2 2 2

 z – z1  π π  z – z1   z – z1 
arg  = e tg =1 ⇒ Re  =Im  
 z – z 2 4 4  z – z 2   z – z2 
⇒ ( x − 10 )( x − 4 ) + ( y − 6 ) = 6 ( y − 6 ) ⇔ ( x − 7) + ( y − 9 ) = 18
2 2 2

( x − 7) + ( y − 9) =
2 2
⇒ z − 7 − 9i= 18= 3 2
.

2ª SOLUÇÃO:
Sejam z1 = (10,6 ) , z2 = ( 4,6 ) e w = ( 7,9 ) .
Sendo A ⋅ B o produto escalar entre A e B, tem-se:
( z1 − w ) ⋅ ( z2 − w ) =( 3, −3) ⋅ ( −3, −3) =−9 + 9 =0 .
Queremos calcular a medida do segmento AB.
Logo, o ângulo entre ( z1 − w ) e ( z2 − w ) mede 90º.
 z – z1  π = AC + CB e CB = 1, falta calcular AC. Daí,
Como AB
Como arg   = , o ângulo entre ( z1 − z ) e ( z2 − z ) mede 45º,
 z – z2  4 =
AC (2 − 0)2 + (0 − ( −1))=
2
=
5 e, portanto, AB 5 + 1.
portanto podemos concluir que os pontos z1 , z2 e z pertencem a
uma circunferência de centro em w e raio z1 − w =
3 2. b) Os triângulos CBD e CAO são semelhantes por AA. Logo,
Logo, z − 7 − 9i = z − w = 3 2 . CD CB 2 BD CB 1
= ⇔ CD = e = ⇔ BD = .
OC AC 5 OA AC 5
3ª SOLUÇÃO:
No plano complexo, sejam z1 = (10,6 ) e z2 = ( 4,6 ) . 2 1 10 + 2 5 5
Portanto, z0 =2+ + i= + i.
5 5 5 5

ANOTAÇÕES

 z – z1  π π π
arg   = ⇒ arg ( z − z1 ) − arg ( z − z2 ) = ⇒ z2zz
ˆ 1=
 z – z2  4 4 4
(sentido trigonométrico)
Logo, z está no semiarco capaz de 45º sobre o segmento z1z2 ,
representado na figura.
Seja O o centro do semiarco capaz, então z1Oz ˆ = 90° . Daí,
 2

=
z1z2 z= 1z2
e =
6 Oz1 Oz=2 z1z2 sen 45= ° 3 2 .
Logo, o raio do semiarco capaz é 3 2 .
 
   z − z
z2O = z2M + MO = 1 2 + ( 0,1) OM = ( 3,3)
2
  
O =z2 + z2O =( 4,6 ) + ( 3,3) =( 7,9 )

O módulo de ( z − 7 − 9i) é a distância de z ao complexo (7,9) que é o


centro do semiarco capaz.
Assim, z − 7 − 9i é igual ao raio do semiarco capaz, que é 3 2 .

PROMILITARES.COM.BR 197
NÚMEROS COMPLEXOS

ANOTAÇÕES

198 PROMILITARES.COM.BR
POLINÔMIOS

POLINÔMIO OPERAÇÕES COM POLINÔMIOS


Chama-se polinômio inteiro em x, a função P: C → C dada por:
P(x) = aoxn + a1xn−1 +a2xn−2 + ... + an−1x + an ADIÇÃO E SUBTRAÇÃO DE POLINÔMIOS:
onde ao, a1, ..., an são chamados coeficientes e podem ser números A adição e a subtração de polinômios são feitas somando-se ou
reais ou complexos. subtraindo-se os coeficientes dos termos de mesmo grau em todas
as variáveis.
MONÔMIO Exemplos:
É o polinômio que possui um único termo. 1) (4x2 − 3x) − (x2 −4x − 3) = 3x2 + x + 3
Exemplo: p(x) = −3x . 3 2) (x3 − 1) + (x4 − x3 +1) = x4

Atenção!
POLINÔMIO COMPLETO
Frequentemente na subtração de polinômios é preciso eliminar
É aquele que não possui coeficientes nulos. Um polinômio
parênteses. Deve-se atentar para o fato do sinal menos incidir sobre
completo de grau n possui n + 1 termos.
todos os termos entre parênteses de acordo com a propriedade
distributiva da multiplicação.
VALOR NUMÉRICO Exemplo: x2 − 3x + 1 (x2 − 5x + 1) = x2 − 3x + 1 − x2 + 5x − 1 = 2x
O valor numérico de p(x) em a (a ∈ C) é a imagem de a pela
função p, ou seja, P(x) = aoan + a1an−1 +a2an−2 + ... + an−1a + an
Exemplos: MULTIPLICAÇÃO DE POLINÔMIOS
P(x) =2x −5x + 2x −x +1 ⇒ P(2) = 2.2 −5.2 +2.2 −2 +1 =1
4 3 2 4 3 2 Para multiplicar polinômios basta aplicar a distributividade da
multiplicação.
P(x) = x −2ix −x + (3i −2) ⇒ P(i) = i −4i.i −i + (3i −2) = 5i −2
3 2 3 2
Exemplo:
P(x) = x3 +3x2 +2x ⇒ P(−1) = (−1)3 +3.(1)2 +2.(−1) = 0
(x3 +2x −1)(x2 + x + 2) = x5+ x4 + 2x3 + 2x3 + 2x2 +4x − x2 − x − 2 =
Observação x5 + x4 + 4x3 + x2 + 3x − 2

P(1) = ao + a1 +a2 + ... + an−1 + an é a soma dos coeficientes. Note que se o produto de dois polinômios é nulo, pelo menos um
P(0) = an é o termo independente. dos polinômios deve ser nulo.
p⋅q = 0 ⇔ p = 0 ou q = 0

Observação
RAÍZES O grau do produto é a soma dos graus dos fatores.
Chamam-se raízes do polinômio P(x) os valores de x ∈ C tais que
gr(p⋅q) = gr(p) +gr(q)
P(x) = 0.
No exemplo acima, o produto de fatores de graus 3 e 2 teve graus
Um polinômio de grau n possui exatamente n raízes reais ou
2 + 3 = 5.
complexas. Desta forma, a quantidade de raízes reais é no máximo n.
Exemplo:
O polinômio P(x) = x3 + 2x2 − x − 2 é um polinômio completo de DIVISÃO DE POLINÔMIOS
grau 3 e possui três raízes reais: 1, 1 e 2.
Dados dois polinômios P(x) e D(x), de graus p e q, respectivamente,
dividir P(x) por D(x) significa é encontrar dois polinômios Q(x) e R(x),
GRAU denominados quociente e resto, respectivamente, que satisfazem
Dado um polinômio P(x) com pelo menos um termo de coeficiente P(x) = D(x) ⋅ Q(x) + R(x)
não nulo, o grau de P, indicado por gr(P) é o maior dos expoentes da onde o grau de R(x) deve ser menor que o grau de D(x) ou R(x) = 0.
variável x nos termos com coeficientes não nulos. Se gr(P) < gr(D), então Q(x) = 0 e R(x) = P(x).
Se P tem todos os coeficientes nulos, não se define o grau de P. Se gr(P) ≥ gr(D), a divisão pode ser efetuada pelo seguinte
Exemplo: algoritmo denominado Método da Chave.
P(x) = 2x3 −x +1 ⇒ gr(P) = 3 I. Ordenam-se P(x) e D(x) segundo as potências decrescentes
P(x) = 1 +2x −x4 ⇒ gr(P) = 4 de x, inclusive com os termos do dividendo que possuem
coeficiente 0.
P(x) = 3 ⇒ gr(P) = 0
II. Divide-se o primeiro termo de P(x) pelo primeiro termo de
P(x) = 0 ⇒ não se define gr(P)
D(x), obtendo-se o primeiro termo do quociente.

PROMILITARES.COM.BR 197
POLINÔMIOS

III. Multiplica-se D(x) pelo primeiro termo do quociente e subtrai- ⇒ x4 + 2x3 + 3x2 + 4x + 5 ≡ ax4 + bx3 + cx2 +(a + d)x + (b + e)
se o resultado de P(x), obtendo-se o primeiro resto parcial.
a = 1
IV. Com o primeiro resto parcial e o divisor D(x) repetem-se b = 2
as operações, obtendo-se o segundo termo do quociente 
e assim sucessivamente até se encontrar um resto de grau ⇒ c = 3 ⇒ Q(x) = x + 2 e R(x) = 3x2 + 3x + 3
menor que o divisor. a + d = 4 ⇔ d = 3

Exemplo: Calcular (x3 +2x -1) ÷ (x2 + x + 2) b + e = 5 ⇔ e = 3

Exemplo 2: Determine p e q de modo que x3 − 6x2 + px − 1 seja


x3 +0x2 + 2x − 1 x2 + x + 2
divisível por x2 + 3x − q.
−x3 − x2 − 2x x−1 Devemos fazer o resto R(x) = 0 e adotar um quociente Q(x) = ax + b
do primeiro grau. Assim,
− x2 + 0x − 1 x3 − 6x2 + px − 1 = (x2 + 3x − q)⋅(ax + b) ⇔
x3 − 6x2 + px − 1 = ax3 +(b + 3a)x2 + (3b − aq)x − bq
x2 + x + 2
Igualando os coeficientes dos termos de mesmo grau, temos:
x+1 a=1
b + 3a = b + 3⋅1 = −6 ⇔ b = −9
Q(x) = x − 1 e R(x) = x + 1 3b − aq = 3(−9) −1⋅q = p ⇔ p + q = −27
−bq = −1 ⇔ −(−9)q = −1 ⇔ q = −1/9
Observação
p = −27 − (−1/9) = −242/9
O grau do quociente é a diferença dos graus do dividendo e do
divisor. Polinômio identicamente nulo: É aquele que é nulo para qualquer
gr(Q) = gr(P) −gr(D) valor da variável. Um polinômio identicamente nulo tem todos os seus
coeficientes iguais a zero.
No exemplo acima, o quociente tem grau 1 = 3 − 2.

Observação
Se um polinômio de grau n possuir mais de n raízes, então ele é
IDENTIDADE DE POLINÔMIOS identicamente nulo.
Dois polinômios são ditos idênticos quando têm sempre o mesmo
valor qualquer que seja o valor atribuído à variável.

Importante! TEOREMA DE D’ALEMBERT


Dois polinômios idênticos são sempre de mesmo grau e têm todos O resto da divisão de um polinômio P(x) por ax +b, com a ≠ 0, é
os coeficientes iguais. igual a P(−b/a).
Demonstração:
Exemplo 1: Calcular a, b e c de modo que se tenha, ∀x ∈ R, Na divisão de P(x) por ax +b o resto deve ter grau zero. Assim,
ax4 +(b +1)x2 + (2c −1) = x2 +1. podemos dizer que a divisão terá um quociente Q(x) e resto
R(x) = R = constante. Logo,
A igualdade se verifica ∀x ∈ R se os polinômios forem idênticos,
assim: P(x) = (ax +b)⋅Q(x) + R(x) ⇔ P(x) = (ax +b)⋅Q(x) +R
a = 0 Fazendo x = −b/a, teremos

ax4 +(b +1)x2 + (2c −1) ≡ x2 +1 ⇔ b + 1 = 1 ⇔ b = 0 P(a) = (a(−b/a) +b)⋅Q(−b/a) + R ⇔ R = P(−b/a)
2c − 1 = 1 ⇔ c = 2
 Exemplo: Calcule o resto de P(x) = x3 + x2 + x + 1 por x + 1.
1 A B O resto será P(−1) = (−1)3 + (−1)2 + (−1) + 1 = 0. Logo, −1 é raiz
Exemplo 2: Obtenha A e B de forma que = +
para todo x ≠ 0 e x ≠ −1. x(x + 1) x x +1 de P(x).

1 A B O polinômio P(x) é divisível por ax + b, com a ≠ 0, se, e somente


= + ⇔ 1 = A(x+1) + Bx ⇔ 1 = (A+B)x + A se, P(−b/a) = 0.
x(x + 1) x x + 1
Igualando os coeficientes temos: Exemplo: Determine m para que o polinômio P(x) = x3 +2x2 +mx −10
A=1 seja divisível por x −2.
A + B = 1 + B = 0 ⇔ B = −1 P(2) = 23 + 2 ⋅ 22 +m⋅2 −10 = 0 ⇔ m = −3

A divisão de polinômios também pode ser efetuada pelo método


de Descartes ou método dos coeficientes a determinar, que é uma REGRA DE RUFFINI-HORNER
aplicação da identidade de polinômios. Nesse método, parte-se Numa divisão de um polinômio P(x) por x − a, temos:
da expressão P(x) = D(x) ⋅ Q(x) + R(x), onde gr(Q) = gr(P) −gr(D) e
1º. dispomos a e os coeficientes de P(x) inclusive os nulos
gr(R)MAX = gr(D) 1. O quociente e o resto são obtidos então igualando-
se os coeficientes dos dois lados. 2º. o coeficiente do primeiro termo do quociente é igual ao
coeficiente do primeiro termo do dividendo;
Exemplo 1: Dividir P(x) = x4 +2x3 +3x2 +4x +5 e D(x) = x3 +1.
3º. o coeficiente do segundo termo do quociente é igual ao
Supondo Q(x) = ax +b e R(x) = cx2 + dx + e, temos: coeficiente do segundo termo do dividendo mais o produto
P ≡ QD + R do coeficiente do primeiro termo do quociente pelo segundo
termo do binômio tomado com o sinal trocado;
⇒ x4 + 2x3 + 3x2 + 4x + 5 ≡ (ax + b)⋅(x3 + 1) + (cx2 + dx + e)

198 PROMILITARES.COM.BR
POLINÔMIOS

4º. em geral, o coeficiente do termo de ordem p do quociente é 08.


igual ao coeficiente do termo da mesma ordem do dividendo, a) é dado o polinômio do 30grau P(x) = x3 − 3x2 −9x + λ, onde λ é um
mais o produto do coeficiente do termo antecedente do número real não negativo. Determine o menor valor do parâmetro
quociente pelo segundo termo do binômio tomado com o λ de modo que o polinômio p(x) possua 3 raízes reais e distintas.
sinal trocado;
b) Encontre um polinômio de grau mínimo tal que p(i)= −1,
5º. finalmente, obtém-se o resto da divisão multiplicando o p(1) = 2 + i e P(0) = 1.
coeficiente do termo constante do quociente pelo segundo
termo do binômio tomado com o sinal trocado e adicionando
09. Mostre que é raiz tripla do polinômio x5 – 5x4 + 7x3 – 2x2 + 4x – 8.
a esse produto o coeficiente do termo constante do dividendo.
Exemplo: Dividir 2x3 − 5x2 + 3x − 4 por x − 2 10. Calcular as raízes iguais da equação, f(x) ≡ x4 – 8x3 + 22x2 – 24x + 9 = 0
Inicialmente alocar no dispositivo os coeficientes do dividendo e
o segundo termo do binômio com o sinal trocado e então proceder
11. (FUVEST 2012) O polinômio p(x) = x + ax + bx + cx − 8 , em
4 3 2
como acima:
que a, b, c são números reais, tem o número complexo 1 + i como
raiz, bem como duas raízes simétricas.
2 −5 3 −4 a) Determine a, b, c e as raízes de p(x).

2 2 −1 1 −2 b) Subtraia 1 de cada uma das raízes de p(x) e determine todos os


polinômios com coeficientes reais, de menor grau, que possuam
esses novos valores como raízes.
↓ ↓ ↓

2⋅2 + (−5) 2⋅(−1) + 3 2⋅1 + (−4) 12. (FGV 2012) A figura mostra o gráfico da função
f(x) = 2x 3 − 3x 2 − 36x + 81.
Q(x) = 2x2 − x + 1 e R = −2
a) Resolva a equação 2x 3 − 3x 2 − 36x + 81 =
0.

EXERCÍCIOS DE
b) Para que valores de x tem-se f(x) ≤ 0 ?

TREINAMENTO 13. (UFPE 2012) O polinômio x + ax + bx + 19 tem coeficientes a, b


3 2

números inteiros, e suas raízes são inteiras e distintas. Indique |a| + |b|.

01. (FUVEST 2004) O produto de duas das raízes do polinômio 14. (FUVEST 2011) As raízes da equação do terceiro grau
p(x) = 2x3 −mx2 +4x +3 é igual a −1. Determinar x3 -14x2 + kx – 64 = 0
a) o valor de m. são todas reais e formam uma progressão geométrica. Determine
b) as raízes de p. a) as raízes da equação;
b) o valor de k.
02. (UFRJ 1999) Encontre as raízes de x3 + 15x2 + 66x + 80 = 0,
sabendo que são reais e estão em progressão aritmética.
x 2 − 2x + 4 A B C
15. (UFPE 2011) Sabendo que =+ + ,
assinale A + B + 2C. x 3 + x 2 − 2x x x + 2 x − 1
03. (FGV 2002)
a) Sejam a, b e c as raízes da equação x3 − 4x2 + 6x − 1 = 0. Calcule
1 1 1 16. (UFPE 2011) Se o numero complexo 3 + 2i é raiz da equação
o valor da expressão: + + . x³ – 23x + c = 0, com c sendo uma constante real, qual o valor de c?
ab ac bc
b) Resolva a equação x3 − 2x2 −5x + 6 = 0, sabendo que a soma de 17. (UNESP 2010) Uma raiz da equação x3 – (2a – 1)x2 – a(a + 1)x +
duas raízes vale 4. 2a2(a – 1) = 0 é (a – 1). Quais são as outras duas raízes dessa equação?

04. Verificar qual é a multiplicidade da raiz −3 na equação 18. (UERJ 2010) As seis soluções da equação z6 + z3 + 1 = 0 são
x4 + 6x3 + 11x2 + 12x + 18 = 0 e obter as outras raízes. números complexos que possuem módulos iguais e argumentos
distintos.
05. (IME 2004) Considere o polinômio P(x) = x3 + ax + b de coeficientes O argumento θ, em radianos, de uma dessas soluções pertence ao
reais, com b ≠ 0. Sabendo que suas raízes são reais, demonstre que
π 
a < 0. intervalo  , π .
2 
06. (ITA-79) Se a, b, c são raízes da equação x3 – rx + 20 = 0, onde Determine a medida de θ.
r ∈ R, podemos afirmar que o valor de a3 + b3 + c3 é:
6


a) – 60 d) 62 + r3
19. (ITA 2010) Considere o polinômio p(x) = anxn, com
b) 62 + r e) 62 – r
n= 0
c) 62 + r2 coeficientes reais, sendo a0 ≠ 0 e a6 = 1. Sabe-se que se r é raiz de p, – r
também é raiz de p. Analise a veracidade ou falsidade das afirmações:
07. (ITA-81) Considere a equação x3 + px2 + qx + r = 0, de coeficientes I. Se r1 e r2, r1 ≠ r2 , são raízes reais é r3 e raiz não real de p, então
reais, cujas raízes estão em progressão geométrica. Qual das relações r3 é imaginário puro.
é verdadeira?
II. Se r é raiz dupla de p, então r é real ou imaginário puro.
a) p2 = rq d) p3 = rq3
III. a0 < 0.
b) 2p + r = q e) q3 = rp3
c) 3p = r q
2 2

PROMILITARES.COM.BR 199
POLINÔMIOS

20. (IME) Prove que o polinômio 08. (ITA 2008) Um polinômio P é dado pelo produto de 5 polinômios
cujos graus formam uma progressão geométrica. Se o polinômio de
x 9999 + x 8888 + x 7777 +  + x 2222 + x1111 + 1 é divisível por
menor grau tem grau igual a 2 e o grau de P é 62, então o de maior
x9 + x8 + x7 +  + x2 + x + 1 . grau tem grau igual a
a) 30 c) 34 e) 38
EXERCÍCIOS DE

COMBATE
b) 32 d) 36

09. (IME 2018) Seja P(x) o polinômio de menor grau que passa pelos
pontos A(2, − 4 + 3 3), B(1, 3 2 − 2),
C( 2, 3) e D( 3, 2). O resto da divisão de P(x) por (x – 3) é:
01. (EN 2017) Seja P(x) = x 6 + bx5 + cx 4 + dx 3 + ex 2 + fx + g um
polinômio de coeficientes inteiros e que P( 2 + 3 3) = 0. O polinômio a) 8 3 −5 2 −6
R(x) é o resto da divisão de P(x) por x³ – 3x – 1. Determine a soma dos 6 3 − 4 2 −1
b)
coeficientes de R(x) e assinale a opção correta.
a) –51 b) –52 c) –53 d) –54 e) –55 c) 9 3−8 2−2
d) 4 3 − 10 2 − 3
2(k 2 + 1)2 25k 2 e) 4 3 − 2 −6
02. (EN 2015) Em uma P.G., a4 = e a1 = , onde
5k 4(k 2 + 1)
k ∈ + . Para o valor médio M de k, no intervalo onde a P.G. é decrescente,
*
10. (IME 2017) O polinômio P(x) =x 3 − bx 2 + 80x − c possui três raízes
5 5 inteiras positivas distintas. Sabe-se que duas das raízes do polinômio
o resto da divisão do polinômio P(x) = x5 − x 4 + 25x 2 − 10 pelo
4 2 são divisoras de 80 e que o produto dos divisores positivos de c
 15 
binômio  Mx −  é menores do que c é c². Qual é o valor de b?
 8 
a) 11 c) 17 e) 29
1039 1103 1103
a) c) e) b) 13 d) 23
32 32 16
1231 1487
b) d) 11. (ITA 2018) Seja p(x) um polinômio não nulo. Se x − 4x + 5x − 2
3 2
16 32
e x 3 − 5x 2 + 8x − 4 são divisores de p(x), determine o menor grau
03. (EN 2014) Considere P(x) = (m − 4 )(m + 4 )x + x + kx + 1 um
2 5 2 possível de p(x).
polinômio na variável real x, em que m e k são constantes reais. Quais
os valores das constantes m e k para que P(x) não admita raiz real? 12. (ITA 2017) Considere o polinômio
a) m = 4 e 2< k < 2 d) m = 4 e |k| > 2 p(x) = x 4 − (1 + 2 3)x 3 + (3 + 2 3)x 2 − (1 + 4 3)x + 2.
b) m = – 4 e k > 2 e) m = –2 e k > –2 a) Determine os números reais a e b tais que
c) m = –2 e –2 < k < 2 p(x) = (x 2 + ax + 1)(x 2 + bx + 2).

04. (EN 2013) Sejam F(x) = x³ + ax + b e G(x) = 2x² + 2x – 6 dois b) Determine as raízes de p(x).
polinômios na variável real x, com a e b números reais. Qual valor de
F(x) 13. (ITA 2015) Seja S o conjunto de todos os polinômios de grau 4 que
(a + b) para que a divisão seja exata? têm três dos seus coeficientes iguais a 2 e os outros dois iguais a 1.
G(x)
a) –2 b) –1 c) 0 d) 1 e) 2 a) Determine o número de elementos de S.
b) Determine o subconjunto de S formado pelos polinômios que têm
05. (ITA 2011) Se 1 é uma raiz de multiplicidade 2 da equação –1 como uma de suas raízes.
x4 + x2 + ax + b = 0, com a, b ∈ IR, então a2 – b3 é igual a
a) – 64. c) – 28. e) 27. 14. (IME 2018) Resolva a inequação abaixo, onde x é uma variável real.
b) – 36. d) 18. 2 | x 3 | −6x 2 + 3 | x | +2 < 0

15

06. (ITA 2010) Considere o polinômio p(x) = ∑


n= 0
an xn com 15. (ITA 2016) Sejam a, b, c números reais com a ≠ 0.
1
a) Mostre que a mudança = x+
z transforma a equação
coeficientes a0 = – 1 e an = 1 + ian – l, n = 1, 2, ..., 15. Das afirmações: x
I. p(– 1) ∉  , ax + bx + cx + bx + a =
4 3 2
0 numa equação de segundo grau.
b) Determine todas as raízes da equação x + 3x − 2x + 3x + 1 =
4 3 2
II. |p(x)| ≤ 4 (3 + 2 + 5 ), ∀x ∈ [– 1, 1], 0.
III. a8 = a4,
16. (ITA 2016) Determine o termo constante do resto da divisão do
é (são) verdadeira(s) apenas
polinômio (1 + x + x ) por (1 + x)³.
2 40

a) I. b) II. c) III. d) I e II. e) II e III.


17. (ITA 2015) Considere o polinômio p dado por
07. (ITA 2008) Considere o polinômio p(x) = a5x5 + a4x4 + a3x3 + a2x2 - a1,
= 18z3 + βz2 − 7z − β em que β é um número real.
p(z)
em que uma das raízes é x = -1. Sabendo-se que a1, a2, a3, a4 e a5 são
reais e formam, nesta ordem, uma progressão aritmética com a4 = 1/2, a) Determine todos os valores de β sabendo-se que p tem uma raiz
então p(-2) é igual a de módulo igual a 1 e parte imaginária não nula.
a) - 25 b) - 27 c) - 36 d) - 39 e) - 40 b) Para cada um dos valores de β obtidos no item anterior, determine
todas as raízes do polinômio p.

200 PROMILITARES.COM.BR
POLINÔMIOS

18. (FUVEST 2014) Os coeficientes a, b e c do polinômio ⇒ P(x) = (x +3)2⋅(x2 +2) ⇒ −3 tem multiplicidade 2
p(x) = x 3 + ax 2 + bx + c são reais. Sabendo que –1 e 1 + αi com α >
0, são raízes da equação p(x) = 0 e que o resto da divisão de p(x) por 05. 1ª Solução:
(x – 1) é 8, determine
P(x) não tem raiz tripla, pois como a soma das raízes é nula, a raiz tripla
a) o valor de α; seria 0, contradizendo b ≠ 0.
b) o quociente de p(x) por (x + 1). Logo P(x) tem pelo menos duas raízes reais distintas.
i é a unidade imaginária, i² = –1.
Se a = 0, P(x) = x3 + b tem uma única raiz real, 3
−b (as outras são
19. (IME 2014) O polinômio P(x) =x − 3x + 10x − 30x + 81x − 243
5 4 3 2  1 3 
3
−b  − ± i ), logo a ≠ 0.
possui raízes complexas simétricas e uma raiz com valor igual ao  2 2 
módulo das raízes complexas. Determine todas as raízes do polinômio.
Se a > 0, a derivada P’(x) = 3x2 + a é positiva para todo x ∈ R, o
20. (FUVEST 2013) Considere o polinômio p ( x=
) x + 1. 4 que significa que a função P : R → R é estritamente crescente,
contradizendo o fato de P(x) ter pelo menos duas raízes reais distintas.
a) Ache todas as raízes complexas de p(x).
Logo, como a ∈ R, devemos ter a < 0.
b) Escreva p(x) como produto de dois polinômios de segundo grau,
com coeficientes reais.
2ª Solução:
Suponha r1, r2 e r3 as raízes de P(x). Pelas relações de Girard:
GABARITO
r1 . r2 . r3 = -b ≠ 0 (I)
EXERCÍCIOS DE TREINAMENTO r1 + r2 + r3 = 0 (II)
01. raízes: a, b, c b ⋅ c = −1 r1r2 + r1r3 + r2r3 = a (III)
Pelas relações de Girard:
A equação (I) nos dá r12 + r22 + r32 > 0.
⇒ a⋅b⋅c = −3/2 ⇒ a⋅(−1) = −3/2 ⇒ a = 3/2
Como (r1 + r2 + r3)2 = r12 + r22 + r32 + 2(r1r2 + r2r3 + r1r3), então:
3
ab + ac + bc = 2 ⇒ a(b +c) +bc = 2 ⇒ (b + c) − 1 =2 ⇒b+c=2
2 (r12 + r22 + r32 )
3 m r1r2 + r1r3 + r2r3 = a = − < 0.
a +b +c = +2= ⇒m=7 2
2 2
06. Substituindo a,b e c na equação acima e somando obtemos:
b + c = 2
 ⇒ x2 −2x −1 = 0 ⇒ x = 1 ± 2 a3 + b3 + c3 - r(a + b + c) + 60 = 0 ⇒ a3 + b3 + c3 = -60
bc = −1
S = {3/2, 1 + 2,1 − 2 }
07.

02. raízes: a −r, a, a + r a + b + c =−p (I)


ab + ac + bc = q (II)
soma = 3a = −15 ⇒ a = −5 
abc = −r (III)
produto = (−5 −r)⋅(−5)⋅(−5 +r) = −80 ⇒ r2 −25 = −16 ⇒ r = ±3 
b2 = ac (IV)
raízes: −8, −5, −3 ⇒ S = {−8, −5, −3}
por III e IV b3 = −r
03. a) Pelas Relações de Girard: a +b +c = 4 e abc = 1 por II e IV b(a+b+c)=q ⇔ -bp=q ⇔ -b3.p3 =q3 ⇔ r.p3 =q3
1 1 1 c+b+a 4
+ + = = =4
ab ac bc abc 1

08. Considere o polinômio t(x)=x3-3x2-9x


3
2
1± 5 }
raízes {0, ( )
b) raízes: a, b,c b +c = 4
Logo a derivada de t(x) é t’(x)=3x −6x −9 raízes {−1,3}
2

Pelas relações de Girard: a +b +c = 2 ⇒ a +4 = 2 ⇒ a = −2


t(−1) = 5 e t(3) = -27
gráfico de t(x)
1 −2 −5 6

−2 1 −4 3 0

⇒ (x +2)⋅(x2 −4x +3) = 0 ⇒ raízes: −2, 1, 3 ⇒ S = {−2, 1, 3}

04.

1 6 11 12 18

−3 1 3 2 6 0

−3 1 0 2 0

−3 1 −3 11

PROMILITARES.COM.BR 201
POLINÔMIOS

como p(x) = t(x) + λ e λ ≠ 0 → o gráfico de t(x) é exatamente o mesmo b) De acordo com o gráfico e a raiz encontrada no item (a), temos:
gráfico de T(x) deslocado de λ para cima .
logo se λ < 27 o polinômio terá 3 raízes distintas
se λ = 27 o polinômio terá 1 raiz simples e uma raiz dupla igual a 3 .
logo o menor valor de λ é igual a zero.
b) p(x) = a(x − i)⋅(x− (2+ i))⋅(x − 0).

09. p(2) = p’(2) = p”(2) = 0 e p’’’(2) ≠ 0.

10. Temos f’(x) = 5x4 + 4x3 – 15x2 – 2x + 8 e o m.d.c. entre f(x) e f’ (x)
é ∆ = x3 – 3x + 2. Apliquemos novamente o processo, determinando
o m.d.c. entre os polinômios ∆ e ∆’ = 3x2 – 3. Achamos x – 1, como
resultado. Logo, pondo x – 1 = 0, vê-se que a unidade é raiz dupla de
9
∆ = x3 – 3x + 2 = 0 cuja outra raiz simples é –2. Portanto, a unidade f(x) ≤ 0 para x ∈  / x ≤ − .
é raiz tripla e –2 raiz dupla para a equação considerada. Suas raízes 2
serão pois: 1, 1, 1, –2, –2.
13. Pelo Teorema das Raízes Racionais, segue que as raízes racionais
do polinômio pertencem ao conjunto { ±1, ± 19}.
11. a) Como os coeficientes são reais, as raízes complexas aparecem
com suas respectivas conjugadas, então (1 + i), (1 - i), r e – r são raízes Pelas Relações de Girard, temos que o produto das raízes do polinômio
de P(x) 19
é dado por − = −19.
Utilizando, agora, a relação do produto das raízes, temos: 1

−8 Dessa forma, como as raízes são inteiras e distintas, temos que essas
(1 + i) ⋅ (1 − i) ⋅ r ⋅ ( −r) = ⇔ −2.r 2 = −8 ⇔ r = ±2
1 raízes só podem ser –1, 1 e 19. Logo, pelas Relações de Girard, vem
Portanto, as raízes de p(x) são (1 + i), (1 - i), 2 e -2 a
−1 + 1 + 19 =− ⇔ a =−19
1
Escrevendo o polinômio na forma fatorada, temos:
e
P ( x )= 1.( x − (1 + i) ) .(x − (1 − i) . ( x − 2) . ( x + 2)
b
P ( x ) = x 4 − 2x 3 − 2x 2 + 8x − 8 ( −1) ⋅ 1 + ( −1) ⋅ 19 + 1⋅ 19 = ⇔ b =−1.
1

Logo, a =
−2, c =
−2 e c=
8. Portanto, | a | + | b | =| −19 | + | 1| =20.

b) Subtraindo 1 de cada uma das raízes, temos; 14.


1 − i − 1 =−i −( −64)
Multiplicando as raízes, encontramos x3 = ⇔x=4
1 + i − 1 =i 1
2 − 1= 1 b) Fazendo x = 4, encontramos o valor de k.
−2 − 1 =−3
43 – 14.42 + 4.k – 64 = 0 ⇔ k = 56
Portanto,
) k.( x − ( −i)) .( x − i) .( x – 1) .( x − ( −3))
q ( x= a) Considerando k = 56 e aplicando Briott Ruffini:

( )
q ( x ) = k. x 2 + 1 . ( x − 1) . ( x + 3)

Para k diferente de zero.

12.
a) No gráfico percebe-se que 3 é raiz com multiplicidade par, o que
nos leva a concluir que 3 é raiz dupla, já que a equação é de terceiro
grau. Resolvendo, agora a equação x2 – 10x + 16 = 0 as outras raízes são
Podemos, então, determinar a terceira raiz, aplicando o dispositivo de 2 e 8.
Briot-Ruffini: Logo, as raízes são 2, 4 e 8.

15. Temos que


x 2 − 2x + 4 A B C
=+ + ⇔
x 3 + x 2 − 2x x x + 2 x − 1
x 2 − 2x + 4 A(x + 2)(x − 1) + Bx(x − 1) + Cx(x + 2)
= ⇔
x(x + 2)(x − 1) x(x + 2)(x − 1)
Concluímos que 2x 3 − 3x 2 − 36x + 81 = (x − 3)2 (2x + 9) = 0. Logo, a x 2 − 2x + 4= A(x + 2)(x − 1) + Bx(x − 1) + Cx(x + 2)
terceira raiz será x = –9/2
Fazendo x = –2, obtemos
S = {–9/2, 3}.

202 PROMILITARES.COM.BR
POLINÔMIOS

( −2)2 − 2 ⋅ ( −2) + 4 = B ⋅ ( −2) ⋅ ( −2 − 1) ⇔ 6B = 12 ⇔ B = 2. π 


θ ∈  , π ,
2 
Para x = 1, encontramos 12 − 2 ⋅ 1 + 4 = C ⋅ 1⋅ (1 + 2) ⇔ 3C = 3 ⇔ C = 1. 8π
=θ arg(w
= 1)
Finalmente, para x = 0, vem 9

02 − 2 ⋅ 0 + 4 = A ⋅ (0 + 2) ⋅ (0 − 1) ⇔ −2A = 4 ⇔ A = −2.
19. I. verdadeira
Portanto, A + B + 2C =−2 + 2 + 2 ⋅ 1 =2.
Se r1 é raiz então – r1 também é raiz.
Se r2 é raiz então – r2 também é raiz
16. Se 3 + 2i é raiz da equação x 3 − 23x + c =0 então 3 – 2i também Se r3 é raiz não real então o conjugado de r3 também será raiz e igual
é raiz. Logo, o trinômio (x − 3 − 2i)(x − 3 + 2i) = x 2 − 6x + 13 é divisor a - r3 , portanto r3 é imaginário puro.

de x 3 − 23x + c. II. Falsa. Supondo 3 + i como raiz dupla , teremos


S = {3+i, 3+i, 3- i, 3- i, -3 -i , -3 + i}
Aplicando o método da chave, obtemos
III. Falsa
x 3 − 23x + c x 2 − 6x + 13
− x 3 + 6x 2 − 13x x + 6 a0
No exemplo acima o produto das raízes é = 100 logo ao = 100 > 0
1
6x − 36x + c
2

−6x 2 + 36x − 78 20. B = x


9999
+ x 8888 + x 7777 +  + x 2222 + x1111 + 1
c − 78
A = x + x + x ++ x + x +1
9 8 7 2

Portanto, como o resto deve ser zero, vem que c − 78 = 0 ⇔ c = 78. B – A = (x9999 −x9) +(x8888 −x8) +(x7777 −x7) +... +(x2222 −x2) +(x1111 −x)
B −A = x9[(x10)999 −1] +x8[(x10)888 −1] + ... + x2[(x10)222 −1] +x[(x10)111 −1]
17. Dividindo o primeiro membro da equação por[ x –(a-1)] temos: Cada um dos termos em colchete é divisível por x10 −1 = (x −1)⋅(
a–1 1 -2a + 1 -a2 –a 2.a3 – 2.a2 x9 + x8 + x7 +  + x2 + x + 1 )
1 -a -2.a2
0 ⇒ B −A = A⋅Q(x) ⇒ B = A⋅(Q(x) +1)
x .-a.x – 2.a = 0
2 2 Logo, B é divisível por A

a ± 9a2 x = 2a ou
=
Resolvendo temos: x ⇔ EXERCÍCIOS DE COMBATE
2 x = −a
Resposta: -a e 2a 01. B 05. C 09. A
02. ANULADA 06. E 10. E
18. Substitui-se z3 por y na equação z6 + z3 + 1 = 0: 03. A 07. A 11. 4
04. B 08. B
−1 ± 1 − 4x1x1 −1 ± −3
2
y2 + y + 1 = 0 ⇒ y = = ⇒ a = −2 3
2x1 2 12. a)
−1 3 −1 3 b = −1
⇒ y1 = + i ou y 2 = − i
2 2 2 2
=
x 3± 2
Para determinar as raízes cúbicas de um número complexo b) 1± 7 i
w = ρ(cosθ + isenθ), usa-se a seguinte relação: x=
2
 θ 2kπ    θ 2kπ  13. a) 10
3 ρ cos 
wk =   +  + ixsen  +  ,k ∈ {0,1,2} .
 3 3   3 3  b)

Portanto, as raízes cúbicas do número complexo {x 4


}
+ 2x 3 + 1x 2 + 2x + 2, x 4 + 2x 3 + 2x 2 + 2x + 1, 2x 4 + 2x 3 + 1x 2 + 2x + 1
1 3  2π 2π 
y1= − +=i 1 cos + isen  são determinadas por:
2 2  3 3  1+ 3 1+ 3 
14. S = x ∈  : −2 < x < − ou < x < 2 .
  2π   2π     8π   8π    2 2 
w0 =
cos  9  + ixsen  9   ,w1 = cos  9  + ixsen  9   ,
   
  14 π   14 π   15. x =−2 + 3 ou x =−2 − 3
=w 2 cos  + ixsen 
  9   9   1+ 3 ⋅ i 1− 3 ⋅ i
=x = ou x .
Analogamente, as raízes cúbicas do número complexo 2 2

1 3  4π 4π 
y2 =− − i=1 cos + isen  são determinadas por: 16. 781
2 2  3 3
  4π   4π     10π   10π   17. a) 15 e –15
w3 =
cos  9  + ixsen  9   ,w 4 =
cos  9  + ixsen  9   ,
   
5 5 + i ⋅ 11
  16 π   16 π   b) z =− ,z = e
=w5 cos  + ixsen  6 6
  9   9  
5 − i ⋅ 11
z= .
Como 6

PROMILITARES.COM.BR 203
POLINÔMIOS

18. a) α = 2
p(x) (x + 1)(x 2 − 2x + 5)
b) = = x 2 − 2x + 5.
x +1 x +1

19. 3, 7 + 2 ⋅ i, 7 − 2 ⋅ i,

− 7 + 2 ⋅ i e − 7 − 2 ⋅ i.

2 + 2i 2 − 2i
20. a) x 2 − 2 ⋅ x + 1=
=0, temos x = ou x
2 2

− 2 + 2i − 2 − 2i
  x 2 − 2 ⋅ x +=
1=0, temos x = ou x
2 2

b) P(x) = (x 2 + 1) − ( 2 ⋅ x)2
P(x)= (x 2 − 2 ⋅ x + 1)(x 2 + 2 ⋅ x + 1)

ANOTAÇÕES

204 PROMILITARES.COM.BR
GEOMETRIA ANALÍTICA I

EQUAÇÃO VETORIAL Se fizermos x = 0 na equação reduzida da reta obteremos y = p.


 Logo, o ponto (O, p) é o ponto de interseção de (r) com o eixo dos y e
Suponhamos que dados um ponto P0 do plano e um vetor u seja |p| é a distância deste ponto à origem. O parâmetro p é denominado
pedido achar a equação da reta (r) que passa por P0 e tenha a direção coeficiente linear da reta (r).

de u . Para
 resolver o problema seja P um ponto qualquer de (r). O
vetor v − v 0 deve ter a direção de u , pois queremos que (r) tenha
 
a direção de u . Decorre daí que para cada v = OP da reta existe um
VETOR NORMAL À UMA RETA
      Suponhamos agora, uma reta (r) que passa pelo ponto P = (x1,y1)
número real α tal que v − v 0 = αu isto é v − v 0 = αu , a ∈  chamada 
a equação vetorial da reta. possui direção dada pelo vetor u e está representada pela sua equação
geral ax + by + c = 0. Se Q = (x2,y2) é outro ponto qualquer da reta (r),
 
então claramente PQ = αu com PQ = Q − P = (x 2 − x1, y 2 − y1) . Como
EQUAÇÕES PARAMÉTRICAS P e Q são pontos de (r), temos também ax1 + by1 + c = 0 e ax2 + by2 +
  c = 0. Subtraindo membro a membro a primeira da segunda temos-se:
= Se u a',b') e v ( x, y )
(= obteremos, a partir da equação vetorial a(x2 – x1) + b(y2 – y1) = 0.
que: A expressão acima pode ser identificada com o produto escalar
 
x= x 0 + αa' do vetor PQ =( x 2 − x1, y 2 − y1) com o vetor n = (a,b) . Fazendo esta
( x, y ) ( x 0 , y 0 ) + α ( a',b') ou 
= α ∈ identificação obtemos:
y= y 0 + αb'  
n ⋅ PQ= a ( x 2 − x1) + b ( y 2 − y1)= 0
que são as equações paramétricas da reta sendo α o parâmetro.
qualquer que seja o ponto Q = (x2,y2) da reta. Concluímos daí que se
a reta estiver representada pela sua equação geral ax + by + c = 0, os
EQUAÇÃO SIMÉTRICA números a e b são as componentes de um vetor n perpendicular à

Se a’ ≠ 0 e b’ ≠ 0 as equações paramétricas podem ser escritas direção de (r), isto é perpendicular ao vetor αn para todo a ∈ .
na forma
x − x0 y − y0
a'
=
b'
PARALELISMO E
PERPENDICULARISMO
que é a chamada equação simétrica da reta.
Suponhamos agora duas retas (r) e (s) representadas por suas
equações gerais ax + by + c= 0 e a’x +b’y + c’ = 0, coeficientes angulares
EQUAÇÃO GERAL m e m’ e vetores normais n = (a,b) e n’ = (a’,b’) respectivamente. Logo,
Da equação simétrica obtém-se b’(x – x0) = a’(y – y0) ou b’x – a’y +   a b
I. Se (r) || (s) então n || n’ e daí = ou ainda m = m’.
(a’y0 – b’x0) = 0, fazendo b’ = a; –a’ = b e c = a’y0 – b’x0 resulta que ax a' b'
  1
+ by + c = 0 denominada equação geral da reta (r). II. Se (r) ⊥ (s) então n ⊥ n’ e daí aa’ + bb’ = 0 ou ainda m = − .
m'

EQUAÇÃO REDUZIDA DISTÂNCIA DE PONTO À RETA


a c
Supondo b ≠ 0 podemos escrever y = − x− e pondo Sejam dados a reta (r), representada pela sua equação geral,
b b
a c ax + by + c = 0 e P0 = (x0, y0) determinemos agora a distância d do
m= − e p= − segue-se que y = mx + p conhecida como ponto à reta. Pelo que vimos anteriormente, a direção da normal a (r)
b b 
n 
equação reduzida da reta. é dada por n =  onde n = (a,b). Considere um ponto P qualquer da
n
Os parâmetros m e p que aparecem na equação da reduzida da reta  
possuem um significado geométrico simples. De fato, suponhamos ( )
reta (r) e seja θ o ângulo entre a direção de u − v e n . A distância d
que a reta (r) passa pelos pontos P e Q com P = (x1,y1) e Q = (x2,y2) procurada é expressa por:
     
   
e seja R = (x2,y1). Logo, w − u= ( x 2 − x1,0) e v − w= ( 0, y 2 − y1) . ( ) ( )
d= u − v cos θ= u − v cos θ ou seja d = u − v ⋅ n ( )
Como a reta (r) passa pelos pontos P e Q tem-se que y1 = mx1 + p e Em coordenadas a fórmula acima se torna
y 2 − y1 ax 0 + by 0 + c
y2 = mx2 + p e daí, y2 – y1 = m(x2 – x1) ou seja m = que é igual à d=
x 2 − x1
a2 + b2
tangente do ângulo θ que a reta (r) faz com o semi-eixo positivo dos x.

O parâmetro m é denominado coeficiente angular da reta (r) e é uma


medida da inclinação da reta em relação ao eixo dos x.

PROMILITARES.COM.BR 205
GEOMETRIA ANALÍTICA I

CONDIÇÃO DE ALINHAMENTO DE 3 ÁREA DE UM TRIÂNGULO


PONTOS Dado um triângulo ABC, de vértices A(x1,y1), B(x2,y2), C(x3,y3),
desejamos expressar sua área em função das coordenadas de A, B e C.
Sejam os pontos P1(x1, y1) ≠ P2(x2, y2) e P3(x3, y3).
Seja r a reta suporte do seguimento BC. A equação de r é dada por:
Sabemos que os pontos P1 e P2 determinam a reta (r) da equação
x − x1 y − y1
=
x 2 − x1 y 2 − y1
Para P3 pertencer à reta (r) é necessário e suficiente que suas
coordenadas satisfaçam sua equação ⇒
x1 y1 1
x −x y 3 − y1
⇒ 3 1
= =
ou x2 y2 1 0
x 2 − x1 y 2 − y1
x3 y3 1
x1 x 2 x3 x1
ou =0
y1 y 2 y3 y1

POSIÇÕES RELATIVAS DE DUAS RETAS


Duas retas (r1) e (r2) do plano R2 serão:
a) paralelas distintas: (r1) ∩ (r2) = 0
b) paralelas coincidentes: (r1) ∩ (r2) = (r1) = (r2)
x y 1
c) concorrentes: (r1) ∩ (r2) = {P}
x2 y2 1 = 0 (1)
x3 y3 1
ÂNGULO DE DUAS RETAS | ax1 + by1 + c |
A distância de A à reta r é: d(A, r) =
Sejam as retas (r1) A1x + B1y + C1 = 0 e (r2) A2x + B2y + C2 = 0 a2 + b2
 
referidas num sistema {0, i , j } , de vetores diretores onde, de acordo com (1), a = y2 – y3 ; b = x3 – x2 e c = x2y3 – x3y2.
 
V1 = ( −B1, A1) e V2 = ( −B2 , A 2 ) e θ o ângulo agudo entre elas. x1 y1 1
x2 y2 1
x3 y3 1
=
Assim, d(A, r) =
(x 3 − x 2 )2 + (y 2 − y 3 )2
1
a área do triângulo ABC é igual a (ABC) = d(A, r) . d(B, C)
2
1  x2 y 2 x1 y1 x1 y1 
= desenvolvendo obtemos =  − + =

x1 y1 2  x3 y3 x3 y3 x2 y2 

1 2x y 2
(ABC) = logo a área do triângulo ABC é igual a metade
    2 x3 y3
Do produto
= V1 × V2 | V1 || V2 | cos θ ⇒
  x4 y4
V1 V ( −B1)( −B2 ) + A1A 2 do valor absoluto do determinante.
⇒ cos=θ  × 2 ⇒ cos= θ
| V1 | | V2 | A12 + B12 A 22 + B22 O “determinante” acima e pode ser calculado da seguinte
A1A 2 + B1B2 maneira:
⇒ cos θ Fórmula que nos permite calcular o
A12 + B12 A 22 + B22
ângulo θ ou seu suplemento.
O ângulo θ pode ser calculado através do produto vetorial dos 2
vetores:
 
| V1 ^ V2 | A1B2 − A 2B1
= sen θ   ⇒ =
sen θ ou
| V1 || V2 | A12 + B12 A 22 + B22
sen θ A1B2 − A 2B1
=
tg θ = ou, dividindo numerador e
cos θ A1A 2 + B1B2
denominador por B1B2,
A1 A 2

B1 B2 a2 − a1
=tg θ =
A1 A 2 1 + a1a2
. +1
B1 B2
= x1y2 + x2y3 + x3y1 – x2y1 – x3y2 – x1y3

206 PROMILITARES.COM.BR
GEOMETRIA ANALÍTICA I

TRIANGULARIZAÇÃO DE POLÍGONOS mas OP = (x − x o )2 + (y − y o )2


Dado um polígono P qualquer, uma triangularização de P é uma Substituindo r na equação acima e elevando ao quadrado
divisão de P em triângulos, satisfazendo às seguintes condições: (x – x0)² + (y – y0)² = r²
A união de todos os triângulos é igual ao polígono P; Que é a equação reduzida da circunferência.
Dados dois triângulos T e T’, se T ∩ T’ ≠ ∅ , estão T ∩ T’ é um lado Desenvolvendo a equação reduzida temos:
comum a T e T’ ou um vértice comum de T e T’. x² + y² – 2x0x – 2y0y + x0² + y0² – r² = 0
Exemplo: Fazendo
Dado o polígono P = A1A2A3...A7A8, triangularize-o obtendo –2x0 = D, –2y0 = E, x0² + y0² – r² = F
os triângulos T1 = A1A2A8, T2 = A2A3A4, T3 = A4A5A6, T4 = A4A6A7,
obtemos x² + y² + Dx + Ex + F = 0 (equação geral da circunferência)
T5 = A2A4A7, T6 = A2A7A8 (observe a figura). Temos, então (A(P) significa
área de P):  D E
que é uma circunferência de centro  − , −  e raio igual a
 2 2
A (P ) = A ( T1) + A ( T2 ) + A ( T3 ) + A ( T4 ) + A ( T5 ) + A ( T6 ) = 1 2 2
D + E − 4F .
2
Para que a equação x² + y² + Dx + Ex + F = 0 represente uma
circunferância deveremos ter D² + E³ – 4F > 0.

POTÊNCIA DE UM PONTO EM
RELAÇÃO A UMA CIRCUNFERÊNCIA
Dados um ponto P(xp, yp) e uma circunferência (x – x0)2 + (y – y0)2 = R2,
chamamos potência do ponto em relação à circunferência ao número
real k = (xp – x0)2 + (yp – y0)2 = R2.
A soma das duas primeiras parcelas representa o quadrado da
distância do ponto P ao centro C da circunferência, então:
se k > 0 o ponto P é exterior;
se k = 0 o ponto P pertence à circunferência;
se k < 0 o ponto P é interior.

Exemplo: Calcule a área do pentágono ABCDE de vértices: POSIÇÃO RELATIVA DE RETA E


A(3,0), B(1,2), C(-2,2), D(-8,-7) e E(6,-1). CIRCUNFERÊNCIA
A 3 0 Sejam ax + by + c = 0 a equação da reta r e x² + y² + Dx + Ex + F = 0
B 1 2 a equação da circunferância C.
1 C 1 −2 2 1 98
Área ( ABCDE) = = = ( 65 + 33) = = 49u.a. Resolvendo o sistema:
2 D 2 −8 −7 2 2
ax+by+c=0
E 6 −1  2 2
x + y + Dx + Ex + F =0
A 3 0
recai-se numa equação do 2o grau de discriminante ∆. A reta e a
circunferência serão:
CIRCUNFERÊNCIA secantes ⇔ ∆ > 0
A circunferência é o lugar Geométrico dos pontos do plano
equidistantes de um ponto fixo do mesmo plano, chamado de centro. tangentes ⇔ ∆ = 0
exteriores ⇔ ∆ < 0

Posições relativas entre circunferências

Sejam duas circunferências de raios R e r, e d a distância entre


seus centros, tem-se:

circunf. exteriores D>R+r

circunf. tangentes exteriormente D=R+r

circunf. secantes |R – r| < d < R + r

circunf. menor interior à maior 0 < d < |R - r|

circunf. concêntricas d=0

Seja a circunferência de centro C(x0,y0) e raio r e P um ponto Equações paramétricas da circunferência


qualquer (x,y)
Pela definição temos |OP| = r x = a + r⋅cosθ y = b + r⋅senθ

PROMILITARES.COM.BR 207
GEOMETRIA ANALÍTICA I

EXERCÍCIOS DE 5 7 9 11 13

TREINAMENTO
a) b) c) d) e)
2 2 2 2 2

08. Considere os pontos A = (0,–1), B = (0,5) e a reta r: 2x – 3y + 6 = 0.


Das afirmações a seguir:
01. A equação da reta tangente ao gráfico da função f(x) = x²  – 6x + 1, I. d(A,r) = d(B,r).
no ponto (4, –7), é igual a
II. B é simétrico de A em relação à reta r.
a) y = –2x + 1. d) y = –3x + 5.
III. AB é base de um triângulo equilátero ABC, de vértice C = (–3 3 ,2)
b) y = 3x – 19. e) y = 2x – 15.
ou C = (3 3 ,2).
c) y = x – 11.
É (são) verdadeira(s) apenas
02. Considere a definição: duas circunferências são ortogonais a) I. c) I e II. e) II e III.
quando se interceptam em dois pontos distintos e nesses pontos b) II. d) I e III.
suas tangentes são perpendiculares. Com relação às circunferências
C1: x² + (y + 4)² = 7, C2: x² + y² = 9 e C3: (x – 5)² + y² = 16, podemos 09. Seja C uma circunferência tangente simultaneamente às retas
afirmar que r: 3x + 4y – 4 = 0 e s: 3x + 4y – 19 = 0. A área do círculo determinado
a) somente C1 e C2 são ortogonais. por C é igual a
b) somente C1 e C3 são ortogonais. 5π 3π 9π
a) . c) . e) .
c) C2 é ortogonal a C1 e a C3. 7 2 4
4π 8π
d) C1, C2 e C3 são ortogonais duas a duas. b) . d) .
5 3
e) não há ortogonalidade entre as circunferências.
10. Sejam r a circunferência que passa pelos pontos (6,7), (4,1), e (8,5)
03. A forma de uma montanha pode ser descrita pela equação e t a reta tangente à r, que passa por (0,–1) e o ponto de tangência
y = –x² + 17x – 66 (6 ≤ x ≤ 11). Considere um atirador munido de um tem ordenada 5. A menor distância do ponto P(–1,4) à reta t é:
rifle de alta precisão, localizado no ponto (2,0). A partir de que ponto,
a) 3 2 c) 2 3 4 10
na montanha, um indefeso coelho estará 100% seguro? e)
b) 4 d) 3 5
a) (8,9). c) (7,9). e) (7,4).
b) (8,6). d) (7,5). 11. Quantas unidades de área possui a região plana limitada pela
curva de equação x =1 − 1 − y 2 e pelas retas 2y + x – 3 = 0,
04. Duas pessoas combinaram de se encontrar entre 12h00 e 13h00.
2y – x + 3 = 0 e x = 2?
Elas também combinaram de esperar até 20 minutos pela outra pessoa
depois de chegar ao local do encontro. Assumindo que os horários 1 π π 3
a) π+ c) +1 e) +
de chegada ao local de encontro são uniformemente distribuídos no 2 2 2 2
intervalo de uma hora, que vai das 12h00 às 13h00, a probabilidade 3
b) π+ d) p + 3
de que elas se encontrem no intervalo combinado é igual a 2
1 4 5 2 5
a) b) c) d) e) 12. Considere o triângulo ABC no plano cartesiano com vértices
3 9 9 3 6
A = (0,0), B = (3,4) e C = (8,0). O retângulo MNPQ tem os vértices M e
N sobre o eixo das abscissas, o vértice Q sobre o lado AB e o vértice P
05. Considere a reta t mediatriz do segmento cujos extremos são sobre o lado BC. Dentre todos os retângulos construídos desse modo,
os pontos em que a reta s: 2x – 3y + 12 = 0 intercepta os eixos o que tem área máxima é aquele em que o ponto P é
coordenados. Então, a distância do ponto M(1,1) à reta t é
 16   12  8
a) 4, c) 5, e) 6,
a)
13 3
c)
13 11
e)
3 3  5   5   5 
11 13 11
 17   11 
10 13 3 11 b)  ,3 d)  ,2
b) d) 4 2
13 13

13. A equação do círculo localizado no 1º quadrante que tem área


 1 igual a 4π (unidades de área) e é tangente, simultaneamente, às retas
06. Sejam os pontos A(0,0), B(–1,1), C(1,2), D(4,1) e E  3,  . A reta
 2 r: 2x – 2y + 5 = 0 e s: x + y – 4 = 0 é
r passa por A e corta o lado CD, dividindo o pentágono ABCDE em 2 2
a)  3  10 
x −  + y −  = 4.
dois polígonos de mesma área. Determine a soma das coordenadas do  4  4
ponto de interseção da reta r com a reta que liga C e D.
2 2
25 26 27 b)  3   3 
a) c) e)  x −  +  y −  2 2 +   =
4.
7 7 7 4  4 
51 53 2 2
b) d)   3   10 
c)
14 14  x −  2 2 + 4   +  y − 4  =
4.

2 2
07. Duas circunferências com raios 1 e 2 têm centros no primeiro d)   3   13
quadrante do plano cartesiano e ambas tangenciam os dois eixos  x −  2 2 + 4   +  y − 4  =
4.
coordenados. Essas circunferências se interceptam em dois pontos 2 2
distintos de coordenadas (x1,y1) e (x2,y2). e)   3   11
 x −  2 2 + 4   +  y − 4  =
4.
O valor de (x1 + y1)² + (x2 + y2)² é igual a

208 PROMILITARES.COM.BR
GEOMETRIA ANALÍTICA I

14. Considere no plano complexo, o conjunto dos números z = x + yi; a) Calcule a área da região sombreada na figura, em função de x.
{x,y} ⊂  e i² = –1 que satisfazem a condição |z| ≥ |2z + 1|. b) Calcule o perímetro do quadrilátero PQRS, em função de x.
É FALSO afirmar que
a) este conjunto pode ser representado por um círculo de raio igual EXERCÍCIOS DE

COMBATE
1
a
3
b) z = –1 é o elemento de maior módulo, neste conjunto.

c) z = – 1 é o elemento de maior argumento, neste conjunto.


3 01. (ITA 2017) Considere a reta r: y = 2x. Seja A = (3,3) o vértice de
d) não existe z, neste conjunto, que seja imaginário puro. um quadrado ABCD, cuja diagonal BD está contida em r. A área deste
quadrado é
15. Quantas unidades de área possui a região plana limitada pela 9 12 18 21 24
a) . b) . c) . d) . e) .
curva de equação y =− 3 − x 2 − 2x e a reta y = x – 1? 5 5 5 5 5
π 1 c) 3π + 2 e) π – 2
a) − 02. (ITA 2016) Se a reta de equação x = a divide o quadrilátero cujos
4 4
vértices são (0,1), (2,0), (4,0) e (6,4) em duas regiões da mesma área,
π 1 π 1
b) − d) − então o valor de a é igual a
2 4 4 2
a) 2 5 – 1. d) 2 7 – 5.
16. Considere a circunferência de raio 13 e centro (0,0) e a curva b) 2 6 – 1. e) 3 7 – 5.
6
de equação y = . c) 3 5 – 4.
x
a) Determine a equação da circunferência. Esboce, no mesmo
sistema de coordenadas ortogonais, a circunferência e a curva. 03. (ITA 2019) Seja γ a circunferência de equação x² + y² = 4. Se r e s
são duas retas que se interceptam no ponto P = (1,3) e são tangentes
b) Encontre todos os pontos de interseção entre a circunferência e
a γ, então o cosseno do ângulo entre r e s é igual a
a curva.
1 1 2 6
c) Considere o polígono convexo cujos vértices são os pontos de a) . c) . e) .
interseção encontrados no item anterior. Calcule a área deste polígono. 5 2 5
7 2
b) . d) .
17. No plano cartesiano são dados o ponto P = (0,3) e o triângulo de 7 2
vértices A = (0,0), B = (3,0) e C = (3,2). Determine um ponto N sobre o
eixo dos x de modo que a reta que passa por P e N divida o triângulo 04. (ITA 2017) Considere dois círculos no primeiro quadrante:
ABC em duas regiões de mesma área.
π
- C1 com centro (x1,y1), raio r1 e área .
16
18. No plano cartesiano, 0xy a circunferência C tem centro no ponto
P = (2,1), e a reta t é tangente a C no ponto Q = (–1,5). - C2 com centro (x2,y2), raio r2 e área 144π.
a) Determine o raio da circunferência C. Sabendo que (x1,y1,r1) e (x2,y2,r2) são duas progressões geométricas
b) Encontre uma equação para a reta t. com somas dos termos iguais a 7 e 21, respectivamente, então a
c) Calcule a área do triângulo PQR, sendo R o ponto de interseção 4
distância entre os centros de C1 e C2 é igual a
de t com o eixo 0x.
123 131 137
a) . c) . e) .
19. Considere as circunferências 2 2 2

λ1 : x 2 + y 2 − 8x + 4y =
20 129 135
b) . d) .
e 2 2
λ 2 : x 2 + y 2 − 2x − 8y =
8.
05. (ITA 2013) Sobre a parábola definida pela equação x² + 2xy + y² –
O triângulo ABC satisfaz as seguintes propriedades: 2x + 4y + 1 = 0 pode-se afirmar que
a) o lado AB coincide com a corda comum a λ1 e λ2; a) ela não admite reta tangente paralela ao eixo Ox.
b) o vértice B pertence ao primeiro quadrante; b) ela admite apenas uma reta tangente paralela ao eixo Ox.
c) o vértice C pertence a λ1 e a reta que contém AC é tangente a λ2. c) ela admite duas retas tangentes paralelas ao eixo Ox.
d) a abscissa do vértice da parábola é x = –1.
20. Na figura, AC e BD são diagonais do quadrado ABCD de lado x, M
e N são pontos médios de AB e BC, respectivamente. e) a abscissa do vértice da parábola é x = − 2 .
3

06. (ITA 2008) Dada a cônica λ: x2 - y2 = 1, qual das retas abaixo é


perpendicular à λ no ponto P = (2, 3 )?
a) y = 3 x - 1 3
d) y = – x −7
3 5
b) y = x
2 3
e) y = – x –4
2
c) y = 3 x + 1
3

PROMILITARES.COM.BR 209
GEOMETRIA ANALÍTICA I

1 15. (ITA 2013) Determine a área da figura plana situada no primeiro


07. (IME 2015) O lugar geométrico no plano complexo de w= z + ,
z quadrante e delimitada pelas curvas
sendo z número complexo tal que |z| = k e k > 1 é um(a): x
(y − x − 2)(y + =− 2) 0 e x 2 − 2x + y 2=
− 8 0.
a) segmento de reta 2
b) circunferência
c) hipérbole 16. (ITA 2010) Determine uma equação da circunferência inscrita no
d) elipse triangulo cujos vértices são A = (1,1), B = (1,7) e C = (5,4) no plano xOy.
e) parábola
17. (IME 2017) Um triângulo ABC tem o seu vértice A na origem do
sistema cartesiano, seu baricentro é o ponto D(3,2) e seu circuncentro
08. (IME 2018) Seja uma elipse com focos no eixo OX e centrada na
é o ponto E(55/18,18,5/6) Determine:
origem. Seus eixos medem 10 e 20/3 Considere uma hipérbole tal que
- a equação da circunferência circunscrita ao triângulo ABC;
os focos da elipse são os vértices da hipérbole e os focos da hipérbole
são os vértices da elipse. As parábolas que passam pelas interseções - as coordenadas dos vértices B e C.
entre a elipse e a hipérbole e que são tangentes ao eixo OY na origem,
têm as seguintes equações: 18. (IME 2011) Determine o valor da excentricidade da cônica dada
35 35 pela equação x² – 10 3 xy + 11y² + 16 = 0
a) 2
y = ±2 x d) 2
y = ±6 x
7 7
19. (ITA 2007) Considere, no plano cartesiano xy, duas circunferências
5 35 C1 e C2, que se tangenciam exteriormente em P: (5,10). O ponto
b) y 2 = ±4 x e) y 2 = ±8 x
7 63 Q: (10,12) é o centro de C1. Determine o raio da circunferência C2,
5 sabendo que ela tangencia a reta definida pela equação x = y.
c) y 2 = ±6 x
7
20. Determine a equação da parábola que passa pelo ponto P1 = (0,a)
09. (IME 2015) Determine o produto dos valores máximo e mínimo de e é tangente ao eixo x no ponto P2 = (a,0), sabendo que a distância
y que satisfazem às inequações dadas para algum valor de x. de P1 a P2 é igual a 4.
2x² – 12x + 10 ≤ 5y ≤ 10 – 2x
a) –3,2 d) 1,6
GABARITO
b) –1,6 e) 3,2 EXERCÍCIOS DE TREINAMENTO
c) 0 01. E
Equação da reta tangente à parábola no ponto (4, –7).
10. (IME 2014) Uma elipse cujo centro encontra-se na origem e
cujos eixos são paralelos ao sistema de eixos cartesianos possui y – (–7) = m.(x – 4) ⇒ y = mx – 4m – 7
comprimento da semi-distância focal igual a 3 e excentricidade Resolvendo um sistema com as equações da parábola e da reta, temos:
3 x² – 6x + 1 = mx – 4m – 7 ⇒ x² – (m + 6)x + 4m + 8 = 0
igual a . Considere que os pontos A, B, C e D representam as
2
Como existe apenas um ponto de intersecção do discriminante deverá
interseções da elipse com as retas de equações y = x e y = – x. A área
ser zero, ou seja:
do quadrilátero ABCD é ∆=0
16 16
a) 8 c) e) (–(m + 6))² – 4⋅(4m + 8) = 0
3 7
(m – 2)² = 0 ⇒ m = 2
b) 16 16
d)
5 Considerando m = 2, a equação da reta será:
y = 2x – 4⋅2 – 7 ⇒ y = 2x – 15
11. (ITA 2018) No plano cartesiano são dadas as circunferências
C1: x² + y² = 1 e C2:(x –4)² + y² = 4. Determine o centro e o raio de
uma circunferência C tangente simultaneamente a C1 e C2 passando 02. C
pelo ponto A = (3, 3 ).
Do enunciado, temos:

12. (ITA 2018) No plano cartesiano são dados o ponto P = (0,3) e o


triângulo de vértices A = (0,0), B = (3,0) e C = (3,2). Determine um
ponto N sobre o eixo dos x de modo que a reta que passa por P e N
divida o triângulo ABC em duas regiões de mesma área.

13. (ITA 2017) Considere as retas de equações


r: y = 2 x + a e s: y = bx + c,
em que a, b, c são reais. Sabendo que r e s são perpendiculares entre
si, com r passando por (0,1) e s, por ( 2 ,4), determine a área do
triângulo formado pelas retas r, s e o eixo x.

14. (ITA 2015) Sabe-se que a equação 3x² + 5xy – 2y² – 3x + 8y – 6 = 0


representa a reunião de duas retas concorrentes, r e s, formando um
ângulo agudo θ. Determine a tangente de θ.

210 PROMILITARES.COM.BR
GEOMETRIA ANALÍTICA I

B é um dos pontos de intersecção entre C1 e C2. y= x − 2


C é um dos pontos de intersecção entre C1 e C3. 
y =− x 2 + 17x − 66
A é um dos pontos de intersecção entre C2 e C3. x − 2 =− x 2 + 17x − 66
Para que C1 e C2 sejam ortogonais, o triângulo EAF deve ser retângulo x 2 − 16x + 64 =
0
em A.
Temos:
( x − 8) 2 =
0
x=8
AE = 3 AF = 4 EF = 5
Substituindo x = 8 na equação y = x – 2, y = 6
Como 5² = 3² + 4², o triângulo EAF é retângulo em A, logo, C1 e C2
são ortogonais. Se m = 25,

Para que C2 e C3 sejam ortogonais, o triângulo DCF deve ser retângulo =


y 25x − 50

em C. y =− x 2 + 17x − 66
CD = 7 25x − 50 =− x 2 + 17x − 66
CF = 4 x 2 + 8x + 16 =
0
d (D,F) = ( 0 − 5)2 + ( 0 − 4 )2 ⇒ d (D,F) = 41 ( x + 4 )2 =
0
x = −4
2 2
Como 41 ≠ 7 + 42 , o triângulo DCF não é retângulo em C, logo,
Como o ponto que garante a segurança do coelho está no primeiro
C1 e C3 não são ortogonais. quadrante, tal ponto é: (8,6).
EB = 3
Para C2 e C3 sejam ortogonais, o triângulo EBD deve ser retângulo
em B. EB = 3 BD = 7
04. C
EB = 3 BD = 7 ED = 4
Sendo x e y os tempos em minutos entre 12h e 13h em que é possível
BD = 7 2 2 ED = 4 cada uma das pessoas chegarem, pode-se inferir que o encontro
Como 4= 32 + 7 , o triângulo EBD é retângulo em B, logo, C2 e
ED = 4 ocorrerá apenas se:
C3 são ortogonais.
x − y ≤ 20
Dessa forma, C2 é ortogonal às circunferências C1 e C3.
x − y ≤ 20,se x ≥ y

ou
03. B  − x + y ≤ 20,se x ≤ y

Teremos:
Graficamente:

A equação da reta t é dada por:


y = mx + n
O ponto (2,0) é um ponto da reta t, logo,
0 = 2m + n
n = –2m
Então,
(t) y = mx – 2m
O ponto de tangência entre a reta t e a parábola é dado por:
Os possíveis horários de chegada estão compreendidos nos pontos
mx − 2m =− x 2 + 17x − 66 pertencentes ao quadrado destacado, enquanto que os possíveis
x 2 + x (m − 17) + 66 − 2m =
0 horários de chegada que resultem no encontro entre as duas pessoas
estão compreendidos na área hachurada destacada. Assim, pode-se
∆ =0,
calcular:
(m − 17)2 − 4 ⋅ 1⋅ (66 − 2m) =0 402
2
m2 − 34m + 289 − 264 + 8m =
0 Shachurada 60 − 2 ⋅ 2 2000 20 5
=
P(encontro) = = = =
m2 − 26m + 25 =
0 Squadrado 602 3600 36 9
=m 25 = ou m 1

Se m = 1,

PROMILITARES.COM.BR 211
GEOMETRIA ANALÍTICA I

05. B Assim, pode-se escrever:


SAPDE = SABCP
1 0 x 4 3 0 1 0 −1 1 x 0
= → −2x + y − 3 = x − 4y − 1
2 0 y 1 12 0 2 0 1 2 y 0

P ∈1º Q / P ∈ r / P ∈ CD
−2x + y − 3 =x − 4y − 1 → 3x − 5y =−2
1 2
x y
Colineares C, D e P → = 0 → x + 3y = 7
4 1
1 2
Sistema:
 29
x=
3x − 5y = −2  14
 →
x + 3y = 7 y = 23
 14
52 26
x + y= =
14 7

07. C
Se as circunferências tangenciam os dois eixos coordenados e estão no
primeiro quadrante, então as coordenadas de seus centros são iguais
ao comprimento de seu raio. Assim, pode-se escrever:
Intersecção da reta S com o eixo x (y = 0). λ1 → raio =1; C1(1, 1)
2x + 12 = 0 ⇒ x = – 6 ⇒ P(–6,0) λ 2 → raio =2 ; C2 (2, 2 )
Intersecção da reta S com o eixo y (x = 0). λ1 : (x − 1)2 + (y − 1)2 = 12 → x 2 + y 2 − 2x − 2y + 1 = 0
–3y + 12 = 0 ⇒ y = 4 ⇒ Q(0,4)  2 2 2 2 2
λ 2 : (x − 2) + (y − 2) = 2 → x + y − 4x − 4y + 4 = 0
Considerando que N é o ponto médio de PQ, temos: Fazendo λ1 – λ2 tem-se uma reta r que é a reta que passa pelos pontos
−6 + 0 de intersecção das circunferências. Como os pontos (x1,y1) e (x2,y2)
xN = = −3
2 pertencem a essa reta, pode-se escrever:
0+4 3
=
yN = 2 λ1 − λ 2 = r → r : 2x + 2y − 3 = 0 → x + y =
2 2
Portanto, N = (–3,2). 3
x1 + y1 = x 2 + y 2 =
2
A reta S tem coeficiente angular 2/3, portanto a reta t terá coeficiente 2 2
( x1 + y1)2 + ( x 2 + y 2 )2 = 
angular –3/2 pois são perpendiculares. 3  3 18 9
 +   = =
2 2 4 2
Determinando agora a equação da reta t, que passa pelo ponto N e é
perpendicular à reta S, temos:
3 08. D
y − 2 = − ⋅ ( x − ( −3)) ⇒ 3x + 2y + 5 = 0
2 [I] Verdadeira.
Calculando a distância do ponto M(1,1) à reta (t) 3x + 2y + 5 ==
0 2 ⋅ 0 − 3 ⋅ ( −1) + 6 9
d(A,r) =
temos: 22 + ( −3)2 13
3 ⋅ 1+ 2 ⋅ 1+ 5 10 10 ⋅ 13 2 ⋅ 0 − 3 ⋅ (5) + 6 9
=d = = =d(B,r) =
32 + 22 13 13 22 + ( −3)2 13
Portanto d(A,r) = d(B,r).
[II] Falsa. A reta que passa pelos pontos A e B é vertical, portanto não
06. C
é perpendicular à reta r que é oblíqua. Concluímos, então que B não
Segundo o enunciado: é simétrico de A em relação à reta r
[III] Verdadeira.

212 PROMILITARES.COM.BR
GEOMETRIA ANALÍTICA I

5 + ( −1) Substituindo a (eq. 2) na (eq. 1) tem-se:


=yC = 2
2 a + 3b = 17 → b + 1 + 3b = 17 → 4b = 16 → b = 4
6⋅ 3 6⋅ 3 Substituindo o valor de b em (eq. 2), tem-se:
xC = = 3 3 ou x C =
− =
−3 3
2 2 a = b + 1→ a = 4 + 1→ a = 5
Portanto, C = (–3 3 ,2) ou C = (3 3 ,2). Substituindo os valores de a e b em (I), tem-se:
(6 − a)2 + (7 − b)2 =
R2
09. E (6 − 5)2 + (7 − 4)2 = R2 → 12 + 32 = R2 → R2 = 10 → R = 10
Assim, sabe-se que a circunferência r tem centro (5,4) e raio R =
10
O ponto de tangência entre a reta t e a circunferência r tem
coordenadas (x, 5) e, por ser tangente, obedece à equação geral da
circunferência. Logo:
(x − 5)2 + (y − 4)2 =
10
(x − 5)2 + (5 − 4)2 =10 → x 2 − 10x + 25 + 1 − 10 =0 → x 2 − 10x + 16 =0
∆ = ( −10)2 − 4 ⋅ 1⋅ 16 → ∆ = 36
10 ± 36
=x =
→ x 8 ou
= x 2
2
Com as informações até aqui, é possível desenhar os elementos:

Determinando um ponto P da reta r de abscissa x = 0, temos: P = (0,1)


mr = ms = –3/4 ⇒ r//s
Considerando a medida R do raio da circunferência, temos:
3 ⋅ 0 + 4 ⋅ 1 − 19 3
dr,s =dP,s =2R ⇒ 2R = ⇒ 2R =3 ⇒ R =
32 + 42 2
Portanto a área do círculo será dada por:
2
 3 9⋅ π
A = π⋅  =
 2 4
Pela figura percebe-se que o único valor possível de x é 2, pois caso o
10. E mesmo fosse 8, a reta t não seria tangente à circunferência, mas sim
cortaria a mesma. Logo, os pontos conhecidos da reta t são (0,–1) e
Pela equação geral da reta sabe-se que: (2,5). Substituindo estes pontos na equação geral da reta, tem-se:
(x − a)2 + (y − b)2 =
R2 onde : =
y gx + k
(a, b) = centro da circunferência  −1 =g ⋅ 0 + k → k =−1
(x, y) = pontos quaisquer da circunferência 
5 = g ⋅ 2 + k → 5 = 2g − 1 → g = 3
R = raio da circunferência Eq. reta t → y = 3x − 1
Assim, sabendo três pontos quaisquer, ou seja, (6,7), (4,1) e (8,5), A menor distância entre a reta t e o ponto P é um segmento de reta
pode-se escrever: perpendicular à t e passando por P (segmento PN).
(6 − a)2 + (7 − b)2 = R2 Sabendo que a reta perpendicular que passa pelo ponto P e pela reta
 2 2
t – vamos chamá-la de reta z – também tem a equação geral na forma
(4 − a) + (1 − b) = R2 y = gx + k, pode-se escrever:
 2 2
(8 − a) + (5 − b) = R2 Eq. reta t → y = 3x − 1
36 − 12a + a2 + 49 − 14b + b2 = R2 ( Ι ) 1 1
Eq. reta z → y =gz x + k z → gz =− , logo y =− x + k z
 2 3 3
16 − 8a + a + 1 −=2b + b2 R2 ( ΙΙ )
 2
Sabe-se que a reta z passa pelo ponto P(–1,4), portanto:
b2 R2 ( ΙΙΙ )
64 − 16a + a + 25 − 10b +=
1 1 11
y=− x + kz → 4 =− ⋅ ( −1) + k z → k z =
Igualando (I) e (II): 3 3 3

36 − 12a + a2 + 49 − 14b + b2 = 16 − 8a + a2 + 1 − 2b + b2 Assim, tem-se a equação da reta z e a equação da reta z. As


coordenadas do ponto de intersecção das duas retas (ponto N) podem
36 − 16 + 49 − 1 − 12a + 8a − 14b + 2b =0 ser calculadas, pois este ponto é comum a ambas:
68 − 4a − 12b = 0 → 4a + 12b = 68 → a + 3b = 17 (eq.1)
=y 3x − 1 (eq. reta t)

Igualando (I) e (III):  1 11
y = − x+
3 3
(eq. reta z)
36 − 12a + a2 + 49 − 14b + b2 = 64 − 16a + a2 + 25 − 10b + b2 1 11 9 1 11 3 10 14 14 7
3x − 1= − x + → x + x = + → x= → x= → x=
36 − 64 + 49 − 25 − 12a + 16a − 14b + 10b = 0 3 3 3 3 3 3 3 3 10 5
−4 + 4a − 4b = 0 → a − b = 1 → a = b + 1 (eq.2) 1 7 11 11 7 55 − 7 48 16
y =− ⋅ + = − = = →y=
3 5 3 3 15 15 15 5

PROMILITARES.COM.BR 213
GEOMETRIA ANALÍTICA I

13. D
Assim, o ponto N tem coordenadas N  7 , 16  .
5 5 
Logo, a distância entre o ponto P(–1,4) e o ponto N é dada por:
2 2 2 2
 7  16   5 7  20 16 
d =  −1 −  +  4 −  =  − −  +  −  =
 5  5  5 5  5 5
2 2
 12  4 144 16 160
= −  +   = + =
 5  5 25 25 25
160 160 22 ⋅ 22 ⋅ 10 4 10
=
=
d = = →=
d
25 25 5 5

A menor distância entre a reta t e o ponto P é o segmento de reta PN


4 10
de comprimento .
5

As retas são perpendiculares, pois mr⋅ms = 1⋅(–1) = –1.


11. E Considerando o ponto C centro da circunferência de raio 2, pois sua
Desenhando as funções dadas, tem-se: uma semicircunferência de área é 4π.
centro (1,0) e raio 1 tangenciando eixo y; duas retas simétricas que A reta PC é paralela ao eixo x, logo:
se interceptam no ponto (3,0) e cruzam o eixo y nos pontos (3/2,0) yP = yC e xC = xP + k
e (–3/2,0), cada uma, respectivamente; e uma reta paralela ao eixo Para determinar as coordenadas do ponto P basta resolver o sistema
y interceptando o eixo x no ponto (2,0). Essas funções delimitam abaixo:
uma superfície formada por metade de uma circunferência de raio 1
2x + 2y + 5 =0
somado a um trapézio de base maior 2, base menor 1 e altura 1. Logo, 
pode-se escrever:  x+y−4 = 0

( 2 + 1) ⋅ 1 + π ⋅ 12 → S = 3 + π
Portanto, P  , 
S= 3 13
2 2 2 2 4 4 
Determinando o valor de k no triângulo assinalado, temos:
12. D 2
sen 45°= ⇒ k= 2 2
k
Considere a figura.
3 13
Portanto, x c = + 2 2 e yc = .
4 4
Logo, a equação da circunferência será dada por:
2 2
  3   13
x − 2 2 +  +  y −  =
4.
  4   4

yB 4 14. C
A equação da reta AB é dada por y= x ⇔ y= x.
xB 3 Considerando z = x + yi, pode-se escrever:
tem-se Q =
3y   3y  | z | ≥ | 2z + 1|→ x + yi ≥ 2 ( x + yi) + 1 →
=
Logo, , y e M  , 0 , com 0 < y < 4.
 4   4 
( ) 1
2
→ x 2 + y 2 ≥ ( 2x + 1) + 4y 2 → x + 2 3 + y 2 ≤
2
Além disso, a equação da reta BC é
9
yB − y C 4−0
y=
− yC (x − x C ) ⇔ y=
−0 (x − 8)  2 
xB − x C 3−8 Assim, a equação representa um círculo com centro em C  − ,0 e
1
 3 
4 32
⇔ y =− x+ . raio .
5 5 3
Analisando cada uma das alternativas:
 32 − 5y   32 − 5y 
=Daí, P = , y e N  , 0 , com 0 < y < 4. [A] Verdadeira. Vide cálculos acima.
 4   4 
A área do retângulo MNPQ é dada por [B] Verdadeira. A distância entre o centro do círculo e o ponto z(–1,0)
é igual ao raio.
= MN ⋅ PN
(MNPQ)
[C] Falsa. Para todo y maior que zero, z será maior que – 1 .
 32 − 5y 3y  3
=  −  ⋅ (y − 0)
 4 4 [D] Verdadeira. A alternativa é verdadeira porque o círculo não corta
−2 y 2 + 8y
= o eixo y.

=−2 ⋅ [(y − 2)2 − 4)]


15. E
= 8 − 2 ⋅ (y − 2)2.

Portanto, o retângulo MNPQ tem área máxima quando y = 2, ou seja,


 11 
quando P =  , 2 .
2 

214 PROMILITARES.COM.BR
GEOMETRIA ANALÍTICA I

x 2 + y 2 = 13
 36
 6 ⇒ x 2 + 2 =13 ⇒ x 4 + 36 =13x 2 ⇒ x 4 − 13x 2 + 36 =0 ⇒
 y = x
 x
x 2 =9 ⇒ x =±3 ou x 2 =4 ⇒ x =±2

Portanto os pontos de intersecção são:


A(2,3), B(3,2), C(–2,–3) e D(–3,–2).
c) Considerando AC e BD como diâmetros, concluímos que os
ângulos internos do quadrilátero são retos, ou seja, este quadrilátero
é um retângulo, portanto sua área será o produtos de dois lados
perpendiculares.
y=− 3 − x 2 − 2x ⇒ y 2 =−
3 x 2 − 2x AB = (2 − 3)2 + (3 − 2)2 = 2
Logo, x² + 2x + 1 + y² = 3 + 1, temos então uma semicircunferência de AD = 2 2
( −3 − 2) + ( −2 − 3) = 5 ⋅ 2
raio 2 e centro no ponto (–1,0).
Portanto sua área será dada por:
Resolvendo um sistema com as equações da reta e da
semicircunferência, temos os pontos (1,0) e (–1,–2). A =5 ⋅ 2 ⋅ 2
π ⋅ 22 2 ⋅ 2 A = 10 unid2
Logo, a área pedida será dada por: − = π − 2.
4 2

16. 17. Do enunciado,

a) A equação da circunferência será dada por: Q = (c,d): ponto de intersecção da reta r com a reta AC.
2 SABC: área do triângulo ABC.
(x − 0)2 + (y − 0)=
2
13 ⇒ x 2 + y=
2
13
SAQN: área do triângulo AQN.
Esboçando os gráficos da hipérbole e da circunferência temos:
SCQNB: área do quadrilátero CQNB.
SAQN = SCQNB
1
SABC = ⋅ 3 ⋅ 2 ⇒ SABC = 3
2
2 ⋅ SAQN = SABC
3
2 ⋅ SAQN = 3 ⇒ SAQN =
2
yC − y A 2 − 0 2
=
mAC
 = =
xC − x A 3 − 0 3
 2
AC : y = x
3
y − yN 3 − 0 3
mr = P = = −
xP − x N 0 − a a
3 3
r : y − 3 =− ⋅ ( x − 0) ⇒ y =− x + 3
a a

r ∩ AC = {Q}, logo,
 2
y = 3 x (i)

b) Para determinar os pontos comuns devemos resolver um sistema 3
com as equações da hipérbole e da circunferência.
y =

− x+3 (ii)
a
Da equação (i),
3y
x=
2
3y
Substituindo x = na equação (ii),
2
3 3y
y =− ⋅ + 3
a 2
9y
y+ =3
2a
 9
y ⋅ 1+  = 3
 2a 
 2a + 9 
y⋅ =
3
 2a 
6a
y=
2a + 9
Então,

PROMILITARES.COM.BR 215
GEOMETRIA ANALÍTICA I

6a
x + y − 8x + 4y =
2 2
d= 20
2a + 9  2 2
 x + y − 2x − 8y =
8
1
SAQN = ⋅ a ⋅ d
2 Subtraindo as equações obtemos que: x = 2y – 2.
3 1 6a
= ⋅ a⋅ Substituindo o resultado acima na segunda equação do sistema,
2 2 2a + 9
obtemos: 5y² – 20y = 0.
a ⋅ 6a
3= Resolvendo a equação, temos:
2a + 9
3 ⋅ ( 2a + 9) =3 ⋅ 2a2 y = 0 ⇒ x = –2 ⇒ A(–2,0)
2a + 9 =2a2 y = 4 ⇒ x = 6 ⇒ B(6,4) (pertencente ao primeiro quadrante)
2
2a − 2a − 9 =0 Temos então a seguinte figura:

− ( −2) ± ( −2)2 − 4 ⋅ 2 ⋅ ( −9)


a=
2⋅ 2
2 ± 76
a=
4
2 ± 2 19
a=
4
Como a > 0,
2 + 2 19
a=
4
1+ 19
a=
2
 1+ 19 
Logo, N =  , 0 .
 2 

18.
a) Como Q é tangente à circunferência C, então o segmento PQ é Calculando o coeficiente angular da reta que passa pelos pontos A e
igual ao raio. Logo: C1, temos:
4−0 4
r= ( 2 − ( −1))2 + (1− 5)2= 9 + 16= 25 ⇒ r= 5 =
mAC2 =
1 − ( −2) 3
,

b) Como t é tangente à circunferência em Q, sabe-se então que t é portanto, o coeficiente angular da reta que passa pelos pontos A e
perpendicular ao segmento PQ. Assim, os coeficientes angulares da C será:
reta t e do segmento PQ tem a seguinte relação:
3
1 mAC = − ;
α t =− 4
αPQ
Determinando agora, a equação da reta AC, temos:
5 −1 4 3
αPQ
= → αPQ
= → α=
t 3
−1 − 2 −3 4 y − 0 =− ⋅ (x + 2)
Assim, a reta t é dada pela equação 4

3 Finalmente, resolvendo um sistema com as equações da reta que


reta t ⇒ y − 5= ( x + 1) ⇒ 3x − 4y + 23= 0 passa pelos pontos A e C da circunferência de equação x² + y² – 8x +
4
4y = 20, encontraremos as coordenadas do ponto C.
c) Se o ponto R intercepta o eixo x, então suas coordenadas são do
tipo (a,0). Para encontrar o valor de a, basta substituir na equação da  3
 y =− ⋅ (x + 2)
reta:  4
x 2 + y 2 − 8x + 4y =
20
23  23  
3a + 23 =0 ⇒ a =− ⇒ R  − ,0
3  3 
Resolvendo o sistema temos os seguintes pontos:
Assim, a área S do triângulo PQR pode ser escrita como:  38 36 
(–2,0) e  , − 
 5 5
2 1 1
1 1  23 115  1 125 125 Como o ponto (–2,0) já é o ponto A, concluímos que o ponto C é
S= ⋅ −1 5 1 = ⋅ 10 − + + 1 = ⋅ ⇒S=
2 2  3 3  2 3 6  38 36 
23  , −  .
− 0 1 5 5
3

20.
19. A circunferência de equação x² + y² – 8x + 4y = 20 possui centro
a) Queremos calcular
no ponto C1(4,–2) e a circunferência de equação x² + y² – 2x – 8y = 8
2⋅[(ACM) – (PQRS)].
possui centro no ponto C2(1,4).
Como AC é diagonal do quadrado e M é ponto médio de AB, segue-
Determinando os pontos A e B (pertencente ao primeiro quadrante)
se que
onde as circunferências se intersectam, temos o seguinte sistema.

216 PROMILITARES.COM.BR
GEOMETRIA ANALÍTICA I

1 2 2
(ACM)= ⋅ (ABC)  2x x   x x
2 RS=  −  +  − 
5 3 5 3
1 1
= ⋅ ⋅ (ABCD) x 5
2 2 =
x2 15
= .
4 e
Agora vamos calcular (PQRS). 2 2
 x x  x x
PS =  −  +  − 
Adotando convenientemente um sistema de eixos cartesianos 2 3 2 3
com origem no ponto C, vem A = (x,x), B = (x,0), C = (0,0),
x 2
 x x   x x = .
=M = x, , N  , 0 e P =  ,  . 6
 2 2   2 2
Fácil ver que a equação da reta AC é Y = X. Portanto,
A equação da reta DN é tal que x 2 x 5 2x 5
2pPQRS =
2⋅ + +
0−x  x 6 15 15
Y −0 = ⋅  X −  ⇔ Y =−2X + x. (5 2 + 3 5) x
x  2  = u.c.
−0
2 15
Logo, como o ponto S é a interseção das retas AC e DN, temos EXERCÍCIOS DE COMBATE
x  x x 01. C 03. A 05. B 07. D 09. A
X =−2X + x ⇔ X = , o que implica em S =  ,  .
3  3 3 02. D 04. E 06. E 08. E 10. D
BM 1
A equação da reta CM é dada por Y = ⋅ X ⇔ Y = ⋅ X. 11. Do enunciado, sem perda de generalidade, observemos a figura:
BC 2
Desse modo, como o ponto R é a interseção das retas CM e DN segue-
1 2x
se que ⋅ X =−2X + x ⇔ X = , implicando em R =  ,  .
2x x
2 5  5 5
A equação da reta BD é igual a Y = tg135º ⋅ X + x ⇒ y = –X + x.
Desse modo, como Q é a interseção das retas BD e CM vem
1 2x  2x x 
⋅ X =− X + x ⇔ X = , o que implica em Q =  ,  .
2 3  3 3
Portanto,
x 2x 2x x x
1 2 3 5 3 2
(PQRS)= ⋅
2 x x x x x
2 3 5 3 2
1 x 2 2x 2 2x 2 x 2 2x 2 2x 2 x 2 x 2
=⋅ + + + − − − −
2 6 15 15 6 6 15 15 6

=
x2 ( )
A reta s passa pelos pontos A 3, 3 , B ( 4, 0 ) e C ( x C , y C ) , onde C é
20 o centro da circunferência λ.
e, finalmente, 3−0
ms = ⇒ ms = − 3
 x2 x2  3− 4
2 ⋅ [(ACM) − (PQRS)] =⋅
2  − 
 4 20  s : y − 0 =− 3 ⋅ ( x − 4 )
2x 2 − 3 (x − 4)
y=
= .
5
Como C ∈ s,
b) De (a) temos
2 2
(
C =a, − 3 ( a − 4 ) )
 x 2x   x x
PQ = − + − 
 2 3   2 3 De ms = ˆ =
− 3, CBD 60°.

x 2 No triângulo CBD,
= ,
6 (r + 1)2 = (r + 2)2 + 42 − 2 ⋅ (r + 2) ⋅ 4 ⋅ cos60°
1
 2x 2x 
2
 x x
2 r 2 + 2r + 1 = r 2 + 4r + 4 + 16 − 2 ⋅ (r + 2) ⋅ 4 ⋅
QR=  −  +  −  2
3 5 3 5 2r + 1 = 4r + 20 − 4r − 8
2x 5 2r = 11
= ,
15 11
r=
2
dA, C = r
11
(a − 3)2 + ( − 3(a − 4) − 3)2 =
2

( 121
)
2
(a − 3) + − 3 ⋅ (a − 4 + 1) =
2

4
2 121
(a − 3)2 + 3 ⋅ (a − 3)2 =
4
121 PROMILITARES.COM.BR 217
(a − 3) ⋅ (1 + 3) =
2

4
11
r=
2
GEOMETRIA ANALÍTICA I
dA, C = r
11
(a − 3)2 + ( − 3(a − 4) − 3)2 =
2

( ) 121
2
(a − 3)2 + − 3 ⋅ (a − 4 + 1) =  2
4 y = 3 x ( i)
121 
2
(a − 3) + 3 ⋅ (a − 3) =
2 2
y = 3
4 
− x+3 (ii)
a
121
(a − 3) ⋅ (1 + 3) =
2
Da equação (i),
4
112 3y
( a − 3) =
2 x=
42 2
3y
11 11 Substituindo x = na equação (ii),
a − 3 = ou a − 3 =− 2
4 4 3 3y
y =− ⋅ +3
11 a 2
De a − 3 = ,
4 9y
y+ = 3
23 2a
a=
4  9
y ⋅ 1 +  = 3
11  2a 
De a − 3 =− ,
4  2a + 9 
1 y ⋅ = 3
a=  2a 
4 6a
y=
Assim, há duas possibilidades para λ. 2a + 9
  11 Então,
λ1: Centro no ponto  23 , − 7 3  e raio igual a
 4 4  2 6a
  d=
 1 15 3  11 2a + 9
λ2: Centro no ponto  , e raio igual a
4 4  2
1
SAQN = ⋅ a ⋅ d
  2
3 1 6a
= ⋅a⋅
12. Do enunciado, 2 2 2a + 9
a ⋅ 6a
3=
2a + 9
3 ⋅ ( 2a + 9 ) =3 ⋅ 2a2
2a + 9 =2a2
2a2 − 2a − 9 =0
− ( −2) ± ( −2) − 4 ⋅ 2 ⋅ ( −9 )
2

a=
2⋅2
2 ± 76
a=
4
2 ± 2 19
a=
4
 Como a > 0,
Q = (c,d): ponto de intersecção da reta r com a reta AC.
SABC : área do triângulo ABC. 2 + 2 19
a=
SAQN : área do triângulo AQN. 4
SCQNB : área do quadrilátero CQNB. 1 + 19
a=
2
SAQN = SCQNB
1  1 + 19 
Logo, N = 
SABC =
2
⋅ 3 ⋅ 2 ⇒ SABC = 3  2 , 0  .
 
2 ⋅ SAQN = SABC
3 13. Calculando:
2 ⋅ SAQN = 3 ⇒ SAQN =
2
yC − y A 2 − 0 2
( 0,1) ∈ r ⇒ a =1
=
mAC = =
xC − x A 3 − 0 3 − 2
r ⊥ s ⇒ b 2 =−1 ⇒ b =
 2 2
AC : y = x
3 ( )
2,4 ∈ s ⇒ c =5
yP − y N 3 − 0 3
mr = = = − Desenhando:
xP − x N 0 − a a
3 3
r : y − 3 =− ⋅ ( x − 0 ) ⇒ y =− x + 3
a a

r ∩ AC = {Q} , logo,

218 PROMILITARES.COM.BR
GEOMETRIA ANALÍTICA I

Seja B o ponto em que a reta y = x + 2 intersecta a circunferência,


conforme a figura.

r=
: y 2x + 1
− 2 As coordenadas do ponto B são obtidas do sistema
=
s: y x +5 2
2  2
( 2 ) 121 y= x + 2
2

− 2  ⇒ + =
2
 5 2 +  ⇒ =
2
.
2 6 
  (x − 1) + y =
2 2
 2 ,0  ∈ r
9
 
Logo, sendo B = (1,3), a área pedida corresponde à soma das áreas do
(5 )
2,0 ∈ s  Assim, como:
triângulo ABP e do segmento circular, definido pelo arco PB.
22 121 2 1 0 1 4 0
=S = (ABP)= ⋅
2 12 2 2 3 0 2
1
= ⋅ | −6 |
14. 2
= 3 u.a.
3x 2 + 5xy − 2y 2 − 3x + 8y − 6 = 0 ⇒ −2 ⋅ y 2 + (5x + 8) ⋅ y + 3 ⋅ x 2 − 3 ⋅ x − 6 = 0
e
8y − 6 = 0 ⇒ −2 ⋅ y 2 + (5x + 8) ⋅ y + 3 ⋅ x 2 − 3 ⋅ x − 6 = 0
π ⋅ 32 32
Calculando o discriminante da equação na incógnita y, temos: =
(BP) −
4 2
∆ = (5 ⋅ x + 8)2 − 4 ⋅ ( −2) ⋅ (3 ⋅ x 2 − 3 ⋅ x − 6) ⇒ ∆ = (7 ⋅ x + 4)2 9π − 18
= u.a.,
Aplicando, finalmente a fórmula de Bhaskara para a resolução de uma 4
equação do segundo grau, temos: Segue-se o resultado
−5x − 8 ± (7x + 4) 1 9π − 18 3
y= ⇒ y =− ⋅ x + 1 ou y =3 ⋅ x + 3, 3+ = ⋅ (3π − 2) u.a.
2 ⋅ ( −2) 2 4 4
que são as equações das retas concorrentes r e s, respectivamente.
Portanto,
16. d= d= 5
1 A,C B,C
mr =
− e ms =
3 Aplicando o teorema de Pitágoras no triângulo assinalado, temos:
2
e (4 − r)2 = r 2 + 22 ⇔ 8r = 12 ⇔ r = 3 2.
1 E o centro C(1 + r, 4) =
(5 3, 4)
− −3
mr − ms 2 Logo, a equação da circunferência será:
=t gθ = = 7.
1 + mr ⋅ ms  1 2 2 2
1+  −  ⋅ 3  5  3  5 9
 x −  + ( y − 4 )=   ⇒  x −  + ( y − 4 )=
2 2
 2  3  2  3 4

15. Temos
y −x −2=0
 x 
(y − x − 2)  y + − 2  = 0 ⇔ ou
 2 
x
y+
−2= 0
2
y= x + 2
⇔ ou .
x
y =− + 2 =0
2
A reta y = x + 2 intersecta o eixo das ordenadas no ponto A = (0,2)
17. Calculando a equação da circunferência circunscrita ao triângulo
e o eixo das abscissas no ponto Q = (-2,0), enquanto que a reta
ABC:
x
y =− + 2 =0 intersecta o eixo das ordenadas no ponto A = (0,2) 2 2
2  55  5  1625
e o eixo das abscissas no ponto P = (4,0). =
R  − 0  +  − 0  →=
R
 18  6  162
Completando os quadrados, vem
Equação da circunferência λ:
x 2 − 2x + y 2 − 8 = 0 ⇔ (x − 1)2 + (y − 0)2 = 9, 2 2 2 2
 55   5  1625 55x  55  5y  5  1625
ou seja, a curva x 2 − 2x + y 2 − 8 =0 é uma circunferência centrada em x −  +  y − = → x2 − +   + y2 + +  =
 18   6 162 9  18  3 6 162
C = (1,0) e raio 3.
λ : 9x 2 + 9y 2 − 55x − 15y =0

PROMILITARES.COM.BR 219
GEOMETRIA ANALÍTICA I

2 2 2 2 2

5y  5  1625 55   5  1625 55x  55  5y  5  1625 | 10 − 12 |
+  =  x −  +  y − = → x2 − +   + y2 + +  = = dQ,s = 2
3 6 162 18   6 162 9  18  3 6 162
12 + 12
λ : 9x 2 + 9y 2 − 55x − 15y = 0
PQ= (10 − 5)2 + (12 − 10)2 = 29
Calculando as coordenadas dos vértices B e C:
A +B+C 2
baricentro D → D = 1−
 =
12 − 10 2 5 = 3⇔
3 mPQ = e ms = 1 ⇒ tg θ =
10 − 5 5 2 7
A = ( 0,0 ) 1 + 1.
5
Logo, B + C = (9,6)
7 49 58 58
ctg θ = ⇒ csc2 θ = 1 + ctg2 θ = 1 + = ⇒ csc θ =
Ponto Médio de BC → =
B+C
MBC = 9 ,3
2 2 ( ) 3 9 9 3
29 + r 58 29 + r 58
Cálculo da equação da reta ⊂ EM : =
csc θ =
e csc θ ⇒ = ⇔
r− 2 3 r− 2 3
−1
EM ⊥ BC → coeficiente angular mEM = 3 29 + 3r= r 58 − 2 29 ⇔ r( 58 − 3) = 5 29 ⇔
mBC
5 29 58 + 3 145 2 + 15 29
3− 5 3 2 = r = . u.c.
mEM = 6 → mEM = → mBC = − 58 − 3 58 + 3 49
9 − 55 2 3
2 18
2  9 2x
reta r ⊂ BC : y − 3 =− ⋅  x −  → y =− +6 4 ⇒ ( 0 − a) + ( a − 0 ) =a 2 =4 ⇔ a =±2 2
2 2
3  2 3 20. PP 1 2 =

Intersecção entre r e a circunferência: Toda parábola que tangencia o eixo x em P2 = ( a,0 ) tem equação da
y k ( x − a ) , k ∈ * .
2
9x + 9y − 55x − 15y =
2 2
0 forma =

 2x 1
Como P1 = ( 0,a) está na parábola, temos: a= k ( 0 − a) ⇔ k=
2
y = − +6 .
 3 a
Portanto, há duas parábolas que satisfazem às condições do
9x 2 + 4x 2 − 72x + 324 − 55x + 10x − 90 = 0 → 13x 2 − 117x + 234 = 0
2( 2(
x − 2 2) e y = x + 2 2) .
2 2
B(3,4) e C(6,2) enunciado: = y −
x' = 3 → y = 4 4 4
x 2 − 9x + 18 =0 → → ou
x'' = 6 → y = 2
B(6,2) e C(3,4)
ANOTAÇÕES

18. Vamos inicialmente eliminar o termo x⋅y da equação.


B −10 3
tg2θ= = = 3 ⇒ θ= 30°
A − C 1 − 11
Utilizando rotação de eixos, temos:
3 1 1 3
x = x'⋅ − y'⋅ e y = x'⋅ + y'⋅
2 2 2 2
Substituindo os valores de x e y acima na equação, temos:
x'2 y '2
−16x'2 + 64y'2 + 64 = 0 ⇒ − = 1 (hipérbole)
4 1
Onde a = 2, b = 1 e c = 22 + 12 = 5
c 5
Logo, a excentricidade será dada por e= = .
a 2

19.
s: y = x
2
y

r
Q
2
12
29
p
10
r
θ

x
5 10

220 PROMILITARES.COM.BR
GEOMETRIA ANALÍTICA II

DEFINIÇÃO GERAL DAS CÔNICAS ELEMENTOS DA ELIPSE


Cônica é o lugar geométrico dos pontos de um plano cuja razão
entre as distâncias a um ponto fixo F(foco) e a uma reta fixa d (diretriz)
é igual a uma constante não negativa e (excentricidade da cônica). A
excentricidade “e” é um número não-negativo, pois é a razão de duas
distâncias.
Se e = 1, a cônica é uma parábola.
Se 0 < e < 1, a cônica é uma elipse.
Se e > 1, a cônica é uma hipérbole.

Diretriz

D P

F(foco) x

ELIPSE Pontos principais:


Dados dois pontos fixos F1 e F2 de um plano, tais que, F2F1 = 2c ≠ 0, A2, A1, B2 e B1 – vértices
chamamos elipse ao lugar geométrico dos pontos deste plano, cuja F2 e F1 – focos
soma das suas distâncias aos dois pontos F2 e F1 é constante igual a
C – centro
2a > 2c.

Segmento:
A2A1 – eixo maior – m(A2A1) = 2a
B2B1 – eixo menor – m(B2B1) = 2b
F2F1 – distância focal – m(F2F1) = 2c

Os vetores de origem num dos focos e extremidade em qualquer


 
ponto da elipse são chamados raios vetores: F2P, FP,
1 etc.

RELAÇÕES

c
e= <1 Excentricidade
a
a² = b² + c² Relação notável tirada do
triângulo retângulo B1CF1

b2
p= Parâmetro
a

“Parâmetro de uma cônica é a semicorda focal mínima.”

b2
p=
a

PROMILITARES.COM.BR 225
GEOMETRIA ANALÍTICA II

RETAS: DIRETRIZES POSIÇÕES RELATIVAS ENTRE PONTO E


Diretrizes da elipse são duas retas, (D1) e (d2), perpendiculares ao ELIPSE
a Sejam ε uma elipse de focos F1 e F2, cujo eixo maior mede 2a, e P
suporte do eixo maior, distando do centro da curva.
e um ponto do ponto do plano de ε.
1o caso: A é ponto da elipse.
EQUAÇÃO REDUZIDA A pertence à elipse ε se, e somente se PF1 + PF2 = 2a
2o caso: A é interior à elipse ε. PF1 + PF2 < 2a
EIXO MAIOR NO EIXO X 3o caso: A é exterior à elipse ε. PF1 + PF2 > 2a
 
| F2P | + | FP
1 |=
2a
(x + c)2 + y 2 + (x − c)2 + y 2 = 2a ⇔ POSIÇÕES RELATIVAS ENTRE RETA E ELIPSE
2 2 2 2 2 2 Sendo ε uma elipse e r uma reta, contidas em mesmo plano,
x + 2cx + c + y = 2a − x − 2cx + c + y ⇒
temos:
x 2 + 2cx + c2 + y 2 = 4a2 − 4a x 2 − 2cx + c2 + y 2 ⇔ 1o caso: a reta r é exterior à elipse se, e somente se, r ∩ ε = ∅
a x 2 − 2cx + c2 + y 2 = a2 − cx ⇒ 2o caso: a reta r é tangente à elipse se, e somente se, r ∩ ε = {P}
3o caso: a reta r é secante à elipse se, e somente se r ∩ ε = {P1 , P2}
a2x 2 − 2a2cx + a2c2 + a2y 2 =
a4 − 2a2cx + c2x 2 ⇔
a2x 2 − c2x 2 + a2y 2 =a4 − a2c2 ⇔
PROCESSO PRÁTICO
(a2 − c2 )x 2 + a2y 2= a2 (a2 − c2 ) ⇔ A quantidade de pontos de interseção entre a reta r e a elipse ε
b2x 2 + a2y 2 = a2b2 ⇔ é dada pelo número de soluções do sistema formado pelas equações
de r e ε, resolvido substituindo-se uma variável previamente isolada na
x2 y2
⇔ + =
1 equação de r na equação de ε. Com isso, se:
a2 b2
I. ∆ > 0: r é secante a ε;
a
As equações das diretrizes são x = ± II. ∆ = 0: r é tangente a ε;
e
III. ∆ < 0: r é exterior a ε.
Se Se C = 0, porém e eixo maior está no eixo y.

x2 y2
HIPÉRBOLE
+ =
1 Sejam 2 pontos fixos F1 e F2 de um plano, com F2F1 = 2c ≠ 0,
b2 a2
hipérbole é o lugar geométrico dos pontos deste plano, cujo módulo
da diferença de suas distâncias aos dois pontos F2 e F1 é constante
igual a 2a, com 2a > 2c.
EQUAÇÃO DA ELIPSE QUANDO HÁ TRANSLAÇÃO DE
SISTEMA
ELEMENTOS DA HIPÉRBOLE
1o caso: Elipse com centro O(m , n) e eixo maior horizontal.
(x − m)2 (y − n)2
+ =
1,a > b.
a2 b2

2º caso: Elipse com centro O(m, n) e eixo maior vertical.


(x - m)2 (y - n)2
+ =1, a>b
b2 a2

A1 e A2 – vértices
F2 e F1 – focos
C – Centro
Eixo real: A2A1 = 2a
Eixo imaginário: B2B1 = 2b
Distância focal: F2F1 = 2c
 
Raios vetores: F2P, FP
1

226 PROMILITARES.COM.BR
GEOMETRIA ANALÍTICA II

Relações: 2) A equação da hipérbole de centro na origem, focos no eixo y


c e semieixos real e imaginário iguais a a e b é dada por:
e= >1 Excentricidade
a x2 y2
+ 2 =
1
c² = a² + b² Relação notável −b 2
a
Reta: Diretrizes são duas retas, (d1) e (d2), perpendiculares ao As diretrizes são, agora, paralelas ao eixo x e suas equações são:
a
suporte do eixo real, distando do centro da hipérbole. a
e y= ±
Assíntotas são duas retas, (a1) e (a2), que passam pelo centro da e
hipérbole e posições limites das tangentes a ela, quando os pontos de a
e as assíntotas y = ± x .
contato se afastam indefinidamente. b
Para y = 0 ⇒ ± x = bi a curva não intercepta o eixo dos x e para
x = 0 ⇒ y = ± a, ordenadas dos vértices A1 e A2 .
EQUAÇÕES REDUZIDAS
Seja a hipérbole de eixos real A2A1 e imaginário B2B1 com centro 3) Hipérbole com centro no ponto C (m, n) e A2A1 //Ox
na origem. Sua equação é:
Considere P(x, y) um ponto genérico da curva. (x − m)2 (y − n)2
− =
1
a2 b2
As equações das diretrizes são
a
x= m ±
e
e das assíntotas
b
y= m ± x
a
Quando A2 A1 // Oy ⇒
(x − m)2 (y − n)2
+ =
1
−b2 a2
a
As equações das diretrizes assumem a forma x= m ± e as das
a e
assíntotas y = ± x .
b

4) Equação geral:
A equação geral é obtida pelo desenvolvimento das formas
Da definição temos que: reduzidas.
 
| F2P | − FP 2a .
1 | =

5) Hipérbole Equilátera:
1) Equação reduzida:
Uma hipérbole cujos semieixos são iguais (a = b) é chamada de
(x + c)2 + y 2 − (x − c)2 + y 2 =±2a hipérbole equilátera.
(x + c)2 + y 2 =±2a + (x − c)2 + y 2 As suas equações se simplificam com a substituição de b por a.
2 2 2 2 2 2 2 2 2 2
x + 2cx + c + y = 4a ± 4a x − 2cx + c + y + x − 2cx + c + y
6) Hipérboles Conjugadas:
4cx − 4a2 =±4a x 2 − 2cx + c2 + y 2
Duas hipérboles são conjugadas quando o eixo real (transverso)
cx − a2 =± x 2 − 2cx + c2 + y 2 de uma é o imaginário (não transverso) da outra e vice-versa.
c2x 2 − 2a2cx + a4 =a2x 2 − 2a2cx + a2c2 + a2y 2
7) Posições relativas entre ponto e hipérbole:
c2x 2 − a2x 2 − a2x 2 − a2y 2 = a2c2 − a4
2 2 2 2 2 2 2 2 Uma hipérbole H e um ponto P, coplanares, têm três posições
(c − a )x − a y = a (c − a )
relativas possíveis, onde 2a é a medida do eixo real da hipérbole, e F1
b2x 2 − a2y 2 =
a2b2 e F2 são os focos da hipérbole.
Dividindo ambos os membros por a²b² vem, 1o caso: P é um ponto da hipérbole.
P ∈ H ⇔ |PF1 – PF2| = 2a
x2 y2
− = 1
a2 b2
2o caso: P é ponto interior à hipérbole.
Para y = 0, temos: x = ± a, abscissas dos vértices A1 e A2.
P é interior à H ⇔ |PF1 – PF2| > 2a
Para x = 0, temos: y = ± bi, o que significa que a curva não é
interceptada pelo eixo dos y.
3o caso: P é ponto exterior à hipérbole
a
As equações das diretrizes (d1) e (d2) são x = ± . P é exterior à hipérbole H ⇔ |PF1 – PF2| < 2a
e
As equações das assíntotas, y = tga ⋅ x ⇒
b
⇒ y=
± x
a

PROMILITARES.COM.BR 227
GEOMETRIA ANALÍTICA II

8) Posições relativas entre reta e hipérbole: 3) Relação:


Processo prático: p
VF =
Seja S o sistema formado pelas equações de r e H. Se, ao 2
substituirmos uma das variáveis da equação de r na equação de H,
obtivermos: 4) Reta e eixo:
I. Uma equação do 2o grau com ∆ > 0. r e H são secantes; A reta fixa (d) é a diretriz e e, eixo que passa pelo foco e é
II. Uma equação do 2o grau com ∆ < 0. r e H são exteriores; perpendicular à diretriz, eixo de simetria da parábola.
III. Uma equação do 2o grau com ∆ = 0. r e H são tangentes;
IV. Uma equação do 1o grau. r e H são secantes; EQUAÇÕES
V. Uma equação impossível. r e H são exteriores. Equação espontânea: No sistema foco-diretriz a equação
espontânea da parábola é: FP = MP ou u = v.

9)

RECONHECIMENTO DE UMA CÔNICA


Dada uma equação do 2º grau redutível à forma

( x-x 0 )2 + ( y-y 0 )2 =
1
k1 k2

k1 > 0, k2 > 0 e k1 > k2 elipse de eixo maior horizontal (1)

k1 > 0, k2 > 0 e k1 < k2 elipse de eixo maior vertical

k1 > 0 e k2 < 0 hipérbole de eixo real horizontal

k1 < 0 e k2 > 0 hipérbole de eixo real vertical

PARÁBOLA
1) Equação reduzida:
DEFINIÇÃO
p 
Parábola é o lugar geométrico dos pontos de um plano, situados O ponto F tem coordenadas  , 0 .
2 
a igual distância de uma reta fixa (d) e de um ponto fixo F, não Calculemos u e v
pertencente a (d), do plano considerado. 2
 p p
Definição geométrica: é a cônica obtida mediante a secção de u=dFP =  x −  + y 2 e v = + x
um plano secante a um cone quadrático, sendo o plano paralelo a  2 2
uma e somente uma geratriz do cone.
Igualando, conforme (1), vem:
2
ELEMENTOS DA PARÁBOLA  p 2 p
 x −  + y = + x
2 2
p2 p2
x 2 − px + + y2 = + px + x 2
4 4
y 2 = 2px
p
Equação da diretriz: x = − .
2

2) Se V = 0 e o eixo de simetria coincidir com o eixo dos y


 p
As coordenadas do foco passam a ser  0,  , então a equação
 2
da parábola toma a forma
x² = 2py
e a da diretriz
p
y= −
2
1) Pontos principais:
F – foco 3) Quando a parábola tem V(m, n), portanto,V ≠ 0 e o eixo de
V – vértice simetria paralelo ao eixo 0x, vem:
(y’)² = 2px
e aplicando a translação de eixos de I resulta:
2) Segmentos:
V’F = p – parâmetro (semi-corda focal mínima) (y – n)2 = 2p(x – m) ou (y – n)2 = –2p(x – m)

FP – raio vetor

228 PROMILITARES.COM.BR
GEOMETRIA ANALÍTICA II

3x + y ≤ 2
2 2
EXERCÍCIOS DE
07. A solução gráfica do sistema de inequações  2 é a

TREINAMENTO
2
x + y ≤ 1
região sombreada em
a) d)

01. Os pontos M(0,y), com y ≥ 0 e N( 3 ,4) pertencem a uma


circunferência de centro C(0,2). Considere o ponto P, do gráfico de
=
f(x) x + 2, que possui ordenada y igual à do ponto M.

b) e)

A abscissa x do ponto P é igual a c)


a) 7. c) 7. e) 12.
b) 7 + 2. d) 9.

02. A equação da reta tangente ao gráfico da função f(x) = x² – 6x + 1,


no ponto (4,–7), é igual a
a) y = –2x + 1. d) y = –3x + 5.
b) y = 3x – 19. e) y = 2x – 15.
c) y = x – 11. 08. Seja uma elipse com focos no eixo OX e centrada na origem. Seus
eixos medem 10 e 20/3. Considere uma hipérbole tal que os focos
03. Considere a definição: duas circunferências são ortogonais da elipse são os vértices da hipérbole e os focos da hipérbole são os
quando se interceptam em dois pontos distintos e nesses pontos vértices da elipse. As parábolas que passam pelas interseções entre a
suas tangentes são perpendiculares. Com relação às circunferências elipse e a hipérbole e que são tangentes ao eixo OY, na origem, têm
C1: X² + (y + 4)² = 7, C2: x² + y² = 9 e C3: (x – 5)² + y² = 16, podemos as seguintes equações:
afirmar que
35 35
a) somente C1 e C2 são ortogonais. a) y 2 = ±2 x d) y 2 = ±6 x
7 7
b) somente C1 e C3 são ortogonais.
5 35
b) y 2 = ±4 x e) y 2 = ±8 x
c) C2 é ortogonal a C1 e a C3. 7 63
d) C1, C2 e C3 são ortogonais duas a duas. 5
c) y 2 = ±6 x
e) não há ortogonalidade entre as circunferências. 7

04. A forma de uma montanha pode ser descrita pela equação 09. Duas circunferências com raios 1 e 2 têm centros no primeiro
y = –x² + 17x – 66 (6 ≤ x ≤ 11). Considere um atirador munido de um quadrante do plano cartesiano e ambas tangenciam os dois eixos
rifle de alta precisão, localizado no ponto (2,0). A partir de que ponto, coordenados. Essas circunferências se interceptam em dois pontos
na montanha, um indefeso coelho estará 100% seguro? distintos de coordenadas (x1,y1) e (x2 e y2).
a) (8,9). d) (7,5). O valor de (x1 + y1)² + (x2 + y2)² é igual a
b) (8,6). e) (7,4). 5 9 e) 13
a) c)
c) (7,9). 2 2 2
7 11
b) d)
05. Uma circunferência com centro na origem está tangenciando 2 2
duas retas paralelas de equações y = –2x + b e y = –2x + c Nesse caso,
o valor de b + c é 10. Seja λ: 3x² + 3y² – 6x – 12y + k = 0, uma circunferência que no
a) 0 c) –1 e) 2 plano cartesiano tem intersecção vazia com os eixos coordenados.
b) –2 d) 1 Considerando k ∈  correto afirmar que
 k k
06. O ponto A(3,4) pertence a uma circunferência λ cujo centro tem a) P  ,  é interior a λ.
 3 3
abscissa 7 e ordenada inteira. Uma reta r passa pelo ponto O(0,0) e
pelo ponto A e a distância de r até o centro de λ é igual a 2. O raio b) existem apenas dois valores inteiros para k.
da circunferência λ é c) a reta r: x = k intersecta λ.
a) 2 c) 2 2 d) se c é o comprimento de λ, então c > 2π unidades de comprimento.
b) 5 d) 2 5

PROMILITARES.COM.BR 229
GEOMETRIA ANALÍTICA II

18. O gráfico representa uma hipérbole, dada pela função real


11. A área da região limitada pelos gráficos das funções =
y 9 − x2 ,
3 2 + 2x 3
y = |x| e y = é igual a: f(x)= x + . Sabe-se que ABCD é um retângulo, que EC é
4 2− x
3 diagonal do retângulo EBCF e que a área da região indicada em rosa
a) 3 2 (3π − 2) d) (3π − 2) é igual a 4,7 cm².
4 4

b) 3 3
( π − 2) e) (3π − 2 2)
4 4
3
c) ( π − 2 2)
4

12. Analise as proporções abaixo e escreva V para a(s) verdadeira(s) e


F para a(s) falsa(s).
( ) A distância entre o vértice e o foco da parábola y² + 4x – 4 = 0 é
igual a 1 unidade de comprimento.
( ) Numa hipérbole equilátera, as assíntotas são perpendiculares
entre si.
( ) A equação 2x² + y² – 4x – 4y + 4 = 0 representa uma elipse que
tem um dos focos no ponto P(1,4).
A sequência correta é
a) F - F - V b) V - F - V c) F-V-F d) V - V - F
a) Determine as coordenadas (x,y) do ponto A.
13. A curva plana C é representada pelo gráfico da função real b) Calcule a área da região indicada em amarelo no gráfico.
f(x) = xcosx e tem uma reta tangente no ponto de abscissa x = π. Essa
reta tangente, o eixo y e o arco de curva x² + y² – 2πx = 0 situado
19. Sejam C um subconjunto não vazio e P um ponto, ambos em
abaixo do eixo x, determinam uma região R, cuja área vale
um mesmo plano, tais que P ∈ C. Diz-se que “P enxerga C sob um
a) π(π + 1) π2  4 ângulo α” se α for a medida do menor ângulo com vértice em P que
d)  π + 2 
2  π contenha C. Por exemplo, na figura, o ponto P enxerga o quadrado C
π2  4 sob o ângulo α indicado.
b)  π −  e) π(π + 2)
2  π

c) π2  4
 π + 
2  π

14. Sejam r a circunferência que passa pelos pontos (6,7), (4,1) e (8,5)
e t a reta tangente à r, que passa por (0,–1) e o ponto de tangência
tem ordenada 5. A menor distância do ponto P(–1,4) à reta t é: a) Se C for um círculo de raio r, centrado na origem de um plano
a) 3 2 c) 2 3 4 10 cartesiano real, determine o lugar geométrico dos pontos que
e) enxergam C sob um ângulo de 60º.
b) 4 d) 3 5
b) Se C for a união dos segmentos OA e OB, em que O = (0,0),
A = (a,0) e B = (0,b), com a,b > 0, determine o lugar geométrico
15. Considere as afirmações a seguir:
dos pontos que enxergam C sob um ângulo de 90º.
I. O lugar geométrico do ponto médio de um segmento AB, com
comprimento 1 fixado, cujos extremos se deslocam livremente 20. Pelo ponto P de coordenadas (–1,0) traçam-se as tangentes t e
sobre os eixos coordenados é uma circunferência. s à parábola y² = 2x. A reta t intercepta a parábola em A e a reta s
II. O lugar geométrico dos pontos (x,y) tais que 6x³ + x²y – xy² – 4x² intercepta a parábola em B. Pelos pontos A e B traçam-se paralelas
– 2xy = 0 é um conjunto finito no plano cartesiano ². às tangentes encontrando a parábola em outros pontos C e D
III. Os pontos (2,3), (4,–1) e (3,1) pertencem a uma circunferência. respectivamente. Calcule o valor da razão AB .
Destas, é (são) verdadeira(s) CD
a) apenas I. c) apenas III. e) I e III.
EXERCÍCIOS DE

COMBATE
b) apenas II. d) I e II.

16. Considere a circunferência de raio 13 e centro (0,0) e a curva


6
de equação y = .
x 01. (EN 2009) Seja P o ponto de interseção entre as retas r e s de
a) Determine a equação da circunferência. Esboce, no mesmo equações 3x – 2y + 4 = 0 e –4x + 3y – 7 = 0, respectivamente. Seja Q o
sistema de coordenadas ortogonais, a circunferência e a curva. centro da circunferência de equação x² + y² + 24 = 6x + 8y. A medida
do segmento PQ é igual à quarta parte do comprimento do eixo maior
b) Encontre todos os pontos de interseção entre a circunferência e a
da elipse de equação
curva.
a) 2x² + y² – 8x – 2y + 7 = 0 d) x² + 2y² – 2x – 8y + 1 = 0
c) Considere o polígono convexo cujos vértices são os pontos de
interseção encontrados no item anterior. Calcule a área deste b) 2x² + y² – 4x – 2y – 1 = 0 e) x² + 2y² – 4x + 8y + 8 = 0
polígono. c) x² + 4y² – 4x – 24y + 36 = 0

230 PROMILITARES.COM.BR
GEOMETRIA ANALÍTICA II

02. (EN 2011) A curva de equação x² – 14 = y² + 2x intercepta a reta c) o par de retas dadas por y = ± (3x – 1)
4y + 1 = x nos pontos A e B. Seja C a circunferência com centro no d) a parábola de equação y² = 4x + 4
ponto médio do segmento AB e cujo raio é a medida do maior eixo
e) a circunferência centrada em (9,5) e raio 120
da curva de equação x 2 + 2y=
2
2 3x − 8y − 2 . A circunferência C tem
por equação 07. (IME 2010) Uma hipérbole de excentricidade 2 tem centro na
origem e passa pelo ponto ( 5 ,1). A equação de uma reta tangente
35 − x 2 − y 2 x 2 + y 2 − 35 a esta hipérbole e paralela a y = 2x é:
a) x= d) x=
2 2 a) 3y=2 3x+6
2
20 − x − y 2 2
25 − x − y 2 b) y = –2x + 3 3
b) x= e) x=
2 2 c) 3y = 6x + 2 3

x 2 + y 2 − 25 d) 3y=2 3x+4
c) x=
2 e) y = 2x + 3
x2 y2
03. (IME 2012) Os triângulos ABC e DEF são equiláteros com lados 08. A circunferência x² + y² – 8x = 0 e a hipérbole − =1
iguais a m. A área da figura FHCG é igual à metade da área da figura 9 4
ABHFG. Determine a equação da elipse de centro na origem e eixos interceptam-se nos pontos A e B. A equação da tangente comum à
formados pelos segmentos FC e GH. circunferência e à hipérbole com coeficiente angular positivo é
a) 2x – 5 y – 20 = 0
b) 2x – 5y+4=0
c) 3x – 4y + 8 = 0
d) 4x – 3y + 4 = 0
e) 5 x – 2y + 4 = 0

09. Uma reta tangente à hipérbole de equação x² – 4y² – 100 = 0 é


paralela à reta x – y – 3 = 0 forma um triângulo com as assíntotas da
hipérbole. A área desse triângulo é igual a
a) 48x² + 36y² – 2 m² = 0 a) 10 3 d) 50
b) 8x² + 16y² – 3 m² = 0 b) 20 e) 60 2
c) 16x² + 48y² – 3m² = 0 c) 30 2
d) 8x² + 24y² – m² = 0
e) 16x² – 24y² – m² = 0 10. (ITA 1998) As retas y = 0 e 4x + 3y + 7 = 0 são retas suportes das
diagonais de um paralelogramo. Sabendo que estas diagonais medem
04. (ITA 2011) A expressão 4e2x + 9e2y – 16ex – 54ey + 61 = 0, com 4 cm e 6 cm, então a área deste paralelogramo, em cm², vale:
x e y reais, representa
a) o conjunto vazio. a) 36 d) 48
5 3
b) um conjunto unitário.
c) um conjunto não-unitário com um número finito de pontos. b) 27 e) 48
d) um conjunto com um número infinito de pontos. 4 5
e) o conjunto {(x,y) ∈ ²| 2(ex – 2)² + 3(ey – 3)² = 1}. c) 44
3
05. Sejam uma circunferência M de centro M(0, 0) e raio 10 e uma
11. (ITA 2019) Determine o lugar geométrico de todos os pares
circunferência P de centro P(8, 0) e raio 2. Uma circunferência de
ordenados (a,b) ∈ ² que tornam impossível o sistema linear
centro Q é tangente internamente à circunferência M e externamente
à circunferência P. O maior valor de y do lugar geométrico dos pontos  − x + 5y = 10
Q é: 
S :  a2 2  .
a) 2 d) 2 5  5 + 5b  x + 10aby =
1

b) 2 3
e) 2 7
c) 4 12. (IME 2016) Determine o lugar geométrico dos pontos em ²
equidistantes às retas de equações 4x + 3y – 2 = 0 e 12x – 16y + 5 = 0.
06. (ITA) Considere a hipérbole H e a parábola T, cujas equações são,
respectivamente, 5(x + 3)² – 4(y – 2)² = –20 e (y – 3)² = 4(x – 1). 13. (ITA 2016) Sejam S um subconjunto de ² e P = (a,b) um ponto de
Então, o lugar geométrico dos pontos P, cuja soma dos quadrados das ². Define-se distância de P a S, d(P, S), como a menor das distâncias
distâncias de P a cada um dos focos da hipérbole H é igual ao triplo do d(P, Q), com Q ∈ S:
quadrado da distância de P ao vértice da parábola T, é:
d(P,S) = min{d(P,Q): Q ∈ S}.

a) a elipse de equação
( x − 3) + ( y + 2) =
2
1
2
Sejam S1 = {(x,y) ∈ ² : x = 0 e y ≥ 2} e S2 = {(x,y) ∈ ² : y = 0}.
4 3 a) Determine d(P,S1) quando P = (1,4) e d(Q,S1) quando Q = (–3,0).
b) Determine o lugar geométrico dos pontos do plano equidistantes
b) a hipérbole de equação
( y + 1)2 − ( x − 3)2 =
1
de S1 e de S2.
5 4

PROMILITARES.COM.BR 231
GEOMETRIA ANALÍTICA II

14. Determine as coordenadas dos vértices, dos focos, a equação Define-se o ponto P sobre C’ de forma que no início do movimento
reduzida e as equações das diretrizes da elipse de C’ o ponto P coincide com o ponto de tangência (4,0), conforme
figura a. Após certo deslocamento, o ângulo de entre o eixo x e a reta
x =3 + 20 cos θ
 que une o centro das circunferências é α. conforme figura b.
 y =
2 + 16 sen θ
a) Determine as coordenadas do ponto P marcado sobre C’ em
função do ângulo α.
15. (UNESP 2016) Em um plano cartesiano ortogonal são dadas uma b) Determine a equação em coordenadas cartesianas do lugar
reta d, de equação x = –3, e um ponto F, de coordenadas (–1,2). Nesse geométrico do ponto P quando α varia no intervalo [0,2π).
plano, o conjunto dos pontos que estão à mesma distância do ponto
F e da reta d forma uma parábola. Na figura, estão nomeados dois
20. (IME 2012) É dada uma parábola de parâmetro p. Traça-se a corda
pontos dessa parábola: o vértice V, de coordenadas (–2,2), e o ponto
focal MN, que possui uma inclinação de 60° em relação ao eixo de
P, de coordenadas (0,yp).
simetria da parábola. A projeção do ponto M sobre a diretriz é o ponto
Q, e o prolongamento da corda MN intercepta a diretriz no ponto R.
Determine o perímetro do triângulo MQR em função de p, sabendo
que N encontra-se no interior do segmento MR.

GABARITO
EXERCÍCIOS DE TREINAMENTO
01. C
O primeiro passo é determinar o raio da circunferência, calculando a
distância entre os pontos N e C.

( )
2
3 − 0 + ( 4 − 2)=
2
=
r 7
Portanto, a ordenada do ponto M será yM= 2 + 7
Como o ponto P tem a mesma ordenada do ponto M, podemos
escrever que:
2 + 7= x + 2 ⇒ x= 7 ⇒ x= 7 .
Portanto, a abscissa do ponto P é 7.

02. E
Equação da reta tangente à parábola no ponto (4,–7).
Determine as coordenadas de dois pontos quaisquer dessa parábola
y − ( −7) = m ⋅ (x − 4) ⇒ y = mx − 4m − 7
que sejam diferentes de V e de P. Em seguida, calcule yp.
Resolvendo um sistema com as equações da parábola e da reta, temos:
16. (ITA 2008) Considere a parábola de equação y = ax2 + bx + c, x 2 − 6x + 1 = mx − 4m − 7 ⇒ x 2 − (m + 6)x + 4m + 8 = 0
que passa pelos pontos (2, 5), (- 1, 2) e tal que a, b, c formam, nesta Como existe apenas um ponto de intersecção do discriminante deverá
ordem, uma progressão aritmética. Determine a distância do vértice ser zero, ou seja:
da parábola à reta tangente à parábola no ponto (2, 5).
∆ =0

17. (ITA 2006) Sabendo que 9y2 - 16x2 - 144y + 224x - 352 = 0 é a ( −(m + 6))2 − 4 ⋅ (4m + 8) =0
equação de uma hipérbole, calcule sua distância focal. (m − 2)2 = 0 ⇒ m = 2

18. (ITA 2003) Sabe-se que uma elipse de equação (x2/a2) + (y2/b2) = 1 Considerando m = 2, a equação da reta será:
tangencia internamente a circunferência de equação x2 + y2 = 5 e y = 2x − 4 ⋅ 2 − 7 ⇒ y = 2x − 15
que a reta de equação 3 x+ 2y = 6 é tangente à elipse no ponto P.
Determine as coordenadas de P.
03. C
19. (IME 2016) A circunferência C tem equação x² + y² = 16. Seja Do enunciado, temos:
C’ uma circunferência de raio 1 que se desloca tangenciando interna
mente a circunferência C, sem escorregamento entre os pontos de
contato, ou seja, C’ rola internamente sobre C.

232 PROMILITARES.COM.BR
GEOMETRIA ANALÍTICA II

B é um dos pontos de intersecção entre C1 e C2. Se m = 1,


C é um dos pontos de intersecção entre C1 e C3. y= x − 2
A é um dos pontos de intersecção entre C2 e C3. 
y =− x 2 + 17x − 66
Para que C1 e C2 sejam ortogonais, o triângulo EAF deve ser retângulo x − 2 =− x 2 + 17x − 66
em A.
x 2 − 16x + 64 =
0
AE = 3
AF = 4 ( x − 8) 2 =
0
EF = 5 x=8
Como 5² = 3² + 4², o triângulo EAF é retângulo em A, logo, C1 e C2 Substituindo x = 8 na equação y = x – 2, y = 6
são ortogonais. Se m = 25,
Para que C1 e C3 sejam ortogonais, o triângulo DCF deve ser retângulo =
y 25x − 50
em C 
y =− x 2 + 17x − 66
CD = 7
25x − 50 =− x 2 + 17x − 66
CF = 4
x 2 + 8x + 16 =
0
d (D,F) = ( 0 − 5)2 + ( 0 − 4 )2 ⇒ d (D,F) = 41
( x + 4 )2 =
0
2 2
2
x = −4
Como 41 ≠ 7 + 4 , o triângulo DCF não é retângulo em C, logo,
C1 e C3 não são ortogonais. Como o ponto que garante a segurança do coelho está no primeiro
quadrante, tal ponto é: (8,6).
Para C2 e C3 sejam ortogonais, o triângulo EBD deve ser retângulo
em B.
EB = 3 05. A
BD = 7
ED = 4

2 2
Como 4= 32 + 7 , o triângulo EBD é retângulo em B, logo, C2 e
C3 são ortogonais.
Dessa forma, C2 é ortogonal às circunferências C1 e C3.

04. B
Teremos:

A equação da reta t é dada por:


OB = OC (raios)
y = mx + n ˆ = CODˆ (retos)
AOB
O ponto (2,0) é um ponto da reta t, logo,
α = β (alternos internos)
0 = 2m + n
n = –2m Portanto, DAOB ≅ DDOC (caso LAA0)
Então, (t) y = mx – 2m Portanto, AO = DO, ou seja b e c tem o mesmo tamanho, porém sinais
O ponto de tangência entre a reta t e a parábola é dado por: contrários. Então, sua soma será: b + c = 0.

mx − 2m =− x 2 + 17x − 66
06. D
x 2 + x (m − 17) + 66 − 2m =
0
Calculando:
∆ =0,
 4 − 0 4
(m − 17)2 − 4 ⋅ 1⋅ (66 − 2m) =0 =
r:y 
 3 − 0 
=
x⇒y
3
x

m2 − 34m + 289 − 264 + 8m =


0 4 ⋅ 7 − 3c
λ ⇒ C ( 7, a) ⇒ dCr = = 2 ⇒ a= 6 ∈ 
m2 − 26m + 25 =
0 42 + ( −3)2
=m 25 = ou m 1
R = dAC = (3 − 7)2 + (4 − 6)2 = 20 = 2 5

PROMILITARES.COM.BR 233
GEOMETRIA ANALÍTICA II

07. C 5 9y 2
As equações apresentadas representam uma elipse e uma + =
1
7 100
circunferência de raio 1. A solução gráfica de ser a intersecção de
9y 2 2
duas áreas. Calculando: =
100 7
3x 2 y 2 6 200 10 14
3x 2 + y 2 ≤ 2 ⇒ + ≤ 1 ⇒ raio menor =  6 y2 = ⇒ y =±
2 2 3 ⇒ <1 63 21
2 2  3
x + y ≤ 1 ⇒ raio =1 
Portanto, os pontos de intersecção são
Assim, a solução gráfica é a região sombreada representada em [C]  5 35 10 14   5 35 10 14 
(eixo menor da elipse é menor que o diâmetro da circunferência).  7 , 21  ,  − 7 , − 21  ,
   
 5 35 10 14   5 35 10 14 
08. E
 7 , − 21  e  − 7 , 21  .
Do enunciado, temos:    

x2 y2 Assim, as equações das parábolas são dadas por:


+ = 1 (equação da elipse)
a2 b2 y² = ± px
a = 5 e b = 10  5 35 10 14 
3 Como  , é um ponto da parábola,
Então, a equação da elipse é dada por:  7 21 
2
x 2 9y 2  10 14  5 35
+ =
1  21  = p ⋅ 7
25 100  
10 8 35
De a2 =b2 + c2 , a =5eb= , p=
3 63
2
 10  Então, as equações das parábolas são:
=52   + c2
 3 8 35
y2 = ± x
2 125 63
c =
9
5 5
c= ± 09. C
3
Se as circunferências tangenciam os dois eixos coordenados e estão no
Dessa forma, os focos da elipse são primeiro quadrante, então as coordenadas de seus centros são iguais
ao comprimento de seu raio. Assim, pode-se escrever:
5 5   5 5 
F1  , 0 e F2  − , 0 . λ1 → raio =1; C1(1, 1)
 3   3 
λ 2 → raio =2 ; C2 (2, 2 )
Os vértices da elipse são V1(5,0) e V2(–5,0).
λ1 : (x − 1) + (y − 1) = 1 → x + y − 2x − 2y + 1 = 0
2 2 2 2 2

Assim, pelo enunciado, a equação da hipérbole é dada por  2 2 2 2 2


λ 2 : (x − 2) + (y − 2) = 2 → x + y − 4x − 4y + 4 = 0
x2 y2
− = 1, de tal forma que:
α 2 β2 Fazendo λ1 – λ2 tem-se uma reta r que é a reta que passa pelos pontos
2
de intersecção das circunferências. Como os pontos (x1,y1) e (x2,y2)
10 5 5 5 5 5 10 pertencem a essa reta, pode-se escrever:
2=
α ⇒=
α e=
β 52 −  = 3 .
3 3  3  3
λ1 − λ 2 = r → r : 2x + 2y − 3 = 0 → x + y =
2
Portanto, a equação da hipérbole é dada por:
3
x1 + y1 = x 2 + y 2 =
9x 2 9y 2 2
− =
1
125 100 2 2
( x1 + y1)2 + ( x 2 + y 2 )2 = 
3  3 18 9
 +   = =
As intersecções entre a elipse e a hipérbole são dadas pelas soluções 2 2 4 2
do sistema abaixo:
 x 2 9y 2
 + =1 10. B
25 100
 2 2
Colocando na equação geral da circunferência:
 9x − 9y =
125 100 1 3x 2 + 3y 2 − 6x − 12y + k =0

Somando membro a membro as equações do sistema,


( ) ( )
3 ⋅ x 2 − 2x + 3 ⋅ y 2 − 4y + k =0 →

14x 2 ( ) ( )
→ 3 ⋅ x 2 − 2x + 1 + 3 ⋅ y 2 − 4y + 4 = 15 − k
=2
125 15 − k
( x − 1) + ( y − 2=
2
) 2
= R2
125 5 35 3
x2 = ⇒ x =±
7 7 Assim, conclui-se que o centro da circunferência será em (1,2) e que
2 2 para que a mesma possua intersecção vazia com os eixos coordenados
5 35 x 9y
Substituindo x = na equação + =
1, é necessário que:
7 25 100

234 PROMILITARES.COM.BR
GEOMETRIA ANALÍTICA II

0 < R < 1 → 0 < R2 < 1 [II] VERDADEIRA. A proposição é verdadeira pois esta é justamente
a definição de hipérbole equilátera: ter as assíntotas perpendiculares
15 − k entre si.
0< < 1 → 0 < 15 − k < 3 → 12 < k < 15 com k ∈ 
3
[III] FALSA. Podemos reescrever a equação dada de modo a facilitar
Analisando as alternativas conclui-se que apenas a alternativa [B] as conclusões:
é a correta, pois entre o intervalo 12 e 15 há apenas dois números
inteiros: 13 e 14. (2x 2 − 4x) + (y 2 − 4y + 4) =0 →
( x − 1)2 + ( y − 2)2 =1
1 2

11. D Comparando esta equação com a equação geral de uma elipse, pode-
se concluir que a equação dada trata-se de uma elipse de centro (1,2),
3 2 + 2x
Sendo A e B os pontos de intersecção entre y = e y = |x|, semi-eixo menor b = 1 e semi-eixo maior a = 2. A elipse pode ser
pode-se escrever: 4
representada graficamente como na figura a seguir:
 2 2
 3 2 + 2x A− , 
y = 3 2 + 2x  2 2 
 4 =→ ±x →
y = | x | 4  3 2 3 2
 B ,
 2 2 

Desenhando os gráficos das funções e os pontos calculados, tem-se:

Sabendo que a² = b² + c², então c = 1. Daí pode-se deduzir que os


focos da elipse serão (1,3) e (1,1). A proposição é falsa.

13. D

dist AC =  −
2 
2
 2
− 0 + 

2

− 0 → dist AC =1
f(x) = x cos x → f( π ) = x cos π =π −1 → Tangência = T π , π −1 ( )
 2   2  derivada → =
f'(x) x (cos x) −1
( x
⋅ cos x − x ⋅ log ⋅ sen π )
reta → y − y 0 = a ⋅ ( x − x 0 )
2 2
3 2  3 2 
dist=
BC  2 − 0 +  2 − 0 → dist=
BC 3

3 ⋅1
  
( )
a = f'( π ) = x (cos π ) −1 ⋅ cos π − π ⋅ logπ ⋅ sen π → a =−
1
π2
SACB = → SACB =3 2 1 1 −x 2
2 y− = − 2 ⋅ ( x − π) → y = 2 +
π ⋅ 32 3 π π π π
=
S Ssetor − SACB
= S 3 4 ⋅ ( 3π − 2)
− →=
circunferência → x 2 + y 2 − 2πx = 0 → ( x − π ) + y 2 = π 2
2
4 2
= C ( π,0)
centro

12. D raio = π

Analisando as proposições:
[I] VERDADEIRA. Podemos reescrever a equação da parábola dada:
1 2
y 2 + 4x − 4 =0 → x =− y +1
4
Assim, temos que quando x = 0, y = ±2, e quando y = 0, x = 1. Com
isso pode-se construir um gráfico e identificar que trata-se de uma
parábola com concavidade voltada para a esquerda, que corta o eixo
y nos pontos +2 e –2, cujo vértice tem coordenadas (0,1). Conclui-se
também que eixo de simetria da parábola é o próprio eixo x(x = 0).
O foco de uma parábola fica sempre sobre o eixo de simetria (portanto, =S Striângulo + Ssemicircunferência
nesse caso, x = 0), com y = k + p onde k será a coordenada y do vértice
1 2π ⋅ 2 π π ⋅ π 2 π2  4
e p= . =
S + →= S  π + 2 
4a 2 2 2  π
Assim, a coordenada y do foco será:
k =1
14. E
1
p= = −1 Pela equação geral da reta sabe-se que:
4 ⋅ −14
y =k + 1 → y =1 − 1 → y = 0

Logo, as coordenadas do foco serão (0,0) e sua distância até o vértice


é igual a 1. A alternativa é verdadeira.

PROMILITARES.COM.BR 235
GEOMETRIA ANALÍTICA II

(x − a)2 + (y − b)2 =
R2 onde :
(a, b) = centro da circunferência
(x, y) = pontos quaisquer da circunferência
R = raio da circunferência

Assim, sabendo três pontos quaisquer, ou seja, (6,7), (4,1) e (8,5),


pode-se escrever:
(6 − a)2 + (7 − b)2 =R2
 2 2
(4 − a) + (1 − b) = R2
 2 2
(8 − a) + (5 − b) =R2
36 − 12a + a2 + 49 − 14b + b2 = R2 ( Ι )
 2
16 − 8a + a + 1 −= 2b + b2 R2 ( ΙΙ )
 2 2 2
64 − 16a + a + 25 − 10b +=b R ( ΙΙΙ )

Igualando (I) e (II):


36 − 12a + a2 + 49 − 14b + b2 = 16 − 8a + a2 + 1 − 2b + b2
36 − 16 + 49 − 1 − 12a + 8a − 14b + 2b =0 Pela figura percebe-se que o único valor possível de x é 2, pois caso o
68 − 4a − 12b = 0 → 4a + 12b = 68 → a + 3b = 17 (eq.1) mesmo fosse 8, a reta t não seria tangente à circunferência, mas sim
cortaria a mesma. Logo, os pontos conhecidos da reta t são (0,–1)
Igualando (I) e (III): e (2,5). Substituindo estes pontos na equação geral da reta, tem-se:
36 − 12a + a2 + 49 − 14b + b2 = 64 − 16a + a2 + 25 − 10b + b2 =
y gx + k
36 − 64 + 49 − 25 − 12a + 16a − 14b + 10b = 0  −1 =g ⋅ 0 + k → k =−1

−4 + 4a − 4b = 0 → a − b = 1 → a = b + 1 (eq.2) 5 = g ⋅ 2 + k → 5 = 2g − 1 → g = 3
Substituindo a (eq. 2) na (eq. 1), tem-se: Eq. reta t → y = 3x − 1

a + 3b = 17 → b + 1 + 3b = 17 → 4b = 16 → b = 4 A menor distância entre a reta t e o ponto P é um segmento de reta


perpendicular à t e passando por P (segmento PN).
Substituindo o valor de b em (eq. 2), tem-se:
Sabendo que a reta perpendicular que passa pelo ponto P e pela reta
a = b + 1→ a = 4 + 1→ a = 5 t – vamos chamá-la de reta z – também tem a equação geral na forma
Substituindo os valores de a e b em (I), tem-se: y = gx + k, pode-se escrever:
(6 − a)2 + (7 − b)2 =
R2 Eq. reta t → y = 3x − 1
2 2 2 2 2 2 2 1 1
(6 − 5) + (7 − 4) = R → 1 + 3 = R → R = 10 → R = 10 Eq. reta z → y =gz x + k z → gz =− , logo y =− x + k z
3 3
Assim, sabe-se que a circunferência r tem centro (5,4) e raio R = 10. Sabe-se que a reta z passa pelo ponto P(–1,4), portanto:
O ponto de tangência entre a reta t e a circunferência r tem 1 1 11
coordenadas (x,5) e, por ser tangente, obedece à equação geral da y=− x + kz → 4 =− ⋅ ( −1) + k z → k z =
3 3 3
circunferência. Logo:
(x − 5)2 + (y − 4)2 =
10 Assim, tem-se a equação da reta t e a equação da reta z. As
coordenadas do ponto de intersecção das duas retas (ponto N) podem
(x − 5)2 + (5 − 4)2 = 10 → ser calculadas, pois este ponto é comum a ambas:
→ x 2 − 10x + 25 + 1 − 10 =0 → x 2 − 10x + 16 =0 =y 3x − 1 (eq. reta t)

∆ = ( −10)2 − 4 ⋅ 1⋅ 16 → ∆ = 36  1 11
10 ± 36 y = − x+
3 3
(eq. reta z)
=x =
→ x 8 ou
= x 2
2 1 11 9 1 11 3
3x − 1 = − x + → x + x =+ →
Com as informações até aqui, é possível desenhar os elementos: 3 3 3 3 3 3
10 14 14 7
→ x= → x= → x=
3 3 10 5
1 7 11 11 7 55 − 7 48 16
y =− ⋅ + = − = = →y=
3 5 3 3 15 15 15 5

Assim, o ponto N tem coordenadas N  7 , 16  .


5 5 

Logo, a distância entre o ponto P(–1,4) e o ponto N é dada por:

236 PROMILITARES.COM.BR
GEOMETRIA ANALÍTICA II

2 2 2 2
 7  16   5 7  20 16 
d =  −1 −  +  4 −  =  − −  +  −  =
 5  5  5 5  5 5
2 2
 12  4 144 16 160
= −  +   = + =
 5  5 25 25 25
160 160 22 ⋅ 22 ⋅ 10 4 10
=
=
d = = →=
d
25 25 5 5

A menor distância entre a reta t e o ponto P é o segmento de reta PN


4 10
de comprimento .
5

15. A
[I] Verdadeira.
Vamos admitir os pontos médios da forma M(x,y) e O a origem. Como
os pontos A e B estão sobre os eixos, concluímos que o triângulo AOB
I
é retângulo de hipotenusa I, portanto, OM = .
2
Daí, temos:
2 b) Para determinar os pontos comuns devemos resolver um sistema
 I I2 com as equações da hipérbole e da circunferência.
OM = x2 + y2 =   ⇒ x2 + y2 =
 2 4

Portanto, uma circunferência de raio I/2.


[II] Falsa.
6x 3 + x 2y − xy 2 − 4x 2 − 2xy =
0
x ⋅ (6x 2 + xy − y 2 − 4x − 2y) =
0
x ⋅ (4x 2 − y 2 + 2x 2 + xy − 4x − 2y) = 0
x ⋅ ((2x + y) ⋅ (2x − y) + x ⋅ (2x + y) − 2 ⋅ (2x + y)) =
0
x ⋅ (2x + y) ⋅ (2x − y + x − 2) = 0
x ⋅ (2x + y) ⋅ (3x − y − 2) = 0

Temos então três equações de reta:


x=0
2x + y = 0
3x − y − 2 =0

Portanto, temos infinitos pontos.


[III] Falsa. Os pontos estão alinhados, pois:
2 3 1
4 −1 1 =−2 + 9 + 4 + 3 − 2 − 12 =0
3 1 1
x 2 + y 2 = 13
 36
 6 ⇒ x 2 + 2 = 13 ⇒
 y= x
16.  x
a) A equação da circunferência será dada por: ⇒ x 4 + 36 =13x 2 ⇒ x 4 − 13x 2 + 36 =0 ⇒
2 x 2 =9 ⇒ x =±3 ou x 2 =4 ⇒ x =±2
(x − 0)2 + (y − 0)=
2
13 ⇒ x 2 + y=
2
13
Portanto os pontos de intersecção são:
Esboçando os gráficos da hipérbole e da circunferência temos:
A(2,3), B(3,2), C(–2,–3) e D(–3,–2).
c) Considerando AC e BD como diâmetros, concluímos que os
ângulos internos do quadrilátero são retos, ou seja, este quadrilátero
é um retângulo, portanto sua área será o produtos de dois lados
perpendiculares.

AB = (2 − 3)2 + (3 − 2)2 = 2
AD = ( −3 − 2)2 + ( −2 − 3)2 = 5 ⋅ 2

Portanto sua área será dada por:


A =5 ⋅ 2 ⋅ 2
A = 10 unid2

PROMILITARES.COM.BR 237
GEOMETRIA ANALÍTICA II

17. Do enunciado, temos: MM'= m + n


b2 b2
=
MM' +
a + c ⋅ cos θ a − c ⋅ cos θ
b2 ⋅ ( a − c ⋅ cos θ) + b2 ⋅ ( a + c ⋅ cos θ)
MM' =
( a + c ⋅ cos θ) ⋅ ( a − c ⋅ cos θ)
b2 ⋅ ( a − c ⋅ cos θ + a + c ⋅ cos θ)
MM' =
( a + c ⋅ cos θ) ⋅ ( a − c ⋅ cos θ)
2ab2
MM' = (i)
( a + c ⋅ cos θ) ⋅ ( a − c ⋅ cos θ)
Se CD = k, as coordenadas do ponto D são xD = k⋅cosθ e yD = k⋅senθ.
Como D é um ponto da elipse,
(k ⋅ cos θ)2 + (k ⋅ senθ)2 = 1
a2 b2
2 2 2 2
k ⋅ cos θ k ⋅ sen θ
x2 y2 + = 1
+ =
1 a2 b2
a2 b2
k 2 cos2 θ ⋅ b2 + k 2sen2θ ⋅ a2
F e F’ são os focos da elipse. M, D, M’ e D’ são pontos da elipse. =1
a2b2
Assim,
(
k 2 ⋅ b2 cos2 θ + a2sen2θ = a2b2 )
MF + MF’ = 2a e M’F + M’F’ = 2a 2 2
ab
Seja MF = m k2 =
b2 cos2 θ + a2sen2θ
De MF + MF’ = 2a e MF = m,
CD’ = CD, logo, CD’ = k.
MF’ = 2a – m
Então,
No triângulo MF’F,
DD' = 2k
( 2a − m
=)2 ( 2c )2 + m2 − 2 ⋅ 2c ⋅ m ⋅ cos (180 − θ)
DD'2 = 4k 2
4a2 − 4am + m= 2
4c2 + m2 − 4 ⋅ m ⋅ c ⋅ ( − cos θ)
4a2b2
2 2 DD'2 =
4a − 4m ⋅ a= 4c + 4 ⋅ m ⋅ c ⋅ cos θ b cos θ + a2 sen2 θ
2 2

⋅ a 4a2 − 4c2
4 ⋅ m ⋅ c ⋅ cos θ + 4m= 2 2ab2
DD'= 2a ⋅ 2
m ⋅ c ⋅ cos θ + m ⋅ a= a2 − c2 b cos θ + a2 sen2 θ
2

2ab2
) a2 − c2
m ⋅ ( c ⋅ cos θ + a= 2
DD'= 2a ⋅ 2 2
a2 − c2
( )
a − c ⋅ cos2 θ + a2 sen2 θ
m=
c ⋅ cos θ + a 2 2ab2
DD'= 2a ⋅
a cos θ − c cos2 θ + a2 sen2 θ
2 2 2
Como a² = b² + c², a² – c2 = b².
2 2ab2
Logo, DD'= 2a ⋅
b2
( )
a2 cos2 θ + sen2 θ − c2 cos2 θ
m= 2ab 2
a + c ⋅ cos θ 2
DD'= 2a ⋅
a2 − c2 cos2 θ
Seja M1F = n
2ab2
De M’F + M’F’ = 2a e M’F = n,
2
DD'= 2a ⋅ (ii)
( a + c ⋅ cos θ) ⋅ ( a − c ⋅ cos θ)
M’F’ = 2a – n
Das equações (i) e (ii),

No triângulo M’F’F, DD’² = MM’⋅2a

) ( 2c )
( 2a − n= 2 2 2
+ n − 2 ⋅ 2c ⋅ n ⋅ cos θ
18.
4a − 4an + n= 4c2 + n2 − 4 ⋅ n ⋅ c ⋅ cos θ
2 2
a) Desde que a abscissa do ponto A corresponde a um dos zeros de
4a2 − 4n ⋅ a= 4c2 − 4 ⋅ n ⋅ c ⋅ cos θ f, temos
θ 4a2 − 4c2
4n ⋅ a − 4 ⋅ n ⋅ c ⋅ cos= 3
x+ = 0 ⇒ x 2 − 2x − 3 = 0
n ⋅ a − n ⋅ c ⋅ cos θ= a − c 2 2 2− x
⇒ x =−1 ou x =3.
n ⋅ ( a − c ⋅ cos θ) = a2 − c2
a2 − c2 Logo, como o ponto A se encontra no semieixo negativo das abscissas,
n= vem A = (–1,0).
a − c ⋅ cos θ
b) Observando que C e D possuem a mesma ordenada, encontramos
b2
n= 3
a − c ⋅ cos θ 4= x+ ⇒ x 2 − 6x + 5 = 0
2− x
Dessa forma, ⇒= x 1 ou= x 5.

238 PROMILITARES.COM.BR
GEOMETRIA ANALÍTICA II

Assim, temos xC = 5 e, portanto, se G é o centro do retângulo ABCD, 2


 a 2 a2
então  x −  + y= ,y>0
2 4
 x + xC y A + yC  ou
G=  A , 
 2 2  2
 b b2
 −1+ 5 0 + 4  x2 +  y − = ,x<0
= ,   2  4
 2 2 
ou
= (2, 2).
2 2
 a  b a2 + b2
x −  + y =
−  , x > 0 e y > 0.
Portanto, como o ponto de encontro das assíntotas é o centro da  2   2  4
hipérbole, segue que xE = xG = 2.
Finalmente, como a hipérbole é simétrica em relação a G, podemos
concluir que a área pedida é dada por 20. Pelas informações do enunciado pode-se desenhar:
3⋅ 4
(EBC) − 4,7 = − 4,7
2
= 1,3cm2 .

19.
a) Considere a figura.

Sabendo que a reta s passa pelo ponto P e pela parábola descrita,


pode-se calcular o coeficiente angular a da mesma:
y − 0 = a ⋅ (x + 1) → pois P( −1, 0) ∈ s
=y ax + a
 2
y = 2x
y2
x=
2
As retas tangentes a C conduzidas por P formam um ângulo MP N y2
y= a + a → ay 2 − 2y + 2a= 0
cuja medida é 60º. Desse modo, sendo OP bissetriz de MP N , temos 2
∆ = ( −2)2 − 4 ⋅ a ⋅ 2a = 4 − 8a2
MP O
= 30°. Logo, segue que
mas ∆ =0, logo
 = OM ⇔ 1 = r ⇔ OP = 2r.
senMPO
OP 2 OP 2
4 − 8a2 = 0 → 2a2 = 1 → a =
2
Em consequência, o lugar geométrico procurado é a circunferência de
centro na origem e raio 2r, cuja equação é x² + y² = 4r². Sabendo o coeficiente angular e como as retas s e t são simétricas,
suas equações serão:
b) Considere a figura.
2 2
reta s → =
y x+
2 2
2 2
reta t → y =− x−
2 2
O ponto A pertence à reta s e à parábola descrita, portanto pode-se
escrever:
 2 2
=
y x+
 2 2
y 2 = 2x

y2
x=
O lugar geométrico dos pontos P que enxergam a união dos 2
segmentos OA e OB sob um ângulo de 90º corresponde à reunião de 2 y2 2
y= ⋅ + → y 2 2 − 4y + 2 2 = 0
três semicircunferências com centros em  a , 0 ,  0, b  e  a , b  , e 2 2 2
 2   2  2 2
∆ = ( −4)2 − 4 ⋅ 2 ⋅ 2 2 = 16 − 16

raios respectivamente iguais a a , b e a2 + b2 4 4 2


. =
y = →=
y 2
2 2 2 2 2 4
2
Portanto, o lugar geométrico dos pontos P = (x,y) é dado por y2 2
x= →x= →x= 1
2 2

PROMILITARES.COM.BR 239
GEOMETRIA ANALÍTICA II

Portanto, as coordenadas do ponto A serão A 1, 2 e do ponto B, ( ) Verificação:

por simetria, B 1, − 2 . ( ) Como a = - 5b,

Acerca da reta paralela a s, que chamaremos de reta s’, sabe-se que −x + 5y =10 x − 5y =10 ( i)
S: ⇔
ela passa pela parábola já descrita e pelo ponto B. Como se tratam 1 10b ⋅ ( x − 5y ) =
10b x − 50b y =
2 2 2
1 (ii)
de retas paralelas, possuem o mesmo coeficiente angular. Portanto,
pode-se escrever: Das equações (i) e (ii),
10b2 ⋅ ( −10 ) =1
y − ( − 2)=
2
2
(
⋅ (x − 1) → pois B 1, − 2 ∈ s' ) 1
b2 = − (impossível)
2 2 100
y + 2= x−
Assim, o lugar geométrico de todos os pares ordenados ( a, b ) ∈ 
2
2 2
2 3 2 que tornam impossível o sistema linear dado, é uma reta de equação
reta s' → =y x− a = −5b.
2 2
Quanto às coordenadas do ponto C, este pertence à parábola e à reta 12. O lugar geométrico pedido pode ser descrito pela equação:
s’, logo: 4x + 3y − 2 12x − 16y + 5 4x + 3y − 2 12x − 16y + 5
= → =
 2 3 2 42 + 32 122 + 162 5 20
=
y x−
 2 2
Desenvolvendo esta equação, tem-se:
y 2 = 2x

 12x − 16y + 5
y2 =
4x + 3y − 2 → 4x +
= 28y − 13 0
x= 4
2 
ou
2 y2 3 2  −12x + 16y − 5
y= ⋅ − → 2y 2 − 4y − 6 2 = 0 =4x + 3y − 2 → 28x
= − 4y − 3 0
2 2 2  4
∆= ( −4)2 − 4 ⋅ 2 ⋅ ( −6 2) = 16 + 48 → ∆ = 64
Sabendo que ambas as equações acima são nulas, seu produto
4±8 também será nulo. Assim, pode-se escrever:
=
y →= y 3 2 (obs. : solução negativa descartada)
2 2
( 4x + 28y − 13) ⋅ ( 28x − 4y − 3=) 112x 2 + 768xy − 376x − 112y 2 − 32y + 39
= 0
y2 (3 2)2
x= →x= →x= 9
2 2

(
Portanto, as coordenadas do ponto C serão C 9 , 3 2 e do ponto D, ) 13. Representando as regiões no plano, temos a seguinte figura:

(
por simetria, D 9 , − 3 2 . )
Assim, com as coordenadas dos pontos A, B, C e D é possível calcular
os comprimentos dos segmentos AB e CD:
AB= (
2 − − 2 → AB= 2 2 )
= 3 2 − −3 2 → CD
CD (
= 6 2 )
Por fim, o valor da razão AB/CD será:
AB 2 2 AB 1
= → =
CD 6 2 CD 3

EXERCÍCIOS DE COMBATE
01. D 06. E
02. D 07. A
a) d((P,S1) = 1 (abscissa do ponto P)
03. D 08. B
04. D 09. D d((Q,S1) = 22 + 33 = 13 (hipotenusa do triângulo retângulo de
vértices ( −3, 0), (0, 0) e (0, 2)).
05. D 10. E
b) São equidistantes de S1 e S2 os pontos com coordenadas maiores
11. Uma condição necessária para que o sistema linear seja impossível ou iguais a dois e pertencentes às bissetrizes dos quadrantes pares
é: e ímpares e também os pontos com coordenadas menores que 2
pertencentes à parábola com foco no ponto (0,2) e diretriz na reta
−1 5
y = 0.
a 2 =0
+ 5b2 10ab Determinando a equação da parábola , temos:
5
x2
 a2  y= (x − 0)2 + (y − 2)2 ⇒ y 2 = x 2 + y 2 − 4y + 4 ⇒ y = +1
−10ab − 5 ⋅  + 5b2  =0 4
 5 
−10ab − a2 − 25b2 =0 14. Transformemos as equações paramétricas na forma reduzida:
a2 + 10ab + 25b2 =
0 x−3 y −2
= cos θ e = sen θ
( a + 5b )
2
=
0 20 16
a = −5b Elevemos ao quadrado ambos os membros das duas equações:

240 PROMILITARES.COM.BR
GEOMETRIA ANALÍTICA II

(x − 3)2
= cos2 θ e
(y − 2)2 = OO' + O'P
OP = ( 3cos α,3sen α ) + ( 3cos α, −3sen α=)
= sen2 θ .
400 256
Somando membro a membro,
= ( 3cos α + 4 cos3 α − 3cosα,3sen α − 3sen α + 4sen3α )
(x − 3)2 (y − 2)2 (
Assim, P =( x,y ) = 4 cos3 α, 4sen3α )
+ =cos 2
θ + sen2 θ
 ou
400 256 1
b) Da relação fundamental da trigonometria, tem-se:
(x − 3)2 (y − 2)2
+ =
1
400 256 sen2α + cos2 α =1
2 2
 x  y 2 2
da qual tiramos:
 4  +  4  =→
1 x 3 +y 3 =3
3 3 16
a2 = 400 ⇒ a = 20    
b2 = 256 ⇒ b = 16 C(3, 2) e da relação notável a2 = b2 + c2
20. Considere a figura.
c= 400 − 256 = 12
Então, A2(-17, 2) e A1(23, 2); B2(3, -14) e B1(3, 18) e F2(-9, 2) e F1(15, 2).
As equações das diretrizes para o eixo maior horizontal são
a 20 100
x =m± ⇒ x =3± ou x= 3 ±
e 12 3
20

15. Sendo V = ( −2, 2) e p = 2, tem-se que a equação da parábola é


(y − 2)2 =2 ⋅ 2(x − ( −2)) ⇔ (y − 2)2 =4(x + 2).

Tomando arbitrariamente x = −1, encontramos


Sejam F o foco da parábola, H a projeção de F sobre MQ e F’ a projeção
(y − 2)2 =4( −1 + 2) ⇔ y − 2 =± 4 ⇔ y =0 ou y =4. de F sobre a diretriz d.
Logo, segue que ( −1, 0) e ( −1, 4) são pontos da parábola. Como M pertence à parábola e MQ é paralelo ao eixo de simetria e da
 ≡ RFF'
parábola, segue que MF = MQ. Além disso, FF’ = p e RMQ  =60°.
Desde que yp > 0, temos
Logo, do triângulo RFF’, obtemos
(yp − 2)2 = 4(0 + 2) ⇒ yp − 2 = 8 ⇒ yp = 2(1 + 2).
 = FF' ⇔ FR =
cosRFF'
p
FR cos60°
⇔ FR = 2p.
16. 5 u.c.
5 Do triângulo MRQ, vem
17. 10  MQ MQ
=
cosRMQ ⇔ cos60
= °
MF + FR MQ + 2p
18. P (8/9, 5/3) ⇔ MQ =2p =MF.

19. Assim, MR = 4p e, portanto,


a) Se a circunferência C’ deslocou-se α, então ela percorreu uma = QR ⇔ sen60
senRMQ = °
QR
distância d igual a: MR 4p
α d ⇔ QR =
2p 3.
2π 2πR
O resultado pedido é 2p + 4p + 2p 3 = 2p(3 + 3).
α ⋅ 2π ⋅ 4
d= → d = 4α

ANOTAÇÕES

Pode-se escrever:

PROMILITARES.COM.BR 241
GEOMETRIA ANALÍTICA II

ANOTAÇÕES

242 PROMILITARES.COM.BR
GEOMETRIA ESPACIAL I

PIRÂMIDE REGULAR a) As arestas laterais são todas congruentes.


Uma pirâmide é dita regular se sua base for um polígono regular b) As faces laterais são triângulos isósceles, mas não todos
e se o pé de sua altura estiver sobre o centro da base. congruentes entre si.
c) As arestas laterais formam ângulos iguais com o plano da base.
PROPRIEDADES DE UMA PIRÂMIDE REGULAR d) A projeção ortogonal da aresta lateral é igual ao raio do círculo
circunscrito à base.
1- As arestas laterais são iguais;
Na figura abaixo V-ABCD é uma pirâmide de base retangular.
2- As faces laterais são congruentes;
3- As arestas laterais formam ângulos iguais com o plano da base;
4- As faces laterais formam ângulos iguais com o plano da base e
também com a altura.
5- As alturas das faces laterais (apótemas da pirâmide) são
congruentes.
6- A projeção ortogonal do apótema da pirâmide sobre o plano da
base é o apótema da base.
7- A projeção ortogonal da aresta lateral sobre o plano da base é o
raio do círculo circunscrito à base.
Elementos
VM = apótema da pirâmide= m
PM = apótema da base=r
VP = altura da pirâmide=h
PD = raio do círculo circunscrito à base=R
VD = aresta lateral da pirâmide=a

VOLUME
Todo prisma triangular pode ser decomposto em três pirâmides
triangulares equivalentes.
Consideremos um prisma triangular qualquer, de bases ABC e
DEF.

Relações fundamentais
No VPM : h2 + r 2 =
m2 ;
No VPD : R2 + h2 =
a2.

Algumas pirâmides não regulares, que não têm como base Seccionando o prisma pelo plano (EAB), obtemos a pirâmide
polígonos regulares, apresentam propriedades similares aos das triangular E-ABC, que tem bases e altura iguais ao do prisma. Retirada
pirâmides regulares, são as que possuem polígonos inscritíveis em um esta primeira pirâmide, restará o sólido EABDF.
círculo e cuja a projeção ortogonal do vértice da pirâmide é o centro
desse círculo.

PROMILITARES.COM.BR 235
GEOMETRIA ESPACIAL I

Consideramos o plano (DEF) que divide o sólido em duas


pirâmides equivalentes, E-BDF e E-ABD, pois tem a mesma altura e
bases congruentes, os triângulos ABD e DEF que são metades do
paralelogramo ABDF.
Portanto todo prisma triangular pode ser decomposto em três
pirâmides triangulares equivalentes. Daí concluímos que o seu volume
é dado pela fórmula:
Bh
V= onde B é área da sua base e h é a sua altura.
3
A partir deste teorema é possível demonstrar que o volume de
qualquer pirâmide é igual a um terço do produto da área da base pela Se a pirâmide possui altura h e a seção está a uma distância h’
altura- para justificar tal fato basta observar que qualquer pirâmide do vértice, a semelhança das duas pirâmides implica as seguintes
pode ser dividida em pirâmides de base triangular. Esta divisão é feita relações:
dividindo-se a base da pirâmide por triângulos justapostos por meio
VG = h e VH=h'
de diagonais e definindo cada plano de divisão da pirâmide por uma
dessas diagonais da base e pelo vértice da pirâmide. VAB VDE; VAC VDF; VBC VEF ⇒ABC DEF ⇒
Corolário: Seja P uma pirâmide de área da base B e altura h então AB AC BC h
= = =
o volume da pirâmide é dado pela fórmula: DE DF EF h'
2
Bh S ABC  h 
V= . = 
3 SDEF  h'
3
VV − ABC  h 
Exemplo: = 
VV − DEF  h'
Uma pirâmide hexagonal dividida em quatro pirâmides
triangulares. As relações são válidas para pirâmides n-gonais, isto é a base pode
ser um polígono de n lados.
A obtenção das características da pirâmide menor possibilita
encontrar as características do tronco de pirâmide.

TRONCO DE PIRÂMIDE BASES


PARALELAS
Tronco de pirâmide de bases paralelas é o sólido limitado pela
superfície de um ângulo poliédrico e situado entre duas seções planas
e paralelas deste ângulo.
Podemos defini-lo também como o sólido obtido de uma pirâmide,
sendo a porção de pirâmide compreendida entre a base e um plano
que secciona a pirâmide em todas as arestas laterais, tendo por bases:
a seção de interseção e a base da pirâmide.
Os troncos de pirâmides podem ser de primeira ou de segunda
espécie.
Primeira espécie: todas as faces laterais são trapézios e as bases
são polígonos semelhantes.
SÓLIDOS SEMELHANTES
Dois sólidos são semelhantes quando suas faces são respectivamente
semelhantes e seus ângulos sólidos são respectivamente iguais.
Quando seccionamos uma pirâmide por um plano paralelo ao de
sua base, uma outra pirâmide semelhante à primeira é formada.
Uma seção transversal divide a pirâmide em duas partes: a que
contém o vértice é uma pirâmide semelhante à original e a outra parte
é chamada tronco de pirâmide de bases paralelas.

236 PROMILITARES.COM.BR
1 1 1
V1 − V2 =⋅ (Bh1 − bh2 ) =⋅ (B(h + h2 ) − bh2 ) =⋅GEOMETRIA
(Bh + (B − b) ⋅ hESPACIAL
2) I
3 3 3
h B h B− b h b
Por outro lado 1 = ⇒ = ⇒ h2 =
h2 b h2 b B− b
Segunda espécie: O tronco de pirâmide de segunda espécie é 1  h b  1
formado por duas pirâmides semelhantes e homotéticas em relação V1 − V2 = ⋅ Bh + (B − b) ⋅
3 
(
= ⋅ Bh + ( B + b) ⋅ h b ⇒
B − b  3
)
ao vértice V.
1
V = V1 − V2 = ⋅ h ⋅ (B + Bb + b)
3

Quando as bases são polígonos regulares, temos um tronco de


pirâmide regular, no qual é possível encontrar uma fórmula fechada
para a área lateral e para área total.

VOLUME DO TRONCO DE PIRÂMIDE


DE BASE PARALELA DE PRIMEIRA
ESPÉCIE
Sejam:
2P → perímetro da base maior
2p → perímetro da base menor
M → apótema da base maior
m → apótema da base menor
m’ → apótema do tronco
A → área lateral do tronco
At → área total do tronco
Al = (P + p) ⋅ m'
A t = (P + p) ⋅ m'+ P ⋅ M + p ⋅ m

TRONCO DE PRISMA
Tronco de prisma pode ser definido como cada um dos sólidos
obtidos, quando um prisma é seccionado por um plano não paralelo
às bases e que corte todas as suas arestas laterais.

Sejam:
VOLUME DO PRISMA TRIANGULAR
B = área da base maior
b = área da base menor
h= altura do tronco de pirâmide
h1 = altura da pirâmide maior
h2 = altura da pirâmide menor
V1 = volume da pirâmide maior
V2 = volume da pirâmide menor

Calculando o volume do tronco como a diferença entre os


volumes das pirâmides:

1 1 1 a, b, c → arestas laterais do tronco


V1 − V2 =⋅ (Bh1 − bh2 ) =⋅ (B(h + h2 ) − bh2 ) =⋅ (Bh + (B − b) ⋅ h2 )
3 3 3 S → seção reta do prisma
h B h B− b h b
Por outro lado 1 = ⇒ = ⇒ h2 =  a + b + c
h2 b h2 b B− b V = S 
 3 
1  h b  1
V1 − V2 = ⋅ Bh + (B − b) ⋅
3 
(
= ⋅ Bh + ( B + b) ⋅ h b ⇒
B − b  3
) Para calcular o volume de troncos de outras naturezas, basta
dividi-los em troncos de prismas triangulares.
1
V = V1 − V2 = ⋅ h ⋅ (B + Bb + b)
3

PROMILITARES.COM.BR 237
GEOMETRIA ESPACIAL I

EXERCÍCIOS DE
a) d) 5t
6 t

TREINAMENTO b)
c)
5 3t
6 3t
e) 3t

01. (AFA 1999) Qual deve ser a medida da altura de um prisma reto, 05. O recipiente da figura, que contém água, é um prisma reto cujas
cuja base é um triângulo eqüilátero de lado a, para que seu volume bases são triângulos equiláteros de altura 2. A superfície da água é
tenha valor a3? paralela á face ABCD. Se o volume ocupado pela água é metade do
volume do prisma, o valor de h é:
a 3 a 3
a) c)
4 3

3a 3 4a 3
b) d)
4 3

02. A figura abaixo corresponde à planificação de um prisma regular


hexagonal de altura 2a e perímetro da base igual a 3a.

6 1
a) d)
5 2
3
b) 3 e)
4

c) 2

06. (UNICAMP 2005) A figura abaixo apresenta um prisma reto cujas


bases são hexágonos regulares. Os lados dos hexágonos medem 5 cm
cada um e a altura do prisma mede 10 cm.
a) Calcule o volume do prisma.
b) Encontre a área da secção desse prisma pelo plano que passa
pelos pontos A, C e A’.
Determine a distância entre os pontos P e Q no prisma.

03. Uma caixa de cartolina em forma de um tronco de prisma


retangular reto foi planificada, obtendo-se o recorte de cartolina
indicado na figura abaixo. Para recuperar a caixa basta dobrar a
cartolina nas linhas pontilhadas. As dimensões das arestas, em
unidades de comprimentos, são como estão indicadas na figura.

07. No cubo de aresta  da figura, M e N são pontos médios de AB e


CD, respectivamente. Calcular a medida de MN.

a) Calcule o volume da caixa original.


b) Calcule a área da cartolina.

04. (EFOMM 1999) Um prisma quadrangular regular cuja área da base


vale t é equivalente a um prisma hexagonal regular de aresta da base
igual à diagonal da base do prisma quadrangular e cuja altura é o
dobro da aresta desse prisma. Então, a altura do prisma quadrangular
regular é:

238 PROMILITARES.COM.BR
GEOMETRIA ESPACIAL I

 2  2 12. Um cristal com a forma de um prisma hexagonal regular, após


a) d) ser cortado e polido, deu origem a um sólido de 12 faces triangulares
2 4
congruentes. Os vértices desse poliedro são os centros das faces do
 3  5 prisma, conforme representado na figura.
b) e)
2 4

 5
c)
2
08. (EN 2013) Num prisma hexagonal regular a área lateral é 75% da
área total. A razão entre a aresta lateral e a aresta da base é

2 5 2 3
a) d)
3 5
3 3 5 2
b) e)
2 3
5 3
c)
2 Calcule a razão entre os volumes do sólido e do prisma.

09. (AFA 2006) O produto da maior diagonal pela menor diagonal de 13. (INSPER 2011) Dois faraós do antigo Egito mandaram construir
um prisma hexagonal regular de área lateral igual a 144 cm² e volume seus túmulos, ambos na forma de pirâmides quadrangulares regulares,
igual a 144 3 cm³ é: num mesmo terreno plano, com os centros de suas bases distando
120 m. As duas pirâmides têm o mesmo volume, mas a área da base
a) 10 7 c) 10 21
de uma delas é o dobro da área da base da outra. Se a pirâmide mais
b) 20 7 d) 20 21 alta tem 100 m de altura, então a distância entre os vértices das duas
pirâmides, em metros, é igual a
10. (ITA 1998) Um poliedro convexo de 16 arestas é formado por a) 100. c) 130. e) 160.
faces triangulares e quadrangulares. Seccionando-o por um plano b) 120. d) 150.
convenientemente escolhido, dele se destaca um novo poliedro
convexo, que possui apenas faces quadrangulares. Este novo
14. Um poliedro convexo de 2n vértices apresenta faces triangulares
poliedro possui um vértice a menos que o original e uma face a
e quadrangulares. O número de faces quadrangulares, o número de
mais que o número de faces quadrangulares do original. Sendo m
faces triangulares e o número total de faces formam, nesta ordem,
e n, respectivamente, o número de faces e o número de vértices do
uma progressão aritmética. O número de arestas é:
poliedro original, então:
a) 5n – 5 c) 4n e) 4n + 4
a) m = 9, n = 7 d) m = 10, n = 8
b) 5n – 2 d) 4n – 2
b) m = n = 9 e) m = 7, n = 9
c) m = 8, n = 10 15. A razão entre a área de uma das faces e a área da base de uma
pirâmide regular de base hexagonal é k. Sabendo que o volume da
11. Uma empresa fabrica porta-joias com a forma de prisma pirâmide é de x³, temos que a altura da pirâmide mede:
hexagonal regular, com uma tampa no formato de pirâmide regular,
como mostrado na figura. 3
108k 2 − 1
3
54k 2 − 1
a) ⋅x d) 6
⋅x
6
6 12

108k 2 − 3 3
108k 2 − 3
b) 3 ⋅x e) ⋅x
12 6
12

108k 2 − 1
c) 3 ⋅x
12

16. (ITA 2018) A aresta lateral de uma pirâmide reta de base quadrada
25π
mede 13 cm e a área do círculo inscrito na base mede cm2.
2
Dois planos, π1 e π2, paralelos à base, decompõem a pirâmide em
três sólidos de mesmo volume. Determine a altura de cada um desses
As faces laterais do porta-joias são quadrados de lado medindo 6 cm sólidos.
e a altura da tampa também vale 6 cm. A parte externa das faces
laterais do porta-joias e de sua tampa são revestidas com um adesivo
17. (OBM 2005 F2) Um prisma é reto e tem como base um triângulo
especial, sendo necessário determinar a área total revestida para
equilátero. Um plano corta o prisma mas não corta nenhuma de suas
calcular o custo de fabricação do produto. A área da parte revestida,
bases, determinando uma secção triangular de lados a, b e c. Calcule
em cm2, é igual a
o lado da base do prisma em função de a, b e c.
a) 72(3 + 3). d) 27(8 + 7).
18. (IME 1968) Dado um prisma reto cuja base é um quadrado de
b) 36(6 + 5). e) 54(4 + 7). lado 10 m e a altura é 18 m, passa-se um plano que corta o prisma
c) 108(2 + 5). de modo que as três arestas consecutivas ficam medindo 10 m, 12 m
e 14  m. Calcular, em metros quadrados, a área lateral do prisma
truncado assim formado.

PROMILITARES.COM.BR 239
GEOMETRIA ESPACIAL I

19. (IME 1975) A figura abaixo mostra um prisma em que uma seção 07. (IME 2009) Os centros das faces de um tetraedro regular são os
π vértices de um tetraedro interno. Se a razão entre os volumes dos
reta é o triângulo retângulo isósceles ABC, no qual  = e AB = b.
2 m
tetraedros interno e original vale , onde m e n são inteiros positivos
A base superior do prisma é o triângulo equilátero MNP, de lado a. A n
base inferior do prisma é o triângulo RST, sendo E o ponto médio de primos entre si, então o valor de m + n é
RT e sendo SE = b, por construção. A menor distância entre as bases
a) 20 b) 24 c) 28 d) 30 e) 32
= NA + AS , sendo, por construção, NA
se encontra sobre a aresta NS
= b. O comprimento AS = d é escolhido de tal forma que o volume V1,
08. (ITA 2007) Considere uma pirâmide regular de base hexagonal,
do semi-prisma superior BACMNP, seja igual ao volume V2, do semi-
prisma inferior BACRST. Calcule: cujo apótema da base mede 3 cm. Seleciona-se a pirâmide por um
a) V1 em função de b. plano paralelo à base, obtendo-se um tronco de volume igual a 1 cm3
e uma nova pirâmide. Dado que a razão entre as alturas das pirâmides
b) d em função de b.
é 1 / 2 , a altura do tronco, em centímetros, é igual a
20. O pé de uma das alturas de um tetraedro é o ortocentro da face
oposta. Prove que quaisquer duas arestas opostas são ortogonais. a) ( 6 − 2) / 4 d) (3 2−2 3 /6 )
EXERCÍCIOS DE
b) ( 6 − 3) / 3 e) (2 )
6 − 2 / 22

COMBATE c) (3 3 − 6 ) / 21
09. (IME 2008) Um plano corta um cubo com aresta de comprimento
1 passando pelo ponto médio de três arestas concorrentes no vértice
01. (EN 2001) Um poliedro convexo de 25 arestas tem faces A e formando uma pirâmide, conforme a figura a seguir. Este processo
triangulares, quadrangulares e pentagonais. O número de faces é repetido para todos os vértices. As pirâmides obtidas são agrupadas
quadrangulares vale o dobro do número de faces pentagonais e o formando um octaedro cuja área da superfície externa é igual a:
número de faces triangulares excede o de faces quadrangulares em 4
unidades. Pode-se afirmar que o número de vértices deste poliedro é:
a) 14 b) 13 c) 11 d) 10

02. (ITA 1999) Um poliedro convexo de 10 vértices apresenta faces


triangulares e quadrangulares. O número de faces quadrangulares, o
número de faces triangulares e o número total de faces formam, nesta
ordem, uma progressão aritmética. O número de arestas é:
a) 10 b) 17 c) 20 d) 22 e) 23

03. (ITA 1990) Considere um prisma triangular regular cuja aresta da 3


base mede x cm. Sua altura é igual ao menor lado de um triângulo a) d) 2
2
ABC inscritível num círculo de raio x cm. Sabendo-se que o triângulo
ABC é semelhante ao triângulo de lados 3 cm, 4 cm e 5 cm, o volume b) 3 e) 2 2
do prisma em cm3 é:

2 3 3 3 c) 1
a) x d) x
3 10
10. (IME 2012) Uma pirâmide regular possui como base um
2 2 3 dodecágono de aresta a. As faces laterais fazem um ângulo de 15º
b) x e) n.d.a.
5 com o plano da base. Determine o volume desta pirâmide em função
3 3 3 de a.
c) x
10
a3 3+2 3−2
04. (ITA 1995) Dado um prisma hexagonal regular, sabe-se que sua a) ⋅ d) a3 ⋅
altura mede 3 cm e que sua área lateral é o dobro da área de sua base. 2 2− 3 2+ 3
O volume deste prisma, em cm3, é:
a3 3−2 2− 3
e) b) ⋅ e) a3 ⋅
a) 27 3 c) 54 3 17 5 2 2+ 3 3+2
b) 13 2 d) 12
3+2
c) a3 ⋅
05. (EN 2005) Um octaedro regular está inscrito num cubo de aresta 2− 3
a. A razão entre o volume do cubo e o volume do octaedro é:
a) 2 b) 3 c) 4 d) 5 e) 6 11. (EN 2003)Numa pirâmide triangular regular, o apótema mede 39
cm e o apótema da base mede 15 cm. Interceptando-se esta pirâmide
06. (IME 2007 F1) O volume do octaedro cujos vértices são os pontos por um plano paralelo a sua base, distante 24 cm de seu vértice,
médios das arestas de um tetraedro regular de Volume V é: obtém-se um tronco de pirâmide. Calcule o volume deste tronco.

V V V 2 3 12. (IME 2010) A área da superfície lateral de uma pirâmide


a) b) c) d) V e) V quadrangular regular SABCD é duas vezes maior do que a área de
2 4 8 2 2
sua base ABCD. Nas faces SAD e SDC traçam-se as medianas AQ e DP
Calcule o ângulo entre estas medianas.

240 PROMILITARES.COM.BR
GEOMETRIA ESPACIAL I

13. (IME 2004) Considere uma pirâmide regular de altura h, cuja base
é um hexágono ABCDEF de lado a. Um plano perpendicular à base e GABARITO
contendo os pontos médios das arestas AB e BC divide a pirâmide em
EXERCÍCIOS DE TREINAMENTO
dois poliedros. Calcule a razão entre os volumes destes dois poliedros.
01.
14. (ITA 2017) Considere o cubo ABCDEFGH de aresta 2 tal que: a2 3
ABCD é o quadrado da base inferior; EFGH, o quadrado da base V = SB ⋅ h = ⋅ h = a3
4
superior e AE, BF, CG e DH são as arestas verticais. Sejam L, M e N os
4a 4 3
pontos médios das arestas AB, CG e GH, respectivamente. Determine ⇔ h= = a
a área do triângulo LMN. 3 3

15. (ITA 2018) Um poliedro convexo tem faces triangulares e a


quadrangulares. Sabe-se que o número de arestas, o número de faces 02. O lado do hexágono é .
2
triangulares e o número de faces quadrangulares formam, nessa
ordem, uma progressão aritmética de razão –5. Determine o número A diagonal do retângulo na planificação é:
de vértices do poliedro. d=
2
( 3a)2 + ( 2a)2 ⇒ =
d 13a

16. (FUVEST 2002) Um bloco retangular (isto é, um paralelepípedo d 13


PQ= = a
reto-retângulo) de base quadrada de lado 4 cm e altura 20 3 , com 2 2
2
de seu volume cheio de água, está inclinado sobre uma das arestas
3 03.
da base, formando um ângulo de 30º com o solo (ver seção lateral a) A figura é um prisma reto cuja base é um trapézio retângulo.
abaixo). Determine a altura h do nível da água em relação ao solo.

 10 + 4 
=V  ⋅ 8=
 ⋅ 7 392 cm
3
 2

 10 + 4 
=
b) ST  ⋅ 8 + 32
= ⋅ 7 280cm2
 2 

04. Aresta da base do prisma quadrangular: t


17. Uma das diagonais de um paralelepípedo reto retângulo forma Diagonal da base do prisma quadrangular : 2t
com as arestas que partem de um dos seus extremos os ângulos α, β
VQ = VH ⇒ SBQ ⋅ hQ = SBH ⋅ hH
e γ. Calcule o valor de sen2 α + sen2 β + sen2 γ .
( 2t )
2
3
18. (IME 2011) A base de um prisma reto ABCA1B1C1 é um triângulo ⇒ t ⋅ hQ = 6 ⋅ ⋅ ( 2 t ) ⇔ hQ = 6 3t
4
com o lado AB igual ao lado AC. O valor do segmento CD vale x,
onde D é o ponto médio da aresta lateral AA1. Sabendo que α é o
ângulo ACB e β é o ângulo DCA, determine a área lateral do prisma 05. A água ocupa o volume de um prisma reto de mesma altura que
em função de x, α e β . o recipiente.
Assim, a razão entre o volume ocupado pela água e o volume do
19. (IME A interseção de um plano com as arestas de um prisma recipiente é igual à razão entre as bases do prismas.
reto triangular determina, a partir da base, segmentos de 3, 4 e x
metros sobre as arestas. Calcule o valor de x para que os dois volumes As bases são triângulos equiláteros semelhantes de alturas h e 2. Logo,
resultantes sejam equivalentes, sendo a aresta do prisma igual a 10 Vágua 2
 h 1
metros. =   = ⇔h= 2
Vrecip.  2 2

20. (EN 1998) A altura de um paralelepípedo retângulo mede 60 cm


e sua base é um quadrado. A diagonal do paralelepípedo forma um 06.
ângulo de 60º com o plano da base. O volume do paralelepípedo
retângulo é em cm3  52 3 
a) V =  6 ⋅  ⋅ 10 = 375 3 cm
3

a) 12000  4 
b) 18000 b) AC=
2 ˆ
AB2 + BC2 − 2 ⋅ AB ⋅ BC ⋅ cos ABC
c) 24000  1
⇔ AC2 = 52 + 52 − 2 ⋅ 5 ⋅ 5 ⋅  − 
d) 36000  2
⇔ AC =
5 3
S = AC ⋅ AA' = 5 3 ⋅ 10 = 50 3 cm2

PROMILITARES.COM.BR 241
GEOMETRIA ESPACIAL I

07. F'= f4=' f4 + 1


V'= V − 1
4f4 ' = 2A' ⇒ 4F' = 2A' ⇔ A' = 2F'
V'+ F' = A'+ 2 ⇒ V' = 2F'− F'+ 2 = F'+ 2
⇒ V − 1 = f4 + 3

Voltando ao poliedro original:


V + f3 + f4 =
18

⇒ 3f3 + 4f4= 32 ⇒ f3= 4, f4= 5 e V= 9
V − f = 4
 4

⇒ m =n = 9
Um poliedro convexo que satisfaz as condições do enunciado é o da
figura abaixo.
Seja P o ponto médio de AE, então o triângulo MPN é retângulo.
 2
MP =    52  5
2  ⇒ MN =   +  = ⇒ MN =
2 2
 2  4 2
PN =  

08. Considerando x a medida da aresta da base, y a medida da aresta


lateral, temos:
11. Considere a figura, em que V é o vértice da pirâmide, O é o centro
A área lateral é 75% da área total: 6xy = 75% ⋅ AT
da base e M é o ponto médio da aresta AB
6 ⋅ x2 3
A área da base é 12,5% da área total: = 12,5% ⋅ A T
4
6xy 75% ⋅ A T y 3 3
= = ⇒
x 2 ⋅ 3 12,5% ⋅ A T x 2
6
4

09.

Desse modo, como AB = 6 cm, vem


AB 6
OM= ⇔ OM= = 3 3 cm.
2tg30° 3
2⋅
3
Aplicando o Teorema de Pitágoras no triângulo OVM, encontramos
2 2 2 2
VM =OV + OM ⇒ VM =62 + (3 3)2
A maior diagonal é AB e a manor é AE. ⇒ VM =
3 7 cm.
SL = 6ah = 144 ⇔ ah = 24
Portanto, o resultado pedido é dado por
a2 3
V = SB ⋅ h = 6⋅ h = 144 3 ⇔ a2h = 96  2 AB ⋅ VM
4 6 ⋅  AB + = 6 ⋅ (62 + 3 ⋅ 3 7)

= a 4 e=h 6  2 
∆ABC : AB2 = ( 2a) + h2 = 64 + 36 ⇔ AB = 10 = 54(4 + 7)cm2 .
2

∆ADE : AE2 = ( a 3 ) + h2 = 48 + 36 ⇔ AE = 2 21
2

⇒ AB ⋅ AE =
20 21 12. Sendo 2 a medida da aresta da base do prisma, considere a
seguinte vista superior.

10. No poliedro original, temos:


3f3 + 4f4 = 2A = 32
V + F = A + 2 ⇒ V + F = 18
No novo poliedro, temos:

242 PROMILITARES.COM.BR
GEOMETRIA ESPACIAL I

Aplicando a Lei dos Cossenos no triângulo ABC, obtemos Seja ap o apótema da pirâmide, então a área de cada face lateral é
 1  ⋅ ap
x 2 = 2 + 2 − 2 ⋅  ⋅  ⋅ cos120° ⇔ x 2 = 22 − 22 ⋅  −  Sf = .
 2 2
 ⋅ ap
⇒x= 3,
Sf 2
Do enunciado, temos: =k ⇔ =k ⇔ ap =3 3  ⋅ k .
em que x é a medida da aresta da base das pirâmides hexagonais Sb 3 3 2
regulares obtidas pelo corte. 2
Portanto, se h é a altura do prisma, segue que a razão pedida é dada Aplicando o teorema de Pitágoras no triângulo formado pelo apótema
por da base, altura da pirâmide e apótema da pirâmide, temos:
1 3 ⋅ (  3)2 h 2
2⋅ ⋅ ⋅   3  3
h2 + ab2 = ap2 ⇔ h2 = ( 3 3 ⋅ k ) − 
2
3 2 2 = 1.  2 
=  27k 2 −  2 ⇔

3 ⋅ (2 ) 2
4 4
⋅h 4h2
2 ⇒ 2 = 2
108k − 3 .
O volume da pirâmide é
13. Sejam A1 e A2, as áreas das bases das pirâmides.
Como os volumes são iguais, temos 1 1 3 3 2 3 4h2
V= ⋅ Sb ⋅ h = ⋅ ⋅ h = x3 ⇔ ⋅ ⋅ h = x3
3 3 2 2 108k 2 − 3
1 1
⋅ A1 ⋅ O1V1 =⋅ A 2 ⋅ O2V2 ⇔ A1 ⋅ O1V1 =A 2 ⋅ O2V2. 108k 2 − 3 3 3
108k 2 − 3
3 3 =
⇔ h3 ⋅ x=
⇔h ⋅x
6
2 3 12
Dado que A1 = 2 ⋅ A2, vem
2 ⋅ A 2 ⋅ O1V1 =A 2 ⋅ O2V2 ⇔ O2V2 =2 ⋅ O1V1.
16. Do enunciado, temos:
Assim, a pirâmide mais alta tem a base menor e, portanto,
O2V2 = 100 m ⇒ O1V1= 50 m.

 = 2r
25π
πr2 =
2
25
Como o terreno é plano, segue que V1P||O1P, sendo P o pé da r =
2

2
perpendicular baixada de V1 sobre O2V2. Daí, =
VP
1 O=
1O2 120 m e 2 ⋅ 25
V2P = 100 − 50 = 50 m. r =
2

4
Finalmente, aplicando o Teorema de Pitágoras no triângulo V1PV2, 5 2
r=
obtemos 2
2
V1V2 =502 + 1202 ⇒ V1V2 =130 m. =5 2

14. V = 2n
PA : f4 , f3 , F ⇔ 2f3 =f4 + F
F = f3 + f4 ⇒ 2f3 = f4 + ( f3 + f4 ) ⇔ f3 = 2f4 ∧ F = 3f4
3f3 + 4f4 = 3 ( 2f4 ) + 4f4 =
2A ⇔ 2A =⋅ 10f4 ⇔ A =5f4

Relação de Euler:
V + F = A + 2 ⇔ 2n + ( 3f4 ) = 5f4 + 2 ⇔ f4 = n − 1
⇒ A = 5f4 = 5 ⋅ (n − 1) = 5n − 5

15. Seja a base da pirâmide um hexágono de lado , então o apótema


 3 2 3 3 3 2
da base é ab = e a área da base será Sb =
6⋅ = .
2 4 2

PROMILITARES.COM.BR 243
GEOMETRIA ESPACIAL I

1 Podemos supor, sem perda de generalidade, a configuração acima e,


OC = ⋅ 5 2 ⋅ 2 portanto, pelo teorema de Pitágoras:
2
OC = 5
( )
2
2 + b2 − 2 − a2 − 2 = c2 ⇔
No triângulo AOC,
13=
2
52 + ( AO)
2 ( )( )
2 b2 − 2 a2 − 2 = a2 + b2 − c2 − 2 ⇔

AO = 12 (
4 b2a2 − b22 − a22 +  4 = )
Os sólidos AJKLM e ABCDE são semelhantes, logo,  4 + a4 + b4 + c 4 − 2a22 − 2b22 + 2c22 + 2a2b2 − 2a2c2 − 2b2c2

V  h1 
3 (
⇔ 3 4 − 2 a2 + b2 + c2 2 )
= 
3V  12 −(a4 + b4 + c 4 − 2a2b2 − 2a2c2 − 2b2c2 ) =
0
3
1  h1  O discriminante da equação do segundo grau acima, em ², é
= 
3  12 2
∆ =  −2(a2 + b2 + c2 )
1 h
= 1
3
3 12 (
+4 ⋅ 3 ⋅ a4 + b4 + c 4 − 2a2b2 − 2a2c2 − 2b2c2 = )
12 16(a4 + b4 + c 4 − a2b2 − a2c2 − b2c2 ).
h1 = 3 cm
3
2(a2 + b2 + c2 ) ± 16(a4 + b4 + c 4 − a2b2 − a2c2 − b2c2 )
Logo 2 =
Os sólidos AJKLM e AFGHI são semelhantes, logo, 2⋅ 3
3
V  h1  (a2 + b2 + c2 ) ± 2 a4 + b4 + c 4 − a2b2 − a2c2 − b2c2
= ⇔ 2 =
2V  h1 + h2  3
3 De fato, observando que  é menor ou igual a min {a, b, c}, temos
1  h1 
=
2  h1 + h2  a2 + b2 + c2
2 ≤ . Portanto
1 h1 3
=
3
2 h1 + h2
(a2 + b2 + c2 ) − 2 a4 + b4 + c 4 − a2b2 − a2c2 − b2c2
h1 + h2 =
3
2 h1 =
3
=
h2 3
2 h1 − h1
h2 =h1 ⋅ ( 3
2 −1 ) Observação: Outra maneira de obter as equações é trabalhar em R3,
supondo, sem perda de generalidade, que C = (0, 0, 0), A = (,0,h) e
12
Como h1 = ,   3 
3
3 B= , ,z , com h,z ≥ 0 . Obteríamos, então, as equações
2 2 
h2 =
12
3
3
⋅ ( 3
2 −1 ) 2 + h=
2
a2 , 2 + z=
2
b2 e 2 + (z − h)=
2
c2 , que nos leva à mesma
2 −1
3
equação da solução acima.
=h2 3
⋅ 12 cm
3
h1 + h2 + h3 = 12 18. Seja a base o quadrado ABCD e a seção o quadrilátero EFGH, onde
AE = 10, BF = 12 e CG = 14.
Então, Sendo M o ponto médio de EG e N o ponto médio de AC, então

( ) 3− 32
3
12 12 AE + CG 10 + 14
3
+ ⋅ 3
2 − 1 + h3 = 12 ⇒ h3 = cm =
MN = = 12
3 33 3
3 2 2
Resposta: As alturas dos sólidos formados são: Como BF = MN = 12, então DH = 12.
12 3
2 −1 3
3− 32 Logo a área lateral é dada por
=h1 =
cm, h2 ⋅ 12 cm e h3 = 3
cm.
3
3 3
3 3  10 + 12 12 + 14 
SL =2 ⋅  ⋅ 10 + ⋅ 10 =480 m2
 2 2 
17. (Referência Revista Eureka N° 24 de 2006)
19.
a) Consideremos o plano (A’B’C’) paralelo ao plano (ABC), onde
A’ ≡ N.
∆NB'M ≡ ∆NC'P ⇒ MB' =PC'
⇒ MP  BC ⇒ MP = BC = b 2
⇒ MN =NP =MP =b 2
⇒ MB' =PC' =b
 BM + AN + CP 
V1 =
SABC ⋅   =
 3
b2  2b + b + 2b  5 3
= ⋅  = b
2  3 6

244 PROMILITARES.COM.BR
GEOMETRIA ESPACIAL I

b) Consideremos o plano (A”B”C”) paralelo ao plano (ABC), onde


A” ≡ S.
b 2
Seja F ponto médio de B”C”, então SF = .
2
No triângulo SEF, temos:
2
 b 2 b 2
EF2 = b2 −  ⇒ EF =
 2  2
Como EF é base média do trapézio B”C”TR, então
RB"+ TC" =⋅
2 EF

 AS + BR + CT 
V2 = SABC ⋅  =
 3 
 AS + BB"+ B"R + CC''+ C"T 
= SABC ⋅   =
 3
 3d + b 2 
= SABC ⋅  
 3
5b  3d + b 2 
V1 = V2 ⇒ SABC ⋅ = SABC ⋅  
3  3
5− 2
⇔d= b
3

20. Cada aresta da base do tetraedro é perpendicular a uma das


alturas da base e ortogonal à altura do tetraedro, logo cada aresta da
base é perpendicular ao plano determinado pela altura da base citada
e pela altura do tetraedro, mas esse plano contém a aresta oposta.
Assim duas arestas opostas são sempre ortogonais.

EXERCÍCIOS DE COMBATE
01. B 3
12. θ =arc cos
02. C 13
03. C 13. 95

04. C 14. 3 u.a.


05. E 15. Seis vértices
06. A 16. 21 cm
07. C 17. 2
08. C =
18. SL 2x 2 sen2β (1 + cos α )
09. B 19. 8
10. A 20. D
11. 5700 3 cm3

ANOTAÇÕES

PROMILITARES.COM.BR 245
GEOMETRIA ESPACIAL I

ANOTAÇÕES

246 PROMILITARES.COM.BR
GEOMETRIA ESPACIAL II

CILINDRO Seção meridiana de um cilindro circular (quadrilátero ABCD) são


as seções produzidas por planos que contém o eixo OO’ do cilindro.

DEFINIÇÃO
Superfície cilíndrica é a superfície gerada por uma reta g (geratriz)
de direção constante que se move apoiada numa linha fixa (diretriz),
portanto a natureza de uma superfície cilíndrica depende da sua
diretriz. Existem superfícies cilíndricas circulares, elíticas, parabólicas,
etc. A superfície será aberta se a sua diretriz for aberta, fechada, se
sua diretriz for fechada.
Superfície cilíndrica de revolução é a superfície gerada pela
rotação de uma reta g (geratriz) ao redor de uma reta fixa paralela
OO’ (eixo). A distância constante entre g e OO’ é o raio.
Cilindro é o sólido limitado por uma superfície cilíndrica fechada e
por dois planos paralelos entre si que intersectam todas as geratrizes A seção meridiana de um cilindro oblíquo é um paralelogramo e
da superfície e produzem duas seções planas paralelas. Essas seções do cilindro reto é um retângulo.
são denominadas bases do cilindro e a distância entre elas é a sua
Cilindro equilátero é o cilindro circular reto no qual seção
altura.
meridiana é um quadrado.
h = 2r

ÁREA E VOLUME
Planificando o cilindro circular reto obtemos:

Seção reta do cilindro é a seção reta da superfície cilíndrica que


o limita.
Um cilindro é reto ou oblíquo conforme suas geratrizes
sejam perpendiculares ou oblíquas aos planos de suas bases e são
denominados circulares se suas bases forem circulares.
No cilindro reto a geratriz é igual à altura. No cilindro oblíquo a
geratriz é diferente da sua altura.
O cilindro circular reto também é conhecido como cilindro
de revolução, pois é o sólido gerado pela rotação completa de um O desenvolvimento da superfície lateral de um cilindro é um
retângulo em torno de um de seus lados. retângulo de base igual a 2⋅π⋅r, que é o comprimento da base do
cilindro, e a altura h. Daí a sua área lateral é igual a A1 = 2π⋅r⋅h.
E a sua área total é dada pela soma da área lateral com as áreas
das duas bases.
área total=A t = Al + 2A b= 2 ⋅ π ⋅ r ⋅ h + 2 ⋅ π ⋅ r 2 = 2 ⋅ π ⋅ r ⋅ (r + h) .

VOLUME DO CILINDRO CIRCULAR


O volume de um cilindro circular reto pode ser considerado como
o limite do volume de um prisma regular inscrito no cilindro, quando o
número de faces laterais cresce indefinidamente. Portanto, o volume
do cilindro circular reto é igual a:

Vcilindro circular =Ab ⋅ h =πr 2 ⋅ h


Ab = área da base
h= Altura

PROMILITARES.COM.BR 251
GEOMETRIA ESPACIAL II

Para o cilindro oblíquo vale raciocínio análogo e a fórmula é a A reta que passa pelos pontos A e V, centro da base e o vértice é
mesma. chamada de eixo do cone.
No caso em o segmento AV é perpendicular à base, o cone é
TRONCO DE CILINDRO circular reto, caso contrário será oblíquo.

Tronco de cilindro é o sólido limitado por uma superfície cilíndrica


e duas seções não paralelas secantes às geratrizes. CONE CIRCULAR RETO OU CONE DE
REVOLUÇÃO
TEOREMA No cone circular reto o segmento que une os pontos A e V é
chamado de altura e o segmento que une os pontos B e V é chamado
A seção produzida numa superfície cilíndrica de revolução por um
de geratriz, V é o vértice do cone e o círculo R é a base do cone, a
plano obliquo às suas geratrizes é uma elipse.
distância de A até B é o raio da base.
Dado um tronco de cilindro circular de raio r e sendo d a distância
entre os centros das seções, é possível obter um cilindro circular reto
equivalente e de mesma área lateral com raio r e altura d.

No cone circular reto vale a seguinte relação


g² = r² + h²
A medida d é a base média do trapézio da seção meridiana.

Vtronco =πr 2 ⋅ d DEFINIÇÃO


Alateral do tronco = 2πr ⋅ d O cone circular reto é criado pela rotação de um triângulo
retângulo em torno de um dos seus catetos. Este é o motivo pelo qual
A total do tronco = Alateral + A seção 1 + A seção 2
a superfície de um cone recebe o nome de superfície de revolução.

As seções que constituem as bases de um tronco de cilindro


circular são elipses. A área das seções pode ser calculada através
da fórmula da área da elipse de eixo maior 2a e eixo menor 2b:
Selipse = π⋅a⋅b.

CONE
DEFINIÇÃO
Superfície cônica é a superfície gerada por uma reta, geratriz, que
se desloca passando por um ponto fixo chamada de vértice, apoiando-
se por sobre uma curva, chamada de diretriz.
Cone é o sólido limitado por uma superfície cônica fechada e por
um plano que não contém o vértice e intersecta todas as geratrizes
da superfície. A seção produzida pelo plano é a base, e a distância do
vértice à base é a altura do cone. O cone será circular, elítico, ou de um
outro formato de acordo com a natureza da sua base. Estudaremos
os cones circulares que podem também serem definidos da seguinte
forma: considere um círculo R contido num plano α, e V um ponto
que não está em α. O conjunto de todos os segmentos de reta que
unem o os pontos de R ao ponto B formam um cone circular.

SEÇÃO MERIDIANA
Interseção do cone com um plano que contém a reta que liga o
vértice ao centro da base.

252 PROMILITARES.COM.BR
GEOMETRIA ESPACIAL II

Na figura abaixo o triângulo VBC é a seção meridiana. TRONCO DE CONE DE BASES


PARALELAS
A rotação completa do trapézio retângulo, em A e B’ , em torno
do eixo, gera o sólido conhecido como tronco de cone de primeira
espécie.

No cone oblíquo a seção meridiana é um triângulo escaleno. No


cone reto a seção meridiana é um triângulo isósceles.

CONE EQUILÁTERO
É aquele cuja seção meridiana é um triângulo equilátero. Nesse
caso, g = 2r e h = r 3 .

ÁREA LATERAL (CONE CIRCULAR RETO)


O desenvolvimento da superfície lateral de um cone circular reto
é um setor circular de raio g e comprimento igual ao da circunferência
da base do cone, 2πR.

2πR =αg onde α é o ângulo central do setor circular


2πR α 2πR
α= ⇒ AL = ⋅ g2 = ⋅ g2 = π ⋅R ⋅ g
g 2 2g
O Volume do tronco de cone de bases paralelas de primeira
AL = πRg
πh ( 2
espécie é igual a =
V R + Rr + r2 )
3
ÁREA TOTAL (CONE CIRCULAR RETO) Com a demonstração análoga a utilizada no cálculo do volume
É a soma da área lateral com a área da base. do tronco de pirâmide.

A total = πr ( g + r )
Alateral + Abase =
ESFERA
CÍRCULOS DE UMA ESFERA
VOLUME
O volume do cone circular reto é igual ao limite do volume da TEOREMA
pirâmide regular inscrita, quando o número de faces laterais cresce A seção de uma esfera por um plano é um círculo.
indefinidamente. Portanto o volume do cone é igual ao produto de R = raio da esfera;
um terço da área da base pela altura.
r = raio do círculo secção (círculo menor);
1 1
V= Abase ⋅ h = πr 2h d = distância do plano secante ao centro da esfera;
3 3
=r R2 − d2
(A fórmula também é válida para um cone circular oblíquo)

PROMILITARES.COM.BR 253
GEOMETRIA ESPACIAL II

POLOS E DISTÂNCIA POLAR – NOTAÇÕES


p1, p2 = distâncias polares de um ponto M

p1 + p2 = 4R2 Volume gerado pelo triângulo qualquer


1
p1.p=
2 2R ⋅ r =( ABC) ( área gerada BC) ⋅ h (12)
3
p=
2
1 2R ⋅ PS
1

p1 = distância polar em relação ao polo mais próximo do círculo VOLUME GERADO POR UM SETOR
considerado,
POLIGONAL REGULAR
=p1 2R (R − d)

ÁREA GERADA PELO SEGMENTO DE RETA


A área da superfície gerada por um segmento de reta, quando
efetua uma volta completa em torno deum eixo situada no seu plano
e não o atravessando é igual ao produto da sua projeção ortogonal
sobre o eixo pela circunferência cujo raio é a parte da mediatriz do
segmento gerador, compreendida entre este e o eixo. (I, II, III e IV)

Volume gerado pelo setor poligonal regular


1
(O − ABCD) = ⋅ ( área ABCD) ⋅ OM
3

1 2
Volume (O − ABCD)= ⋅ ( 2πOM ⋅) OM= π ⋅ OM ⋅ h
3 3

VOLUME DAS PARTES DA ESFERA


Há três posições diferentes (I, II, III) 2 2
Volume do setor esférico = πR ⋅ h ;
3
Área gerada pelo segmento de reta AB = 2π OM ⋅ h = 2π m h
4 3
Área gerada pela linha poligonal regular ( ABCD) =π
2 OM ⋅ h Volume da esfera = πR ;
3
m = OM = apótema da poligonal (IV)
1 3
Volume da esfera = πD ;
6
ÁREA DAS PARTES DA ESFERA
R = raio da esfera; πR3 ⋅ n°
Volume da cunha = ;
270°
D = diâmetro da esfera;
h = altura da zona; 1
Volume do anel esférico = πAB2 ⋅ h ;
6
AB = corda arco gerador da calota.
Área da zona esférica = 2πrh1;
Área da calota = π . AB²;
Área da esfera = 4πR²;
Área da esfera = πD²;
πR2 ⋅ n°
Área fuso esférico cujo ângulo mede n° =
90°

VOLUME GERADO POR UM TRIÂNGULO


O volume gerado por um triângulo que gira em torno de um eixo
situado no seu plano, sem o atravessar e passando por um de seus
vértices é igual ao produto da área gerada pelo lado oposto ao vértice
sobre o eixo por um terço da altura correspondente e esse lado (I, II, III). V1 = volume do segmento esférico de duas bases;
Devemos considerar três distintas: V2 = volume do segmento esférico de uma base;

254 PROMILITARES.COM.BR
GEOMETRIA ESPACIAL II

06. O diâmetro de uma das bases de um tronco de cone é 100 mm. Se


o diâmetro é aumentado de 21%, mantendo-se a altura é o tamanho
da outra base inalterados, o volume do tronco de cone também
aumentará 21%. Qual o diâmetro da outra base?
a) 90 mm c) 105 mm e) 121 mm
b) 100 mm d) 110 mm

07. (FUVEST 2017) Um reservatório de água tem o formato de um


1 3 1 ( 2
V1 = πh + πh m + n2 ) cone circular reto. O diâmetro de sua base (que está apoiada sobre o
6 2 chão horizontal) é igual a 8 m. Sua altura é igual a 12 m. A partir de
1 3 1 um instante em que o reservatório está completamente vazio, inicia-se
V2 = πh + πm2 ⋅ h (n = 0)
6 2 seu enchimento com água a uma vazão constante de 500 litros por
1 2 minuto.
V2 =πh 3R − h)
(
3 O tempo gasto para que o nível de água atinja metade da altura do
reservatório é de, aproximadamente,
EXERCÍCIOS DE Dados:

TREINAMENTO
- π é aproximadamente 3,14.
- O volume V do cone circular reto de altura h e raio da base r é
1 2
V= π r h.
3
01. (AFA 2011) Uma vinícola armazena o vinho produzido em um a) 4 horas e 50 minutos. d) 6 horas e 20 minutos.
tanque cilíndrico (reto) com sua capacidade máxima ocupada. b) 5 horas e 20 minutos. e) 6 horas e 50 minutos.
Esse vinho será distribuído igualmente em barris idênticos também
cilíndricos (retos) e vendidos para vários mercados de uma cidade. c) 5 horas e 50 minutos.
Sabe-se que cada mercado receberá 2 barris de vinho, com altura igual
08. (FUVEST 2016) Cada aresta do tetraedro regular ABCD mede 10.
a 1/5 da altura do tanque e com diâmetro da base igual a 1/4 do
Por um ponto P na aresta AC, passa o plano α paralelo às arestas AB e
diâmetro da base do tanque. Nessas condições, a quantidade de x de
CD Dado que AP = 3 o quadrilátero determinado pelas interseções de
mercados que receberão os barris (com capacidade máxima ocupada)
α com as arestas do tetraedro tem área igual a
é tal que x pertence ao intervalo
a) 0 < x < 20 b) 20 ≤ x < 40 c) 40 ≤ x < 60 d) 60 ≤ x < 80 21 2 30 30 3
a) 21 b) c) 30 d) e)
2 2 2
6
02. (ITA 2010) Um cilindro reto de altura cm está inscrito num 09. (FUVEST 2011) A esfera ε, de centro O e raio r > 0, é tangente
3
tetraedro regular e tem sua base em uma das faces do tetraedro. Se ao plano α. O plano β é paralelo a α e contém O. Nessas condições, o
as arestas do tetraedro medem 3 cm, o volume do cilindro, em cm3, volume da pirâmide que tem como base um hexágono regular inscrito
é igual a na intersecção de ε com β e, como vértice, um ponto em α, é igual a
3r 3 5 3r 3 3 3r 3 7 3r 3 3r 3
π 3 π 3 π 6 π 6 π a) b) c) d) e)
a) b) c) d) e) 4 16 8 16 2
4 6 6 9 3

10. (ITA 2013) No sistema xOy os pontos A = (2,0), B = (2,5) e C = (0,1)


03. (EN 2011) Três cilindros circulares retos e iguais têm raio da base são vértices de um triângulo inscrito na base de um cilindro circular
R, são tangentes entre si dois a dois e estão apoiados verticalmente volume
sobre um plano. Se os cilindros têm altura H, então o volume do sólido reto de altura 8. Para este cilindro, a razão ,
área total da superfície
compreendido entre os cilindros vale
em unidade de comprimento, é igual a
R2H ( 4 3 − π ) R2H ( 4 3 − π ) R2H ( 2 3 − π ) 100 10 100 5
a) c) e) a) 1. b) . c) . d) . e) .
4 2 2 105 11 115 6

3π 3R2H R2H ( 3 3 − π )
b) d)
2 2 11. (ITA 2012) A superfície lateral de um cone circular reto é um setor
circular de 120º e área igual a 3π cm². A área total e o volume deste
04. Uma esfera de raio r está sobre uma mesa. Uma fonte de luz cone medem, em cm² e cm³, respectivamente
pontual está posicionada diretamente sobre o centro da esfera a uma 2π 2 c) 4π e π 2 e) π e 2π 2
altura h acima da mesa. A sombra da esfera tem área igual à área da a) 4π e
3
superfície da esfera. O valor de h, em função do raio r, é:
2π 2
π 2 d) 3π e
4r 8r 8r b) 4π e 3
a) 2r b) c) d) 3r e) 3
3 3 3
12. (ITA 2012) Em um plano estão situados uma circunferência ω de
05. (ITA 2005) A circunferência inscrita num triângulo equilátero com
lados de 6 cm de comprimento é a interseção de uma esfera de raio raio 2 cm e um ponto P que dista 2 2 cm do centro de ω. Considere os
igual a 4 cm com o plano do triângulo. Então, a distância do centro da segmentos PA e PB tangentes a ω nos pontos A e B, respectivamente.
esfera aos vértices do triângulo é (em cm): Ao girar a região fechada delimitada pelos segmentos PA e PB e pelo
c) 5 e) arco menor AB em torno de um eixo passando pelo centro de ω e
a) 3 3 2 5
perpendicular ao segmento PA, obtém-se um sólido de revolução.
b) 6 d) 4
Determine:

PROMILITARES.COM.BR 255
GEOMETRIA ESPACIAL II

a) A área total da superfície do sólido. a) Calcule o volume de S, em cm³, quando α = 30º.


b) O volume do sólido. π
b) Considere < α < π . Seccionando S por um plano que contém
2
13. (ITA 2011) Uma esfera está inscrita em uma pirâmide regular ED e é perpendicular a AB dividimos S em dois sólidos, S1 e S2
10 Sendo R a razão entre o maior volume dentre os dois sólidos e o
hexagonal cuja altura mede 12 cm e a aresta da base mede 3cm.
3 menor, determine R em função de cos α.
Então o raio da esfera, em cm, é igual a
10 13 15 10
a) 3. b) . c) . d) 2 3. e) . EXERCÍCIOS DE
3 3 4 3

14. (ITA 2010) As superfícies de duas esferas se interceptam COMBATE


ortogonalmente (isto é, em cada ponto da intersecção os respectivos
planos tangentes são perpendiculares). Sabendo que os raios destas
3 01. (ITA 2019) A superfície lateral de um cone circular reto corresponde
esferas medem 2 cm e cm, respectivamente, calcule a um setor circular de 216º, quando planificada. Se a geratriz do cone
2
a) a distância entre os centros das duas esferas. mede 10 cm, então a medida de sua altura, em cm, é igual a
b) a área da superfície do sólido obtido pela intersecção das duas a) 5. b) 6. c) 7. d) 8. e) 9.
esferas.
02. (ESPCEX (AMAN) 2019) O volume de uma esfera inscrita em um
15. (IME 2018) Seja um cubo regular, onde os centros de suas cubo com volume 216 cm³ é igual a
faces são vértices de um octaedro. Por sua vez, os centros das a) 38π cm³. d) 32π cm³.
faces deste octaedro formado são vértices de outro cubo. Obtendo
b) 36π cm³. e) 30π cm³.
consecutivamente octaedros e cubos infinitamente, determine a razão
da soma do volume de todos os poliedros inscritos pelo volume do c) 34π cm³.
cubo inicial.
03. (ITA 2019) Os volumes de um tronco de cone, de uma esfera de
16. (IME 2017) Em um cone equilátero são inscritas duas esferas raio 5 cm e de um cilindro de altura 11 cm formam nessa ordem uma
3 −1 progressão aritmética. O tronco de cone é obtido por rotação de um
de raios R e R conforme a figura abaixo. Um plano secante trapézio retângulo, de altura 4 cm e bases medindo 5 cm e 9 cm, em
3 +1
torno de uma reta passando pelo lado de menor medida. Então, o raio
ao cone é traçado de forma que este seja tangente às duas esferas.
da base do cilindro é, em cm, igual a
Determine em termos de R o maior segmento possível que une dois
pontos da curva formada pela interseção do referido plano com o cone. a) 2 2. d) 2 5.
b) 2 3. e) 2 6.
c) 4

04. (EPCAR (AFA) 2018) Considere o sólido geométrico obtido pela


rotação de 360º do triângulo ABC em torno da reta que passa por C
e é paralela ao lado AB.
Sabe-se que este triângulo é isósceles, com AC ≡ BC =
R 2 m, AB = 2R m
(sendo R uma constante real não nula), e que o volume do sólido
obtido é V= 4 π 3 m .
3

A medida de R, em metros, é igual a


6 3 3
a) 3 b) 3 c) 9 d) 3
17. (IME 2015) Um tetraedro regular, com arestas de comprimento
igual a d, é cortado por 2 planos paralelos entre si e a uma das bases,
dividindo-o em 3 sólidos de volumes iguais. Determine a altura de 05. (EPCAR (AFA) 2017) Se uma pirâmide hexagonal regular está
cada um destes 3 sólidos em função de d. 10 3
inscrita num cone equilátero cujo volume é igual a π cm3 ,
7
18. (FGV 2012) Um losango ABCD de lado 12 cm e medida do ângulo então o volume dessa pirâmide, em cm³, é igual a
BÂD igual a α é rotacionado por um eixo sobre AB, gerando um sólido
45 15 3 30 3 135
de revolução denotado por S. a) b) c) d)
7 7 7 7

06. (EN 2016) Um cilindro circular reto tem área total A, raio da
base R e altura h. Se o volume máximo desse cilindro é expresso
por um número real m e a função f da variável real x é definida por
1
f(x) =(2πx 2 ) 3 + 1, pode-se dizer que f(m).
1 1
a) A d) (A − 3)
3 3
b) A + 3 2
e) A +1
3
1
c) (A + 3)
3

256 PROMILITARES.COM.BR
GEOMETRIA ESPACIAL II

07. (ITA 2016) Uma esfera S1, de raio R > 0, está inscrita num cone 15. Os raios das bases de um tronco de cone de revolução medem
circular reto K. Outra esfera, S2, de raio r, com 0 < r < R, está contida no 1 cm e 7 cm e a geratriz 10 cm. A que distância da base menor se
interior de K e é simultaneamente tangente à esfera S1 e à superfície deve traçar um plano paralelo à base para que se tenha dois troncos
lateral de K. O volume de K é igual a de cone de mesma área lateral?
πR5 πR5 5πR5
a) . c) . e) . 16. (ITA 2011) Considere uma esfera Ω com centro em C e raio
3r(R − r) r(R − r) 3r(R − r)
r = 6 cm e um plano Σ que dista 2 cm de C. Determine a área da
2πR5 4 πR5 intersecção do plano Σ com uma cunha esférica de 30º em Ω que
b) . d) . tenha aresta ortogonal a Σ.
3r(R − r) 3r(R − r)

17. (ITA 2012) Em um plano estão situados uma circunferência ω de


08. (ITA 2015) Uma taça em forma de cone circular reto contém um
raio 2 cm e um ponto P que dista 2 2 cm do centro de ω. Considere os
certo volume de um líquido cuja superfície dista h do vértice do cone.
segmentos PA e PB tangentes a ω nos pontos A e B, respectivamente.
Adicionando-se um volume idêntico de líquido na taça, a superfície do
Ao girar a região fechada delimitada pelos segmentos PA e PB e pelo
líquido, em relação à original, subirá de
arco menor AB  em torno de um eixo passando pelo centro de ω
a) 3
2 − h. c) ( 3 2 − 1)h. h
e) . e perpendicular ao segmento PA, obtém-se um sólido de revolução.
d) h 2 Determine:
b) 3
2 − 1.
a) A área total da superfície do sólido.
09. (ESPCEX (AMAN) 2015) Um cone de revolução tem altura 4 cm b) O volume do sólido.
e está circunscrito a uma esfera de raio 1 cm. O volume desse cone
(em cm³) é igual a
18. (IME 2010) Sejam ABC um triângulo equilátero de lado 2 cm e
1 4 e) 3π. r uma reta situada no seu plano, distante 3 cm do seu baricentro.
a) π. c) π.
3 3 Calcule a área da superfície gerada pela rotação deste triângulo em
torno da reta r.
2 8
b) π. d) π.
3 3
19. Dada uma esfera S de diâmetro AB = 2R. Considera-se o cone C
de altura AB e de raio R. Calcular o volume do sólido comum à esfera
10. (EN 2015) Um prisma quadrangular regular tem área lateral 36 6 S e ao cone C.
unidades de área. Sabendo que suas diagonais formam um ângulo de
60º com suas bases, então a razão do volume de uma esfera de raio 20. (IME 2019) Um cubo com diagonal principal AG é interceptado
241/6 unidades de comprimento para o volume do prisma é pelo plano α, perpendicular à AG, formando uma seção hexagonal
8 8π 81 regular. Calcule, em função da aresta a do cubo:
a) c) e)
81π 81 8π a) o apótema dessa seção hexagonal;
81π 8π b) o raio da esfera que é tangente a essa seção e às faces do cubo
b) d)
8 27 que contém o vértice A.

11. (EN 1999) Para executar a manutenção dos aviões A4 recentemente


adquiridos, a Marinha Brasileira está construindo na Base Aérea Naval GABARITO
de São Pedro D`Aldeia, um hangar que tem a forma de uma seção de
EXERCÍCIOS DE TREINAMENTO
um cilindro circular reto, como mostra a figura abaixo:
01. Para o cilindro da vinícola, temos que Vtotal =Sbase × H =πR2H .
Por outro lado, para o cilindro dos mercados, temos que
2
 R  H πR H
2
V'total =S'base × H' =π  =
 4 5 80

Cada mercado recebe dois cilindros, logo cada mercado deve receber
πR2H
um volume de vinho de .
40
Sabendo que H = 15 m, L = 30 3 m e C = 100 m, encontre o volume Conclui-se então que 40 mercados são suficientes para esgotar todo
do hangar. o vinho da vinícola.
02.
12. Em um cone de revolução inscreve-se uma semiesfera, cujo círculo
máximo descansa sobre a base do cone. Determinar o ângulo do
vértice do cone, sabendo que a razão entre a área total do cone e a
18
área da semiesfera é .
5

13. Calcular os raios das esferas circunscrita e inscrita a um cone de


revolução, dados o raio da base r e a altura h.

14. Um reservatório cilíndrico C de raio igual a r contém um líquido


até o nível H. Este líquido cabe todo num reservatório T, tronco cônico,
3
de altura igual a H e menor raio igual ao raio de C. Sendo R o raio
7
maior de T, determine o menor valor possível de R.
Seja o retângulo EFGH a interseção do cilindro com o plano ABM.

PROMILITARES.COM.BR 257
GEOMETRIA ESPACIAL II

6 Como a sombra da esfera tem área igual à área da superfície da


Seja a = 3 aresta e H a altura do tetraedro, r o raio e h = a altura esfera, temos: π ⋅ R2 = 4 π ⋅ r 2 ⇔ R = 2r
do cilindro. 3
Aplicando o teorema de Pitágoras no triângulo retângulo ATI, temos:
2 2
 1 a 3  a 3 2 2 6 π ⋅ R2 = 4 π ⋅ r 2 ⇔ R = 2r
H + ⋅
2
=  ⇔ H2= a ⇔ H= a
 3 2   2  3 3
r h2 − 2rh 1 h (h − 2r )
1 1 a 3 a 3 ∆ATI ~ ∆AMC ⇒
= =
⇒ ⇔
OM = ⋅ BM = ⋅ = 2r h 4 h2
3 3 2 6
8r
6 ⇔ h = 4h − 8r ⇔ h =
a−h 3
AI IE 3 r
∆AIE ~ ∆AOM ⇒ = ⇔ = ⇔
AO OM 6 3
a a
3 6 05.
6
6 ⋅3− 3 ⋅
3 3
⇔r= =
6 2 3
2
 3 6 π 6 3
VCILINDRO = πr 2 ⋅ h = π ⋅  ⋅ = cm
 3  3 9

03. Seja a figura abaixo a seção reta do sólido formado.

O raio da circunferência inscrita é igual à terça parte da altura do


1 3
triângulo equilátero, ou seja, r =⋅ 6 =3 .
3 2
A distância do centro da esfera ao plano do triângulo é dada por:
h2 = R2 − r 2 = 42 − ( 3 ) = 13 ⇔ h =
2
13

A distância d do centro da esfera a um vértice do triângulo é:

d2 = h2 + x 2 = ( 13 ) + ( 2 3 ) = 25 ⇔ d = 5 cm
2 2

Note que x é igual ao raio da circunferência circunscrita ao triângulo


equilátero que é o dobro do raio da circunferência inscrita.
O sólido pedido é uma superfície cilíndrica reta de seção transversal
dada pela área sombreada e altura H. 06.
A área da seção sombreada é igual à área de um triângulo equilátero 1
de lado 2R menos a área de três setores circulares de 60º e raio R, ou V =πh (R12 + R1R2 + R22 )
3
( 2R)2 3 1  π 2 R1 = 50
seja,=S − 3 ⋅ ⋅ π=
R2  3 −  ⋅R .
4 6  2 R1 ' = 1,21⋅ 50 = 60,5 ⇒ V' = 1,21⋅ V

R2H ( 2 3 − π )
1 1

Logo, o volume pedido é dado por V=  3 −
π 2 πh (60,52 + 60,5 ⋅ R2 + R22 ) = 1,21⋅ πh (502 + 50 ⋅ R2 + R22 )
  ⋅ R ⋅ H
= . 3 3
2 2
⇔ 3660,25 + 60,5 ⋅ R2 + R= 2
2 3025 + 60,5 ⋅ R2 + 1,21R22
04. A figura abaixo é a seção meridiana da esfera e do cone de luz, ⇔ 0,21R=
2
2 635,25 ⇔ R=
2
2 3025 ⇔ R=
2 55
onde A é a fonte de luz pontual.
Logo, o diâmetro é 110 mm.

07. De acordo com o enunciado:

258 PROMILITARES.COM.BR
GEOMETRIA ESPACIAL II

Considerando: 10. Determinando os coeficientes angulares das retas BC e AC


V = volume total do cone 5 −1 0 −1 1
=
mBC = 2 e=
mAC = −
v' = volume cheio (tronco) 2−0 2−0 2
v'' = volume vazio (topo)
Como mBC ⋅ mAC = −1, concluímos que o triângulo ABC é retângulo
=
H 12 = altura total em C. Logo, o segmento AB = 5 é diâmetro da circunferência
h= 6= altura topo / altura tronco circunscrita no triângulo.
Pode-se calcular: 5
Portanto, o raio dessa circunferência é .
V  H
3 3 2
 12 V
=   →  = → V = 8v''  5
2
v''  h   6 v'' π.   .8
volume  2 100
V 7 Logo, = =
v'+ v'' = V → v'+ = V → v' = V área total da superfície 5  5
2
105
8 8 2π ⋅ ⋅ 8 + 2.π ⋅  
1 1 2  2
V= ⋅ π ⋅ R ⋅ H=
2
⋅ 3,14 ⋅ 4 ⋅ 12 → V= 200,96
2

3 3
7 7
v' = V = ⋅ 200,96 → v' = 175,85 m3
8 8
Tempo : 500 L / min = 0,5 m3 / min
1min 0,5 m3
t 175,85 m3
=t 351,7 min ≈ 5h e 50 min

08. Considere a figura.

11. Considere a figura abaixo.

Sejam Q, R e S, respectivamente, as interseções de α com as arestas


BC, BD e AD. Desde que α é paralelo à aresta AB, temos SR e PQ
paralelos a AB. Analogamente, concluímos que PS e QR são paralelos
a CD. Ademais, sabendo que arestas opostas de um tetraedro regular
são ortogonais, tem-se que o quadrilátero PQRS é um retângulo.
Sendo ABCD regular, os triângulos APS e CQP são equiláteros, e,
portanto, a área pedida é igual a 3 ⋅ 7 – 21 m².

09.
Sabendo que a área do setor circular VAB é 3π cm², segue que
2
π ⋅ VA ⋅ 120°
= 3π ⇔ VA = 3cm.
360°
 é dado por
O comprimento do arco AB

ˆ ⋅ VA = 2π ⋅ 3 = 2π cm.
AVB
3

Desse modo, como o comprimento do arco AB  corresponde ao


comprimento da base cone, obtemos 2π ⋅ r = 2π ⇔ r = 1 cm, em que
r é o raio da base do cone.
Portanto, a área total do cone é 3π + π ⋅ r² = 3π + π ⋅ 1² = 4 πcm².
1 Como VA é a geratriz do cone, temos que
V= Ab .h
3 2
h2 = VA − r 2 ⇔ h2 = 32 − 12 ⇒ h = 2 2 cm, sendo h a altura do cone.
1 6.r 3 3 Por conseguinte, temos que o volume desse cone mede
V= . .r
3 4
1 1 2π 2 3
3
r 3 ⋅ π ⋅ r 2 ⋅ h= ⋅ π ⋅ 12 ⋅ 2 2= cm .
V= 3 3 3
2

PROMILITARES.COM.BR 259
GEOMETRIA ESPACIAL II

12. 14. Aplicando o teorema de Pitágoras na figura, temos:


a) A área total será igual à soma das seguintes áreas. 25 5
a) d2 = 22 + (3/2)2 ⇔ d2 = ⇔d=
4 2

b)

Área da base do cilindro:


Ab =π.22 =4. π cm2
8 5 8 9
Área lateral do cilindro: 22 = (5/2) ⋅ m ⇔ m = e n = − =
5 2 5 10
AL =2.π.2.2 =8. π cm2 Área de uma calota esférica:

Área da parte da esfera, interna ao cilindro (metade da superfície


esférica):
Ai =2.π.22 = 8. π cm2

Logo, a área total será:


8π 2
A = 4 π + 8π + 8π = 20 π cm2 Primeira calota: A1 = 2π ⋅ 2(2-8/5) = cm
5
b) O volume será dado pelo volume do cilindro menos o volume do 9π 2
hemisfério. Segunda calota: A2 = 2π ⋅ 2(3/2 – 9/10) = cm
5
Volume do cilindro:
8π 2 9π 2 17π 2
V=
π.22.2 =
8π cm3 Logo a área total será cm + cm = cm
5 5 5
Volume do hemisfério:
2 16π 2 15. Do enunciado, temos a figura abaixo:
VH = π.23 = cm
3 3
Volume do sólido:
16π 8π 3
V = 8π − = cm
3 3

10 3 3
13. Aresta da base a = . =5
3 2
12 − r r
Por semelhança de triângulos, temos = ⇔ r = 10 / 3cm
13 5

Vcubo = a3

260 PROMILITARES.COM.BR
GEOMETRIA ESPACIAL II

Dessa forma, temos a figura abaixo:

No triângulo P1Q1M,
2 2
 x  x
(PQ1)
2
No triângulo JMP, 1=   +  
2 2
2 2
 a  a x 2
=
x2   +  
 2  2 1 1 =
PQ
2
a 2
x= Da semelhança entre os triângulos IP1Q1 e IA1D1,
2
IA1 y
=
VIJKLM: Volume da pirâmide de base quadrada JKLM e vértice I. IP1 PQ 1 1

Voctaedro= 2 ⋅ VIJKLM 2 y
Voctaedro= 2 ⋅ VIJKLM =
1 a 3 x 2
Voctaedro = 2 ⋅ 1 ⋅ x 22 ⋅ a
Voctaedro = 2 ⋅ 3 ⋅ x ⋅ 2 2
3 2
a ⋅ x2 2 x 2
Voctaedro = a ⋅ x 2 y= ⋅
Voctaedro = 3 3 2
3 2
 a 2 2 x 2
a ⋅  a 2  y=
a⋅  2  3
Voctaedro =  2 
Voctaedro = 3 Assim, o volume do cubo A1B1C1D1E1F1G1H1 é dado por:
3
a a22 ⋅ 2 3
Voctaedro= a ⋅ a ⋅ 2  x 2
Voctaedro= 3 ⋅ 4 VA1B1C1D1E11F G1H= y=
3
 3 
3 4
1
1
 
Voctaedro= 1 ⋅ a33
Voctaedro= 6 ⋅ a x3 ⋅ 2 2
6 VA1B1C1D1E11F G1H1 =
1 27
Voctaedro = 1 Vcubo
Voctaedro = 6 Vcubo
6 a 2
Mas, x = , logo,
Agora, observemos o octaedro e o cubo inscrito nele. 2
3
1  a 2
VA1B1C1D1E11F G1H1 =⋅  ⋅2 2
27  2 
1 1
VA1B1C1D1E11F G1H1 = ⋅ a3 ⋅ 2 2 ⋅ 2 2 ⋅
27 8
1 3
VA1B1C1D1E11F G1H1 = a
27
2  1 3
VA1B1C1D1E11F G1H1= ⋅ a 
9 6 

2
Então, o volume do cubo inscrito no octaedro equivale a do volume
do octaedro. 9
Dessa forma, sendo V o volume do primeiro cubo, temos:
1
Volume do primeiro octaedro: V
6
2 1 V
Volume do segundo cubo: ⋅ V=
9 6 27
A1 é baricentro do triângulo IJM. 1 2 1 V
Volume do segundo octaedro: ⋅ ⋅ V=
D1 é baricentro do triângulo ILM. 6 9 6 162

PROMILITARES.COM.BR 261
MV
GEOMETRIA ESPACIAL II = =
3 tg 60 ° → MV
= R 3
R

NV =R ⋅
3 −1
⋅ 3 → NV =
R⋅ 3− 3 ( )
3 +1 3 +1

Volume do terceiro cubo:


2 1 2 1
⋅ ⋅ ⋅ V=
V
MN =
R 3−
(
R⋅ 3− 3 )=
R ⋅ (3 + 3 −3+ 3 )=
2R 3
9 6 9 6 729
3 +1 3 +1 3 +1
1 2 1 2 1
Volume do terceiro octaedro: ⋅ ⋅ ⋅ ⋅ V =
6 9 6 9 6
V
4374
( )
MN = R 3 − 3 → maior segmento possível

Sendo r a razão pedida, temos:


V V V   V V  17. Sabendo que trata-se de um tetraedro regular, sua altura total
 + + + ... +  +
  27 729
+ ...
 será igual a:
6 162 4374
r=
V d 6
V V htotal =
3
6 + 27
1 1 Considerando como h1 a altura do tetraedro formado no topo
1− 1− (primeiro plano de corte formando pirâmide de base triangular), a
r= 27 27
V relação entre a altura h1 e a altura total do tetraedro pode ser escrita
11 como:
r= 3
52  h1  1 1 d 6
 h  = 3 → h1 = 3 3 ⋅ htotal → h1 = 3 3 3
11 total
Resposta:
52 Considerando como h1 + h2 a altura do tetraedro formado no segundo
plano de corte (formando também uma pirâmide de base triangular),
16. Teremos: a relação entre a altura h1 + h2 e a altura total do tetraedro pode ser
escrita como:
3
 h1 + h2  2 3
2
 h  = 3 → h1 + h2 = 3 3 ⋅ htotal →
total
3
2 d 6 d 63 2
→ h1 + h2 = ⋅ → h1 + h2 =
3
3 3 33 3

=
h2
d 63 2
− h1 →=
h2
d 63 2 − d 6
→=
h2
d 6 3 2 −1 ( )
3 3
3 3 3 3 33 3
Por fim, sabe-se que a soma de cada uma das alturas dos sólidos
gerados é igual a altura total do tetraedro, portanto:
htotal = h1 + h2 + h3 → h3 = htotal − (h1 + h2 )

h3=
d 6 d 63 2
− → h =
d 63 3 − d 63 2
→ h =
d 6 33− 32 ( )
3 3
3 33 3 33 3 33 3
Assim, a altura de cada um destes três sólidos em função de d será:
Como A, B, X e Y são pontos de tangência, podemos escrever:
PA = PX =h1
d 6
= h2
d 6 3 2 −1
= h3
(
d 6 33− 32 ) ( )
3 3
3 3 3 3 33 3
PB = PY
PA + PB = PX + PY = XY → geratriz do tronco de cone
18. a)
Assim, podemos desenhar:

R= 12 ⋅ sen30°= 12 ⋅ 0,5= 6cm


O volume pedido é igual ao volume do cilindro da figura:
V = π ⋅ 62 ⋅ 12 = 432πcm2.

π
b) Considerando < α < π não existe um plano que passa por DE
2
que seja perpendicular ao segmento AB. Portanto não será possível
MV resolver o item [B].
= =
3 tg 60 ° → MV
= R 3
R

NV =R ⋅
3 −1
⋅ 3 → NV =
R⋅ 3− 3 ( )
3 +1 3 +1

MN =
R 3−
(
R⋅ 3− 3 )=
R ⋅ (3 + 3 −3+ 3 )=
2R 3
3 +1 3 +1 3 +1
262 ( )
MN = R 3 − 3 → maior segmento possível
PROMILITARES.COM.BR
GEOMETRIA ESPACIAL II

EXERCÍCIOS DE COMBATE
01. D 14. R = 2r
02. B 16
15. x = cm
03. B 3
04. D 8π
16. cm2
05. A 3
06. C 17. a) 20p cm²
07. B 8π
b) cm3
08. C 3
18. 36p cm²
09. D
19. está sem gabarito
10. C

(
11. 7500 4 π − 3 3 m3 ) 20. a) ap =
a 6
4

12. 2arcsen
5
ou 2arcsen
1
b) r =
(3 − 3 ) ⋅ a
6 6
4

13.=
x
r
h
( h +r −r
2 2
)
ANOTAÇÕES

PROMILITARES.COM.BR 263
GEOMETRIA ESPACIAL II

ANOTAÇÕES

264 PROMILITARES.COM.BR

Você também pode gostar